LayerRx Mapping ID
318
Slot System
Featured Buckets
Featured Buckets Admin
Medscape Lead Concept
3029493

How to refine your approach to peripheral arterial disease

Article Type
Changed
Tue, 05/03/2022 - 15:07

Peripheral arterial disease (PAD), the progressive disorder that results in ischemia to distal vascular territories as a result of atherosclerosis, spans a wide range of presentations, from minimally symptomatic disease to limb ischemia secondary to acute or chronic occlusion.

The prevalence of PAD is variable, due to differing diagnostic criteria used in studies, but PAD appears to affect 1 in every 22 people older than age 40.1 However, since PAD incidence increases with age, it is increasing in prevalence as the US population ages.1-3

PAD is associated with increased hospitalizations and decreased quality of life.4 Patients with PAD have an estimated 30% 5-year risk for myocardial infarction, stroke, or death from a vascular cause.3

Screening. Although PAD is underdiagnosed and appears to be undertreated,3 population-based screening for PAD in asymptomatic patients is not recommended. A Cochrane review found no studies evaluating the benefit of ­asymptomatic population-based screening.5 Similarly, in 2018, the USPSTF performed a comprehensive review and found no studies to support routine screening and determined there was insufficient evidence to recommend it.6,7

Risk factors and associated comorbidities

PAD risk factors, like the ones detailed below, have a potentiating effect. The presence of 2 risk factors doubles PAD risk, while 3 or more risk factors increase PAD risk by a factor of 10.1

Increasing age is the greatest single risk factor for PAD.1,2,8,9 Researchers using data from the National Health and Nutrition Examination Survey (NHANES) found that the prevalence of PAD increased from 1.4% in individuals ages 40 to 49 years to almost 17% in those age 70 or older.1

JFP06912500_300x300.JPG


 

Patients with PAD have an estimated 30% 5-year risk for myocardial infarction, stroke, or death from a vascular cause.

Demographic characteristics. Most studies demonstrate a higher risk for PAD in men.1-3,10 African-American patients have more than twice the risk for PAD, compared with Whites, even after adjustment for the increased prevalence of associated diseases such as hypertension and diabetes in this population.1-3,10

 

Continue to: Genetics...

 

 



Genetics. A study performed by the National Heart Lung and Blood Institute suggested that genetic correlations between twins were more important than environmental factors in the development of PAD.11

Smoking. Most population studies show smoking to be the greatest modifiable risk factor for PAD. An analysis of the NHANES data yielded an odds ratio (OR) of 4.1 for current smokers and of 1.8 for former smokers.1 Risk increases linearly with cumulative years of smoking.1,2,9,10

Diabetes is another significant modifiable risk factor, increasing PAD risk by 2.5 times.2 Diabetes is also associated with increases in functional limitation from claudication, risk for acute coronary syndrome, and progression to amputation.1

Hypertension nearly doubles the risk for PAD, and poor control further increases this risk.2,9,10

Chronic kidney disease (CKD). Patients with CKD have a progressively higher prevalence of PAD with worsening renal function.1 There is also an association between CKD and increased morbidity, revascularization failure, and increased mortality.1

Two additional risk factors that are less well understood are dyslipidemia and chronic inflammation. There is conflicting data regarding the role of individual components of cholesterol and their effect on PAD, although lipoprotein (a) has been shown to be an independent risk factor for both the development and progression of PAD.12 Similarly, chronic inflammation has been shown to play a role in the initiation and progression of the disease, although the role of inflammatory markers in evaluation and treatment is unclear and assessment for these purposes is not currently recommended.12,13

Continue to: Diagnosis...

 

 

Diagnosis

Clinical presentation

Lower extremity pain is the hallmark symptom of PAD, but presentation varies. The classic presentation is claudication, pain within a defined muscle group that occurs with exertion and is relieved by rest. Claudication is most common in the calf but also occurs in the buttock/thigh and the foot.

 

African- American patients have more than twice the risk for PAD, compared with Whites, even after adjustment for the increased prevalence of associated diseases in this population.

However, most patients with PAD present with pain that does not fit the definition of claudication. Patients with comorbidities, physical inactivity, and neuropathy are more likely to present with atypical pain.14 These patients may demonstrate critical or acute limb ischemia, characterized by pain at rest and most often localized to the forefoot and toes. Patients with critical limb ischemia may also present with nonhealing wounds/ulcers or gangrene.15

Physical exam findings can support the diagnosis of PAD, but none are reliable enough to rule the diagnosis in or out. Findings suggestive of PAD include cool skin, presence of a bruit (iliac, femoral, or popliteal), and palpable pulse abnormality. Multiple abnormal physical exam findings increase the likelihood of PAD, while the absence of a bruit or palpable pulse abnormality makes PAD less likely.16 In patients with PAD, an associated wound/ulcer is most often distal in the foot and usually appears dry.17

The differential diagnosis for intermittent leg pain is broad and includes neurologic, musculoskeletal, and venous etiologies. Table 118 lists some common alternate diagnoses for patients presenting with leg pain or claudication.

 

JFP06912500_t1.JPG

 

Continue to: Diagnostic testing...

 

 

Diagnostic testing

An ankle-brachial index (ABI) test should be performed in patients with history or physical exam findings suggestive of PAD. A resting ABI is performed with the patient in the supine position, with measurement of systolic blood pressure in both arms and ankles using a Doppler ultrasound device. Table 213 outlines ABI scoring and interpretation.

JFP06912500_t2.JPG

 

An ABI > 1.4 is an invalid measurement, indicating that the arteries are too calcified to be compressed. These highly elevated ABI measurements are common in patients with diabetes and/or advanced CKD. In these patients, a toe-brachial index (TBI) test should be performed, because the digital arteries are almost always compressible.13

Patients with symptomatic PAD who are under consideration for revascularization may benefit from radiologic imaging of the lower extremities with duplex ultrasound, computed tomography angiography, or magnetic resonance angiography to determine the anatomic location and severity of stenosis.13

 

Management of PAD

Lifestyle interventions

For patients with PAD, lifestyle modifications are an essential—but challenging—component of disease management.

Continue to: Smoking cessation...

 

 

Smoking cessation. As with other atherosclerotic diseases, PAD progression is strongly correlated with smoking. A trial involving 204 active smokers with PAD showed that 5-year mortality and amputation rates dropped by more than half in those who quit smoking within a year, with numbers needed to treat (NNT) of 6 for mortality and 5 for amputation.19 Because of this dramatic effect, American College of Cardiology/American Heart Association (ACC/AHA) guidelines encourage providers to address smoking at every visit and use cessation programs and medication to increase quit rates.13

Exercise may be the most important intervention for PAD. A 2017 Cochrane review found that supervised, structured exercise programs increase pain-free and maximal walking distances by at least 20% and also improve physical and mental quality of life.20 In a trial involving 111 patients with aortoiliac PAD, supervised exercise plus medical care led to greater functional improvement than either revascularization plus medical care or medical care alone.21 In a 2018 Cochrane review, neither revascularization or revascularization added to supervised exercise were better than supervised exercise alone.22 ACC/AHA guidelines recommend supervised exercise programs for claudication prior to considering revascularization.13TABLE 313 outlines the components of a structured exercise program.

JFP06912500_t3.JPG



Unfortunately, the benefit of these programs has been difficult to reproduce without supervision. Another 2018 Cochrane review demonstrated significant improvement with supervised exercise and no clear improvement in patients given home exercise or advice to walk.23 A recent study examined the effect of having patients use a wearable fitness tracker for home exercise and demonstrated no benefit over usual care.24

Diet. There is some evidence that dietary interventions can prevent and possibly improve PAD. A large randomized controlled trial showed that a Mediterranean diet lowered rates of PAD over 1 year compared to a low-fat diet, with an NNT of 336 if supplemented with extra-virgin olive oil and 448 if supplemented with nuts.25 A small trial of 25 patients who consumed non-soy legumes daily for 8 weeks showed average ABI improvement of 6%, although there was no control group.26

Medical therapy to address peripheral and cardiovascular events

Standard medical therapy for coronary artery disease (CAD) is recommended for patients with PAD to reduce cardiovascular and limb events. For example, treatment of hypertension reduces cardiovascular and cerebrovascular events, and studies verify that lowering blood pressure does not worsen claudication or limb perfusion.

A trial involving 204 active smokers with PAD showed that 5-year mortality and amputation rates dropped by more than half in those who quit smoking within a year.

13TABLE 413,27-30 outlines the options for medical therapy.

 

JFP06912500_t4.JPG

 

Continue to: Statins...

 

 

Statins reduce cardiovascular events in PAD patients. A large study demonstrated that 40 mg of simvastatin has an NNT of 21 to prevent a coronary or cerebrovascular event in PAD, similar to the NNT of 23 seen in treatment of CAD.27 Statins also reduce adverse limb outcomes. A registry of atherosclerosis patients showed that statins have an NNT of 56 to prevent amputation in PAD and an NNT of 28 to prevent worsening claudication, critical limb ischemia, revascularization, or amputation.28

Antiplatelet therapy with low-dose aspirin or clopidogrel is recommended for symptomatic patients and for asymptomatic patients with an ABI ≤ 0.9.13 A Cochrane review demonstrated significantly reduced mortality with nonaspirin antiplatelet agents vs aspirin (NNT = 94) without increase in major bleeding.29 Only British guidelines specifically recommend clopidogrel over aspirin.31

Dual antiplatelet therapy has not shown consistent benefits over aspirin alone. ACC/AHA guidelines state that dual antiplatelet therapy is not well established for PAD but may be reasonable after revascularization.13

Voraxapar is a novel antiplatelet agent that targets the thrombin-binding receptor on platelets. However, trials show no significant coronary benefit, and slight reductions in acute limb ischemia are offset by increases in major bleeding.13

For patients receiving medical therapy, ongoing evaluation and treatment should be based on claudication symptoms and clinical assessment.

Medical therapy for claudication

Several medications have been proposed for symptomatic treatment of intermittent claudication. Cilostazol is a phosphodiesterase inhibitor with the best risk-benefit ratio. A Cochrane review showed improvements in maximal and pain-free walking distances compared to placebo and improvements in quality of life with cilostazol 100 mg taken twice daily.32 Adverse effects included headache, dizziness, palpitations, and diarrhea.29

Continue to: Pentoxifylline...

 

 

Pentoxifylline is another phosphodiesterase inhibitor with less evidence of improvement, higher adverse effect rates, and more frequent dosing. It is not recommended for treatment of intermittent claudication.13,33

Supplements. Padma 28, a Tibetan herbal formulation, appears to improve maximal walking distance with adverse effect rates similar to placebo.34 Other supplements, including vitamin E, ginkgo biloba, and omega-3 fatty acids, have no evidence of benefit.35-37

When revascularizationis needed

Patients who develop limb ischemia or lifestyle-limiting claudication despite conservative therapy are candidates for revascularization. Endovascular techniques include angioplasty, stenting, atherectomy, and precise medication delivery. Surgical approaches mainly consist of thrombectomy and bypass grafting. For intermittent claudication despite conservative care, ACC/AHA guidelines state endovascular procedures are appropriate for aortoiliac disease and reasonable for femoropopliteal disease, but unproven for infrapopliteal disease.13

Acute limb ischemia is an emergency requiring immediate intervention. Two trials revealed identical overall and amputation-free survival rates for percutaneous thrombolysis and surgical thrombectomy.38,39 ACC/AHA guidelines recommend anticoagulation with heparin followed by the revascularization technique that will most rapidly restore arterial flow.13

For chronic limb ischemia, a large trial showed angioplasty had lower initial morbidity, length of hospitalization, and cost than surgical repair. However, surgical mortality was lower after 2 years.40 ACC/AHA guidelines recommend either surgery or endovascular procedures and propose initial endovascular treatment followed by surgery if needed.13 After revascularization, the patient should be followed periodically with a clinical evaluation and ABI measurement with further consideration for routine duplex ultrasound surveillance.13

For chronic limb ischemia, a large trial showed angioplasty had lower initial morbidity, length of hospitalization, and cost than surgical repair. Surgical mortality was lower after 2 years.

Outcomes

Patients with PAD have variable outcomes. About 70% to 80% of patients with this diagnosis will have a stable disease process with no worsening of symptoms, 10% to 20% will experience worsening symptoms over time, 5% to 10% will require revascularization within 5 years of diagnosis, and 1% to 5% will progress to critical limb ischemia, which has a 5-year amputation rate of 1% to 4%.2 Patients who require amputation have poor outcomes: Within 2 years, 30% are dead and 15% have had further amputations.18

In addition to the morbidity and mortality from its own progression, PAD is an important predictor of CAD and is associated with a significant elevation in morbidity and mortality from CAD. One small but well-designed prospective cohort study found that patients with PAD had a more than 6-fold increased risk of death from CAD than did patients without PAD.41

Acknowledgement
The authors thank Francesca Cimino, MD, FAAFP, for her help in reviewing this manuscript.

CORRESPONDENCE
Dustin K. Smith, DO, 2080 Child Street, Jacksonville, FL 32214; dustinksmith@yahoo.com

References

1. Eraso LH, Fukaya E, Mohler ER 3rd, et al. Peripheral arterial disease, prevalence and cumulative risk factor profile analysis. Eur J Prev Cardiol. 2014;21:704-711.

2. Pasternak RC, Criqui MH, Benjamin EJ, et al; American Heart Association. Atherosclerotic Vascular Disease Conference: Writing Group I: epidemiology. Circulation. 2004;109:2605-2612.

3. Hirsch AT, Criqui MH, Treat-Jacobson D, et al. Peripheral arterial disease detection, awareness, and treatment in primary care. JAMA. 2001;286:1317-1324.

4. Olin JW, Sealove BA. Peripheral artery disease: current insight into the disease and its diagnosis and management. Mayo Clin Proc. 2010;85:678-692.

5. Andras A, Ferkert B. Screening for peripheral arterial disease. Cochrane Database Syst Rev. 2014;(4):CD010835.

6. Guirguis-Blake JM, Evans CV, Redmond N, et al. Screening for peripheral artery disease using ankle-brachial index: updated evidence report and systematic review for the US Preventive Services Task Force. JAMA. 2018;320:184-196.

7. US Preventive Services Task Force. Screening for peripheral artery disease and cardiovascular disease risk assessment with ankle-brachial index: US Preventive Services Task Force recommendation statement. JAMA. 2018;230:177-183.

8. American Heart Association Writing Group 2. Atherosclerotic Peripheral Vascular Disease Symposium II: screening for atherosclerotic vascular diseases: should nationwide programs be instituted? Circulation. 2008;118:2830-2836.

9. Berger JS, Hochman J, Lobach I, et al. Modifiable risk factor burden and the prevalence of peripheral artery disease in different vascular territories. J Vasc Surg. 2013;58:673-681.

10. Joosten MM, Pai JK, Bertoia ML, et al. Associations between conventional cardiovascular risk factors and risk of peripheral artery disease in men. JAMA. 2012;308:1660-1667.

11. Carmelli D, Fabsitz RR, Swan GE, et al. Contribution of genetic and environmental influences to ankle-brachial blood pressure index in the NHLBI Twin Study. National Heart, Lung, and Blood Institute. Am J Epidemiol. 2000;151:452-458.

12. Aboyans V, Criqui MH, Denenberg JO, et al. Risk factors for progression of peripheral arterial disease in large and small vessels. Circulation. 2006;113:2623-2629.

13. Gerald-Herman MD, Gornik HL, Barrett C, et al. 2016 AHA/ACC guideline on the management of patients with lower extremity peripheral artery disease: a report of the American College of Cardiology/American Heart Association Task Force on Clinical Practice Guidelines. Circulation. 2017;135:e726-e779.

14. McDermott MM, Greenland P, Liu K, et al. Leg symptoms in peripheral arterial disease: associated clinical characteristics and functional impairment. JAMA. 2001;286:1599-1606.

15. Cranley JJ. Ischemic rest pain. Arch Surg. 1969;98:187-188.

16. Khan NA, Rahim SA, Anand SS, et al. Does the clinical examination predict lower extremity peripheral arterial disease? JAMA. 2006;295:536-546.

17. Wennberg PW. Approach to the patient with peripheral arterial disease. Circulation. 2013;128:2241-2250.

18. Norgren L, Hiatt WR, Dormandy JA, et al. Inter-society consensus for the management of peripheral arterial disease (TASC II). Eur J Vas Endovasc Surg. 2007;33:S1-S75.

19. Armstrong EJ, Wu J, Singh GD, et al. Smoking cessation is associated with decreased mortality and improved amputation-free survival among patients with symptomatic peripheral artery disease. J Vasc Surg. 2014;60:1565-1571.

20. Lane R, Harwood A, Watson L, et al. Exercise for intermittent claudication. Cochrane Database Syst Rev. 2017;(12):CD000990.

21. Murphy TP, Cutlip DE, Regensteiner JG, et al; CLEVER Study Investigators. Supervised exercise versus primary stenting for claudication resulting from aortoiliac peripheral artery disease: six-month outcomes from the claudication: exercise versus endoluminal revascularization (CLEVER) study. Circulation. 2012;125:130-139.

22. Fakhry F, Fokkenrood HJP, Pronk S, et al. Endovascular revascularization versus conservative management for intermittent claudication. Cochrane Database Syst Rev. 2018;(3):CD010512.

23. Hageman D, Fokkenrood HJ, Gommans LN, et al. Supervised exercise therapy versus home-based exercise therapy versus walking advice for intermittent claudication. Cochrane Database Syst Rev. 2018;(4):CD005263.

24. McDermott MM, Spring B, Berger JS, et al. Effect of a home-based exercise intervention of wearable technology and telephone coaching on walking performance in peripheral artery disease: the HONOR randomized clinical trial. JAMA. 2018;319:1665-1676.

25. Ruiz-Canela M, Estruch R, Corella D, et al. Association of Mediterranean diet with peripheral artery disease: the PREDIMED randomized trial. JAMA. 2014;311:415-417.

26. Zahradka P, Wright B, Weighell W, et al. Daily non-soy legume consumption reverses vascular impairment due to peripheral artery disease. Atherosclerosis. 2013;230:310-314.

27. Heart Protection Study Collaborative Group. Randomized trial of the effects of cholesterol-lowering with simvastatin on peripheral vascular and other major vascular outcomes in 20536 people with peripheral arterial disease and other high-risk conditions. J Vasc Surg. 2007;45:645-655.

28. Kumbhani DJ, Steg G, Cannon CP, et al. Statin therapy and long-term adverse limb outcomes in patients with peripheral artery disease: insights from the REACH registry. Eur Heart J. 2014;35:2864-2872.

29. Wong PF, Chong LY, Mikhailidis DP, et al. Antiplatelet agents for intermittent claudication. Cochrane Database Syst Rev. 2011;(11):CD001272.

30. Critical Leg Ischaemia Prevention Study (CLIPS) Group, Catalano M, Born G, Peto R. Prevention of serious vascular events by aspirin amongst patients with peripheral arterial disease: randomized, double-blind trial. J Intern Med. 2007;261:276-284.

31. Morley RL, Sharma A, Horsch AD, et al. Peripheral artery disease. BMJ. 2018;360:j5842.

32. Bedenis R, Stewart M, Cleanthis M, et al. Cilostazol for intermittent claudication. Cochrane Database Syst Rev. 2014;(10):CD003748.

<--pagebreak-->

33. Salhiyyah K, Forster R, Senanayake E, et al. Pentoxifylline for intermittent claudication. Cochrane Database Syst Rev. 2015;(9):CD005262.

34. Stewart M, Morling JR, Maxwell H. Padma 28 for intermittent claudication. Cochrane Database Syst Rev. 2016;(3):CD007371.

35. Kleijnen J, Mackerras D. Vitamin E for intermittent claudication. Cochrane Database Syst Rev. 1998;(1):CD000987.

36. Nicolai SPA, Kruidenior LM, Bendermacher BLW, et al. Ginkgo biloba for intermittent claudication. Cochrane Database Syst Rev. 2013;(6):CD006888.

37. Campbell A, Price J, Hiatt WR. Omega-3 fatty acids for intermittent claudication. Cochrane Database Syst Rev. 2013;(7):CD003833.

38. American Surgical Association, New York Surgical Society, Philadelphia Academy of Surgery, Southern Surgical Association (US), Central Surgical Association. Results of a prospective randomized trial evaluating surgery versus thrombolysis for ischemia of the lower extremity: the STILE trial. Ann Surg. 1994;220:251-268.

39. Ouriel K, Veith FJ, Sasahara AA. Thrombolysis or peripheral arterial surgery: phase I results. TOPAS Investigators. J Vasc Surg. 1996;23:64-73.

40. Bradbury AW, Ruckley CV, Fowkes FGR, et al. Bypass versus angioplasty in severe ischaemia of the leg (BASIL): multicentre, randomised, controlled trial. Lancet. 2005;366:1925-1934.

41. Criqui MH, Langer RD, Fronek A, et al. Mortality over a period of 10 years in patients with peripheral arterial disease. N Engl J Med. 1992;326:381-386.

Article PDF
Author and Disclosure Information

Jacksonville Family Medicine Residency Program, Naval Hospital Jacksonville, FL ; Uniformed Services University of the Health Sciences, Bethesda, MD ; Naval Hospital Naples, Italy 
dustinksmith@yahoo.com

 

The authors reported no potential conflict of interest relevant to this article.


The views expressed in this article are those of the authors and do not necessarily reflect the official policy or position of the Department of the Navy, Uniformed Services University of the Health Sciences, Department of Defense, or the United States government.


 

Issue
The Journal of Family Practice - 69(10)
Publications
Topics
Page Number
500-506
Sections
Author and Disclosure Information

Jacksonville Family Medicine Residency Program, Naval Hospital Jacksonville, FL ; Uniformed Services University of the Health Sciences, Bethesda, MD ; Naval Hospital Naples, Italy 
dustinksmith@yahoo.com

 

The authors reported no potential conflict of interest relevant to this article.


The views expressed in this article are those of the authors and do not necessarily reflect the official policy or position of the Department of the Navy, Uniformed Services University of the Health Sciences, Department of Defense, or the United States government.


 

Author and Disclosure Information

Jacksonville Family Medicine Residency Program, Naval Hospital Jacksonville, FL ; Uniformed Services University of the Health Sciences, Bethesda, MD ; Naval Hospital Naples, Italy 
dustinksmith@yahoo.com

 

The authors reported no potential conflict of interest relevant to this article.


The views expressed in this article are those of the authors and do not necessarily reflect the official policy or position of the Department of the Navy, Uniformed Services University of the Health Sciences, Department of Defense, or the United States government.


 

Article PDF
Article PDF

Peripheral arterial disease (PAD), the progressive disorder that results in ischemia to distal vascular territories as a result of atherosclerosis, spans a wide range of presentations, from minimally symptomatic disease to limb ischemia secondary to acute or chronic occlusion.

The prevalence of PAD is variable, due to differing diagnostic criteria used in studies, but PAD appears to affect 1 in every 22 people older than age 40.1 However, since PAD incidence increases with age, it is increasing in prevalence as the US population ages.1-3

PAD is associated with increased hospitalizations and decreased quality of life.4 Patients with PAD have an estimated 30% 5-year risk for myocardial infarction, stroke, or death from a vascular cause.3

Screening. Although PAD is underdiagnosed and appears to be undertreated,3 population-based screening for PAD in asymptomatic patients is not recommended. A Cochrane review found no studies evaluating the benefit of ­asymptomatic population-based screening.5 Similarly, in 2018, the USPSTF performed a comprehensive review and found no studies to support routine screening and determined there was insufficient evidence to recommend it.6,7

Risk factors and associated comorbidities

PAD risk factors, like the ones detailed below, have a potentiating effect. The presence of 2 risk factors doubles PAD risk, while 3 or more risk factors increase PAD risk by a factor of 10.1

Increasing age is the greatest single risk factor for PAD.1,2,8,9 Researchers using data from the National Health and Nutrition Examination Survey (NHANES) found that the prevalence of PAD increased from 1.4% in individuals ages 40 to 49 years to almost 17% in those age 70 or older.1

JFP06912500_300x300.JPG


 

Patients with PAD have an estimated 30% 5-year risk for myocardial infarction, stroke, or death from a vascular cause.

Demographic characteristics. Most studies demonstrate a higher risk for PAD in men.1-3,10 African-American patients have more than twice the risk for PAD, compared with Whites, even after adjustment for the increased prevalence of associated diseases such as hypertension and diabetes in this population.1-3,10

 

Continue to: Genetics...

 

 



Genetics. A study performed by the National Heart Lung and Blood Institute suggested that genetic correlations between twins were more important than environmental factors in the development of PAD.11

Smoking. Most population studies show smoking to be the greatest modifiable risk factor for PAD. An analysis of the NHANES data yielded an odds ratio (OR) of 4.1 for current smokers and of 1.8 for former smokers.1 Risk increases linearly with cumulative years of smoking.1,2,9,10

Diabetes is another significant modifiable risk factor, increasing PAD risk by 2.5 times.2 Diabetes is also associated with increases in functional limitation from claudication, risk for acute coronary syndrome, and progression to amputation.1

Hypertension nearly doubles the risk for PAD, and poor control further increases this risk.2,9,10

Chronic kidney disease (CKD). Patients with CKD have a progressively higher prevalence of PAD with worsening renal function.1 There is also an association between CKD and increased morbidity, revascularization failure, and increased mortality.1

Two additional risk factors that are less well understood are dyslipidemia and chronic inflammation. There is conflicting data regarding the role of individual components of cholesterol and their effect on PAD, although lipoprotein (a) has been shown to be an independent risk factor for both the development and progression of PAD.12 Similarly, chronic inflammation has been shown to play a role in the initiation and progression of the disease, although the role of inflammatory markers in evaluation and treatment is unclear and assessment for these purposes is not currently recommended.12,13

Continue to: Diagnosis...

 

 

Diagnosis

Clinical presentation

Lower extremity pain is the hallmark symptom of PAD, but presentation varies. The classic presentation is claudication, pain within a defined muscle group that occurs with exertion and is relieved by rest. Claudication is most common in the calf but also occurs in the buttock/thigh and the foot.

 

African- American patients have more than twice the risk for PAD, compared with Whites, even after adjustment for the increased prevalence of associated diseases in this population.

However, most patients with PAD present with pain that does not fit the definition of claudication. Patients with comorbidities, physical inactivity, and neuropathy are more likely to present with atypical pain.14 These patients may demonstrate critical or acute limb ischemia, characterized by pain at rest and most often localized to the forefoot and toes. Patients with critical limb ischemia may also present with nonhealing wounds/ulcers or gangrene.15

Physical exam findings can support the diagnosis of PAD, but none are reliable enough to rule the diagnosis in or out. Findings suggestive of PAD include cool skin, presence of a bruit (iliac, femoral, or popliteal), and palpable pulse abnormality. Multiple abnormal physical exam findings increase the likelihood of PAD, while the absence of a bruit or palpable pulse abnormality makes PAD less likely.16 In patients with PAD, an associated wound/ulcer is most often distal in the foot and usually appears dry.17

The differential diagnosis for intermittent leg pain is broad and includes neurologic, musculoskeletal, and venous etiologies. Table 118 lists some common alternate diagnoses for patients presenting with leg pain or claudication.

 

JFP06912500_t1.JPG

 

Continue to: Diagnostic testing...

 

 

Diagnostic testing

An ankle-brachial index (ABI) test should be performed in patients with history or physical exam findings suggestive of PAD. A resting ABI is performed with the patient in the supine position, with measurement of systolic blood pressure in both arms and ankles using a Doppler ultrasound device. Table 213 outlines ABI scoring and interpretation.

JFP06912500_t2.JPG

 

An ABI > 1.4 is an invalid measurement, indicating that the arteries are too calcified to be compressed. These highly elevated ABI measurements are common in patients with diabetes and/or advanced CKD. In these patients, a toe-brachial index (TBI) test should be performed, because the digital arteries are almost always compressible.13

Patients with symptomatic PAD who are under consideration for revascularization may benefit from radiologic imaging of the lower extremities with duplex ultrasound, computed tomography angiography, or magnetic resonance angiography to determine the anatomic location and severity of stenosis.13

 

Management of PAD

Lifestyle interventions

For patients with PAD, lifestyle modifications are an essential—but challenging—component of disease management.

Continue to: Smoking cessation...

 

 

Smoking cessation. As with other atherosclerotic diseases, PAD progression is strongly correlated with smoking. A trial involving 204 active smokers with PAD showed that 5-year mortality and amputation rates dropped by more than half in those who quit smoking within a year, with numbers needed to treat (NNT) of 6 for mortality and 5 for amputation.19 Because of this dramatic effect, American College of Cardiology/American Heart Association (ACC/AHA) guidelines encourage providers to address smoking at every visit and use cessation programs and medication to increase quit rates.13

Exercise may be the most important intervention for PAD. A 2017 Cochrane review found that supervised, structured exercise programs increase pain-free and maximal walking distances by at least 20% and also improve physical and mental quality of life.20 In a trial involving 111 patients with aortoiliac PAD, supervised exercise plus medical care led to greater functional improvement than either revascularization plus medical care or medical care alone.21 In a 2018 Cochrane review, neither revascularization or revascularization added to supervised exercise were better than supervised exercise alone.22 ACC/AHA guidelines recommend supervised exercise programs for claudication prior to considering revascularization.13TABLE 313 outlines the components of a structured exercise program.

JFP06912500_t3.JPG



Unfortunately, the benefit of these programs has been difficult to reproduce without supervision. Another 2018 Cochrane review demonstrated significant improvement with supervised exercise and no clear improvement in patients given home exercise or advice to walk.23 A recent study examined the effect of having patients use a wearable fitness tracker for home exercise and demonstrated no benefit over usual care.24

Diet. There is some evidence that dietary interventions can prevent and possibly improve PAD. A large randomized controlled trial showed that a Mediterranean diet lowered rates of PAD over 1 year compared to a low-fat diet, with an NNT of 336 if supplemented with extra-virgin olive oil and 448 if supplemented with nuts.25 A small trial of 25 patients who consumed non-soy legumes daily for 8 weeks showed average ABI improvement of 6%, although there was no control group.26

Medical therapy to address peripheral and cardiovascular events

Standard medical therapy for coronary artery disease (CAD) is recommended for patients with PAD to reduce cardiovascular and limb events. For example, treatment of hypertension reduces cardiovascular and cerebrovascular events, and studies verify that lowering blood pressure does not worsen claudication or limb perfusion.

A trial involving 204 active smokers with PAD showed that 5-year mortality and amputation rates dropped by more than half in those who quit smoking within a year.

13TABLE 413,27-30 outlines the options for medical therapy.

 

JFP06912500_t4.JPG

 

Continue to: Statins...

 

 

Statins reduce cardiovascular events in PAD patients. A large study demonstrated that 40 mg of simvastatin has an NNT of 21 to prevent a coronary or cerebrovascular event in PAD, similar to the NNT of 23 seen in treatment of CAD.27 Statins also reduce adverse limb outcomes. A registry of atherosclerosis patients showed that statins have an NNT of 56 to prevent amputation in PAD and an NNT of 28 to prevent worsening claudication, critical limb ischemia, revascularization, or amputation.28

Antiplatelet therapy with low-dose aspirin or clopidogrel is recommended for symptomatic patients and for asymptomatic patients with an ABI ≤ 0.9.13 A Cochrane review demonstrated significantly reduced mortality with nonaspirin antiplatelet agents vs aspirin (NNT = 94) without increase in major bleeding.29 Only British guidelines specifically recommend clopidogrel over aspirin.31

Dual antiplatelet therapy has not shown consistent benefits over aspirin alone. ACC/AHA guidelines state that dual antiplatelet therapy is not well established for PAD but may be reasonable after revascularization.13

Voraxapar is a novel antiplatelet agent that targets the thrombin-binding receptor on platelets. However, trials show no significant coronary benefit, and slight reductions in acute limb ischemia are offset by increases in major bleeding.13

For patients receiving medical therapy, ongoing evaluation and treatment should be based on claudication symptoms and clinical assessment.

Medical therapy for claudication

Several medications have been proposed for symptomatic treatment of intermittent claudication. Cilostazol is a phosphodiesterase inhibitor with the best risk-benefit ratio. A Cochrane review showed improvements in maximal and pain-free walking distances compared to placebo and improvements in quality of life with cilostazol 100 mg taken twice daily.32 Adverse effects included headache, dizziness, palpitations, and diarrhea.29

Continue to: Pentoxifylline...

 

 

Pentoxifylline is another phosphodiesterase inhibitor with less evidence of improvement, higher adverse effect rates, and more frequent dosing. It is not recommended for treatment of intermittent claudication.13,33

Supplements. Padma 28, a Tibetan herbal formulation, appears to improve maximal walking distance with adverse effect rates similar to placebo.34 Other supplements, including vitamin E, ginkgo biloba, and omega-3 fatty acids, have no evidence of benefit.35-37

When revascularizationis needed

Patients who develop limb ischemia or lifestyle-limiting claudication despite conservative therapy are candidates for revascularization. Endovascular techniques include angioplasty, stenting, atherectomy, and precise medication delivery. Surgical approaches mainly consist of thrombectomy and bypass grafting. For intermittent claudication despite conservative care, ACC/AHA guidelines state endovascular procedures are appropriate for aortoiliac disease and reasonable for femoropopliteal disease, but unproven for infrapopliteal disease.13

Acute limb ischemia is an emergency requiring immediate intervention. Two trials revealed identical overall and amputation-free survival rates for percutaneous thrombolysis and surgical thrombectomy.38,39 ACC/AHA guidelines recommend anticoagulation with heparin followed by the revascularization technique that will most rapidly restore arterial flow.13

For chronic limb ischemia, a large trial showed angioplasty had lower initial morbidity, length of hospitalization, and cost than surgical repair. However, surgical mortality was lower after 2 years.40 ACC/AHA guidelines recommend either surgery or endovascular procedures and propose initial endovascular treatment followed by surgery if needed.13 After revascularization, the patient should be followed periodically with a clinical evaluation and ABI measurement with further consideration for routine duplex ultrasound surveillance.13

For chronic limb ischemia, a large trial showed angioplasty had lower initial morbidity, length of hospitalization, and cost than surgical repair. Surgical mortality was lower after 2 years.

Outcomes

Patients with PAD have variable outcomes. About 70% to 80% of patients with this diagnosis will have a stable disease process with no worsening of symptoms, 10% to 20% will experience worsening symptoms over time, 5% to 10% will require revascularization within 5 years of diagnosis, and 1% to 5% will progress to critical limb ischemia, which has a 5-year amputation rate of 1% to 4%.2 Patients who require amputation have poor outcomes: Within 2 years, 30% are dead and 15% have had further amputations.18

In addition to the morbidity and mortality from its own progression, PAD is an important predictor of CAD and is associated with a significant elevation in morbidity and mortality from CAD. One small but well-designed prospective cohort study found that patients with PAD had a more than 6-fold increased risk of death from CAD than did patients without PAD.41

Acknowledgement
The authors thank Francesca Cimino, MD, FAAFP, for her help in reviewing this manuscript.

CORRESPONDENCE
Dustin K. Smith, DO, 2080 Child Street, Jacksonville, FL 32214; dustinksmith@yahoo.com

Peripheral arterial disease (PAD), the progressive disorder that results in ischemia to distal vascular territories as a result of atherosclerosis, spans a wide range of presentations, from minimally symptomatic disease to limb ischemia secondary to acute or chronic occlusion.

The prevalence of PAD is variable, due to differing diagnostic criteria used in studies, but PAD appears to affect 1 in every 22 people older than age 40.1 However, since PAD incidence increases with age, it is increasing in prevalence as the US population ages.1-3

PAD is associated with increased hospitalizations and decreased quality of life.4 Patients with PAD have an estimated 30% 5-year risk for myocardial infarction, stroke, or death from a vascular cause.3

Screening. Although PAD is underdiagnosed and appears to be undertreated,3 population-based screening for PAD in asymptomatic patients is not recommended. A Cochrane review found no studies evaluating the benefit of ­asymptomatic population-based screening.5 Similarly, in 2018, the USPSTF performed a comprehensive review and found no studies to support routine screening and determined there was insufficient evidence to recommend it.6,7

Risk factors and associated comorbidities

PAD risk factors, like the ones detailed below, have a potentiating effect. The presence of 2 risk factors doubles PAD risk, while 3 or more risk factors increase PAD risk by a factor of 10.1

Increasing age is the greatest single risk factor for PAD.1,2,8,9 Researchers using data from the National Health and Nutrition Examination Survey (NHANES) found that the prevalence of PAD increased from 1.4% in individuals ages 40 to 49 years to almost 17% in those age 70 or older.1

JFP06912500_300x300.JPG


 

Patients with PAD have an estimated 30% 5-year risk for myocardial infarction, stroke, or death from a vascular cause.

Demographic characteristics. Most studies demonstrate a higher risk for PAD in men.1-3,10 African-American patients have more than twice the risk for PAD, compared with Whites, even after adjustment for the increased prevalence of associated diseases such as hypertension and diabetes in this population.1-3,10

 

Continue to: Genetics...

 

 



Genetics. A study performed by the National Heart Lung and Blood Institute suggested that genetic correlations between twins were more important than environmental factors in the development of PAD.11

Smoking. Most population studies show smoking to be the greatest modifiable risk factor for PAD. An analysis of the NHANES data yielded an odds ratio (OR) of 4.1 for current smokers and of 1.8 for former smokers.1 Risk increases linearly with cumulative years of smoking.1,2,9,10

Diabetes is another significant modifiable risk factor, increasing PAD risk by 2.5 times.2 Diabetes is also associated with increases in functional limitation from claudication, risk for acute coronary syndrome, and progression to amputation.1

Hypertension nearly doubles the risk for PAD, and poor control further increases this risk.2,9,10

Chronic kidney disease (CKD). Patients with CKD have a progressively higher prevalence of PAD with worsening renal function.1 There is also an association between CKD and increased morbidity, revascularization failure, and increased mortality.1

Two additional risk factors that are less well understood are dyslipidemia and chronic inflammation. There is conflicting data regarding the role of individual components of cholesterol and their effect on PAD, although lipoprotein (a) has been shown to be an independent risk factor for both the development and progression of PAD.12 Similarly, chronic inflammation has been shown to play a role in the initiation and progression of the disease, although the role of inflammatory markers in evaluation and treatment is unclear and assessment for these purposes is not currently recommended.12,13

Continue to: Diagnosis...

 

 

Diagnosis

Clinical presentation

Lower extremity pain is the hallmark symptom of PAD, but presentation varies. The classic presentation is claudication, pain within a defined muscle group that occurs with exertion and is relieved by rest. Claudication is most common in the calf but also occurs in the buttock/thigh and the foot.

 

African- American patients have more than twice the risk for PAD, compared with Whites, even after adjustment for the increased prevalence of associated diseases in this population.

However, most patients with PAD present with pain that does not fit the definition of claudication. Patients with comorbidities, physical inactivity, and neuropathy are more likely to present with atypical pain.14 These patients may demonstrate critical or acute limb ischemia, characterized by pain at rest and most often localized to the forefoot and toes. Patients with critical limb ischemia may also present with nonhealing wounds/ulcers or gangrene.15

Physical exam findings can support the diagnosis of PAD, but none are reliable enough to rule the diagnosis in or out. Findings suggestive of PAD include cool skin, presence of a bruit (iliac, femoral, or popliteal), and palpable pulse abnormality. Multiple abnormal physical exam findings increase the likelihood of PAD, while the absence of a bruit or palpable pulse abnormality makes PAD less likely.16 In patients with PAD, an associated wound/ulcer is most often distal in the foot and usually appears dry.17

The differential diagnosis for intermittent leg pain is broad and includes neurologic, musculoskeletal, and venous etiologies. Table 118 lists some common alternate diagnoses for patients presenting with leg pain or claudication.

 

JFP06912500_t1.JPG

 

Continue to: Diagnostic testing...

 

 

Diagnostic testing

An ankle-brachial index (ABI) test should be performed in patients with history or physical exam findings suggestive of PAD. A resting ABI is performed with the patient in the supine position, with measurement of systolic blood pressure in both arms and ankles using a Doppler ultrasound device. Table 213 outlines ABI scoring and interpretation.

JFP06912500_t2.JPG

 

An ABI > 1.4 is an invalid measurement, indicating that the arteries are too calcified to be compressed. These highly elevated ABI measurements are common in patients with diabetes and/or advanced CKD. In these patients, a toe-brachial index (TBI) test should be performed, because the digital arteries are almost always compressible.13

Patients with symptomatic PAD who are under consideration for revascularization may benefit from radiologic imaging of the lower extremities with duplex ultrasound, computed tomography angiography, or magnetic resonance angiography to determine the anatomic location and severity of stenosis.13

 

Management of PAD

Lifestyle interventions

For patients with PAD, lifestyle modifications are an essential—but challenging—component of disease management.

Continue to: Smoking cessation...

 

 

Smoking cessation. As with other atherosclerotic diseases, PAD progression is strongly correlated with smoking. A trial involving 204 active smokers with PAD showed that 5-year mortality and amputation rates dropped by more than half in those who quit smoking within a year, with numbers needed to treat (NNT) of 6 for mortality and 5 for amputation.19 Because of this dramatic effect, American College of Cardiology/American Heart Association (ACC/AHA) guidelines encourage providers to address smoking at every visit and use cessation programs and medication to increase quit rates.13

Exercise may be the most important intervention for PAD. A 2017 Cochrane review found that supervised, structured exercise programs increase pain-free and maximal walking distances by at least 20% and also improve physical and mental quality of life.20 In a trial involving 111 patients with aortoiliac PAD, supervised exercise plus medical care led to greater functional improvement than either revascularization plus medical care or medical care alone.21 In a 2018 Cochrane review, neither revascularization or revascularization added to supervised exercise were better than supervised exercise alone.22 ACC/AHA guidelines recommend supervised exercise programs for claudication prior to considering revascularization.13TABLE 313 outlines the components of a structured exercise program.

JFP06912500_t3.JPG



Unfortunately, the benefit of these programs has been difficult to reproduce without supervision. Another 2018 Cochrane review demonstrated significant improvement with supervised exercise and no clear improvement in patients given home exercise or advice to walk.23 A recent study examined the effect of having patients use a wearable fitness tracker for home exercise and demonstrated no benefit over usual care.24

Diet. There is some evidence that dietary interventions can prevent and possibly improve PAD. A large randomized controlled trial showed that a Mediterranean diet lowered rates of PAD over 1 year compared to a low-fat diet, with an NNT of 336 if supplemented with extra-virgin olive oil and 448 if supplemented with nuts.25 A small trial of 25 patients who consumed non-soy legumes daily for 8 weeks showed average ABI improvement of 6%, although there was no control group.26

Medical therapy to address peripheral and cardiovascular events

Standard medical therapy for coronary artery disease (CAD) is recommended for patients with PAD to reduce cardiovascular and limb events. For example, treatment of hypertension reduces cardiovascular and cerebrovascular events, and studies verify that lowering blood pressure does not worsen claudication or limb perfusion.

A trial involving 204 active smokers with PAD showed that 5-year mortality and amputation rates dropped by more than half in those who quit smoking within a year.

13TABLE 413,27-30 outlines the options for medical therapy.

 

JFP06912500_t4.JPG

 

Continue to: Statins...

 

 

Statins reduce cardiovascular events in PAD patients. A large study demonstrated that 40 mg of simvastatin has an NNT of 21 to prevent a coronary or cerebrovascular event in PAD, similar to the NNT of 23 seen in treatment of CAD.27 Statins also reduce adverse limb outcomes. A registry of atherosclerosis patients showed that statins have an NNT of 56 to prevent amputation in PAD and an NNT of 28 to prevent worsening claudication, critical limb ischemia, revascularization, or amputation.28

Antiplatelet therapy with low-dose aspirin or clopidogrel is recommended for symptomatic patients and for asymptomatic patients with an ABI ≤ 0.9.13 A Cochrane review demonstrated significantly reduced mortality with nonaspirin antiplatelet agents vs aspirin (NNT = 94) without increase in major bleeding.29 Only British guidelines specifically recommend clopidogrel over aspirin.31

Dual antiplatelet therapy has not shown consistent benefits over aspirin alone. ACC/AHA guidelines state that dual antiplatelet therapy is not well established for PAD but may be reasonable after revascularization.13

Voraxapar is a novel antiplatelet agent that targets the thrombin-binding receptor on platelets. However, trials show no significant coronary benefit, and slight reductions in acute limb ischemia are offset by increases in major bleeding.13

For patients receiving medical therapy, ongoing evaluation and treatment should be based on claudication symptoms and clinical assessment.

Medical therapy for claudication

Several medications have been proposed for symptomatic treatment of intermittent claudication. Cilostazol is a phosphodiesterase inhibitor with the best risk-benefit ratio. A Cochrane review showed improvements in maximal and pain-free walking distances compared to placebo and improvements in quality of life with cilostazol 100 mg taken twice daily.32 Adverse effects included headache, dizziness, palpitations, and diarrhea.29

Continue to: Pentoxifylline...

 

 

Pentoxifylline is another phosphodiesterase inhibitor with less evidence of improvement, higher adverse effect rates, and more frequent dosing. It is not recommended for treatment of intermittent claudication.13,33

Supplements. Padma 28, a Tibetan herbal formulation, appears to improve maximal walking distance with adverse effect rates similar to placebo.34 Other supplements, including vitamin E, ginkgo biloba, and omega-3 fatty acids, have no evidence of benefit.35-37

When revascularizationis needed

Patients who develop limb ischemia or lifestyle-limiting claudication despite conservative therapy are candidates for revascularization. Endovascular techniques include angioplasty, stenting, atherectomy, and precise medication delivery. Surgical approaches mainly consist of thrombectomy and bypass grafting. For intermittent claudication despite conservative care, ACC/AHA guidelines state endovascular procedures are appropriate for aortoiliac disease and reasonable for femoropopliteal disease, but unproven for infrapopliteal disease.13

Acute limb ischemia is an emergency requiring immediate intervention. Two trials revealed identical overall and amputation-free survival rates for percutaneous thrombolysis and surgical thrombectomy.38,39 ACC/AHA guidelines recommend anticoagulation with heparin followed by the revascularization technique that will most rapidly restore arterial flow.13

For chronic limb ischemia, a large trial showed angioplasty had lower initial morbidity, length of hospitalization, and cost than surgical repair. However, surgical mortality was lower after 2 years.40 ACC/AHA guidelines recommend either surgery or endovascular procedures and propose initial endovascular treatment followed by surgery if needed.13 After revascularization, the patient should be followed periodically with a clinical evaluation and ABI measurement with further consideration for routine duplex ultrasound surveillance.13

For chronic limb ischemia, a large trial showed angioplasty had lower initial morbidity, length of hospitalization, and cost than surgical repair. Surgical mortality was lower after 2 years.

Outcomes

Patients with PAD have variable outcomes. About 70% to 80% of patients with this diagnosis will have a stable disease process with no worsening of symptoms, 10% to 20% will experience worsening symptoms over time, 5% to 10% will require revascularization within 5 years of diagnosis, and 1% to 5% will progress to critical limb ischemia, which has a 5-year amputation rate of 1% to 4%.2 Patients who require amputation have poor outcomes: Within 2 years, 30% are dead and 15% have had further amputations.18

In addition to the morbidity and mortality from its own progression, PAD is an important predictor of CAD and is associated with a significant elevation in morbidity and mortality from CAD. One small but well-designed prospective cohort study found that patients with PAD had a more than 6-fold increased risk of death from CAD than did patients without PAD.41

Acknowledgement
The authors thank Francesca Cimino, MD, FAAFP, for her help in reviewing this manuscript.

CORRESPONDENCE
Dustin K. Smith, DO, 2080 Child Street, Jacksonville, FL 32214; dustinksmith@yahoo.com

References

1. Eraso LH, Fukaya E, Mohler ER 3rd, et al. Peripheral arterial disease, prevalence and cumulative risk factor profile analysis. Eur J Prev Cardiol. 2014;21:704-711.

2. Pasternak RC, Criqui MH, Benjamin EJ, et al; American Heart Association. Atherosclerotic Vascular Disease Conference: Writing Group I: epidemiology. Circulation. 2004;109:2605-2612.

3. Hirsch AT, Criqui MH, Treat-Jacobson D, et al. Peripheral arterial disease detection, awareness, and treatment in primary care. JAMA. 2001;286:1317-1324.

4. Olin JW, Sealove BA. Peripheral artery disease: current insight into the disease and its diagnosis and management. Mayo Clin Proc. 2010;85:678-692.

5. Andras A, Ferkert B. Screening for peripheral arterial disease. Cochrane Database Syst Rev. 2014;(4):CD010835.

6. Guirguis-Blake JM, Evans CV, Redmond N, et al. Screening for peripheral artery disease using ankle-brachial index: updated evidence report and systematic review for the US Preventive Services Task Force. JAMA. 2018;320:184-196.

7. US Preventive Services Task Force. Screening for peripheral artery disease and cardiovascular disease risk assessment with ankle-brachial index: US Preventive Services Task Force recommendation statement. JAMA. 2018;230:177-183.

8. American Heart Association Writing Group 2. Atherosclerotic Peripheral Vascular Disease Symposium II: screening for atherosclerotic vascular diseases: should nationwide programs be instituted? Circulation. 2008;118:2830-2836.

9. Berger JS, Hochman J, Lobach I, et al. Modifiable risk factor burden and the prevalence of peripheral artery disease in different vascular territories. J Vasc Surg. 2013;58:673-681.

10. Joosten MM, Pai JK, Bertoia ML, et al. Associations between conventional cardiovascular risk factors and risk of peripheral artery disease in men. JAMA. 2012;308:1660-1667.

11. Carmelli D, Fabsitz RR, Swan GE, et al. Contribution of genetic and environmental influences to ankle-brachial blood pressure index in the NHLBI Twin Study. National Heart, Lung, and Blood Institute. Am J Epidemiol. 2000;151:452-458.

12. Aboyans V, Criqui MH, Denenberg JO, et al. Risk factors for progression of peripheral arterial disease in large and small vessels. Circulation. 2006;113:2623-2629.

13. Gerald-Herman MD, Gornik HL, Barrett C, et al. 2016 AHA/ACC guideline on the management of patients with lower extremity peripheral artery disease: a report of the American College of Cardiology/American Heart Association Task Force on Clinical Practice Guidelines. Circulation. 2017;135:e726-e779.

14. McDermott MM, Greenland P, Liu K, et al. Leg symptoms in peripheral arterial disease: associated clinical characteristics and functional impairment. JAMA. 2001;286:1599-1606.

15. Cranley JJ. Ischemic rest pain. Arch Surg. 1969;98:187-188.

16. Khan NA, Rahim SA, Anand SS, et al. Does the clinical examination predict lower extremity peripheral arterial disease? JAMA. 2006;295:536-546.

17. Wennberg PW. Approach to the patient with peripheral arterial disease. Circulation. 2013;128:2241-2250.

18. Norgren L, Hiatt WR, Dormandy JA, et al. Inter-society consensus for the management of peripheral arterial disease (TASC II). Eur J Vas Endovasc Surg. 2007;33:S1-S75.

19. Armstrong EJ, Wu J, Singh GD, et al. Smoking cessation is associated with decreased mortality and improved amputation-free survival among patients with symptomatic peripheral artery disease. J Vasc Surg. 2014;60:1565-1571.

20. Lane R, Harwood A, Watson L, et al. Exercise for intermittent claudication. Cochrane Database Syst Rev. 2017;(12):CD000990.

21. Murphy TP, Cutlip DE, Regensteiner JG, et al; CLEVER Study Investigators. Supervised exercise versus primary stenting for claudication resulting from aortoiliac peripheral artery disease: six-month outcomes from the claudication: exercise versus endoluminal revascularization (CLEVER) study. Circulation. 2012;125:130-139.

22. Fakhry F, Fokkenrood HJP, Pronk S, et al. Endovascular revascularization versus conservative management for intermittent claudication. Cochrane Database Syst Rev. 2018;(3):CD010512.

23. Hageman D, Fokkenrood HJ, Gommans LN, et al. Supervised exercise therapy versus home-based exercise therapy versus walking advice for intermittent claudication. Cochrane Database Syst Rev. 2018;(4):CD005263.

24. McDermott MM, Spring B, Berger JS, et al. Effect of a home-based exercise intervention of wearable technology and telephone coaching on walking performance in peripheral artery disease: the HONOR randomized clinical trial. JAMA. 2018;319:1665-1676.

25. Ruiz-Canela M, Estruch R, Corella D, et al. Association of Mediterranean diet with peripheral artery disease: the PREDIMED randomized trial. JAMA. 2014;311:415-417.

26. Zahradka P, Wright B, Weighell W, et al. Daily non-soy legume consumption reverses vascular impairment due to peripheral artery disease. Atherosclerosis. 2013;230:310-314.

27. Heart Protection Study Collaborative Group. Randomized trial of the effects of cholesterol-lowering with simvastatin on peripheral vascular and other major vascular outcomes in 20536 people with peripheral arterial disease and other high-risk conditions. J Vasc Surg. 2007;45:645-655.

28. Kumbhani DJ, Steg G, Cannon CP, et al. Statin therapy and long-term adverse limb outcomes in patients with peripheral artery disease: insights from the REACH registry. Eur Heart J. 2014;35:2864-2872.

29. Wong PF, Chong LY, Mikhailidis DP, et al. Antiplatelet agents for intermittent claudication. Cochrane Database Syst Rev. 2011;(11):CD001272.

30. Critical Leg Ischaemia Prevention Study (CLIPS) Group, Catalano M, Born G, Peto R. Prevention of serious vascular events by aspirin amongst patients with peripheral arterial disease: randomized, double-blind trial. J Intern Med. 2007;261:276-284.

31. Morley RL, Sharma A, Horsch AD, et al. Peripheral artery disease. BMJ. 2018;360:j5842.

32. Bedenis R, Stewart M, Cleanthis M, et al. Cilostazol for intermittent claudication. Cochrane Database Syst Rev. 2014;(10):CD003748.

<--pagebreak-->

33. Salhiyyah K, Forster R, Senanayake E, et al. Pentoxifylline for intermittent claudication. Cochrane Database Syst Rev. 2015;(9):CD005262.

34. Stewart M, Morling JR, Maxwell H. Padma 28 for intermittent claudication. Cochrane Database Syst Rev. 2016;(3):CD007371.

35. Kleijnen J, Mackerras D. Vitamin E for intermittent claudication. Cochrane Database Syst Rev. 1998;(1):CD000987.

36. Nicolai SPA, Kruidenior LM, Bendermacher BLW, et al. Ginkgo biloba for intermittent claudication. Cochrane Database Syst Rev. 2013;(6):CD006888.

37. Campbell A, Price J, Hiatt WR. Omega-3 fatty acids for intermittent claudication. Cochrane Database Syst Rev. 2013;(7):CD003833.

38. American Surgical Association, New York Surgical Society, Philadelphia Academy of Surgery, Southern Surgical Association (US), Central Surgical Association. Results of a prospective randomized trial evaluating surgery versus thrombolysis for ischemia of the lower extremity: the STILE trial. Ann Surg. 1994;220:251-268.

39. Ouriel K, Veith FJ, Sasahara AA. Thrombolysis or peripheral arterial surgery: phase I results. TOPAS Investigators. J Vasc Surg. 1996;23:64-73.

40. Bradbury AW, Ruckley CV, Fowkes FGR, et al. Bypass versus angioplasty in severe ischaemia of the leg (BASIL): multicentre, randomised, controlled trial. Lancet. 2005;366:1925-1934.

41. Criqui MH, Langer RD, Fronek A, et al. Mortality over a period of 10 years in patients with peripheral arterial disease. N Engl J Med. 1992;326:381-386.

References

1. Eraso LH, Fukaya E, Mohler ER 3rd, et al. Peripheral arterial disease, prevalence and cumulative risk factor profile analysis. Eur J Prev Cardiol. 2014;21:704-711.

2. Pasternak RC, Criqui MH, Benjamin EJ, et al; American Heart Association. Atherosclerotic Vascular Disease Conference: Writing Group I: epidemiology. Circulation. 2004;109:2605-2612.

3. Hirsch AT, Criqui MH, Treat-Jacobson D, et al. Peripheral arterial disease detection, awareness, and treatment in primary care. JAMA. 2001;286:1317-1324.

4. Olin JW, Sealove BA. Peripheral artery disease: current insight into the disease and its diagnosis and management. Mayo Clin Proc. 2010;85:678-692.

5. Andras A, Ferkert B. Screening for peripheral arterial disease. Cochrane Database Syst Rev. 2014;(4):CD010835.

6. Guirguis-Blake JM, Evans CV, Redmond N, et al. Screening for peripheral artery disease using ankle-brachial index: updated evidence report and systematic review for the US Preventive Services Task Force. JAMA. 2018;320:184-196.

7. US Preventive Services Task Force. Screening for peripheral artery disease and cardiovascular disease risk assessment with ankle-brachial index: US Preventive Services Task Force recommendation statement. JAMA. 2018;230:177-183.

8. American Heart Association Writing Group 2. Atherosclerotic Peripheral Vascular Disease Symposium II: screening for atherosclerotic vascular diseases: should nationwide programs be instituted? Circulation. 2008;118:2830-2836.

9. Berger JS, Hochman J, Lobach I, et al. Modifiable risk factor burden and the prevalence of peripheral artery disease in different vascular territories. J Vasc Surg. 2013;58:673-681.

10. Joosten MM, Pai JK, Bertoia ML, et al. Associations between conventional cardiovascular risk factors and risk of peripheral artery disease in men. JAMA. 2012;308:1660-1667.

11. Carmelli D, Fabsitz RR, Swan GE, et al. Contribution of genetic and environmental influences to ankle-brachial blood pressure index in the NHLBI Twin Study. National Heart, Lung, and Blood Institute. Am J Epidemiol. 2000;151:452-458.

12. Aboyans V, Criqui MH, Denenberg JO, et al. Risk factors for progression of peripheral arterial disease in large and small vessels. Circulation. 2006;113:2623-2629.

13. Gerald-Herman MD, Gornik HL, Barrett C, et al. 2016 AHA/ACC guideline on the management of patients with lower extremity peripheral artery disease: a report of the American College of Cardiology/American Heart Association Task Force on Clinical Practice Guidelines. Circulation. 2017;135:e726-e779.

14. McDermott MM, Greenland P, Liu K, et al. Leg symptoms in peripheral arterial disease: associated clinical characteristics and functional impairment. JAMA. 2001;286:1599-1606.

15. Cranley JJ. Ischemic rest pain. Arch Surg. 1969;98:187-188.

16. Khan NA, Rahim SA, Anand SS, et al. Does the clinical examination predict lower extremity peripheral arterial disease? JAMA. 2006;295:536-546.

17. Wennberg PW. Approach to the patient with peripheral arterial disease. Circulation. 2013;128:2241-2250.

18. Norgren L, Hiatt WR, Dormandy JA, et al. Inter-society consensus for the management of peripheral arterial disease (TASC II). Eur J Vas Endovasc Surg. 2007;33:S1-S75.

19. Armstrong EJ, Wu J, Singh GD, et al. Smoking cessation is associated with decreased mortality and improved amputation-free survival among patients with symptomatic peripheral artery disease. J Vasc Surg. 2014;60:1565-1571.

20. Lane R, Harwood A, Watson L, et al. Exercise for intermittent claudication. Cochrane Database Syst Rev. 2017;(12):CD000990.

21. Murphy TP, Cutlip DE, Regensteiner JG, et al; CLEVER Study Investigators. Supervised exercise versus primary stenting for claudication resulting from aortoiliac peripheral artery disease: six-month outcomes from the claudication: exercise versus endoluminal revascularization (CLEVER) study. Circulation. 2012;125:130-139.

22. Fakhry F, Fokkenrood HJP, Pronk S, et al. Endovascular revascularization versus conservative management for intermittent claudication. Cochrane Database Syst Rev. 2018;(3):CD010512.

23. Hageman D, Fokkenrood HJ, Gommans LN, et al. Supervised exercise therapy versus home-based exercise therapy versus walking advice for intermittent claudication. Cochrane Database Syst Rev. 2018;(4):CD005263.

24. McDermott MM, Spring B, Berger JS, et al. Effect of a home-based exercise intervention of wearable technology and telephone coaching on walking performance in peripheral artery disease: the HONOR randomized clinical trial. JAMA. 2018;319:1665-1676.

25. Ruiz-Canela M, Estruch R, Corella D, et al. Association of Mediterranean diet with peripheral artery disease: the PREDIMED randomized trial. JAMA. 2014;311:415-417.

26. Zahradka P, Wright B, Weighell W, et al. Daily non-soy legume consumption reverses vascular impairment due to peripheral artery disease. Atherosclerosis. 2013;230:310-314.

27. Heart Protection Study Collaborative Group. Randomized trial of the effects of cholesterol-lowering with simvastatin on peripheral vascular and other major vascular outcomes in 20536 people with peripheral arterial disease and other high-risk conditions. J Vasc Surg. 2007;45:645-655.

28. Kumbhani DJ, Steg G, Cannon CP, et al. Statin therapy and long-term adverse limb outcomes in patients with peripheral artery disease: insights from the REACH registry. Eur Heart J. 2014;35:2864-2872.

29. Wong PF, Chong LY, Mikhailidis DP, et al. Antiplatelet agents for intermittent claudication. Cochrane Database Syst Rev. 2011;(11):CD001272.

30. Critical Leg Ischaemia Prevention Study (CLIPS) Group, Catalano M, Born G, Peto R. Prevention of serious vascular events by aspirin amongst patients with peripheral arterial disease: randomized, double-blind trial. J Intern Med. 2007;261:276-284.

31. Morley RL, Sharma A, Horsch AD, et al. Peripheral artery disease. BMJ. 2018;360:j5842.

32. Bedenis R, Stewart M, Cleanthis M, et al. Cilostazol for intermittent claudication. Cochrane Database Syst Rev. 2014;(10):CD003748.

<--pagebreak-->

33. Salhiyyah K, Forster R, Senanayake E, et al. Pentoxifylline for intermittent claudication. Cochrane Database Syst Rev. 2015;(9):CD005262.

34. Stewart M, Morling JR, Maxwell H. Padma 28 for intermittent claudication. Cochrane Database Syst Rev. 2016;(3):CD007371.

35. Kleijnen J, Mackerras D. Vitamin E for intermittent claudication. Cochrane Database Syst Rev. 1998;(1):CD000987.

36. Nicolai SPA, Kruidenior LM, Bendermacher BLW, et al. Ginkgo biloba for intermittent claudication. Cochrane Database Syst Rev. 2013;(6):CD006888.

37. Campbell A, Price J, Hiatt WR. Omega-3 fatty acids for intermittent claudication. Cochrane Database Syst Rev. 2013;(7):CD003833.

38. American Surgical Association, New York Surgical Society, Philadelphia Academy of Surgery, Southern Surgical Association (US), Central Surgical Association. Results of a prospective randomized trial evaluating surgery versus thrombolysis for ischemia of the lower extremity: the STILE trial. Ann Surg. 1994;220:251-268.

39. Ouriel K, Veith FJ, Sasahara AA. Thrombolysis or peripheral arterial surgery: phase I results. TOPAS Investigators. J Vasc Surg. 1996;23:64-73.

40. Bradbury AW, Ruckley CV, Fowkes FGR, et al. Bypass versus angioplasty in severe ischaemia of the leg (BASIL): multicentre, randomised, controlled trial. Lancet. 2005;366:1925-1934.

41. Criqui MH, Langer RD, Fronek A, et al. Mortality over a period of 10 years in patients with peripheral arterial disease. N Engl J Med. 1992;326:381-386.

Issue
The Journal of Family Practice - 69(10)
Issue
The Journal of Family Practice - 69(10)
Page Number
500-506
Page Number
500-506
Publications
Publications
Topics
Article Type
Sections
Inside the Article

PRACTICE RECOMMENDATIONS

❯ Use the ankle-brachial index for diagnosis in patients with history/physical exam findings suggestive of peripheral arterial disease (PAD). A

Strongly encourage smoking cessation in patients with PAD as doing so reduces 5-year mortality and amputation rates. B

Use structured exercise programs for patients with intermittent claudication prior to consideration of revascularization; doing so offers similar benefit and lower risks. A

Recommend revascularization for patients who have limb ischemia or lifestyle-limiting claudication despite medical and exercise therapy. B

Strength of recommendation (SOR)

A Good-quality patient-oriented evidence
B Inconsistent or limited-quality patient-oriented evidence
Consensus, usual practice, opinion, disease-oriented evidence, case series

Disallow All Ads
Content Gating
No Gating (article Unlocked/Free)
Alternative CME
Disqus Comments
Default
Use ProPublica
Hide sidebar & use full width
render the right sidebar.
Conference Recap Checkbox
Not Conference Recap
Clinical Edge
Display the Slideshow in this Article
Medscape Article
Article PDF Media
Image
Teambase ID
18001A4E.SIG
Disable zoom
Off

Pediatric cholestatic liver disease: Successful transition of care

Article Type
Changed
Mon, 07/01/2019 - 09:00
Display Headline
Pediatric cholestatic liver disease: Successful transition of care

Thanks to advances in medical science and our understanding of inherited and acquired liver disease, many more children with acquired or congenital liver disease survive into adulthood than they did 2 decades ago. Improvements in immunosuppression and surgery have increased the chances of pediatric liver transplant recipients reaching adulthood, with a survival rate of 75% at 15 to 20 years.1

selvakumar_pediatriccholestaticliverdisease_f1.jpg
%3Cp%3EFigure%201.%20Cholestasis%20in%20children%20can%20be%20caused%20by%20biliary%20tract%20obstruction%20such%20as%20in%20biliary%20atresia%20(top)%20or%20defects%20in%20forming%20and%20excreting%20bile%20acids%20and%20other%20components%20of%20bile%20(bottom).%3C%2Fp%3E
The causes of cholestasis in children are different from those in adults (Figure 1). Pediatric cholestasis is caused either by genetic defects that affect the process of synthesis, processing, or secreting bile, or by mechanical obstruction of the biliary tract, the classic example being biliary atresia.

With the growing number of adult patients with pediatric-onset liver disease, internists and adult hepatologists need to be aware of these liver diseases and develop expertise to manage this challenging group of patients. Moreover, young adults with pediatric-onset chronic liver disease pose distinct challenges such as pregnancy, adherence to medical regimens, and psychosocial changes in life.

These patients need a “transition of care” rather than a “transfer of care.” Transition of care is a multifaceted process that takes the medical, educational, and psychosocial needs of the patient into consideration before switching their care to adult care physicians, whereas transfer of care is simply an administrative process of change to adult care without previous knowledge of the patients.2

selvakumar_pediatriccholestaticliverdisease_t1.jpg
In this article, we discuss relatively common types of inherited childhood-onset cholestatic disease and their implications and management in adulthood (Table 1). Other acquired or rare cholestatic diseases are beyond the scope of this article.

BILIARY ATRESIA

Biliary atresia is a progressive inflammatory fibrosclerosing cholangiopathy of unknown cause. Its prevalence varies with geographic location, ranging from 1 in 6,000 to 1 in 19,000, with the highest prevalence in Taiwan.3

Biliary atresia usually presents within the first few weeks of life, with progressive cholestasis leading to failure to thrive and to fat-soluble vitamin deficiency. Approximately 20% of patients have congenital splenic, gastrointestinal, genitourinary, cardiac, and venous malformations.4,5 Untreated, biliary atresia progresses to end-stage liver disease and death within 2 years.

The first-line treatment for biliary atresia is to establish biliary outflow with the Kasai procedure (hepatic portoenterostomy), in which a jejunal limb is anastomosed in a Roux-en-Y with the liver. The outcomes of the Kasai procedure depend on the timing of surgery, so timely diagnosis of biliary atresia is crucial. When the Kasai procedure is performed within 60 days of birth, biliary flow is achieved in up to 70% of patients; but if performed after 90 days, biliary flow is achieved in fewer than 25%.6

Long-term outcomes of biliary atresia in patients with their native liver have been reported in a few studies.

In a French study,7 743 patients with biliary atresia underwent the Kasai procedure at a median age of 60 days. Survival rates were 57.1% at 2 years, 37.9% at 5 years, 32.4% at 10 years, and 28.5% at 15 years. In other studies,4–9 the 20-year transplant-free survival rate ranged from 23% to 46%. Therefore, at least one-third of children with biliary atresia survive to adulthood with their native liver.

Implications of biliary atresia in adulthood

Although the Kasai procedure improves biliary outflow, up to 70% of patients develop complications of biliary atresia such as progressive fibrosis, cirrhosis, portal hypertension, cholangitis, and hepatocellular carcinoma, even after a successful Kasai procedure.10

Portal hypertension with evidence of splenomegaly, thrombocytopenia, or ascites is found in two-thirds of long-term survivors of biliary atresia with a native liver, with variceal hemorrhage occurring in 30%.11 Therefore, patients with biliary atresia who have evidence of portal hypertension should be screened for varices with upper endoscopy on an annual basis. Management of variceal hemorrhage in these patients includes the use of octreotide, antibiotics, variceal ligation, and sclerotherapy; primary prophylaxis can be achieved with beta-blockers and endoscopic variceal ligation.12

Cholangitis is frequent, occurring in 40% to 60% of biliary atresia patients after the Kasai procedure, and about one-fourth of these patients have multiple episodes.13 The number of episodes of cholangitis negatively affects transplant-free survival.14 Patients with cholangitis should be adequately treated with oral or intravenous antibiotics depending on the severity of presentation. The role of prophylaxis with antibiotics remains unclear.15

Pulmonary complications such as hepato­pulmonary syndrome and portopulmonary hypertension can also occur in biliary atresia patients with a native liver. It is important for physicians to be aware of these complications and to screen for them, for example, with agitated saline echocardiography for hepatopulmonary syndrome and with echocardiography for portopulmonary hypertension. Timely screening is crucial, as the outcome of liver transplant depends on the severity at the time of transplant in these conditions, especially portopulmonary hypertension.

Hepatocellular carcinoma has been rarely reported in children with biliary atresia,16 so well-defined guidelines for screening in young adults with biliary atresia are lacking. Most centers recommend screening with ultrasonography of the abdomen and alpha-fetoprotein measurement every 6 months or annually starting soon after the Kasai procedure, since hepatocellular carcinoma has been reported in children as young as age 2.16

Transplant. Adult hepatologists are faced with the challenging task of deciding when it is time for transplant, balancing the long-term complications of biliary atresia with the risk of long-term immunosuppression after transplant. In addition, young adults with these complications may have preserved synthetic function, resulting in low Model for End-Stage Liver Disease (MELD) scores, which may complicate the process of listing for transplant.

Neurocognitive deficits are reported in children with biliary atresia,17 but young adults with biliary atresia generally have reasonable cognitive function and prospects for education and employment.

Pregnancy with successful outcomes has been reported.8

 

 

ALAGILLE SYNDROME

Alagille syndrome is an autosomal-dominant multisystemic disease caused by mutations in the JAG1 gene (accounting for > 95% of cases) and the NOTCH2 gene, with highly variable expression.18

Extrahepatic manifestations include butterfly vertebral defects, facial dysmorphism (eg, deep-set and low-set eyes, with characteristic “triangular” facies), posterior embryotoxon (a congenital defect of the eye characterized by an opaque ring around the margin of the cornea), peripheral pulmonary stenosis, renal abnormalities, and vascular malformations.

Hepatic manifestations vary from asymptomatic laboratory abnormalities to progressive cholestasis starting in early infancy with intractable pruritus, xanthomas, failure to thrive, and end-stage liver disease requiring liver transplant in childhood in 15% to 20% of patients.19

Implications of Alagille syndrome in adulthood

Transplant. Interestingly, the phenotype of hepatic disease is already established in childhood, with minimal or no progression in adulthood. Most children with minimal liver disease experience spontaneous resolution, whereas those with significant cholestasis might ultimately develop progressive liver fibrosis or cirrhosis requiring liver transplant in childhood. Only a small subset of children with minimal cholestasis progress to end-stage liver disease in late childhood or early adulthood.20 Therefore, liver transplant for progressive liver disease from significant cholestasis almost always occurs in childhood, usually between ages 1 and 4.21

In a retrospective study comparing posttransplant outcomes in children with Alagille syndrome and biliary atresia, 1-year patient survival was excellent overall in children with Alagille syndrome, although slightly lower than in children with biliary atresia, most likely owing to extrahepatic morbidities of Alagille syndrome and especially the use of immunosuppression in those with renal disease.21 Similarly, 1- and 5-year patient and graft survival outcomes of liver transplant in adults with Alagille syndrome were also excellent compared with those who received a liver transplant in childhood for Alagille syndrome or in adulthood for biliary atresia.22

Hepatocellular carcinoma has occurred in these patients in the absence of cirrhosis, which makes implementation of prognostic and surveillance strategies almost impossible to design for them. Annual ultrasonography with alpha-fetoprotein testing might be applicable in Alagille syndrome patients. However, deciding which patients should undergo this testing and when it should start will be challenging, given the paucity of data.

Cardiovascular disease. Cardiac phenotype is also mostly established in childhood, with the pulmonary vasculature being most commonly involved.19 In contrast, renal and other vascular abnormalities can manifest in adulthood. Renal manifestations vary and include structural anomalies such as hyperechoic kidneys or renal cysts, which can manifest in childhood, and some abnormalities such as hypertension and renal artery stenosis that can manifest in adulthood.23,24

Vasculopathy is reported to involve the intracranial, renal, and intra-abdominal blood vessels.25 Neurovascular accidents such as stroke and intracranial hemorrhage can occur at any age, with significant rates of morbidity and death.26 Therefore, some experts recommend magnetic resonance angiography every 5 years and before any major intervention to prevent these devastating complications.20

Pregnancy. Successful pregnancies have been reported. Preexisting cardiac and hepatic disease can complicate pregnancy depending on the severity of the disease. Because of the autosomal-dominant pattern of inheritance, infants have a 50% risk of the disease, so genetic counseling should be seriously considered before conception.27 Prenatal diagnosis is possible, but the lack of genotype-phenotype correlation precludes its use in clinical practice.

PROGRESSIVE FAMILIAL INTRAHEPATIC CHOLESTASIS

Progressive familial intrahepatic cholestasis (PFIC) is a heterogeneous group of autosomal-recessive conditions associated with disruption of bile formation causing cholestatic liver disease in infants and young children. Three types have been described, depending on the genetic mutation in the hepatobiliary transport pathway:

  • PFIC 1 (Byler disease) is caused by impaired bile salt secretion due to mutations in the ATP8B1 gene encoding for the familial intrahepatic cholestasis 1 (FIC 1) protein
  • PFIC 2 is caused by impaired bile salt secretion due to mutations in the ABCB11 gene encoding for the bile salt export pump (BSEP) protein
  • PFIC 3 is caused by impaired biliary phospholipid secretion due to a defect in ABCB4 encoding for multidrug resistance 3 (MDR3) protein.28

PFIC 1 and 2 manifest with low gamma-glutamyl transferase (GGT) cholestasis, whereas PFIC 3 presents with high GGT cholestasis.

PFIC 1 and PFIC 2 usually cause cholestasis in early infancy, but PFIC 3 can cause cholestasis in late infancy, childhood, and even adulthood.

Because ATP8B1 is expressed in other tissues, PFIC 1 is characterized by extrahepatic manifestations such as sensorineural hearing loss, growth failure, severe diarrhea, and pancreatic insufficiency.

 

 

Implications of PFIC in adulthood

PFIC 1 and 2 (low-GGT cholestasis) are usually progressive and often lead to end-stage liver disease and cirrhosis before adulthood. Therefore, almost all patients with PFIC 1 and 2 undergo liver transplant or at least a biliary diversion procedure before reaching adulthood. Intractable pruritus is one of the most challenging clinical manifestations in patients with PFIC.

First-line management is pharmacologic and includes ursodeoxycholic acid, antihistamines (eg, hydroxyzine), bile acid sequestrants (eg, cholestyramine, colestipol), naltrexone, and rifampin, but these have limited efficacy.10

Most patients, especially those with PFIC 1 and 2, undergo a biliary diversion procedure such as partial external biliary diversion (cholecystojejunocutaneostomy), ileal exclusion, or partial internal biliary diversion (cholecystojejunocolic anastomosis) to decrease enterohepatic circulation of bile salts. The efficacy of these procedures is very limited in patients with established cirrhosis. Excessive losses of bile can occur through the biliary stoma, leading to dehydration in patients with external biliary diversion. In patients who are not candidates for biliary diversion, endoscopic nasobiliary drainage of pancreatobiliary secretions could be achieved by placing a catheter in the common bile duct; this has been reported to be effective in relieving cholestasis in a few cases.29

Liver transplant is needed in patients with progressive liver disease and intractable pruritus despite medical management and biliary diversion. Unlike in biliary atresia, liver transplant is not curative in PFIC 1, due to extrahepatic manifestations: patients with PFIC 1 can still have intractable diarrhea and pancreatitis after liver transplant. More importantly, allograft steatohepatitis with further progression to cirrhosis can occur after liver transplant in patients with PFIC 1. Interestingly, biliary diversion has been reported to improve graft steatosis and diarrhea after liver transplant.30

Although graft survival after transplant is good in patients with PFIC 2, recurrence of low-GGT cholestasis has been reported and is believed to be due to the formation of anti-bile salt export pump (anti-BSEP) antibodies by the host immune system in response to exposure to new proteins from the transplant graft.31

Cancer. The risk of malignancy, especially hepatocellular carcinoma, is also increased in PFIC 2, affecting nearly 15% of patients. Therefore, standard hepatocellular carcinoma surveillance with ultrasonography or alpha-fetoprotein testing or both is recommended in patients with PFIC 2. Cholangiocarcinoma and pancreatic adenocarcinoma have also been reported in patients with PFIC 2.20

Incomplete penetrance of mutations in ATP8B1 and ABCB11 can cause recurrent episodes of cholestasis and pruritus with asymptomatic periods between episodes, referred to as benign recurrent intrahepatic cholestasis. Prognosis is usually good, with no progression to cirrhosis.32

Pregnancy. In contrast to FIC 1 and BSEP deficiency, MDR3 defects lead to a wide phenotypic spectrum depending on the type of mutation. Heterozygous mutation is associated with increased risk of development of cholestasis during pregnancy, which typically presents with generalized pruritus in the third trimester and is associated with adverse fetal outcomes. Intrahepatic cholestasis of pregnancy is usually treated with ursodeoxycholic acid, with reported improvement in pruritus, liver function, and pregnancy outcomes.33

In adults, drug-induced liver injury and idiopathic cirrhosis have also been described with MDR3 defects. Intrahepatic lithiasis and cholesterol gallstones can also occur with MDR3 defects as a result of impaired secretion of biliary phospholipid.32 Despite intrahepatic cholestasis of pregnancy, successful outcomes have been reported in women with PFIC.20

OTHER CHILDHOOD-ONSET INHERITED CHOLESTATIC DISEASES

Cystic fibrosis-associated liver disease

Nearly 40% of patients with cystic fibrosis develop liver disease.34 Cystic fibrosis-associated liver disease encompasses a broad clinical spectrum including asymptomatic elevation of aminotransferases, neonatal cholestasis, hepatic steatosis, focal biliary cirrhosis, and multilobar cirrhosis. Cirrhosis and portal hypertension can occur in 5% to 10% of patients and is the third-leading cause of death in patients with cystic fibrosis.35

Risk factors for cystic fibrosis-associated liver disease include male sex, meconium ileus, and severe CFTR gene mutation (class I–III) with pancreatic insufficiency. Cystic fibrosis-related cirrhosis is more frequent in children and adolescents, whereas noncirrhotic portal hypertension and intrahepatic cholangiopathies are more common in adults.36

Limited available studies support treatment with ursodeoxycholic acid in patients with cholestasis to delay the progression of liver disease, but the impact of this drug on long-term outcome is unknown.29

Most patients remain in compensated cirrhosis for many years before progressing to decompensated cirrhosis requiring liver transplant. Other indications for liver transplant include recurrent intractable variceal bleeding, hepatopulmonary syndrome, and portopulmonary hypertension. Combined liver and lung transplant may be considered in patients with advanced liver and lung disease. Outcomes after isolated liver or liver-lung transplant in cystic fibrosis patients have been comparable to those in patients with other liver diseases.37

Defects in bile acid synthesis

Inherited defects of enzymes required for the synthesis of primary bile acids from cholesterol can cause cholestasis from impaired bile flow and production of hepatotoxic aberrant bile acids. The clinical presentation varies depending on the enzymatic defect and can range from liver disease of varying severity to neurologic manifestations. Idiopathic late-onset cholestasis and cirrhosis of unknown etiology have been reported in adults with bile acid synthesis defects.38,39 Therefore, this diagnosis should be considered in cases of cryptogenic cirrhosis and other cholestatic features.

Treatment with primary bile acids (cholic acid) has been effective in most patients with defective bile acid synthesis.

Primary sclerosing cholangitis

Primary sclerosing cholangitis is characterized by progressive obliteration of intrahepatic and extrahepatic bile ducts and is most commonly seen in patients with inflammatory bowel disease. Sclerosing cholangitis can also be secondary to other diseases in children such as immunodeficiency syndromes, Langerhans cell histiocytosis, cystic fibrosis, or sickle cell anemia.40 Neonatal sclerosing cholangitis is a rare autosomal-recessive disease characterized by a severe form of cholangiopathy in neonates and young infants requiring transplant. It can be associated with Kabuki syndrome and neonatal ichthyosis-sclerosing cholangitis syndrome.

Treatment options are limited. Ursodeoxycholic acid and oral vancomycin have variable efficacy. Liver transplant is needed in patients with decompensated cirrhosis. Patients with primary sclerosing cholangitis, especially adults, are at higher risk of developing cholangiocarcinoma, and therefore screening with ultrasonography or magnetic resonance imaging every 6 to 12 months is recommended.

The risk of preterm and cesarean deliveries may be elevated in women with primary sclerosing cholangitis, though data are limited.33

 

 

PEDIATRIC LIVER TRANSPLANT RECIPIENTS WHO SURVIVE INTO ADULTHOOD

Adolescent rebellion poses risks

Outcomes of liver transplant in children and adolescents have improved tremendously in the past 2 decades with advances in surgical techniques, pre- and postoperative management, organ preservation, and immunosuppression. Now, most pediatric liver transplant recipients survive into adulthood, creating a unique challenge for internists and adult  care hepatologists.41

In rebellious adolescents and young adults, risk-taking behavior, nonadherence to immunosuppressive medications, alcohol intake, and substance abuse increase the risk of graft rejection and loss. Current immunosuppressive drugs such as calcineurin inhibitors (tacrolimus, cyclosporine), mycophenolate mofetil, sirolimus, and corticosteroids have drastically decreased rejection rates in compliant patients.41 Educating patients on the importance of taking their medications and avoiding alcohol and drug abuse is especially important for adolescents and young adults, as rates of nonadherence are high in these age groups.

Although pregnancy is usually successful after liver transplant, it should be considered high-risk due to reported complications such as graft rejection, diabetes, preeclampsia, sepsis, prematurity, and low birth weight. Conception should be avoided for at least 1 year after transplant.42 Appropriate counseling with regard to pregnancy and contraception is important.

There is no consensus on breastfeeding, but it is considered safe in women on low-dose calcineurin inhibitors.43

Life is better with a new liver, but patients have special needs

Liver transplant is life-saving and improves quality of life. However, long-term pediatric liver transplant recipients face challenges such as strict adherence to medications and follow-up visits, avoiding exposure to infections, and fear of graft rejection.

Chronic liver disease in children leads to failure to thrive, growth failure, and even delayed puberty, which resolve in most patients after liver transplant before adulthood in the absence of other comorbidities.44 However, these patients are reported to have lower psychosocial functioning and more psychiatric disorders such as anxiety or posttraumatic disorder.41,44

Therefore, a psychologist or other mental health professional should be part of the management team from the time of pretransplant assessment to identify mental health problems and the need for adjustments before liver transplant. Ongoing psychosocial assessment after liver transplant is equally important to identify risks such as drug or alcohol abuse, depression, posttraumatic stress disorder, and medication nonadherence, all of which can negatively affect posttransplant outcome.45

In addition, assessment of family functioning and structure is important for good long-term outcomes posttransplant; therefore, a social worker should also be a part of the transplant team. Psyschosocial assessment tools can identify high-risk candidates who would benefit from earlier intervention to avoid any negative impact posttransplant.

Neurocognitive development can be delayed in children with chronic liver disease, and the delay may persist even after liver transplant, with reported impairments in intellectual ability, language, verbal, and visuospatial functioning skills.41 In spite of this, a recent study found that more than half the study patients were employed at a median follow-up of 24 years from liver transplant and a median age of 27.46

Remarkably, pediatric liver transplant recipients have reported quality of life comparable to that in the general population,47 and even better than in patients with other chronic illnesses.48

Long-term medical comorbidities in pediatric liver transplant recipients

Favorable outcomes such as long-term survival and good quality of life in pediatric liver transplant recipients are lessened by late complications such as portal vein thrombosis or biliary strictures needing interventions, chronic graft rejection, adverse effects of immunosuppression, and recurrence of the disease.

Split-liver transplant—splitting a deceased-donor allograft to provide grafts for 2 recipients—has revolutionized liver transplant by increasing the donor pool and thereby decreasing waitlist mortality rates, especially in pediatric candidates. Despite this advantage, split-liver transplant is technically challenging and associated with increased perioperative complications compared with whole-liver transplant, especially in adult recipients. Recently, experienced centers have reported favorable outcomes with split-liver transplant comparable to those with whole-liver transplant; therefore, split-liver transplant should be considered after careful evaluation of donor organ and recipient clinical status.49

Old age in the recipient can also adversely affect liver transplant outcomes.50

Interestingly, even in patients whose clinical course is unremarkable and biochemical values are normal, graft hepatitis or fibrosis of unknown cause with progression to cirrhosis has been described in the decade after transplant.41

Chronic rejection with eventual graft loss may be related to nonadherence in adolescents and can be reduced with use of an additional immunosuppressant such as sirolimus or mycophenolate. Chronic kidney disease can occur in about one-third of liver transplant recipients secondary to renal disease associated with primary disease (like Alagille syndrome), hepatorenal syndrome, and most importantly, use of calcineurin inhibitors.45

Components of the metabolic syndrome such as type 2 diabetes, obesity, nonalcoholic fatty liver disease, hypertension, and dyslipidemia are also seen in long-term pediatric liver transplant survivors. Internists are advised to screen for these comorbidities so that interventions can be applied early to improve long-term health outcomes and graft survival.

Of importance, multiple studies have shown a 2-fold increase in the rates of de novo malignancy in liver transplant recipients, including solid-organ and lymphoproliferative cancers, probably due to long-term immunosuppression. Posttransplant lymphoproliferative disorder occurs at lower rates than with other solid-organ transplants; its incidence is greatest in pediatric patients and in the first 12 to 18 months after transplant.51

 

 

TRANSITION TO ADULT CARE

While the number of patients with childhood-onset liver disease and pediatric liver transplant recipients who survive into adulthood is increasing, there are no established guidelines or formal models for transitioning these patients into adult care. Consequently, studies on transitional process have examined various issues such as patient and parent frustration, poor medical knowledge among patients during transition, lack of parental facilitation, and inadequate knowledge on disease process among adult-care hepatologists.52–54

selvakumar_pediatriccholestaticliverdisease_f2.jpg
%3Cp%3EFigure%202.%20Transitional%20model%20in%20childhood-onset%20cholestatic%20diseases.%3C%2Fp%3E
A position paper on transitional care from Europe has put forth important strategies.55 A multidisciplinary team including a pediatric provider, nurse practitioner, transplant coordinator, psychologist, social worker, and adult provider is vital for a successful transition. Key requirements for an adolescent patient include knowledge about the disease, demonstration of responsibility toward disease management, and self-management of healthcare needs such as scheduling appointments or insurance needs. This demanding task could be streamlined if pediatric liver transplant programs could adopt a formal transitional model (Figure 2). The transition could start as early as age 10 to 12, with gradual transfer of health management responsibilities from parents to adolescents from age 13 to 17.56

A prolonged period of transition up to age 25 is preferred in complicated cases. Distinctive consideration for transition should include those with neurocognitive developmental delay from underlying disease or hepatic encephalopathy before transplant. These patients need additional support and time to achieve independence in health management before transition.57 Validated questionnaires are available to assess readiness to transition into adult care,58 implying that the decision to transition should not be based solely on age. 

References
  1. Kelly DA, Bucuvalas JC, Alonso EM, et al; American Association for the Study of Liver Diseases; American Society of Transplantation. Long-term medical management of the pediatric patient after liver transplantation: 2013 practice guideline by the American Association for the Study of Liver Diseases and the American Society of Transplantation. Liver Transpl 2013; 19(8):798–825. doi:10.1002/lt.23697
  2. Rosen DS, Blum RW, Britto M, Sawyer SM, Siegel DM; Society for Adolescent Medicine. Transition to adult health care for adolescents and young adults with chronic conditions: position paper of the Society for Adolescent Medicine. J Adolesc Health 2003; 33(4):309–311. pmid:14519573
  3. Fawaz R, Baumann U, Ekong U, et al. Guideline for the evaluation of cholestatic jaundice in infants: joint recommendations of the North American Society for Pediatric Gastroenterology, Hepatology, and Nutrition and the European Society for Pediatric Gastroenterology, Hepatology, and Nutrition. J Pediatr Gastroenterol Nutr 2017; 64(1):154–168. doi:10.1097/MPG.0000000000001334
  4. Vajro P, Ferrante L, Lenta S, Mandato C, Persico M. Management of adults with paediatric-onset chronic liver disease: strategic issues for transition care. Dig Liver Dis 2014; 46(4):295–301. doi:10.1016/j.dld.2013.10.018
  5. Davenport M, Tizzard SA, Underhill J, Mieli-Vergani G, Portmann B, Hadzic N. The biliary atresia splenic malformation syndrome: a 28-year single-center retrospective study. J Pediatr 2006; 149(3):393–400. doi:10.1016/j.jpeds.2006.05.030
  6. Balistreri WF, Bezerra JA. Whatever happened to “neonatal hepatitis?” Clin Liver Dis 2006; 10(1):27–53. doi:10.1016/j.cld.2005.10.008
  7. Serinet MO, Wildhaber BE, Broué P, et al. Impact of age at Kasai operation on its results in late childhood and adolescence: a rational basis for biliary atresia screening. Pediatrics 2009; 123(5):1280–1286. doi:10.1542/peds.2008-1949
  8. de Vries W, Homan-Van der Veen J, Hulscher JB, Hoekstra-Weebers JE, Houwen RH, Verkade HJ; Netherlands Study Group of Biliary Atresia Registry. Twenty-year transplant-free survival rate among patients with biliary atresia. Clin Gastroenterol Hepatol 2011; 9(12):1086–1091. doi:10.1016/j.cgh.2011.07.024
  9. Lykavieris P, Chardot C, Sokhn M, Gauthier F, Valayer J, Bernard O. Outcome in adulthood of biliary atresia: a study of 63 patients who survived for over 20 years with their native liver. Hepatology 2005; 41(2):366–371. doi:10.1002/hep.20547
  10. Joshi D, Gupta N, Samyn M, Deheragoda M, Dobbels F, Heneghan MA. The management of childhood liver diseases in adulthood. J Hepatol 2017; 66(3):631–644. doi:10.1016/j.jhep.2016.11.013
  11. Shneider BL, Abel B, Haber B, et al; Childhood Liver Disease Research and Education Network. Portal hypertension in children and young adults with biliary atresia. J Pediatr Gastroenterol Nutr 2012; 55(5):567–573. doi:10.1097/MPG.0b013e31826eb0cf
  12. Garcia-Tsao G, Abraldes JG, Berzigotti A, Bosch J. Portal hypertensive bleeding in cirrhosis: risk stratification, diagnosis, and management: 2016 practice guidance by the American Association for the Study of Liver Diseases. Hepatology 2017; 65(1):310–335. doi:10.1002/hep.28906
  13. Shneider BL, Brown MB, Haber B, et al; Biliary Atresia Research Consortium. A multicenter study of the outcome of biliary atresia in the United States, 1997 to 2000. J Pediatr 2006; 148(4):467–474. doi:10.1016/j.jpeds.2005.12.054
  14. Hung PY, Chen CC, Chen WJ, et al. Long-term prognosis of patients with biliary atresia: a 25 year summary. J Pediatr Gastroenterol Nutr 2006; 42(2):190–195. doi:10.1097/01.mpg.0000189339.92891.64
  15. Verkade HJ, Bezerra JA, Davenport M, et al. Biliary atresia and other cholestatic childhood diseases: advances and future challenges. J Hepatol 2016; 65(3):631–642. doi:10.1016/j.jhep.2016.04.032
  16. Hadžic N, Quaglia A, Portmann B, et al. Hepatocellular carcinoma in biliary atresia: King’s College Hospital experience. J Pediatr 2011; 159(4):617–622.e1. doi:10.1016/j.jpeds.2011.03.004
  17. Sokol RJ, Shepherd RW, Superina R, Bezerra JA, Robuck P, Hoofnagle JH. Screening and outcomes in biliary atresia: summary of a National Institutes of Health workshop. Hepatology 2007; 46(2):566–581. doi:10.1002/hep.21790
  18. Li L, Krantz ID, Deng Y, et al. Alagille syndrome is caused by mutations in human Jagged1, which encodes a ligand for Notch1. Nat Genet 1997; 16(3):243–251. doi:10.1038/ng0797-243
  19. Saleh M, Kamath BM, Chitayat D. Alagille syndrome: clinical perspectives. Appl Clin Genet 2016; 9:75–82. doi:10.2147/TACG.S86420
  20. Bass LM, Kamath BM. Inherited disorders of cholestasis in adulthood. Clinical Liver Disease 2013; 2(5):200–203. doi:10.1002/cld.245
  21. Kamath BM, Yin W, Miller H, Anand R, Rand EB, Alonso E, Bucuvalas J; Studies of Pediatric Liver Transplantation. Outcomes of liver transplantation for patients with Alagille syndrome: the studies of pediatric liver transplantation experience. Liver Transpl 2012; 18(8):940–948. doi:10.1002/lt.23437
  22. Arnon R, Annunziato R, Schiano T, et al. Orthotopic liver transplantation for adults with Alagille syndrome. Clin Transplant 2012; 26(2):E94–E100. doi:10.1111/j.1399-0012.2011.01574.x
  23. Salem JE, Bruguiere E, Iserin L, Guiochon-Mantel A, Plouin PF. Hypertension and aortorenal disease in Alagille syndrome. J Hypertens 2012; 30(7):1300–1306. doi:10.1097/HJH.0b013e3283531e1f
  24. Kamath BM, Podkameni G, Hutchinson AL, et al. Renal anomalies in Alagille syndrome: a disease-defining feature. Am J Med Genet A 2012; 158A(1):85–89. doi:10.1002/ajmg.a.34369
  25. Kamath BM, Bason L, Piccoli DA, Krantz ID, Spinner NB. Consequences of JAG1 mutations. J Med Genet 2003; 40(12):891–895. pmid:14684686
  26. Emerick KM, Krantz ID, Kamath BM, et al. Intracranial vascular abnormalities in patients with Alagille syndrome. J Pediatr Gastroenterol Nutr 2005; 41(1):99–107. pmid:15990638
  27. Ferrarese A, Senzolo M, Burra P. Successful pregnancy in Alagille syndrome. Dig Liver Dis 2015; 47(1):86–87. doi:10.1016/j.dld.2014.08.047
  28. Davit-Spraul A, Fabre M, Branchereau S, et al. ATP8B1 and ABCB11 analysis in 62 children with normal gamma-glutamyl transferase progressive familial intrahepatic cholestasis (PFIC): phenotypic differences between PFIC1 and PFIC2 and natural history. Hepatology 2010; 51(5):1645–1655. doi:10.1002/hep.23539
  29. Zellos A, Lykopoulou L, Polydorou A, et al. Nasobiliary drainage in an episode of intrahepatic cholestasis in a child with mild ABCB11 disease. J Pediatr Gastroenterol Nutr 2012; 55(1):88–90. doi:10.1097/MPG.0b013e31822f2bda
  30. Alrabadi LS, Morotti RA, Valentino PL, Rodriguez-Davalos MI, Ekong UD, Emre SH. Biliary drainage as treatment for allograft steatosis following liver transplantation for PFIC-1 disease: a single-center experience. Pediatr Transplant 2018; 22(4):e13184. doi:10.1111/petr.13184
  31. Kubitz R, Dröge C, Kluge S, et al. Autoimmune BSEP disease: disease recurrence after liver transplantation for progressive familial intrahepatic cholestasis. Clin Rev Allergy Immunol 2015; 48(2–3):273–284. doi:10.1007/s12016-014-8457-4
  32. Jacquemin E. Progressive familial intrahepatic cholestasis. Clin Res Hepatol Gastroenterol 2012; 36(suppl 1):S26–S35. doi:10.1016/S2210-7401(12)70018-9
  33. Pataia V, Dixon PH, Williamson C. Pregnancy and bile acid disorders. Am J Physiol Gastrointest Liver Physiol 2017; 313(1):G1–G6. doi:10.1152/ajpgi.00028.2017
  34. Lamireau T, Monnereau S, Martin S, Marcotte JE, Winnock M, Alvarez F. Epidemiology of liver disease in cystic fibrosis: a longitudinal study. J Hepatol 2004; 41(6):920–925. doi:10.1016/j.jhep.2004.08.006
  35. Bolia R, Ooi CY, Lewindon P, et al. Practical approach to the gastrointestinal manifestations of cystic fibrosis. J Paediatr Child Health 2018; 54(6):609–619. doi:10.1111/jpc.13921
  36. Debray D, Kelly D, Houwen R, Strandvik B, Colombo C. Best practice guidance for the diagnosis and management of cystic fibrosis-associated liver disease. J Cyst Fibros 2011; 10(suppl 2):S29–S36. doi:10.1016/S1569-1993(11)60006-4
  37. Fridell JA, Bond GJ, Mazariegos G V, et al. Liver transplantation in children with cystic fibrosis: a long-term longitudinal review of a single center’s experience. J Pediatr Surg 2003; 38(8):1152–1156. pmid:12891484
  38. Fischler B, Bodin K, Stjernman H, et al. Cholestatic liver disease in adults may be due to an inherited defect in bile acid biosynthesis. J Intern Med 2007; 262(2):254–262. doi:10.1111/j.1365-2796.2007.01814.x
  39. Molho-Pessach V, Rios JJ, Xing C, Setchell KD, Cohen JC, Hobbs HH. Homozygosity mapping identifies a bile acid biosynthetic defect in an adult with cirrhosis of unknown etiology. Hepatology 2012; 55(4):1139–1145. doi:10.1002/hep.24781
  40. Mieli-Vergani G, Vergani D. Sclerosing cholangitis in children and adolescents. Clin Liver Dis 2016; 20(1):99–111. doi:10.1016/j.cld.2015.08.008
  41. Kelly D, Wray J. The adolescent liver transplant patient. Clin Liver Dis 2014; 18(3):613–632. doi:10.1016/j.cld.2014.05.006
  42. Westbrook RH, Yeoman AD, Agarwal K, et al. Outcomes of pregnancy following liver transplantation: the King’s College Hospital experience. Liver Transpl. 2015; 21(9):1153–1159. doi:10.1002/lt.24182
  43. Hammoud GM, Almashhrawi AA, Ahmed KT, Rahman R, Ibdah JA. Liver diseases in pregnancy: liver transplantation in pregnancy. World J Gastroenterol 2013; 19(43):7647–7651. doi:10.3748/wjg.v19.i43.7647
  44. Codoner-Franch P, Bernard O, Alvarez F. Long-term follow-up of growth in height after successful liver transplantation. J Pediatr 1994; 124(3):368–373. pmid:8120704
  45. Shemesh E. Assessment and management of psychosocial challenges in pediatric liver transplantation. Liver Transpl 2008; 14(9):1229–1236. doi:10.1002/lt.21582
  46. Martinelli J, Habes D, Majed L, et al. Long-term outcome of liver transplantation in childhood: a study of 20-year survivors. Am J Transplant 2018; 18(7):1680–1689. doi:10.1111/ajt.14626
  47. Roblin E, Audhuy F, Boillot O, Rivet C, Lachaux A. Long-term quality of life after pediatric liver transplantation. Arch Pediatr 2012; 19(10):1039–1052. French. doi:10.1016/j.arcped.2012.06.020
  48. Duffy JP, Kao K, Ko CY, et al. Long-term patient outcome and quality of life after liver transplantation: analysis of 20-year survivors. Ann Surg 2010; 252(4):652–661. doi:10.1097/SLA.0b013e3181f5f23a
  49. Hackl C, Schmidt KM, Süsal C, Döhler B, Zidek M, Schlitt HJ. Split liver transplantation: Current developments. World J Gastroenterol 2018; 24(47):5312–5321. doi:10.3748/wjg.v24.i47.5312
  50. Durand F, Levitsky J, Cauchy F, Gilgenkrantz H, Soubrane O, Francoz C. Age and liver transplantation. J Hepatol 2019; 70(4):745–758. doi:10.1016/j.jhep.2018.12.009
  51. Chandok N, Watt KD. Burden of de novo malignancy in the liver transplant recipient. Liver Transpl 2012; 18(11):1277–1289. doi:10.1002/lt.23531
  52. Ferrarese A, Germani G, Lazzaro S, et al. Short-term outcomes of paediatric liver transplant recipients after transition to Adult Healthcare Service. Liver Int 2018; 38(7):1316–1321. doi:10.1111/liv.13655
  53. Wright J, Elwell L, McDonagh JE, Kelly DA, Wray J. “Are these adult doctors gonna know me?” Experiences of transition for young people with a liver transplant. Pediatr Transplant 2016; 20(7):912–920. doi:10.1111/petr.12777
  54. Heldman MR, Sohn MW, Gordon EJ, et al. National survey of adult transplant hepatologists on the pediatric-to-adult care transition after liver transplantation. Liver Transpl 2015; 21(2):213–223. doi:10.1002/lt.24044
  55. Vajro P, Fischler B, Burra P, et al. The health care transition of youth with liver disease into the adult health system. J Pediatr Gastroenterol Nutr 2018; 66(6):976–990. doi:10.1097/MPG.0000000000001965
  56. Fredericks EM, Lopez MJ. Transition of the adolescent transplant patient to adult care. Clin Liver Dis (Hoboken) 2013; 2(5):223–226. doi:10.1002/cld.243
  57. Kaufman M. Transition of cognitively delayed adolescent organ transplant recipients to adult care. Pediatr Transplant 2006; 10(4):413–417. doi:10.1111/j.1399-3046.2006.00491.x
  58. Sawicki GS, Lukens-Bull K, Yin X, et al. Measuring the transition readiness of youth with special healthcare needs: validation of the TRAQ—Transition Readiness Assessment Questionnaire. J Pediatr Psychol 2011; 36(2):160–171. doi:10.1093/jpepsy/jsp128
Article PDF
Author and Disclosure Information

Praveen Kumar Conjeevaram Selvakumar, MD
Department of Pediatric Gastroenterology and Hepatology, Cleveland Clinic

Vera Hupertz, MD
Department of Pediatric Gastroenterology and Hepatology, Cleveland Clinic; Clinical Assistant Professor, Cleveland Clinic Lerner College of Medicine of Case Western Reserve University, Cleveland, OH

Naveen Mittal, MD
Texas Liver Institute, University of Texas Health, San Antonio, TX

Kris V. Kowdley, MD
Liver Care Network and Organ Care Research, Swedish Medical Center, Seattle, WA

Naim Alkhouri, MD
Director of the Metabolic Center, Texas Liver Institute, University of Texas Health, San Antonio, TX

Address: Naim Alkhouri, MD, Metabolic Center, Texas Liver Institute, University of Texas Health, 607 Camden Street, San Antonio, TX 78215; alkhouri@txliver.com

Dr. Kowdley has disclosed financial relationships (consulting, independent contracting, serving on advisory committee or review panels, teaching and speaking, or holding intellectual property rights) with Enanta Pharmaceuticals, Gilead Sciences, GlaxoSmithKline, High Tide Health, Intercept Pharmaceuticals, and UpToDate.

Issue
Cleveland Clinic Journal of Medicine - 86(7)
Publications
Topics
Page Number
454-464
Legacy Keywords
pediatric cholestatic liver disease, liver, cholestasis, children, biliary atresia, transition of care, Alagille syndrome, progressive familial intrahepatic cholestasis, PFIC, primary sclerosing cholangitis, bile acid synthesis defect, extrahepatic biliary atresia, cystic fibrosis, CF, MDR3, Praveen Kumar Conjeevaram Selvakumar, Vera Hupertz, Naveen Mittal, Kris Kowdley, Naim Alkhouri
Sections
Author and Disclosure Information

Praveen Kumar Conjeevaram Selvakumar, MD
Department of Pediatric Gastroenterology and Hepatology, Cleveland Clinic

Vera Hupertz, MD
Department of Pediatric Gastroenterology and Hepatology, Cleveland Clinic; Clinical Assistant Professor, Cleveland Clinic Lerner College of Medicine of Case Western Reserve University, Cleveland, OH

Naveen Mittal, MD
Texas Liver Institute, University of Texas Health, San Antonio, TX

Kris V. Kowdley, MD
Liver Care Network and Organ Care Research, Swedish Medical Center, Seattle, WA

Naim Alkhouri, MD
Director of the Metabolic Center, Texas Liver Institute, University of Texas Health, San Antonio, TX

Address: Naim Alkhouri, MD, Metabolic Center, Texas Liver Institute, University of Texas Health, 607 Camden Street, San Antonio, TX 78215; alkhouri@txliver.com

Dr. Kowdley has disclosed financial relationships (consulting, independent contracting, serving on advisory committee or review panels, teaching and speaking, or holding intellectual property rights) with Enanta Pharmaceuticals, Gilead Sciences, GlaxoSmithKline, High Tide Health, Intercept Pharmaceuticals, and UpToDate.

Author and Disclosure Information

Praveen Kumar Conjeevaram Selvakumar, MD
Department of Pediatric Gastroenterology and Hepatology, Cleveland Clinic

Vera Hupertz, MD
Department of Pediatric Gastroenterology and Hepatology, Cleveland Clinic; Clinical Assistant Professor, Cleveland Clinic Lerner College of Medicine of Case Western Reserve University, Cleveland, OH

Naveen Mittal, MD
Texas Liver Institute, University of Texas Health, San Antonio, TX

Kris V. Kowdley, MD
Liver Care Network and Organ Care Research, Swedish Medical Center, Seattle, WA

Naim Alkhouri, MD
Director of the Metabolic Center, Texas Liver Institute, University of Texas Health, San Antonio, TX

Address: Naim Alkhouri, MD, Metabolic Center, Texas Liver Institute, University of Texas Health, 607 Camden Street, San Antonio, TX 78215; alkhouri@txliver.com

Dr. Kowdley has disclosed financial relationships (consulting, independent contracting, serving on advisory committee or review panels, teaching and speaking, or holding intellectual property rights) with Enanta Pharmaceuticals, Gilead Sciences, GlaxoSmithKline, High Tide Health, Intercept Pharmaceuticals, and UpToDate.

Article PDF
Article PDF
Related Articles

Thanks to advances in medical science and our understanding of inherited and acquired liver disease, many more children with acquired or congenital liver disease survive into adulthood than they did 2 decades ago. Improvements in immunosuppression and surgery have increased the chances of pediatric liver transplant recipients reaching adulthood, with a survival rate of 75% at 15 to 20 years.1

selvakumar_pediatriccholestaticliverdisease_f1.jpg
%3Cp%3EFigure%201.%20Cholestasis%20in%20children%20can%20be%20caused%20by%20biliary%20tract%20obstruction%20such%20as%20in%20biliary%20atresia%20(top)%20or%20defects%20in%20forming%20and%20excreting%20bile%20acids%20and%20other%20components%20of%20bile%20(bottom).%3C%2Fp%3E
The causes of cholestasis in children are different from those in adults (Figure 1). Pediatric cholestasis is caused either by genetic defects that affect the process of synthesis, processing, or secreting bile, or by mechanical obstruction of the biliary tract, the classic example being biliary atresia.

With the growing number of adult patients with pediatric-onset liver disease, internists and adult hepatologists need to be aware of these liver diseases and develop expertise to manage this challenging group of patients. Moreover, young adults with pediatric-onset chronic liver disease pose distinct challenges such as pregnancy, adherence to medical regimens, and psychosocial changes in life.

These patients need a “transition of care” rather than a “transfer of care.” Transition of care is a multifaceted process that takes the medical, educational, and psychosocial needs of the patient into consideration before switching their care to adult care physicians, whereas transfer of care is simply an administrative process of change to adult care without previous knowledge of the patients.2

selvakumar_pediatriccholestaticliverdisease_t1.jpg
In this article, we discuss relatively common types of inherited childhood-onset cholestatic disease and their implications and management in adulthood (Table 1). Other acquired or rare cholestatic diseases are beyond the scope of this article.

BILIARY ATRESIA

Biliary atresia is a progressive inflammatory fibrosclerosing cholangiopathy of unknown cause. Its prevalence varies with geographic location, ranging from 1 in 6,000 to 1 in 19,000, with the highest prevalence in Taiwan.3

Biliary atresia usually presents within the first few weeks of life, with progressive cholestasis leading to failure to thrive and to fat-soluble vitamin deficiency. Approximately 20% of patients have congenital splenic, gastrointestinal, genitourinary, cardiac, and venous malformations.4,5 Untreated, biliary atresia progresses to end-stage liver disease and death within 2 years.

The first-line treatment for biliary atresia is to establish biliary outflow with the Kasai procedure (hepatic portoenterostomy), in which a jejunal limb is anastomosed in a Roux-en-Y with the liver. The outcomes of the Kasai procedure depend on the timing of surgery, so timely diagnosis of biliary atresia is crucial. When the Kasai procedure is performed within 60 days of birth, biliary flow is achieved in up to 70% of patients; but if performed after 90 days, biliary flow is achieved in fewer than 25%.6

Long-term outcomes of biliary atresia in patients with their native liver have been reported in a few studies.

In a French study,7 743 patients with biliary atresia underwent the Kasai procedure at a median age of 60 days. Survival rates were 57.1% at 2 years, 37.9% at 5 years, 32.4% at 10 years, and 28.5% at 15 years. In other studies,4–9 the 20-year transplant-free survival rate ranged from 23% to 46%. Therefore, at least one-third of children with biliary atresia survive to adulthood with their native liver.

Implications of biliary atresia in adulthood

Although the Kasai procedure improves biliary outflow, up to 70% of patients develop complications of biliary atresia such as progressive fibrosis, cirrhosis, portal hypertension, cholangitis, and hepatocellular carcinoma, even after a successful Kasai procedure.10

Portal hypertension with evidence of splenomegaly, thrombocytopenia, or ascites is found in two-thirds of long-term survivors of biliary atresia with a native liver, with variceal hemorrhage occurring in 30%.11 Therefore, patients with biliary atresia who have evidence of portal hypertension should be screened for varices with upper endoscopy on an annual basis. Management of variceal hemorrhage in these patients includes the use of octreotide, antibiotics, variceal ligation, and sclerotherapy; primary prophylaxis can be achieved with beta-blockers and endoscopic variceal ligation.12

Cholangitis is frequent, occurring in 40% to 60% of biliary atresia patients after the Kasai procedure, and about one-fourth of these patients have multiple episodes.13 The number of episodes of cholangitis negatively affects transplant-free survival.14 Patients with cholangitis should be adequately treated with oral or intravenous antibiotics depending on the severity of presentation. The role of prophylaxis with antibiotics remains unclear.15

Pulmonary complications such as hepato­pulmonary syndrome and portopulmonary hypertension can also occur in biliary atresia patients with a native liver. It is important for physicians to be aware of these complications and to screen for them, for example, with agitated saline echocardiography for hepatopulmonary syndrome and with echocardiography for portopulmonary hypertension. Timely screening is crucial, as the outcome of liver transplant depends on the severity at the time of transplant in these conditions, especially portopulmonary hypertension.

Hepatocellular carcinoma has been rarely reported in children with biliary atresia,16 so well-defined guidelines for screening in young adults with biliary atresia are lacking. Most centers recommend screening with ultrasonography of the abdomen and alpha-fetoprotein measurement every 6 months or annually starting soon after the Kasai procedure, since hepatocellular carcinoma has been reported in children as young as age 2.16

Transplant. Adult hepatologists are faced with the challenging task of deciding when it is time for transplant, balancing the long-term complications of biliary atresia with the risk of long-term immunosuppression after transplant. In addition, young adults with these complications may have preserved synthetic function, resulting in low Model for End-Stage Liver Disease (MELD) scores, which may complicate the process of listing for transplant.

Neurocognitive deficits are reported in children with biliary atresia,17 but young adults with biliary atresia generally have reasonable cognitive function and prospects for education and employment.

Pregnancy with successful outcomes has been reported.8

 

 

ALAGILLE SYNDROME

Alagille syndrome is an autosomal-dominant multisystemic disease caused by mutations in the JAG1 gene (accounting for > 95% of cases) and the NOTCH2 gene, with highly variable expression.18

Extrahepatic manifestations include butterfly vertebral defects, facial dysmorphism (eg, deep-set and low-set eyes, with characteristic “triangular” facies), posterior embryotoxon (a congenital defect of the eye characterized by an opaque ring around the margin of the cornea), peripheral pulmonary stenosis, renal abnormalities, and vascular malformations.

Hepatic manifestations vary from asymptomatic laboratory abnormalities to progressive cholestasis starting in early infancy with intractable pruritus, xanthomas, failure to thrive, and end-stage liver disease requiring liver transplant in childhood in 15% to 20% of patients.19

Implications of Alagille syndrome in adulthood

Transplant. Interestingly, the phenotype of hepatic disease is already established in childhood, with minimal or no progression in adulthood. Most children with minimal liver disease experience spontaneous resolution, whereas those with significant cholestasis might ultimately develop progressive liver fibrosis or cirrhosis requiring liver transplant in childhood. Only a small subset of children with minimal cholestasis progress to end-stage liver disease in late childhood or early adulthood.20 Therefore, liver transplant for progressive liver disease from significant cholestasis almost always occurs in childhood, usually between ages 1 and 4.21

In a retrospective study comparing posttransplant outcomes in children with Alagille syndrome and biliary atresia, 1-year patient survival was excellent overall in children with Alagille syndrome, although slightly lower than in children with biliary atresia, most likely owing to extrahepatic morbidities of Alagille syndrome and especially the use of immunosuppression in those with renal disease.21 Similarly, 1- and 5-year patient and graft survival outcomes of liver transplant in adults with Alagille syndrome were also excellent compared with those who received a liver transplant in childhood for Alagille syndrome or in adulthood for biliary atresia.22

Hepatocellular carcinoma has occurred in these patients in the absence of cirrhosis, which makes implementation of prognostic and surveillance strategies almost impossible to design for them. Annual ultrasonography with alpha-fetoprotein testing might be applicable in Alagille syndrome patients. However, deciding which patients should undergo this testing and when it should start will be challenging, given the paucity of data.

Cardiovascular disease. Cardiac phenotype is also mostly established in childhood, with the pulmonary vasculature being most commonly involved.19 In contrast, renal and other vascular abnormalities can manifest in adulthood. Renal manifestations vary and include structural anomalies such as hyperechoic kidneys or renal cysts, which can manifest in childhood, and some abnormalities such as hypertension and renal artery stenosis that can manifest in adulthood.23,24

Vasculopathy is reported to involve the intracranial, renal, and intra-abdominal blood vessels.25 Neurovascular accidents such as stroke and intracranial hemorrhage can occur at any age, with significant rates of morbidity and death.26 Therefore, some experts recommend magnetic resonance angiography every 5 years and before any major intervention to prevent these devastating complications.20

Pregnancy. Successful pregnancies have been reported. Preexisting cardiac and hepatic disease can complicate pregnancy depending on the severity of the disease. Because of the autosomal-dominant pattern of inheritance, infants have a 50% risk of the disease, so genetic counseling should be seriously considered before conception.27 Prenatal diagnosis is possible, but the lack of genotype-phenotype correlation precludes its use in clinical practice.

PROGRESSIVE FAMILIAL INTRAHEPATIC CHOLESTASIS

Progressive familial intrahepatic cholestasis (PFIC) is a heterogeneous group of autosomal-recessive conditions associated with disruption of bile formation causing cholestatic liver disease in infants and young children. Three types have been described, depending on the genetic mutation in the hepatobiliary transport pathway:

  • PFIC 1 (Byler disease) is caused by impaired bile salt secretion due to mutations in the ATP8B1 gene encoding for the familial intrahepatic cholestasis 1 (FIC 1) protein
  • PFIC 2 is caused by impaired bile salt secretion due to mutations in the ABCB11 gene encoding for the bile salt export pump (BSEP) protein
  • PFIC 3 is caused by impaired biliary phospholipid secretion due to a defect in ABCB4 encoding for multidrug resistance 3 (MDR3) protein.28

PFIC 1 and 2 manifest with low gamma-glutamyl transferase (GGT) cholestasis, whereas PFIC 3 presents with high GGT cholestasis.

PFIC 1 and PFIC 2 usually cause cholestasis in early infancy, but PFIC 3 can cause cholestasis in late infancy, childhood, and even adulthood.

Because ATP8B1 is expressed in other tissues, PFIC 1 is characterized by extrahepatic manifestations such as sensorineural hearing loss, growth failure, severe diarrhea, and pancreatic insufficiency.

 

 

Implications of PFIC in adulthood

PFIC 1 and 2 (low-GGT cholestasis) are usually progressive and often lead to end-stage liver disease and cirrhosis before adulthood. Therefore, almost all patients with PFIC 1 and 2 undergo liver transplant or at least a biliary diversion procedure before reaching adulthood. Intractable pruritus is one of the most challenging clinical manifestations in patients with PFIC.

First-line management is pharmacologic and includes ursodeoxycholic acid, antihistamines (eg, hydroxyzine), bile acid sequestrants (eg, cholestyramine, colestipol), naltrexone, and rifampin, but these have limited efficacy.10

Most patients, especially those with PFIC 1 and 2, undergo a biliary diversion procedure such as partial external biliary diversion (cholecystojejunocutaneostomy), ileal exclusion, or partial internal biliary diversion (cholecystojejunocolic anastomosis) to decrease enterohepatic circulation of bile salts. The efficacy of these procedures is very limited in patients with established cirrhosis. Excessive losses of bile can occur through the biliary stoma, leading to dehydration in patients with external biliary diversion. In patients who are not candidates for biliary diversion, endoscopic nasobiliary drainage of pancreatobiliary secretions could be achieved by placing a catheter in the common bile duct; this has been reported to be effective in relieving cholestasis in a few cases.29

Liver transplant is needed in patients with progressive liver disease and intractable pruritus despite medical management and biliary diversion. Unlike in biliary atresia, liver transplant is not curative in PFIC 1, due to extrahepatic manifestations: patients with PFIC 1 can still have intractable diarrhea and pancreatitis after liver transplant. More importantly, allograft steatohepatitis with further progression to cirrhosis can occur after liver transplant in patients with PFIC 1. Interestingly, biliary diversion has been reported to improve graft steatosis and diarrhea after liver transplant.30

Although graft survival after transplant is good in patients with PFIC 2, recurrence of low-GGT cholestasis has been reported and is believed to be due to the formation of anti-bile salt export pump (anti-BSEP) antibodies by the host immune system in response to exposure to new proteins from the transplant graft.31

Cancer. The risk of malignancy, especially hepatocellular carcinoma, is also increased in PFIC 2, affecting nearly 15% of patients. Therefore, standard hepatocellular carcinoma surveillance with ultrasonography or alpha-fetoprotein testing or both is recommended in patients with PFIC 2. Cholangiocarcinoma and pancreatic adenocarcinoma have also been reported in patients with PFIC 2.20

Incomplete penetrance of mutations in ATP8B1 and ABCB11 can cause recurrent episodes of cholestasis and pruritus with asymptomatic periods between episodes, referred to as benign recurrent intrahepatic cholestasis. Prognosis is usually good, with no progression to cirrhosis.32

Pregnancy. In contrast to FIC 1 and BSEP deficiency, MDR3 defects lead to a wide phenotypic spectrum depending on the type of mutation. Heterozygous mutation is associated with increased risk of development of cholestasis during pregnancy, which typically presents with generalized pruritus in the third trimester and is associated with adverse fetal outcomes. Intrahepatic cholestasis of pregnancy is usually treated with ursodeoxycholic acid, with reported improvement in pruritus, liver function, and pregnancy outcomes.33

In adults, drug-induced liver injury and idiopathic cirrhosis have also been described with MDR3 defects. Intrahepatic lithiasis and cholesterol gallstones can also occur with MDR3 defects as a result of impaired secretion of biliary phospholipid.32 Despite intrahepatic cholestasis of pregnancy, successful outcomes have been reported in women with PFIC.20

OTHER CHILDHOOD-ONSET INHERITED CHOLESTATIC DISEASES

Cystic fibrosis-associated liver disease

Nearly 40% of patients with cystic fibrosis develop liver disease.34 Cystic fibrosis-associated liver disease encompasses a broad clinical spectrum including asymptomatic elevation of aminotransferases, neonatal cholestasis, hepatic steatosis, focal biliary cirrhosis, and multilobar cirrhosis. Cirrhosis and portal hypertension can occur in 5% to 10% of patients and is the third-leading cause of death in patients with cystic fibrosis.35

Risk factors for cystic fibrosis-associated liver disease include male sex, meconium ileus, and severe CFTR gene mutation (class I–III) with pancreatic insufficiency. Cystic fibrosis-related cirrhosis is more frequent in children and adolescents, whereas noncirrhotic portal hypertension and intrahepatic cholangiopathies are more common in adults.36

Limited available studies support treatment with ursodeoxycholic acid in patients with cholestasis to delay the progression of liver disease, but the impact of this drug on long-term outcome is unknown.29

Most patients remain in compensated cirrhosis for many years before progressing to decompensated cirrhosis requiring liver transplant. Other indications for liver transplant include recurrent intractable variceal bleeding, hepatopulmonary syndrome, and portopulmonary hypertension. Combined liver and lung transplant may be considered in patients with advanced liver and lung disease. Outcomes after isolated liver or liver-lung transplant in cystic fibrosis patients have been comparable to those in patients with other liver diseases.37

Defects in bile acid synthesis

Inherited defects of enzymes required for the synthesis of primary bile acids from cholesterol can cause cholestasis from impaired bile flow and production of hepatotoxic aberrant bile acids. The clinical presentation varies depending on the enzymatic defect and can range from liver disease of varying severity to neurologic manifestations. Idiopathic late-onset cholestasis and cirrhosis of unknown etiology have been reported in adults with bile acid synthesis defects.38,39 Therefore, this diagnosis should be considered in cases of cryptogenic cirrhosis and other cholestatic features.

Treatment with primary bile acids (cholic acid) has been effective in most patients with defective bile acid synthesis.

Primary sclerosing cholangitis

Primary sclerosing cholangitis is characterized by progressive obliteration of intrahepatic and extrahepatic bile ducts and is most commonly seen in patients with inflammatory bowel disease. Sclerosing cholangitis can also be secondary to other diseases in children such as immunodeficiency syndromes, Langerhans cell histiocytosis, cystic fibrosis, or sickle cell anemia.40 Neonatal sclerosing cholangitis is a rare autosomal-recessive disease characterized by a severe form of cholangiopathy in neonates and young infants requiring transplant. It can be associated with Kabuki syndrome and neonatal ichthyosis-sclerosing cholangitis syndrome.

Treatment options are limited. Ursodeoxycholic acid and oral vancomycin have variable efficacy. Liver transplant is needed in patients with decompensated cirrhosis. Patients with primary sclerosing cholangitis, especially adults, are at higher risk of developing cholangiocarcinoma, and therefore screening with ultrasonography or magnetic resonance imaging every 6 to 12 months is recommended.

The risk of preterm and cesarean deliveries may be elevated in women with primary sclerosing cholangitis, though data are limited.33

 

 

PEDIATRIC LIVER TRANSPLANT RECIPIENTS WHO SURVIVE INTO ADULTHOOD

Adolescent rebellion poses risks

Outcomes of liver transplant in children and adolescents have improved tremendously in the past 2 decades with advances in surgical techniques, pre- and postoperative management, organ preservation, and immunosuppression. Now, most pediatric liver transplant recipients survive into adulthood, creating a unique challenge for internists and adult  care hepatologists.41

In rebellious adolescents and young adults, risk-taking behavior, nonadherence to immunosuppressive medications, alcohol intake, and substance abuse increase the risk of graft rejection and loss. Current immunosuppressive drugs such as calcineurin inhibitors (tacrolimus, cyclosporine), mycophenolate mofetil, sirolimus, and corticosteroids have drastically decreased rejection rates in compliant patients.41 Educating patients on the importance of taking their medications and avoiding alcohol and drug abuse is especially important for adolescents and young adults, as rates of nonadherence are high in these age groups.

Although pregnancy is usually successful after liver transplant, it should be considered high-risk due to reported complications such as graft rejection, diabetes, preeclampsia, sepsis, prematurity, and low birth weight. Conception should be avoided for at least 1 year after transplant.42 Appropriate counseling with regard to pregnancy and contraception is important.

There is no consensus on breastfeeding, but it is considered safe in women on low-dose calcineurin inhibitors.43

Life is better with a new liver, but patients have special needs

Liver transplant is life-saving and improves quality of life. However, long-term pediatric liver transplant recipients face challenges such as strict adherence to medications and follow-up visits, avoiding exposure to infections, and fear of graft rejection.

Chronic liver disease in children leads to failure to thrive, growth failure, and even delayed puberty, which resolve in most patients after liver transplant before adulthood in the absence of other comorbidities.44 However, these patients are reported to have lower psychosocial functioning and more psychiatric disorders such as anxiety or posttraumatic disorder.41,44

Therefore, a psychologist or other mental health professional should be part of the management team from the time of pretransplant assessment to identify mental health problems and the need for adjustments before liver transplant. Ongoing psychosocial assessment after liver transplant is equally important to identify risks such as drug or alcohol abuse, depression, posttraumatic stress disorder, and medication nonadherence, all of which can negatively affect posttransplant outcome.45

In addition, assessment of family functioning and structure is important for good long-term outcomes posttransplant; therefore, a social worker should also be a part of the transplant team. Psyschosocial assessment tools can identify high-risk candidates who would benefit from earlier intervention to avoid any negative impact posttransplant.

Neurocognitive development can be delayed in children with chronic liver disease, and the delay may persist even after liver transplant, with reported impairments in intellectual ability, language, verbal, and visuospatial functioning skills.41 In spite of this, a recent study found that more than half the study patients were employed at a median follow-up of 24 years from liver transplant and a median age of 27.46

Remarkably, pediatric liver transplant recipients have reported quality of life comparable to that in the general population,47 and even better than in patients with other chronic illnesses.48

Long-term medical comorbidities in pediatric liver transplant recipients

Favorable outcomes such as long-term survival and good quality of life in pediatric liver transplant recipients are lessened by late complications such as portal vein thrombosis or biliary strictures needing interventions, chronic graft rejection, adverse effects of immunosuppression, and recurrence of the disease.

Split-liver transplant—splitting a deceased-donor allograft to provide grafts for 2 recipients—has revolutionized liver transplant by increasing the donor pool and thereby decreasing waitlist mortality rates, especially in pediatric candidates. Despite this advantage, split-liver transplant is technically challenging and associated with increased perioperative complications compared with whole-liver transplant, especially in adult recipients. Recently, experienced centers have reported favorable outcomes with split-liver transplant comparable to those with whole-liver transplant; therefore, split-liver transplant should be considered after careful evaluation of donor organ and recipient clinical status.49

Old age in the recipient can also adversely affect liver transplant outcomes.50

Interestingly, even in patients whose clinical course is unremarkable and biochemical values are normal, graft hepatitis or fibrosis of unknown cause with progression to cirrhosis has been described in the decade after transplant.41

Chronic rejection with eventual graft loss may be related to nonadherence in adolescents and can be reduced with use of an additional immunosuppressant such as sirolimus or mycophenolate. Chronic kidney disease can occur in about one-third of liver transplant recipients secondary to renal disease associated with primary disease (like Alagille syndrome), hepatorenal syndrome, and most importantly, use of calcineurin inhibitors.45

Components of the metabolic syndrome such as type 2 diabetes, obesity, nonalcoholic fatty liver disease, hypertension, and dyslipidemia are also seen in long-term pediatric liver transplant survivors. Internists are advised to screen for these comorbidities so that interventions can be applied early to improve long-term health outcomes and graft survival.

Of importance, multiple studies have shown a 2-fold increase in the rates of de novo malignancy in liver transplant recipients, including solid-organ and lymphoproliferative cancers, probably due to long-term immunosuppression. Posttransplant lymphoproliferative disorder occurs at lower rates than with other solid-organ transplants; its incidence is greatest in pediatric patients and in the first 12 to 18 months after transplant.51

 

 

TRANSITION TO ADULT CARE

While the number of patients with childhood-onset liver disease and pediatric liver transplant recipients who survive into adulthood is increasing, there are no established guidelines or formal models for transitioning these patients into adult care. Consequently, studies on transitional process have examined various issues such as patient and parent frustration, poor medical knowledge among patients during transition, lack of parental facilitation, and inadequate knowledge on disease process among adult-care hepatologists.52–54

selvakumar_pediatriccholestaticliverdisease_f2.jpg
%3Cp%3EFigure%202.%20Transitional%20model%20in%20childhood-onset%20cholestatic%20diseases.%3C%2Fp%3E
A position paper on transitional care from Europe has put forth important strategies.55 A multidisciplinary team including a pediatric provider, nurse practitioner, transplant coordinator, psychologist, social worker, and adult provider is vital for a successful transition. Key requirements for an adolescent patient include knowledge about the disease, demonstration of responsibility toward disease management, and self-management of healthcare needs such as scheduling appointments or insurance needs. This demanding task could be streamlined if pediatric liver transplant programs could adopt a formal transitional model (Figure 2). The transition could start as early as age 10 to 12, with gradual transfer of health management responsibilities from parents to adolescents from age 13 to 17.56

A prolonged period of transition up to age 25 is preferred in complicated cases. Distinctive consideration for transition should include those with neurocognitive developmental delay from underlying disease or hepatic encephalopathy before transplant. These patients need additional support and time to achieve independence in health management before transition.57 Validated questionnaires are available to assess readiness to transition into adult care,58 implying that the decision to transition should not be based solely on age. 

Thanks to advances in medical science and our understanding of inherited and acquired liver disease, many more children with acquired or congenital liver disease survive into adulthood than they did 2 decades ago. Improvements in immunosuppression and surgery have increased the chances of pediatric liver transplant recipients reaching adulthood, with a survival rate of 75% at 15 to 20 years.1

selvakumar_pediatriccholestaticliverdisease_f1.jpg
%3Cp%3EFigure%201.%20Cholestasis%20in%20children%20can%20be%20caused%20by%20biliary%20tract%20obstruction%20such%20as%20in%20biliary%20atresia%20(top)%20or%20defects%20in%20forming%20and%20excreting%20bile%20acids%20and%20other%20components%20of%20bile%20(bottom).%3C%2Fp%3E
The causes of cholestasis in children are different from those in adults (Figure 1). Pediatric cholestasis is caused either by genetic defects that affect the process of synthesis, processing, or secreting bile, or by mechanical obstruction of the biliary tract, the classic example being biliary atresia.

With the growing number of adult patients with pediatric-onset liver disease, internists and adult hepatologists need to be aware of these liver diseases and develop expertise to manage this challenging group of patients. Moreover, young adults with pediatric-onset chronic liver disease pose distinct challenges such as pregnancy, adherence to medical regimens, and psychosocial changes in life.

These patients need a “transition of care” rather than a “transfer of care.” Transition of care is a multifaceted process that takes the medical, educational, and psychosocial needs of the patient into consideration before switching their care to adult care physicians, whereas transfer of care is simply an administrative process of change to adult care without previous knowledge of the patients.2

selvakumar_pediatriccholestaticliverdisease_t1.jpg
In this article, we discuss relatively common types of inherited childhood-onset cholestatic disease and their implications and management in adulthood (Table 1). Other acquired or rare cholestatic diseases are beyond the scope of this article.

BILIARY ATRESIA

Biliary atresia is a progressive inflammatory fibrosclerosing cholangiopathy of unknown cause. Its prevalence varies with geographic location, ranging from 1 in 6,000 to 1 in 19,000, with the highest prevalence in Taiwan.3

Biliary atresia usually presents within the first few weeks of life, with progressive cholestasis leading to failure to thrive and to fat-soluble vitamin deficiency. Approximately 20% of patients have congenital splenic, gastrointestinal, genitourinary, cardiac, and venous malformations.4,5 Untreated, biliary atresia progresses to end-stage liver disease and death within 2 years.

The first-line treatment for biliary atresia is to establish biliary outflow with the Kasai procedure (hepatic portoenterostomy), in which a jejunal limb is anastomosed in a Roux-en-Y with the liver. The outcomes of the Kasai procedure depend on the timing of surgery, so timely diagnosis of biliary atresia is crucial. When the Kasai procedure is performed within 60 days of birth, biliary flow is achieved in up to 70% of patients; but if performed after 90 days, biliary flow is achieved in fewer than 25%.6

Long-term outcomes of biliary atresia in patients with their native liver have been reported in a few studies.

In a French study,7 743 patients with biliary atresia underwent the Kasai procedure at a median age of 60 days. Survival rates were 57.1% at 2 years, 37.9% at 5 years, 32.4% at 10 years, and 28.5% at 15 years. In other studies,4–9 the 20-year transplant-free survival rate ranged from 23% to 46%. Therefore, at least one-third of children with biliary atresia survive to adulthood with their native liver.

Implications of biliary atresia in adulthood

Although the Kasai procedure improves biliary outflow, up to 70% of patients develop complications of biliary atresia such as progressive fibrosis, cirrhosis, portal hypertension, cholangitis, and hepatocellular carcinoma, even after a successful Kasai procedure.10

Portal hypertension with evidence of splenomegaly, thrombocytopenia, or ascites is found in two-thirds of long-term survivors of biliary atresia with a native liver, with variceal hemorrhage occurring in 30%.11 Therefore, patients with biliary atresia who have evidence of portal hypertension should be screened for varices with upper endoscopy on an annual basis. Management of variceal hemorrhage in these patients includes the use of octreotide, antibiotics, variceal ligation, and sclerotherapy; primary prophylaxis can be achieved with beta-blockers and endoscopic variceal ligation.12

Cholangitis is frequent, occurring in 40% to 60% of biliary atresia patients after the Kasai procedure, and about one-fourth of these patients have multiple episodes.13 The number of episodes of cholangitis negatively affects transplant-free survival.14 Patients with cholangitis should be adequately treated with oral or intravenous antibiotics depending on the severity of presentation. The role of prophylaxis with antibiotics remains unclear.15

Pulmonary complications such as hepato­pulmonary syndrome and portopulmonary hypertension can also occur in biliary atresia patients with a native liver. It is important for physicians to be aware of these complications and to screen for them, for example, with agitated saline echocardiography for hepatopulmonary syndrome and with echocardiography for portopulmonary hypertension. Timely screening is crucial, as the outcome of liver transplant depends on the severity at the time of transplant in these conditions, especially portopulmonary hypertension.

Hepatocellular carcinoma has been rarely reported in children with biliary atresia,16 so well-defined guidelines for screening in young adults with biliary atresia are lacking. Most centers recommend screening with ultrasonography of the abdomen and alpha-fetoprotein measurement every 6 months or annually starting soon after the Kasai procedure, since hepatocellular carcinoma has been reported in children as young as age 2.16

Transplant. Adult hepatologists are faced with the challenging task of deciding when it is time for transplant, balancing the long-term complications of biliary atresia with the risk of long-term immunosuppression after transplant. In addition, young adults with these complications may have preserved synthetic function, resulting in low Model for End-Stage Liver Disease (MELD) scores, which may complicate the process of listing for transplant.

Neurocognitive deficits are reported in children with biliary atresia,17 but young adults with biliary atresia generally have reasonable cognitive function and prospects for education and employment.

Pregnancy with successful outcomes has been reported.8

 

 

ALAGILLE SYNDROME

Alagille syndrome is an autosomal-dominant multisystemic disease caused by mutations in the JAG1 gene (accounting for > 95% of cases) and the NOTCH2 gene, with highly variable expression.18

Extrahepatic manifestations include butterfly vertebral defects, facial dysmorphism (eg, deep-set and low-set eyes, with characteristic “triangular” facies), posterior embryotoxon (a congenital defect of the eye characterized by an opaque ring around the margin of the cornea), peripheral pulmonary stenosis, renal abnormalities, and vascular malformations.

Hepatic manifestations vary from asymptomatic laboratory abnormalities to progressive cholestasis starting in early infancy with intractable pruritus, xanthomas, failure to thrive, and end-stage liver disease requiring liver transplant in childhood in 15% to 20% of patients.19

Implications of Alagille syndrome in adulthood

Transplant. Interestingly, the phenotype of hepatic disease is already established in childhood, with minimal or no progression in adulthood. Most children with minimal liver disease experience spontaneous resolution, whereas those with significant cholestasis might ultimately develop progressive liver fibrosis or cirrhosis requiring liver transplant in childhood. Only a small subset of children with minimal cholestasis progress to end-stage liver disease in late childhood or early adulthood.20 Therefore, liver transplant for progressive liver disease from significant cholestasis almost always occurs in childhood, usually between ages 1 and 4.21

In a retrospective study comparing posttransplant outcomes in children with Alagille syndrome and biliary atresia, 1-year patient survival was excellent overall in children with Alagille syndrome, although slightly lower than in children with biliary atresia, most likely owing to extrahepatic morbidities of Alagille syndrome and especially the use of immunosuppression in those with renal disease.21 Similarly, 1- and 5-year patient and graft survival outcomes of liver transplant in adults with Alagille syndrome were also excellent compared with those who received a liver transplant in childhood for Alagille syndrome or in adulthood for biliary atresia.22

Hepatocellular carcinoma has occurred in these patients in the absence of cirrhosis, which makes implementation of prognostic and surveillance strategies almost impossible to design for them. Annual ultrasonography with alpha-fetoprotein testing might be applicable in Alagille syndrome patients. However, deciding which patients should undergo this testing and when it should start will be challenging, given the paucity of data.

Cardiovascular disease. Cardiac phenotype is also mostly established in childhood, with the pulmonary vasculature being most commonly involved.19 In contrast, renal and other vascular abnormalities can manifest in adulthood. Renal manifestations vary and include structural anomalies such as hyperechoic kidneys or renal cysts, which can manifest in childhood, and some abnormalities such as hypertension and renal artery stenosis that can manifest in adulthood.23,24

Vasculopathy is reported to involve the intracranial, renal, and intra-abdominal blood vessels.25 Neurovascular accidents such as stroke and intracranial hemorrhage can occur at any age, with significant rates of morbidity and death.26 Therefore, some experts recommend magnetic resonance angiography every 5 years and before any major intervention to prevent these devastating complications.20

Pregnancy. Successful pregnancies have been reported. Preexisting cardiac and hepatic disease can complicate pregnancy depending on the severity of the disease. Because of the autosomal-dominant pattern of inheritance, infants have a 50% risk of the disease, so genetic counseling should be seriously considered before conception.27 Prenatal diagnosis is possible, but the lack of genotype-phenotype correlation precludes its use in clinical practice.

PROGRESSIVE FAMILIAL INTRAHEPATIC CHOLESTASIS

Progressive familial intrahepatic cholestasis (PFIC) is a heterogeneous group of autosomal-recessive conditions associated with disruption of bile formation causing cholestatic liver disease in infants and young children. Three types have been described, depending on the genetic mutation in the hepatobiliary transport pathway:

  • PFIC 1 (Byler disease) is caused by impaired bile salt secretion due to mutations in the ATP8B1 gene encoding for the familial intrahepatic cholestasis 1 (FIC 1) protein
  • PFIC 2 is caused by impaired bile salt secretion due to mutations in the ABCB11 gene encoding for the bile salt export pump (BSEP) protein
  • PFIC 3 is caused by impaired biliary phospholipid secretion due to a defect in ABCB4 encoding for multidrug resistance 3 (MDR3) protein.28

PFIC 1 and 2 manifest with low gamma-glutamyl transferase (GGT) cholestasis, whereas PFIC 3 presents with high GGT cholestasis.

PFIC 1 and PFIC 2 usually cause cholestasis in early infancy, but PFIC 3 can cause cholestasis in late infancy, childhood, and even adulthood.

Because ATP8B1 is expressed in other tissues, PFIC 1 is characterized by extrahepatic manifestations such as sensorineural hearing loss, growth failure, severe diarrhea, and pancreatic insufficiency.

 

 

Implications of PFIC in adulthood

PFIC 1 and 2 (low-GGT cholestasis) are usually progressive and often lead to end-stage liver disease and cirrhosis before adulthood. Therefore, almost all patients with PFIC 1 and 2 undergo liver transplant or at least a biliary diversion procedure before reaching adulthood. Intractable pruritus is one of the most challenging clinical manifestations in patients with PFIC.

First-line management is pharmacologic and includes ursodeoxycholic acid, antihistamines (eg, hydroxyzine), bile acid sequestrants (eg, cholestyramine, colestipol), naltrexone, and rifampin, but these have limited efficacy.10

Most patients, especially those with PFIC 1 and 2, undergo a biliary diversion procedure such as partial external biliary diversion (cholecystojejunocutaneostomy), ileal exclusion, or partial internal biliary diversion (cholecystojejunocolic anastomosis) to decrease enterohepatic circulation of bile salts. The efficacy of these procedures is very limited in patients with established cirrhosis. Excessive losses of bile can occur through the biliary stoma, leading to dehydration in patients with external biliary diversion. In patients who are not candidates for biliary diversion, endoscopic nasobiliary drainage of pancreatobiliary secretions could be achieved by placing a catheter in the common bile duct; this has been reported to be effective in relieving cholestasis in a few cases.29

Liver transplant is needed in patients with progressive liver disease and intractable pruritus despite medical management and biliary diversion. Unlike in biliary atresia, liver transplant is not curative in PFIC 1, due to extrahepatic manifestations: patients with PFIC 1 can still have intractable diarrhea and pancreatitis after liver transplant. More importantly, allograft steatohepatitis with further progression to cirrhosis can occur after liver transplant in patients with PFIC 1. Interestingly, biliary diversion has been reported to improve graft steatosis and diarrhea after liver transplant.30

Although graft survival after transplant is good in patients with PFIC 2, recurrence of low-GGT cholestasis has been reported and is believed to be due to the formation of anti-bile salt export pump (anti-BSEP) antibodies by the host immune system in response to exposure to new proteins from the transplant graft.31

Cancer. The risk of malignancy, especially hepatocellular carcinoma, is also increased in PFIC 2, affecting nearly 15% of patients. Therefore, standard hepatocellular carcinoma surveillance with ultrasonography or alpha-fetoprotein testing or both is recommended in patients with PFIC 2. Cholangiocarcinoma and pancreatic adenocarcinoma have also been reported in patients with PFIC 2.20

Incomplete penetrance of mutations in ATP8B1 and ABCB11 can cause recurrent episodes of cholestasis and pruritus with asymptomatic periods between episodes, referred to as benign recurrent intrahepatic cholestasis. Prognosis is usually good, with no progression to cirrhosis.32

Pregnancy. In contrast to FIC 1 and BSEP deficiency, MDR3 defects lead to a wide phenotypic spectrum depending on the type of mutation. Heterozygous mutation is associated with increased risk of development of cholestasis during pregnancy, which typically presents with generalized pruritus in the third trimester and is associated with adverse fetal outcomes. Intrahepatic cholestasis of pregnancy is usually treated with ursodeoxycholic acid, with reported improvement in pruritus, liver function, and pregnancy outcomes.33

In adults, drug-induced liver injury and idiopathic cirrhosis have also been described with MDR3 defects. Intrahepatic lithiasis and cholesterol gallstones can also occur with MDR3 defects as a result of impaired secretion of biliary phospholipid.32 Despite intrahepatic cholestasis of pregnancy, successful outcomes have been reported in women with PFIC.20

OTHER CHILDHOOD-ONSET INHERITED CHOLESTATIC DISEASES

Cystic fibrosis-associated liver disease

Nearly 40% of patients with cystic fibrosis develop liver disease.34 Cystic fibrosis-associated liver disease encompasses a broad clinical spectrum including asymptomatic elevation of aminotransferases, neonatal cholestasis, hepatic steatosis, focal biliary cirrhosis, and multilobar cirrhosis. Cirrhosis and portal hypertension can occur in 5% to 10% of patients and is the third-leading cause of death in patients with cystic fibrosis.35

Risk factors for cystic fibrosis-associated liver disease include male sex, meconium ileus, and severe CFTR gene mutation (class I–III) with pancreatic insufficiency. Cystic fibrosis-related cirrhosis is more frequent in children and adolescents, whereas noncirrhotic portal hypertension and intrahepatic cholangiopathies are more common in adults.36

Limited available studies support treatment with ursodeoxycholic acid in patients with cholestasis to delay the progression of liver disease, but the impact of this drug on long-term outcome is unknown.29

Most patients remain in compensated cirrhosis for many years before progressing to decompensated cirrhosis requiring liver transplant. Other indications for liver transplant include recurrent intractable variceal bleeding, hepatopulmonary syndrome, and portopulmonary hypertension. Combined liver and lung transplant may be considered in patients with advanced liver and lung disease. Outcomes after isolated liver or liver-lung transplant in cystic fibrosis patients have been comparable to those in patients with other liver diseases.37

Defects in bile acid synthesis

Inherited defects of enzymes required for the synthesis of primary bile acids from cholesterol can cause cholestasis from impaired bile flow and production of hepatotoxic aberrant bile acids. The clinical presentation varies depending on the enzymatic defect and can range from liver disease of varying severity to neurologic manifestations. Idiopathic late-onset cholestasis and cirrhosis of unknown etiology have been reported in adults with bile acid synthesis defects.38,39 Therefore, this diagnosis should be considered in cases of cryptogenic cirrhosis and other cholestatic features.

Treatment with primary bile acids (cholic acid) has been effective in most patients with defective bile acid synthesis.

Primary sclerosing cholangitis

Primary sclerosing cholangitis is characterized by progressive obliteration of intrahepatic and extrahepatic bile ducts and is most commonly seen in patients with inflammatory bowel disease. Sclerosing cholangitis can also be secondary to other diseases in children such as immunodeficiency syndromes, Langerhans cell histiocytosis, cystic fibrosis, or sickle cell anemia.40 Neonatal sclerosing cholangitis is a rare autosomal-recessive disease characterized by a severe form of cholangiopathy in neonates and young infants requiring transplant. It can be associated with Kabuki syndrome and neonatal ichthyosis-sclerosing cholangitis syndrome.

Treatment options are limited. Ursodeoxycholic acid and oral vancomycin have variable efficacy. Liver transplant is needed in patients with decompensated cirrhosis. Patients with primary sclerosing cholangitis, especially adults, are at higher risk of developing cholangiocarcinoma, and therefore screening with ultrasonography or magnetic resonance imaging every 6 to 12 months is recommended.

The risk of preterm and cesarean deliveries may be elevated in women with primary sclerosing cholangitis, though data are limited.33

 

 

PEDIATRIC LIVER TRANSPLANT RECIPIENTS WHO SURVIVE INTO ADULTHOOD

Adolescent rebellion poses risks

Outcomes of liver transplant in children and adolescents have improved tremendously in the past 2 decades with advances in surgical techniques, pre- and postoperative management, organ preservation, and immunosuppression. Now, most pediatric liver transplant recipients survive into adulthood, creating a unique challenge for internists and adult  care hepatologists.41

In rebellious adolescents and young adults, risk-taking behavior, nonadherence to immunosuppressive medications, alcohol intake, and substance abuse increase the risk of graft rejection and loss. Current immunosuppressive drugs such as calcineurin inhibitors (tacrolimus, cyclosporine), mycophenolate mofetil, sirolimus, and corticosteroids have drastically decreased rejection rates in compliant patients.41 Educating patients on the importance of taking their medications and avoiding alcohol and drug abuse is especially important for adolescents and young adults, as rates of nonadherence are high in these age groups.

Although pregnancy is usually successful after liver transplant, it should be considered high-risk due to reported complications such as graft rejection, diabetes, preeclampsia, sepsis, prematurity, and low birth weight. Conception should be avoided for at least 1 year after transplant.42 Appropriate counseling with regard to pregnancy and contraception is important.

There is no consensus on breastfeeding, but it is considered safe in women on low-dose calcineurin inhibitors.43

Life is better with a new liver, but patients have special needs

Liver transplant is life-saving and improves quality of life. However, long-term pediatric liver transplant recipients face challenges such as strict adherence to medications and follow-up visits, avoiding exposure to infections, and fear of graft rejection.

Chronic liver disease in children leads to failure to thrive, growth failure, and even delayed puberty, which resolve in most patients after liver transplant before adulthood in the absence of other comorbidities.44 However, these patients are reported to have lower psychosocial functioning and more psychiatric disorders such as anxiety or posttraumatic disorder.41,44

Therefore, a psychologist or other mental health professional should be part of the management team from the time of pretransplant assessment to identify mental health problems and the need for adjustments before liver transplant. Ongoing psychosocial assessment after liver transplant is equally important to identify risks such as drug or alcohol abuse, depression, posttraumatic stress disorder, and medication nonadherence, all of which can negatively affect posttransplant outcome.45

In addition, assessment of family functioning and structure is important for good long-term outcomes posttransplant; therefore, a social worker should also be a part of the transplant team. Psyschosocial assessment tools can identify high-risk candidates who would benefit from earlier intervention to avoid any negative impact posttransplant.

Neurocognitive development can be delayed in children with chronic liver disease, and the delay may persist even after liver transplant, with reported impairments in intellectual ability, language, verbal, and visuospatial functioning skills.41 In spite of this, a recent study found that more than half the study patients were employed at a median follow-up of 24 years from liver transplant and a median age of 27.46

Remarkably, pediatric liver transplant recipients have reported quality of life comparable to that in the general population,47 and even better than in patients with other chronic illnesses.48

Long-term medical comorbidities in pediatric liver transplant recipients

Favorable outcomes such as long-term survival and good quality of life in pediatric liver transplant recipients are lessened by late complications such as portal vein thrombosis or biliary strictures needing interventions, chronic graft rejection, adverse effects of immunosuppression, and recurrence of the disease.

Split-liver transplant—splitting a deceased-donor allograft to provide grafts for 2 recipients—has revolutionized liver transplant by increasing the donor pool and thereby decreasing waitlist mortality rates, especially in pediatric candidates. Despite this advantage, split-liver transplant is technically challenging and associated with increased perioperative complications compared with whole-liver transplant, especially in adult recipients. Recently, experienced centers have reported favorable outcomes with split-liver transplant comparable to those with whole-liver transplant; therefore, split-liver transplant should be considered after careful evaluation of donor organ and recipient clinical status.49

Old age in the recipient can also adversely affect liver transplant outcomes.50

Interestingly, even in patients whose clinical course is unremarkable and biochemical values are normal, graft hepatitis or fibrosis of unknown cause with progression to cirrhosis has been described in the decade after transplant.41

Chronic rejection with eventual graft loss may be related to nonadherence in adolescents and can be reduced with use of an additional immunosuppressant such as sirolimus or mycophenolate. Chronic kidney disease can occur in about one-third of liver transplant recipients secondary to renal disease associated with primary disease (like Alagille syndrome), hepatorenal syndrome, and most importantly, use of calcineurin inhibitors.45

Components of the metabolic syndrome such as type 2 diabetes, obesity, nonalcoholic fatty liver disease, hypertension, and dyslipidemia are also seen in long-term pediatric liver transplant survivors. Internists are advised to screen for these comorbidities so that interventions can be applied early to improve long-term health outcomes and graft survival.

Of importance, multiple studies have shown a 2-fold increase in the rates of de novo malignancy in liver transplant recipients, including solid-organ and lymphoproliferative cancers, probably due to long-term immunosuppression. Posttransplant lymphoproliferative disorder occurs at lower rates than with other solid-organ transplants; its incidence is greatest in pediatric patients and in the first 12 to 18 months after transplant.51

 

 

TRANSITION TO ADULT CARE

While the number of patients with childhood-onset liver disease and pediatric liver transplant recipients who survive into adulthood is increasing, there are no established guidelines or formal models for transitioning these patients into adult care. Consequently, studies on transitional process have examined various issues such as patient and parent frustration, poor medical knowledge among patients during transition, lack of parental facilitation, and inadequate knowledge on disease process among adult-care hepatologists.52–54

selvakumar_pediatriccholestaticliverdisease_f2.jpg
%3Cp%3EFigure%202.%20Transitional%20model%20in%20childhood-onset%20cholestatic%20diseases.%3C%2Fp%3E
A position paper on transitional care from Europe has put forth important strategies.55 A multidisciplinary team including a pediatric provider, nurse practitioner, transplant coordinator, psychologist, social worker, and adult provider is vital for a successful transition. Key requirements for an adolescent patient include knowledge about the disease, demonstration of responsibility toward disease management, and self-management of healthcare needs such as scheduling appointments or insurance needs. This demanding task could be streamlined if pediatric liver transplant programs could adopt a formal transitional model (Figure 2). The transition could start as early as age 10 to 12, with gradual transfer of health management responsibilities from parents to adolescents from age 13 to 17.56

A prolonged period of transition up to age 25 is preferred in complicated cases. Distinctive consideration for transition should include those with neurocognitive developmental delay from underlying disease or hepatic encephalopathy before transplant. These patients need additional support and time to achieve independence in health management before transition.57 Validated questionnaires are available to assess readiness to transition into adult care,58 implying that the decision to transition should not be based solely on age. 

References
  1. Kelly DA, Bucuvalas JC, Alonso EM, et al; American Association for the Study of Liver Diseases; American Society of Transplantation. Long-term medical management of the pediatric patient after liver transplantation: 2013 practice guideline by the American Association for the Study of Liver Diseases and the American Society of Transplantation. Liver Transpl 2013; 19(8):798–825. doi:10.1002/lt.23697
  2. Rosen DS, Blum RW, Britto M, Sawyer SM, Siegel DM; Society for Adolescent Medicine. Transition to adult health care for adolescents and young adults with chronic conditions: position paper of the Society for Adolescent Medicine. J Adolesc Health 2003; 33(4):309–311. pmid:14519573
  3. Fawaz R, Baumann U, Ekong U, et al. Guideline for the evaluation of cholestatic jaundice in infants: joint recommendations of the North American Society for Pediatric Gastroenterology, Hepatology, and Nutrition and the European Society for Pediatric Gastroenterology, Hepatology, and Nutrition. J Pediatr Gastroenterol Nutr 2017; 64(1):154–168. doi:10.1097/MPG.0000000000001334
  4. Vajro P, Ferrante L, Lenta S, Mandato C, Persico M. Management of adults with paediatric-onset chronic liver disease: strategic issues for transition care. Dig Liver Dis 2014; 46(4):295–301. doi:10.1016/j.dld.2013.10.018
  5. Davenport M, Tizzard SA, Underhill J, Mieli-Vergani G, Portmann B, Hadzic N. The biliary atresia splenic malformation syndrome: a 28-year single-center retrospective study. J Pediatr 2006; 149(3):393–400. doi:10.1016/j.jpeds.2006.05.030
  6. Balistreri WF, Bezerra JA. Whatever happened to “neonatal hepatitis?” Clin Liver Dis 2006; 10(1):27–53. doi:10.1016/j.cld.2005.10.008
  7. Serinet MO, Wildhaber BE, Broué P, et al. Impact of age at Kasai operation on its results in late childhood and adolescence: a rational basis for biliary atresia screening. Pediatrics 2009; 123(5):1280–1286. doi:10.1542/peds.2008-1949
  8. de Vries W, Homan-Van der Veen J, Hulscher JB, Hoekstra-Weebers JE, Houwen RH, Verkade HJ; Netherlands Study Group of Biliary Atresia Registry. Twenty-year transplant-free survival rate among patients with biliary atresia. Clin Gastroenterol Hepatol 2011; 9(12):1086–1091. doi:10.1016/j.cgh.2011.07.024
  9. Lykavieris P, Chardot C, Sokhn M, Gauthier F, Valayer J, Bernard O. Outcome in adulthood of biliary atresia: a study of 63 patients who survived for over 20 years with their native liver. Hepatology 2005; 41(2):366–371. doi:10.1002/hep.20547
  10. Joshi D, Gupta N, Samyn M, Deheragoda M, Dobbels F, Heneghan MA. The management of childhood liver diseases in adulthood. J Hepatol 2017; 66(3):631–644. doi:10.1016/j.jhep.2016.11.013
  11. Shneider BL, Abel B, Haber B, et al; Childhood Liver Disease Research and Education Network. Portal hypertension in children and young adults with biliary atresia. J Pediatr Gastroenterol Nutr 2012; 55(5):567–573. doi:10.1097/MPG.0b013e31826eb0cf
  12. Garcia-Tsao G, Abraldes JG, Berzigotti A, Bosch J. Portal hypertensive bleeding in cirrhosis: risk stratification, diagnosis, and management: 2016 practice guidance by the American Association for the Study of Liver Diseases. Hepatology 2017; 65(1):310–335. doi:10.1002/hep.28906
  13. Shneider BL, Brown MB, Haber B, et al; Biliary Atresia Research Consortium. A multicenter study of the outcome of biliary atresia in the United States, 1997 to 2000. J Pediatr 2006; 148(4):467–474. doi:10.1016/j.jpeds.2005.12.054
  14. Hung PY, Chen CC, Chen WJ, et al. Long-term prognosis of patients with biliary atresia: a 25 year summary. J Pediatr Gastroenterol Nutr 2006; 42(2):190–195. doi:10.1097/01.mpg.0000189339.92891.64
  15. Verkade HJ, Bezerra JA, Davenport M, et al. Biliary atresia and other cholestatic childhood diseases: advances and future challenges. J Hepatol 2016; 65(3):631–642. doi:10.1016/j.jhep.2016.04.032
  16. Hadžic N, Quaglia A, Portmann B, et al. Hepatocellular carcinoma in biliary atresia: King’s College Hospital experience. J Pediatr 2011; 159(4):617–622.e1. doi:10.1016/j.jpeds.2011.03.004
  17. Sokol RJ, Shepherd RW, Superina R, Bezerra JA, Robuck P, Hoofnagle JH. Screening and outcomes in biliary atresia: summary of a National Institutes of Health workshop. Hepatology 2007; 46(2):566–581. doi:10.1002/hep.21790
  18. Li L, Krantz ID, Deng Y, et al. Alagille syndrome is caused by mutations in human Jagged1, which encodes a ligand for Notch1. Nat Genet 1997; 16(3):243–251. doi:10.1038/ng0797-243
  19. Saleh M, Kamath BM, Chitayat D. Alagille syndrome: clinical perspectives. Appl Clin Genet 2016; 9:75–82. doi:10.2147/TACG.S86420
  20. Bass LM, Kamath BM. Inherited disorders of cholestasis in adulthood. Clinical Liver Disease 2013; 2(5):200–203. doi:10.1002/cld.245
  21. Kamath BM, Yin W, Miller H, Anand R, Rand EB, Alonso E, Bucuvalas J; Studies of Pediatric Liver Transplantation. Outcomes of liver transplantation for patients with Alagille syndrome: the studies of pediatric liver transplantation experience. Liver Transpl 2012; 18(8):940–948. doi:10.1002/lt.23437
  22. Arnon R, Annunziato R, Schiano T, et al. Orthotopic liver transplantation for adults with Alagille syndrome. Clin Transplant 2012; 26(2):E94–E100. doi:10.1111/j.1399-0012.2011.01574.x
  23. Salem JE, Bruguiere E, Iserin L, Guiochon-Mantel A, Plouin PF. Hypertension and aortorenal disease in Alagille syndrome. J Hypertens 2012; 30(7):1300–1306. doi:10.1097/HJH.0b013e3283531e1f
  24. Kamath BM, Podkameni G, Hutchinson AL, et al. Renal anomalies in Alagille syndrome: a disease-defining feature. Am J Med Genet A 2012; 158A(1):85–89. doi:10.1002/ajmg.a.34369
  25. Kamath BM, Bason L, Piccoli DA, Krantz ID, Spinner NB. Consequences of JAG1 mutations. J Med Genet 2003; 40(12):891–895. pmid:14684686
  26. Emerick KM, Krantz ID, Kamath BM, et al. Intracranial vascular abnormalities in patients with Alagille syndrome. J Pediatr Gastroenterol Nutr 2005; 41(1):99–107. pmid:15990638
  27. Ferrarese A, Senzolo M, Burra P. Successful pregnancy in Alagille syndrome. Dig Liver Dis 2015; 47(1):86–87. doi:10.1016/j.dld.2014.08.047
  28. Davit-Spraul A, Fabre M, Branchereau S, et al. ATP8B1 and ABCB11 analysis in 62 children with normal gamma-glutamyl transferase progressive familial intrahepatic cholestasis (PFIC): phenotypic differences between PFIC1 and PFIC2 and natural history. Hepatology 2010; 51(5):1645–1655. doi:10.1002/hep.23539
  29. Zellos A, Lykopoulou L, Polydorou A, et al. Nasobiliary drainage in an episode of intrahepatic cholestasis in a child with mild ABCB11 disease. J Pediatr Gastroenterol Nutr 2012; 55(1):88–90. doi:10.1097/MPG.0b013e31822f2bda
  30. Alrabadi LS, Morotti RA, Valentino PL, Rodriguez-Davalos MI, Ekong UD, Emre SH. Biliary drainage as treatment for allograft steatosis following liver transplantation for PFIC-1 disease: a single-center experience. Pediatr Transplant 2018; 22(4):e13184. doi:10.1111/petr.13184
  31. Kubitz R, Dröge C, Kluge S, et al. Autoimmune BSEP disease: disease recurrence after liver transplantation for progressive familial intrahepatic cholestasis. Clin Rev Allergy Immunol 2015; 48(2–3):273–284. doi:10.1007/s12016-014-8457-4
  32. Jacquemin E. Progressive familial intrahepatic cholestasis. Clin Res Hepatol Gastroenterol 2012; 36(suppl 1):S26–S35. doi:10.1016/S2210-7401(12)70018-9
  33. Pataia V, Dixon PH, Williamson C. Pregnancy and bile acid disorders. Am J Physiol Gastrointest Liver Physiol 2017; 313(1):G1–G6. doi:10.1152/ajpgi.00028.2017
  34. Lamireau T, Monnereau S, Martin S, Marcotte JE, Winnock M, Alvarez F. Epidemiology of liver disease in cystic fibrosis: a longitudinal study. J Hepatol 2004; 41(6):920–925. doi:10.1016/j.jhep.2004.08.006
  35. Bolia R, Ooi CY, Lewindon P, et al. Practical approach to the gastrointestinal manifestations of cystic fibrosis. J Paediatr Child Health 2018; 54(6):609–619. doi:10.1111/jpc.13921
  36. Debray D, Kelly D, Houwen R, Strandvik B, Colombo C. Best practice guidance for the diagnosis and management of cystic fibrosis-associated liver disease. J Cyst Fibros 2011; 10(suppl 2):S29–S36. doi:10.1016/S1569-1993(11)60006-4
  37. Fridell JA, Bond GJ, Mazariegos G V, et al. Liver transplantation in children with cystic fibrosis: a long-term longitudinal review of a single center’s experience. J Pediatr Surg 2003; 38(8):1152–1156. pmid:12891484
  38. Fischler B, Bodin K, Stjernman H, et al. Cholestatic liver disease in adults may be due to an inherited defect in bile acid biosynthesis. J Intern Med 2007; 262(2):254–262. doi:10.1111/j.1365-2796.2007.01814.x
  39. Molho-Pessach V, Rios JJ, Xing C, Setchell KD, Cohen JC, Hobbs HH. Homozygosity mapping identifies a bile acid biosynthetic defect in an adult with cirrhosis of unknown etiology. Hepatology 2012; 55(4):1139–1145. doi:10.1002/hep.24781
  40. Mieli-Vergani G, Vergani D. Sclerosing cholangitis in children and adolescents. Clin Liver Dis 2016; 20(1):99–111. doi:10.1016/j.cld.2015.08.008
  41. Kelly D, Wray J. The adolescent liver transplant patient. Clin Liver Dis 2014; 18(3):613–632. doi:10.1016/j.cld.2014.05.006
  42. Westbrook RH, Yeoman AD, Agarwal K, et al. Outcomes of pregnancy following liver transplantation: the King’s College Hospital experience. Liver Transpl. 2015; 21(9):1153–1159. doi:10.1002/lt.24182
  43. Hammoud GM, Almashhrawi AA, Ahmed KT, Rahman R, Ibdah JA. Liver diseases in pregnancy: liver transplantation in pregnancy. World J Gastroenterol 2013; 19(43):7647–7651. doi:10.3748/wjg.v19.i43.7647
  44. Codoner-Franch P, Bernard O, Alvarez F. Long-term follow-up of growth in height after successful liver transplantation. J Pediatr 1994; 124(3):368–373. pmid:8120704
  45. Shemesh E. Assessment and management of psychosocial challenges in pediatric liver transplantation. Liver Transpl 2008; 14(9):1229–1236. doi:10.1002/lt.21582
  46. Martinelli J, Habes D, Majed L, et al. Long-term outcome of liver transplantation in childhood: a study of 20-year survivors. Am J Transplant 2018; 18(7):1680–1689. doi:10.1111/ajt.14626
  47. Roblin E, Audhuy F, Boillot O, Rivet C, Lachaux A. Long-term quality of life after pediatric liver transplantation. Arch Pediatr 2012; 19(10):1039–1052. French. doi:10.1016/j.arcped.2012.06.020
  48. Duffy JP, Kao K, Ko CY, et al. Long-term patient outcome and quality of life after liver transplantation: analysis of 20-year survivors. Ann Surg 2010; 252(4):652–661. doi:10.1097/SLA.0b013e3181f5f23a
  49. Hackl C, Schmidt KM, Süsal C, Döhler B, Zidek M, Schlitt HJ. Split liver transplantation: Current developments. World J Gastroenterol 2018; 24(47):5312–5321. doi:10.3748/wjg.v24.i47.5312
  50. Durand F, Levitsky J, Cauchy F, Gilgenkrantz H, Soubrane O, Francoz C. Age and liver transplantation. J Hepatol 2019; 70(4):745–758. doi:10.1016/j.jhep.2018.12.009
  51. Chandok N, Watt KD. Burden of de novo malignancy in the liver transplant recipient. Liver Transpl 2012; 18(11):1277–1289. doi:10.1002/lt.23531
  52. Ferrarese A, Germani G, Lazzaro S, et al. Short-term outcomes of paediatric liver transplant recipients after transition to Adult Healthcare Service. Liver Int 2018; 38(7):1316–1321. doi:10.1111/liv.13655
  53. Wright J, Elwell L, McDonagh JE, Kelly DA, Wray J. “Are these adult doctors gonna know me?” Experiences of transition for young people with a liver transplant. Pediatr Transplant 2016; 20(7):912–920. doi:10.1111/petr.12777
  54. Heldman MR, Sohn MW, Gordon EJ, et al. National survey of adult transplant hepatologists on the pediatric-to-adult care transition after liver transplantation. Liver Transpl 2015; 21(2):213–223. doi:10.1002/lt.24044
  55. Vajro P, Fischler B, Burra P, et al. The health care transition of youth with liver disease into the adult health system. J Pediatr Gastroenterol Nutr 2018; 66(6):976–990. doi:10.1097/MPG.0000000000001965
  56. Fredericks EM, Lopez MJ. Transition of the adolescent transplant patient to adult care. Clin Liver Dis (Hoboken) 2013; 2(5):223–226. doi:10.1002/cld.243
  57. Kaufman M. Transition of cognitively delayed adolescent organ transplant recipients to adult care. Pediatr Transplant 2006; 10(4):413–417. doi:10.1111/j.1399-3046.2006.00491.x
  58. Sawicki GS, Lukens-Bull K, Yin X, et al. Measuring the transition readiness of youth with special healthcare needs: validation of the TRAQ—Transition Readiness Assessment Questionnaire. J Pediatr Psychol 2011; 36(2):160–171. doi:10.1093/jpepsy/jsp128
References
  1. Kelly DA, Bucuvalas JC, Alonso EM, et al; American Association for the Study of Liver Diseases; American Society of Transplantation. Long-term medical management of the pediatric patient after liver transplantation: 2013 practice guideline by the American Association for the Study of Liver Diseases and the American Society of Transplantation. Liver Transpl 2013; 19(8):798–825. doi:10.1002/lt.23697
  2. Rosen DS, Blum RW, Britto M, Sawyer SM, Siegel DM; Society for Adolescent Medicine. Transition to adult health care for adolescents and young adults with chronic conditions: position paper of the Society for Adolescent Medicine. J Adolesc Health 2003; 33(4):309–311. pmid:14519573
  3. Fawaz R, Baumann U, Ekong U, et al. Guideline for the evaluation of cholestatic jaundice in infants: joint recommendations of the North American Society for Pediatric Gastroenterology, Hepatology, and Nutrition and the European Society for Pediatric Gastroenterology, Hepatology, and Nutrition. J Pediatr Gastroenterol Nutr 2017; 64(1):154–168. doi:10.1097/MPG.0000000000001334
  4. Vajro P, Ferrante L, Lenta S, Mandato C, Persico M. Management of adults with paediatric-onset chronic liver disease: strategic issues for transition care. Dig Liver Dis 2014; 46(4):295–301. doi:10.1016/j.dld.2013.10.018
  5. Davenport M, Tizzard SA, Underhill J, Mieli-Vergani G, Portmann B, Hadzic N. The biliary atresia splenic malformation syndrome: a 28-year single-center retrospective study. J Pediatr 2006; 149(3):393–400. doi:10.1016/j.jpeds.2006.05.030
  6. Balistreri WF, Bezerra JA. Whatever happened to “neonatal hepatitis?” Clin Liver Dis 2006; 10(1):27–53. doi:10.1016/j.cld.2005.10.008
  7. Serinet MO, Wildhaber BE, Broué P, et al. Impact of age at Kasai operation on its results in late childhood and adolescence: a rational basis for biliary atresia screening. Pediatrics 2009; 123(5):1280–1286. doi:10.1542/peds.2008-1949
  8. de Vries W, Homan-Van der Veen J, Hulscher JB, Hoekstra-Weebers JE, Houwen RH, Verkade HJ; Netherlands Study Group of Biliary Atresia Registry. Twenty-year transplant-free survival rate among patients with biliary atresia. Clin Gastroenterol Hepatol 2011; 9(12):1086–1091. doi:10.1016/j.cgh.2011.07.024
  9. Lykavieris P, Chardot C, Sokhn M, Gauthier F, Valayer J, Bernard O. Outcome in adulthood of biliary atresia: a study of 63 patients who survived for over 20 years with their native liver. Hepatology 2005; 41(2):366–371. doi:10.1002/hep.20547
  10. Joshi D, Gupta N, Samyn M, Deheragoda M, Dobbels F, Heneghan MA. The management of childhood liver diseases in adulthood. J Hepatol 2017; 66(3):631–644. doi:10.1016/j.jhep.2016.11.013
  11. Shneider BL, Abel B, Haber B, et al; Childhood Liver Disease Research and Education Network. Portal hypertension in children and young adults with biliary atresia. J Pediatr Gastroenterol Nutr 2012; 55(5):567–573. doi:10.1097/MPG.0b013e31826eb0cf
  12. Garcia-Tsao G, Abraldes JG, Berzigotti A, Bosch J. Portal hypertensive bleeding in cirrhosis: risk stratification, diagnosis, and management: 2016 practice guidance by the American Association for the Study of Liver Diseases. Hepatology 2017; 65(1):310–335. doi:10.1002/hep.28906
  13. Shneider BL, Brown MB, Haber B, et al; Biliary Atresia Research Consortium. A multicenter study of the outcome of biliary atresia in the United States, 1997 to 2000. J Pediatr 2006; 148(4):467–474. doi:10.1016/j.jpeds.2005.12.054
  14. Hung PY, Chen CC, Chen WJ, et al. Long-term prognosis of patients with biliary atresia: a 25 year summary. J Pediatr Gastroenterol Nutr 2006; 42(2):190–195. doi:10.1097/01.mpg.0000189339.92891.64
  15. Verkade HJ, Bezerra JA, Davenport M, et al. Biliary atresia and other cholestatic childhood diseases: advances and future challenges. J Hepatol 2016; 65(3):631–642. doi:10.1016/j.jhep.2016.04.032
  16. Hadžic N, Quaglia A, Portmann B, et al. Hepatocellular carcinoma in biliary atresia: King’s College Hospital experience. J Pediatr 2011; 159(4):617–622.e1. doi:10.1016/j.jpeds.2011.03.004
  17. Sokol RJ, Shepherd RW, Superina R, Bezerra JA, Robuck P, Hoofnagle JH. Screening and outcomes in biliary atresia: summary of a National Institutes of Health workshop. Hepatology 2007; 46(2):566–581. doi:10.1002/hep.21790
  18. Li L, Krantz ID, Deng Y, et al. Alagille syndrome is caused by mutations in human Jagged1, which encodes a ligand for Notch1. Nat Genet 1997; 16(3):243–251. doi:10.1038/ng0797-243
  19. Saleh M, Kamath BM, Chitayat D. Alagille syndrome: clinical perspectives. Appl Clin Genet 2016; 9:75–82. doi:10.2147/TACG.S86420
  20. Bass LM, Kamath BM. Inherited disorders of cholestasis in adulthood. Clinical Liver Disease 2013; 2(5):200–203. doi:10.1002/cld.245
  21. Kamath BM, Yin W, Miller H, Anand R, Rand EB, Alonso E, Bucuvalas J; Studies of Pediatric Liver Transplantation. Outcomes of liver transplantation for patients with Alagille syndrome: the studies of pediatric liver transplantation experience. Liver Transpl 2012; 18(8):940–948. doi:10.1002/lt.23437
  22. Arnon R, Annunziato R, Schiano T, et al. Orthotopic liver transplantation for adults with Alagille syndrome. Clin Transplant 2012; 26(2):E94–E100. doi:10.1111/j.1399-0012.2011.01574.x
  23. Salem JE, Bruguiere E, Iserin L, Guiochon-Mantel A, Plouin PF. Hypertension and aortorenal disease in Alagille syndrome. J Hypertens 2012; 30(7):1300–1306. doi:10.1097/HJH.0b013e3283531e1f
  24. Kamath BM, Podkameni G, Hutchinson AL, et al. Renal anomalies in Alagille syndrome: a disease-defining feature. Am J Med Genet A 2012; 158A(1):85–89. doi:10.1002/ajmg.a.34369
  25. Kamath BM, Bason L, Piccoli DA, Krantz ID, Spinner NB. Consequences of JAG1 mutations. J Med Genet 2003; 40(12):891–895. pmid:14684686
  26. Emerick KM, Krantz ID, Kamath BM, et al. Intracranial vascular abnormalities in patients with Alagille syndrome. J Pediatr Gastroenterol Nutr 2005; 41(1):99–107. pmid:15990638
  27. Ferrarese A, Senzolo M, Burra P. Successful pregnancy in Alagille syndrome. Dig Liver Dis 2015; 47(1):86–87. doi:10.1016/j.dld.2014.08.047
  28. Davit-Spraul A, Fabre M, Branchereau S, et al. ATP8B1 and ABCB11 analysis in 62 children with normal gamma-glutamyl transferase progressive familial intrahepatic cholestasis (PFIC): phenotypic differences between PFIC1 and PFIC2 and natural history. Hepatology 2010; 51(5):1645–1655. doi:10.1002/hep.23539
  29. Zellos A, Lykopoulou L, Polydorou A, et al. Nasobiliary drainage in an episode of intrahepatic cholestasis in a child with mild ABCB11 disease. J Pediatr Gastroenterol Nutr 2012; 55(1):88–90. doi:10.1097/MPG.0b013e31822f2bda
  30. Alrabadi LS, Morotti RA, Valentino PL, Rodriguez-Davalos MI, Ekong UD, Emre SH. Biliary drainage as treatment for allograft steatosis following liver transplantation for PFIC-1 disease: a single-center experience. Pediatr Transplant 2018; 22(4):e13184. doi:10.1111/petr.13184
  31. Kubitz R, Dröge C, Kluge S, et al. Autoimmune BSEP disease: disease recurrence after liver transplantation for progressive familial intrahepatic cholestasis. Clin Rev Allergy Immunol 2015; 48(2–3):273–284. doi:10.1007/s12016-014-8457-4
  32. Jacquemin E. Progressive familial intrahepatic cholestasis. Clin Res Hepatol Gastroenterol 2012; 36(suppl 1):S26–S35. doi:10.1016/S2210-7401(12)70018-9
  33. Pataia V, Dixon PH, Williamson C. Pregnancy and bile acid disorders. Am J Physiol Gastrointest Liver Physiol 2017; 313(1):G1–G6. doi:10.1152/ajpgi.00028.2017
  34. Lamireau T, Monnereau S, Martin S, Marcotte JE, Winnock M, Alvarez F. Epidemiology of liver disease in cystic fibrosis: a longitudinal study. J Hepatol 2004; 41(6):920–925. doi:10.1016/j.jhep.2004.08.006
  35. Bolia R, Ooi CY, Lewindon P, et al. Practical approach to the gastrointestinal manifestations of cystic fibrosis. J Paediatr Child Health 2018; 54(6):609–619. doi:10.1111/jpc.13921
  36. Debray D, Kelly D, Houwen R, Strandvik B, Colombo C. Best practice guidance for the diagnosis and management of cystic fibrosis-associated liver disease. J Cyst Fibros 2011; 10(suppl 2):S29–S36. doi:10.1016/S1569-1993(11)60006-4
  37. Fridell JA, Bond GJ, Mazariegos G V, et al. Liver transplantation in children with cystic fibrosis: a long-term longitudinal review of a single center’s experience. J Pediatr Surg 2003; 38(8):1152–1156. pmid:12891484
  38. Fischler B, Bodin K, Stjernman H, et al. Cholestatic liver disease in adults may be due to an inherited defect in bile acid biosynthesis. J Intern Med 2007; 262(2):254–262. doi:10.1111/j.1365-2796.2007.01814.x
  39. Molho-Pessach V, Rios JJ, Xing C, Setchell KD, Cohen JC, Hobbs HH. Homozygosity mapping identifies a bile acid biosynthetic defect in an adult with cirrhosis of unknown etiology. Hepatology 2012; 55(4):1139–1145. doi:10.1002/hep.24781
  40. Mieli-Vergani G, Vergani D. Sclerosing cholangitis in children and adolescents. Clin Liver Dis 2016; 20(1):99–111. doi:10.1016/j.cld.2015.08.008
  41. Kelly D, Wray J. The adolescent liver transplant patient. Clin Liver Dis 2014; 18(3):613–632. doi:10.1016/j.cld.2014.05.006
  42. Westbrook RH, Yeoman AD, Agarwal K, et al. Outcomes of pregnancy following liver transplantation: the King’s College Hospital experience. Liver Transpl. 2015; 21(9):1153–1159. doi:10.1002/lt.24182
  43. Hammoud GM, Almashhrawi AA, Ahmed KT, Rahman R, Ibdah JA. Liver diseases in pregnancy: liver transplantation in pregnancy. World J Gastroenterol 2013; 19(43):7647–7651. doi:10.3748/wjg.v19.i43.7647
  44. Codoner-Franch P, Bernard O, Alvarez F. Long-term follow-up of growth in height after successful liver transplantation. J Pediatr 1994; 124(3):368–373. pmid:8120704
  45. Shemesh E. Assessment and management of psychosocial challenges in pediatric liver transplantation. Liver Transpl 2008; 14(9):1229–1236. doi:10.1002/lt.21582
  46. Martinelli J, Habes D, Majed L, et al. Long-term outcome of liver transplantation in childhood: a study of 20-year survivors. Am J Transplant 2018; 18(7):1680–1689. doi:10.1111/ajt.14626
  47. Roblin E, Audhuy F, Boillot O, Rivet C, Lachaux A. Long-term quality of life after pediatric liver transplantation. Arch Pediatr 2012; 19(10):1039–1052. French. doi:10.1016/j.arcped.2012.06.020
  48. Duffy JP, Kao K, Ko CY, et al. Long-term patient outcome and quality of life after liver transplantation: analysis of 20-year survivors. Ann Surg 2010; 252(4):652–661. doi:10.1097/SLA.0b013e3181f5f23a
  49. Hackl C, Schmidt KM, Süsal C, Döhler B, Zidek M, Schlitt HJ. Split liver transplantation: Current developments. World J Gastroenterol 2018; 24(47):5312–5321. doi:10.3748/wjg.v24.i47.5312
  50. Durand F, Levitsky J, Cauchy F, Gilgenkrantz H, Soubrane O, Francoz C. Age and liver transplantation. J Hepatol 2019; 70(4):745–758. doi:10.1016/j.jhep.2018.12.009
  51. Chandok N, Watt KD. Burden of de novo malignancy in the liver transplant recipient. Liver Transpl 2012; 18(11):1277–1289. doi:10.1002/lt.23531
  52. Ferrarese A, Germani G, Lazzaro S, et al. Short-term outcomes of paediatric liver transplant recipients after transition to Adult Healthcare Service. Liver Int 2018; 38(7):1316–1321. doi:10.1111/liv.13655
  53. Wright J, Elwell L, McDonagh JE, Kelly DA, Wray J. “Are these adult doctors gonna know me?” Experiences of transition for young people with a liver transplant. Pediatr Transplant 2016; 20(7):912–920. doi:10.1111/petr.12777
  54. Heldman MR, Sohn MW, Gordon EJ, et al. National survey of adult transplant hepatologists on the pediatric-to-adult care transition after liver transplantation. Liver Transpl 2015; 21(2):213–223. doi:10.1002/lt.24044
  55. Vajro P, Fischler B, Burra P, et al. The health care transition of youth with liver disease into the adult health system. J Pediatr Gastroenterol Nutr 2018; 66(6):976–990. doi:10.1097/MPG.0000000000001965
  56. Fredericks EM, Lopez MJ. Transition of the adolescent transplant patient to adult care. Clin Liver Dis (Hoboken) 2013; 2(5):223–226. doi:10.1002/cld.243
  57. Kaufman M. Transition of cognitively delayed adolescent organ transplant recipients to adult care. Pediatr Transplant 2006; 10(4):413–417. doi:10.1111/j.1399-3046.2006.00491.x
  58. Sawicki GS, Lukens-Bull K, Yin X, et al. Measuring the transition readiness of youth with special healthcare needs: validation of the TRAQ—Transition Readiness Assessment Questionnaire. J Pediatr Psychol 2011; 36(2):160–171. doi:10.1093/jpepsy/jsp128
Issue
Cleveland Clinic Journal of Medicine - 86(7)
Issue
Cleveland Clinic Journal of Medicine - 86(7)
Page Number
454-464
Page Number
454-464
Publications
Publications
Topics
Article Type
Display Headline
Pediatric cholestatic liver disease: Successful transition of care
Display Headline
Pediatric cholestatic liver disease: Successful transition of care
Legacy Keywords
pediatric cholestatic liver disease, liver, cholestasis, children, biliary atresia, transition of care, Alagille syndrome, progressive familial intrahepatic cholestasis, PFIC, primary sclerosing cholangitis, bile acid synthesis defect, extrahepatic biliary atresia, cystic fibrosis, CF, MDR3, Praveen Kumar Conjeevaram Selvakumar, Vera Hupertz, Naveen Mittal, Kris Kowdley, Naim Alkhouri
Legacy Keywords
pediatric cholestatic liver disease, liver, cholestasis, children, biliary atresia, transition of care, Alagille syndrome, progressive familial intrahepatic cholestasis, PFIC, primary sclerosing cholangitis, bile acid synthesis defect, extrahepatic biliary atresia, cystic fibrosis, CF, MDR3, Praveen Kumar Conjeevaram Selvakumar, Vera Hupertz, Naveen Mittal, Kris Kowdley, Naim Alkhouri
Sections
Inside the Article

KEY POINTS

  • The causes of cholestasis in children are different from those in adults, with genetic inherited causes more common in childhood.
  • Cholestasis in children can be caused by biliary tract obstruction such as in biliary atresia or defects in forming and excreting bile acids and other components of bile.
  • With the growing number of people with childhood-onset liver disease surviving into adulthood, it is important for internists to be aware of unique problems and challenges in continuing management of this population.
  • In addition to medical comorbidities, these patients may also have impaired psychosocial functioning and quality of life.
Disallow All Ads
Content Gating
No Gating (article Unlocked/Free)
Alternative CME
Disqus Comments
Default
Gate On Date
Tue, 06/25/2019 - 07:30
Un-Gate On Date
Tue, 06/25/2019 - 07:30
Use ProPublica
CFC Schedule Remove Status
Tue, 06/25/2019 - 07:30
Hide sidebar & use full width
render the right sidebar.
Article PDF Media

Laboratory tests in rheumatology: A rational approach

Article Type
Changed
Wed, 03/13/2019 - 11:27
Display Headline
Laboratory tests in rheumatology: A rational approach

Laboratory tests are often ordered inappropriately for patients in whom a rheumatologic illness is suspected; this occurs in both primary and secondary care.1 Some tests are available both singly and as part of a battery of tests screening healthy people without symptoms.

The problem: negative test results are by no means always reassuring, and false-positive results raise the risks of unnecessary anxiety for patients and clinicians, needless referrals, and potential morbidity due to further unnecessary testing and exposure to wrong treatments.2 Clinicians should be aware of the pitfalls of these tests in order to choose them wisely and interpret the results correctly.

This article provides practical guidance on requesting and interpreting some common tests in rheumatology, with the aid of case vignettes.

RHEUMATOID FACTOR AND ANTICITRULLINATED PEPTIDE ANTIBODY

A 41-year-old woman, previously in good health, presents to her primary care practitioner with a 6-week history of pain and swelling in her hands and early morning stiffness lasting about 2 hours. She denies having any extraarticular symptoms. Physical examination reveals synovitis across her right metacarpophalangeal joints, proximal interphalangeal joint of the left middle finger, and left wrist. The primary care physician is concerned that her symptoms might be due to rheumatoid arthritis.

Would testing for rheumatoid factor and anticitrullinated peptide antibody be useful in this patient?

Rheumatoid factor is an antibody (immunoglobulin M, IgG, or IgA) targeted against the Fc fragment of IgG.3 It was so named because it was originally detected in patients with rheumatoid arthritis, but it is neither sensitive nor specific for this condition. A meta-analysis of more than 5,000 patients with rheumatoid arthritis reported that rheumatoid factor testing had a sensitivity of 69% and specificity of 85%.4

suresh_rheumatologictests_t1.jpg
Numerous other conditions can be associated with a positive test for rheumatoid factor (Table 1). Hence, a diagnosis of rheumatoid arthritis cannot be confirmed with a positive result alone, nor can it be excluded with a negative result.

Anticitrullinated peptide antibody, on the other hand, is much more specific for rheumatoid arthritis (95%), as it is seldom seen in other conditions, but its sensitivity is similar to that of rheumatoid factor (68%).4–6 A positive result would thus lend strength to the diagnosis of rheumatoid arthritis, but a negative result would not exclude it.

Approach to early arthritis

When faced with a patient with early arthritis, some key questions to ask include7,8:

Is this an inflammatory or a mechanical problem? Inflammatory arthritis is suggested by joint swelling that is not due to trauma or bony hypertrophy, early morning stiffness lasting longer than 30 minutes, and elevated inflammatory markers (erythrocyte sedimentation rate or C-reactive protein). Involvement of the small joints of the hands and feet may be suggested by pain on compression of the metacarpophalangeal and metatarsophalangeal joints, respectively.

Is there a definite identifiable underlying cause for the inflammatory arthritis? The pattern of development of joint symptoms or the presence of extraarticular symptoms may suggest an underlying problem such as gout, psoriatic arthritis, systemic lupus erythematosus, or sarcoidosis.

If the arthritis is undifferentiated (ie, there is no definite identifiable cause), is it likely to remit or persist? This is perhaps the most important question to ask in order to prognosticate. Patients with risk factors for persistent disease, ie, for development of rheumatoid arthritis, should be referred to a rheumatologist early for timely institution of disease-modifying antirheumatic drug therapy.9 Multiple studies have shown that patients in whom this therapy is started early have much better clinical, functional, and radiologic outcomes than those in whom it is delayed.10–12

The revised American College of Rheumatology and European League Against Rheumatism criteria13 include the following factors as predictors of persistence:

  • Number of involved joints (with greater weight given to involvement of small joints)
  • Duration of symptoms 6 weeks or longer
  • Elevated acute-phase response (erythrocyte sedimentation rate or C-reactive protein level)
  • A positive serologic test (either rheumatoid factor or anticitrullinated peptide antibody).

If both rheumatoid factor and anticitrullinated peptide antibody are positive in a patient with early undifferentiated arthritis, the risk of progression to rheumatoid arthritis is almost 100%, thus underscoring the importance of testing for these antibodies.5,6 Referral to a rheumatologist should, however, not be delayed in patients with negative test results (more than one-third of patients with rheumatoid arthritis may be negative for both), and should be considered in those with inflammatory joint symptoms persisting longer than 6 weeks, especially with involvement of the small joints (sparing the distal interphalangeals) and elevated acute-phase response.

Rheumatoid factor in healthy people without symptoms

In some countries, testing for rheumatoid factor is offered as part of a battery of screening tests in healthy people who have no symptoms, a practice that should be strongly discouraged.

Multiple studies, both prospective and retrospective, have demonstrated that both rheumatoid factor and anticitrullinated peptide antibody may be present several years before the clinical diagnosis of rheumatoid arthritis.6,14–16 But the risk of developing rheumatoid arthritis for asymptomatic individuals who are rheumatoid factor-positive depends on the rheumatoid factor titer, positive family history of rheumatoid arthritis in first-degree relatives, and copresence of anticitrullinated peptide antibody. The absolute risk, nevertheless, is still very small. In some, there might be an alternative explanation such as undiagnosed Sjögren syndrome or hepatitis C.

In any event, no strategy is currently available that is proven to prevent the development of rheumatoid arthritis, and there is no role for disease-modifying therapy during the preclinical phase.16

Back to our patient

Blood testing in our patient reveals normal complete blood cell counts, aminotransferase levels, and serum creatinine concentration; findings on urinalysis are normal. Her erythrocyte sedimentation rate is 56 mm/hour (reference range 0–15), and her C-reactive protein level is 26 mg/dL (normal < 3). Testing is negative for rheumatoid factor and anticitrullinated peptide antibody.

Although her rheumatoid factor and anticitrullinated peptide antibody tests are negative, she is referred to a rheumatologist because she has predictors of persistent disease, ie, symptom duration of 6 weeks, involvement of the small joints of the hands, and elevated erythrocyte sedimentation rate and C-reactive protein. The rheumatologist checks her parvovirus serology, which is negative.

The patient is given parenteral depot corticosteroid therapy, to which she responds briefly. Because her symptoms persist and continue to worsen, methotrexate treatment is started after an additional 6 weeks.

 

 

ANTINUCLEAR ANTIBODY

A 37-year-old woman presents to her primary care physician with the complaint of tiredness. She has a family history of systemic lupus erythematosus in her sister and maternal aunt. She is understandably worried about lupus because of the family history and is asking to be tested for it.

Would testing for antinuclear antibody be reasonable?

Antinuclear antibody is not a single antibody but rather a family of autoantibodies that are directed against nuclear constituents such as single- or double-stranded deoxyribonucleic acid (dsDNA), histones, centromeres, proteins complexed with ribonucleic acid (RNA), and enzymes such as topoisomerase.17,18

Protein antigens complexed with RNA and some enzymes in the nucleus are also known as extractable nuclear antigens (ENAs). They include Ro, La, Sm, Jo-1, RNP, and ScL-70 and are named after the patient in whom they were first discovered (Robert, Lavine, Smith, and John), the antigen that is targeted (ribonucleoprotein or RNP), and the disease with which they are associated (anti-ScL-70 or antitopoisomerase in diffuse cutaneous scleroderma).

Antinuclear antibody testing is commonly requested to exclude connective tissue diseases such as lupus, but the clinician needs to be aware of the following points:

Antinuclear antibody may be encountered in conditions other than lupus

These include19:

  • Other autoimmune diseases such as rheumatoid arthritis, primary Sjögren syndrome, systemic sclerosis, autoimmune thyroid disease, and myasthenia gravis
  • Infection with organisms that share the epitope with self-antigens (molecular mimicry)
  • Cancers
  • Drugs such as hydralazine, procainamide, and minocycline.

Antinuclear antibody might also be produced by the healthy immune system from time to time to clear the nuclear debris that is extruded from aging cells.

A study in healthy individuals20 reported a prevalence of positive antinuclear antibody of 32% at a titer of 1/40, 15% at a titer of 1/80, 7% at a titer of 1/160, and 3% at a titer of 1/320. Importantly, a positive result was more common among family members of patients with autoimmune connective tissue diseases.21 Hence, a positive antinuclear antibody result does not always mean lupus.

Antinuclear antibody testing is highly sensitive for lupus

With current laboratory methods, antinuclear antibody testing has a sensitivity close to 100%. Hence, a negative result virtually rules out lupus.

Two methods are commonly used to test for antinuclear antibody: indirect immunofluorescence and enzyme-linked immunosorbent assay (ELISA).22 While human epithelial (Hep2) cells are used as the source of antigen in immunofluorescence, purified nuclear antigens coated on multiple-well plates are used in ELISA.

Although ELISA is simpler to perform, immunofluorescence has a slightly better sensitivity (because the Hep2 cells express a wide range of antigens) and is still considered the gold standard. As expected, the higher sensitivity occurs at the cost of reduced specificity (about 60%), so antinuclear antibody will also be detected in all the other conditions listed above.23

To improve the specificity of antinuclear antibody testing, laboratories report titers (the highest dilution of the test serum that tested positive); a cutoff of greater than 1/80 is generally considered significant.

Do not order antinuclear antibody testing indiscriminately

suresh_rheumatologictests_t2.jpg
If the antinuclear antibody test is requested indiscriminately, the positive predictive value for the diagnosis of lupus is only 11%.24 The test should be requested only when the pretest probability of lupus or other connective tissue disease is high. The positive predictive value is much higher in patients presenting with clinical or laboratory manifestations involving 2 or more organ systems (Table 2).18,25

suresh_rheumatologictests_t3.jpg
Categorization of the specific antigen target improves disease specificity. The antinuclear antibody in patients with lupus may be targeted against single- or double-stranded DNA, histones, or 1 or more of the ENAs. Among these, the presence of anti-dsDNA or anti-Sm is highly specific for a diagnosis of lupus (close to 100%). Neither is sensitive for lupus, however, with anti-dsDNA present in only 60% of patients with lupus and anti-Sm in about 30%.17 Hence, patients with a positive antinuclear antibody and negative anti-dsDNA and anti-Sm may continue to pose a diagnostic challenge. Other examples of specific disease associations are listed in Table 3.

To sum up, the antinuclear antibody test should be requested only in patients with involvement of multiple organ systems. Although a negative result would make it extremely unlikely that the clinical presentation is due to lupus, a positive result is insufficient on its own to make a diagnosis of lupus.

Diagnosing lupus is straightforward when patients present with a specific manifestation such as inflammatory arthritis, photosensitive skin rash, hemolytic anemia, thrombocytopenia, or nephritis, or with specific antibodies such as those against dsDNA or Sm. Patients who present with nonspecific symptoms such as arthralgia or tiredness with a positive antinuclear antibody and negative anti-dsDNA and anti-Sm may present difficulties even for the specialist.25–27

Back to our patient

Our patient denies arthralgia. She has no extraarticular symptoms such as skin rashes, oral ulcers, sicca symptoms, muscle weakness, Raynaud phenomenon, pleuritic chest pain, or breathlessness. Findings on physical examination and urinalysis are unremarkable.

Her primary care physician decides to check her complete blood cell count, erythrocyte sedimentation rate, and thyroid-stimulating hormone level. Although she is reassured that her tiredness is not due to lupus, she insists on getting an antinuclear antibody test.

Her complete blood cell counts are normal. Her erythrocyte sedimentation rate is 6 mm/hour. However, her thyroid-stimulating hormone level is elevated, and subsequent testing shows low free thyroxine and positive thyroid peroxidase antibodies. The antinuclear antibody is positive in a titer of 1/80 and negative for anti-dsDNA and anti-ENA.

We explain to her that the positive antinuclear antibody is most likely related to her autoimmune thyroid disease. She is referred to an endocrinologist.

 

 

ANTIPHOSPHOLIPID ANTIBODIES

A 24-year-old woman presents to the emergency department with acute unprovoked deep vein thrombosis in her right leg, confirmed by ultrasonography. She has no history of previous thrombosis, and the relevant family history is unremarkable. She has never been pregnant. Her platelet count is 84 × 109/L (reference range 150–400), and her baseline activated partial thromboplastin time is prolonged at 62 seconds (reference range 23.0–32.4). The rest of her blood counts and her prothrombin time, liver enzyme levels, and serum creatinine level are normal.

Should this patient be tested for antiphospholipid antibodies?

Antiphospholipid antibodies are important because of their association with thrombotic risk (both venous and arterial) and pregnancy morbidity. The name is a misnomer, as these antibodies are targeted against some proteins that are bound to phospholipids and not only to the phospholipids themselves.

According to the modified Sapporo criteria for the classification of antiphospholipid syndrome,28 antiphospholipid antibodies should remain persistently positive on at least 2 separate occasions at least 12 weeks apart for the result to be considered significant because some infections and drugs may be associated with the transient presence of antiphospholipid antibodies.

Screening for antiphospholipid antibodies should include testing for IgM and IgG anticardiolipin antibodies, lupus anticoagulant, and IgM and IgG beta-2 glycoprotein I antibodies.29,30

Anticardiolipin antibodies

Anticardiolipin (aCL) antibodies may be targeted either against beta-2 glycoprotein I (beta-2GPI) that is bound to cardiolipin (a phospholipid) or against cardiolipin alone; the former is more specific. Antibodies directed against cardiolipin alone are usually transient and are associated with infections and drugs. The result is considered significant only when anticardiolipin antibodies are present in a medium to high titer (> 40 IgG phospholipid units or IgM phospholipid units, or > 99th percentile).

Lupus anticoagulant

The antibody with “lupus anticoagulant activity” is targeted against prothrombin plus phospholipid or beta-2GPI plus phospholipid. The test for it is a functional assay involving 3 steps:

Demonstrating the prolongation of a phospholipid-dependent coagulation assay like the activated partial thromboplastin time (aPTT). (This may explain the prolongation of aPTT in the patient described in the vignette.) Although the presence of lupus anticoagulant is associated with thrombosis, it is called an “anticoagulant” because of this in vitro prolongation of phospholipid-dependent coagulation assays.

Mixing study. The phospholipid-dependent coagulation assay could be prolonged because of either the deficiency of a coagulation factor or the presence of the antiphospholipid antibodies. This can be differentiated by mixing the patient’s plasma with normal plasma (which will have all the clotting factors) in a 1:1 ratio. If the coagulation assay remains prolonged after the addition of normal plasma, clotting factor deficiency can be excluded.

Addition of a phospholipid. If the prolongation of the coagulation assay is due to the presence of an antiphospholipid antibody, addition of extra phospholipid will correct this.

Beta-2 glycoprotein I antibody (anti-beta-2GPI)

The beta-2GPI that is not bound to the cardiolipin can be detected by separately testing for beta-2GPI (the anticardiolipin test only detects the beta-2GPI that is bound to the cardiolipin). The result is considered significant if beta-2GPI is present in a medium to high titer (> 99th percentile).

Studies have shown that antiphospholipid antibodies may be present in 1% to 5% of apparently healthy people in the general population.31 These are usually low-titer anticardiolipin or anti-beta-GPI IgM antibodies that are not associated with thrombosis or adverse pregnancy outcomes. Hence, the term antiphospholipid syndrome should be reserved for those who have had at least 1 episode of thrombosis or pregnancy morbidity and persistent antiphospholipid antibodies, and not those who have asymptomatic or transient antiphospholipid antibodies.

Triple positivity (positive anticardiolipin, lupus anticoagulant, and anti-beta-2GPI) seems to be associated with the highest risk of thrombosis, with a 10-year cumulative incidence of 37.1% (95% confidence interval [CI] 19.9–54.3) for a first thrombotic event,32 and 44.2% (95% CI 38.6–49.8) for recurrent thrombosis.33

The association with thrombosis is stronger for lupus anticoagulant than with the other 2 antibodies, with different studies34 finding an odds ratio ranging from 5 to 16. A positive lupus anticoagulant test with or without a moderate to high titer of anticardiolipin or anti-beta-2GPI IgM or IgG constitutes a high-risk profile, while a moderate to high titer of anticardiolipin or anti-beta-2GPI IgM or IgG constitutes a moderate-risk profile. A low titer of anticardiolipin or anti-beta-2GPI IgM or IgG constitutes a low-risk profile that may not be associated with thrombosis.35

Antiphospholipid syndrome is important to recognize because of the need for long-term anticoagulation to prevent recurrence.36 It may be primary, when it occurs on its own, or secondary, when it occurs in association with another autoimmune disease such as lupus.

Venous events in antiphospholipid syndrome most commonly manifest as lower-limb deep vein thrombosis or pulmonary embolism, while arterial events most commonly manifest as stroke or transient ischemic attack.37 Obstetric manifestations may include not only miscarriage and stillbirth, but also preterm delivery, intrauterine growth retardation, and preeclampsia, all occurring due to placental insufficiency.

The frequency of antiphospholipid antibodies has been estimated as 13.5% in patients with stroke, 11% with myocardial infarction, 9.5% with deep vein thrombosis, and 6% for those with pregnancy morbidity.38

Some noncriteria manifestations have also been recognized in antiphospholipid syndrome, such as thrombocytopenia, cardiac vegetations (Libman-Sachs endocarditis), livedo reticularis, and nephropathy.

suresh_rheumatologictests_t4.jpg
The indications for antiphospholipid antibody testing are listed in Table 4.29 For the patient described in the vignette, it would be appropriate to test for antiphospholipid antibodies because of her unprovoked thrombosis, thrombocytopenia, and prolonged aPTT. Anticoagulant treatment is known to be associated with false-positive lupus anticoagulant, so any blood samples should be drawn before such treatment is commenced.

Back to our patient

Our patient’s anticardiolipin IgG test is negative, while her lupus anticoagulant and beta-2GPI IgG are positive. She has no clinical or laboratory features suggesting lupus.

She is started on warfarin. After 3 months, the warfarin is interrupted for several days, and she is retested for all 3 antiphospholipid antibodies. Her beta-2GPI I IgG and lupus anticoagulant tests are again positive. Because of the persistent antiphospholipid antibody positivity and clinical history of deep vein thrombosis, her condition is diagnosed as primary antiphospholipid syndrome. She is advised to continue anticoagulant therapy indefinitely.

 

 

ANTINEUTROPHIL CYTOPLASMIC ANTIBODY

A 34-year-old man who is an injecting drug user presents with a 2-week history of fever, malaise, and generalized arthralgia. There are no localizing symptoms of infection. Notable findings on examination include a temperature of 38.0°C (100.4°F), needle track marks in his arms, nonblanching vasculitic rash in his legs, and a systolic murmur over the precordium.

His white blood cell count is 15.3 × 109/L (reference range 3.7–11.0), and his C-reactive protein level is 234 mg/dL (normal < 3). Otherwise, results of blood cell counts, liver enzyme tests, renal function tests, urinalysis, and chest radiography are normal.

Two sets of blood cultures are drawn. Transthoracic echocardiography and the antineutrophil cytoplasmic antibody (ANCA) test are requested, as are screening tests for human immunodeficiency virus, hepatitis B, and hepatitis C.

Was the ANCA test indicated in this patient?

ANCAs are autoantibodies against antigens located in the cytoplasmic granules of neutrophils and monocytes. They are associated with small-vessel vasculitides such as granulomatosis with polyangiitis (GPA), microscopic polyangiitis (MPA), eosinophilic granulomatosis with polyangiitis (EGPA), and isolated pauciimmune crescentic glomerulonephritis, all collectively known as ANCA-associated vasculitis (AAV).39

Laboratory methods to detect ANCA include indirect immunofluorescence and antigen-specific enzyme immunoassays. Indirect immunofluorescence only tells us whether or not an antibody that is targeting a cytoplasmic antigen is present. Based on the indirect immunofluorescent pattern, ANCA can be classified as follows:

  • Perinuclear or p-ANCA (if the targeted antigen is located just around the nucleus and extends into it)
  • Cytoplasmic or c-ANCA (if the targeted antigen is located farther away from the nucleus)
  • Atypical ANCA (if the indirect immunofluorescent pattern does not fit with either p-ANCA or c-ANCA).

Indirect immunofluorescence does not give information about the exact antigen that is targeted; this can only be obtained by performing 1 of the antigen-specific immunoassays. The target antigen for c-ANCA is usually proteinase-3 (PR3), while that for p-ANCA could be myeloperoxidase (MPO), cathepsin, lysozyme, lactoferrin, or bactericidal permeability inhibitor. Anti-PR3 is highly specific for GPA, while anti-MPO is usually associated with MPA and EGPA. Less commonly, anti-PR3 may be seen in patients with MPA and anti-MPO in those with GPA. Hence, there is an increasing trend toward classifying ANCA-associated vasculitis into PR3-associated or MPO-associated vasculitis rather than as GPA, MPA, EGPA, or renal-limited vasculitis.40

Several audits have shown that the ANCA test is widely misused and requested indiscriminately to rule out vasculitis. This results in a lower positive predictive value, possible harm to patients due to increased false-positive rates, and increased burden on the laboratory.41–43 At least 2 separate groups have demonstrated that a gating policy that refuses ANCA testing in patients without clinical evidence of systemic vasculitis can reduce the number of inappropriate requests, improve the diagnostic yield, and make it more clinically relevant and cost-effective.44,45

The clinician should bear in mind that:

suresh_rheumatologictests_t5.jpg
ANCA testing should be requested only if the pretest probability of ANCA-associated vasculitis is high. The indications proposed by the International Consensus Statement on ANCA testing46 are listed in Table 5. These criteria have been clinically validated, with 1 study even demonstrating that no cases of ANCA-associated vasculitis would be missed if these guidelines are followed.47

Current guidelines recommend using one of the antigen-specific assays for PR3 and MPO as the primary screening method.48 Until recently, indirect immunofluorescence was used to screen for ANCA-associated vasculitis, and positive results were confirmed by ELISA to detect ANCAs specific for PR3 and MPO,49 but this is no longer recommended because of recent evidence suggesting a large variability between the different indirect immunofluorescent methods and improved diagnostic performance of the antigen-specific assays.

In a large multicenter study by Damoiseaux et al, the specificity with the different antigen-specific immunoassays was 98% to 99% for PR3-ANCA and 96% to 99% for MPO-ANCA.50

ANCA-associated vasculitis should not be considered excluded if the PR3 and MPO-ANCA are negative. In the Damoiseaux study, about 11% to 15% of patients with GPA and 8% to 24% of patients with MPA tested negative for both PR3 and MPO-ANCA.50

If the ANCA result is negative and clinical suspicion for ANCA-associated vasculitis is high, the clinician may wish to consider requesting another immunoassay method or indirect immunofluorescence. Results of indirect immunofluorescent testing results may be positive in those with a negative immunoassay, and vice versa.

suresh_rheumatologictests_t6.jpg
A positive ANCA result is not diagnostic of ANCA-associated vasculitis. Numerous other conditions are associated with ANCA, usually p-ANCA or atypical ANCA (Table 6). The antigens targeted by these ANCAs are usually cathepsin, lysozyme, lactoferrin, and bactericidal permeability inhibitor.

Thus, the ANCA result should always be interpreted in the context of the whole clinical picture.51 Biopsy should still be considered the gold standard for the diagnosis of ANCA-associated vasculitis. The ANCA titer can help to improve clinical interpretation, because the likelihood of ANCA-associated vasculitis increases with higher levels of PR3 and MPO-ANCA.52

Back to our patient

Our patient’s blood cultures grow methicillin-sensitive Staphylococcus aureus in both sets after 48 hours. Transthoracic echocardiography reveals vegetations around the tricuspid valve, with no evidence of valvular regurgitation. The diagnosis is right-sided infective endocarditis. He is started on appropriate antibiotics.

Tests for human immunodeficiency virus, hepatitis B, and hepatitis C are negative. The ANCA test is positive for MPO-ANCA at 28 IU/mL (normal < 10).

The positive ANCA is thought to be related to the infective endocarditis. His vasculitis is most likely secondary to infective endocarditis and not ANCA-associated vasculitis. The ANCA test need not have been requested in the first place.

 

 

HUMAN LEUKOCYTE ANTIGEN-B27

A 22-year-old man presents to his primary care physician with a 4-month history of gradually worsening low back pain associated with early morning stiffness lasting more than 2 hours. He has no peripheral joint symptoms.

In the last 2 years, he has had 2 separate episodes of uveitis. There is a family history of ankylosing spondylitis in his father. Examination reveals global restriction of lumbar movements but is otherwise unremarkable. Magnetic resonance imaging (MRI) of the lumbar spine and sacroiliac joints is normal.

Should this patient be tested for human leukocyte antigen-B27 (HLA-B27)?

The major histocompatibility complex (MHC) is a gene complex that is present in all animals. It encodes proteins that help with immunologic tolerance. HLA simply refers to the human version of the MHC.53 The HLA gene complex, located on chromosome 6, is categorized into class I, class II, and class III. HLA-B is one of the 3 class I genes. Thus, a positive HLA-B27 result simply means that the particular gene is present in that person.

HLA-B27 is strongly associated with ankylosing spondylitis, also known as axial spondyloarthropathy.54 Other genes also contribute to the pathogenesis of ankylosing spondylitis, but HLA-B27 is present in more than 90% of patients with this disease and is by far considered the most important. The association is not as strong for peripheral spondyloarthropathy, with studies reporting a frequency of up to 75% for reactive arthritis and inflammatory bowel disease-associated arthritis, and up to 50% for psoriatic arthritis and uveitis.55

About 9% of healthy, asymptomatic individuals may have HLA-B27, so the mere presence of this gene is not evidence of disease.56 There may be up to a 20-fold increased risk of ankylosing spondylitis among those who are HLA-B27-positive.57

Some HLA genes have many different alleles, each of which is given a number (explaining the number 27 that follows the B). Closely related alleles that differ from one another by only a few amino-acid substitutions are then categorized together, thus accounting for more than 100 subtypes of HLA-B27 (designated from HLA-B*2701 to HLA-B*27106). These subtypes vary in frequency among different racial groups, and the population prevalence of ankylosing spondylitis parallels the frequency of HLA-B27.58 The most common subtype seen in white people and American Indians is B*2705. HLA-B27 is rare in blacks, explaining the rarity of ankylosing spondylitis in this population. Further examples include HLA-B*2704, which is seen in Asians, and HLA-B*2702, seen in Mediterranean populations. Not all subtypes of HLA-B27 are associated with disease, and some, like HLA-B*2706, may also be protective.

When should the clinician consider testing for HLA-B27?

suresh_rheumatologictests_t7.jpg
Not all patients with low back pain need an HLA-B27 test. First, it is important to look for clinical features of axial spondyloarthropathy (Table 7). The unifying feature of spondyloarthropathy is enthesitis (inflammation at the sites of insertion of tendons or ligaments on the skeleton). Inflammation of axial entheses causes spondylitis and sacroiliitis, manifesting as inflammatory back pain. Clinical clues to inflammatory back pain include insidious onset, aggravation with rest or inactivity, prolonged early morning stiffness, disturbed sleep during the second half of the night, relief with movement or activity, alternating gluteal pain (due to sacroiliitis), and good response to anti-inflammatory medication (although nonspecific).

Peripheral spondyloarthropathy may present with arthritis, enthesitis (eg, heel pain due to inflammation at the site of insertion of the Achilles tendon or plantar fascia), or dactylitis (“sausage” swelling of the whole finger or toe due to extension of inflammation beyond the margins of the joint). Other clues may include psoriasis, inflammatory bowel disease, history of preceding gastrointestinal or genitourinary infection, family history of similar conditions, and history of recurrent uveitis.

For the initial assessment of patients who have inflammatory back pain, plain radiography of the sacroiliac joints is considered the gold standard.59 If plain radiography does not show evidence of sacroiliitis, MRI of the sacroiliac joints should be considered. While plain radiography can reveal only structural changes such as sclerosis, erosions, and ankylosis, MRI is useful to evaluate for early inflammatory changes such as bone marrow edema. Imaging the lumbar spine is not necessary, as the sacroiliac joints are almost invariably involved in axial spondyloarthropathy, and lesions seldom occur in the lumbar spine in isolation.60

The diagnosis of ankylosing spondylitis previously relied on confirmatory imaging features, but based on the new International Society classification criteria,61–63 which can be applied to patients with more than 3 months of back pain and age of onset of symptoms before age 45, patients can be classified as having 1 of the following:

  • Radiographic axial spondyloarthropathy, if they have evidence of sacroiliitis on imaging plus 1 other feature of spondyloarthropathy
  • Nonradiographic axial spondyloarthropathy, if they have a positive HLA-B27 plus 2 other features of spondyloarthropathy (Table 7).

These new criteria have a sensitivity of 82.9% and specificity of 84.4%.62,63 The disease burden of radiographic and nonradiographic axial spondyloarthropathy has been shown to be similar, suggesting that they are part of the same disease spectrum. Thus, the HLA-B27 test is useful to make a diagnosis of axial spondyloarthropathy even in the absence of imaging features and could be requested in patients with 2 or more features of spondyloarthropathy. In the absence of imaging features and a negative HLA-B27 result, however, the patient cannot be classified as having axial spondyloarthropathy.

Back to our patient

The absence of radiographic evidence would not exclude axial spondyloarthropathy in our patient. The HLA-B27 test is requested because of the inflammatory back pain and the presence of 2 spondyloarthropathy features (uveitis and the family history) and is reported to be positive. His disease is classified as nonradiographic axial spondyloarthropathy.

He is started on regular naproxen and is referred to a physiotherapist. After 1 month, he reports significant symptomatic improvement. He asks if he can be retested for HLA-B27 to see if it has become negative. We tell him that there is no point in repeating it, as it is a gene and will not disappear.

SUMMARY: CONSIDER THE CLINICAL PICTURE

When approaching a patient suspected of having a rheumatologic disease, a clinician should first consider the clinical presentation and the intended purpose of each test. The tests, in general, might serve several purposes. They might help to:

Increase the likelihood of the diagnosis in question. For example, a positive rheumatoid factor or anticitrullinated peptide antibody can help diagnose rheumatoid arthritis in a patient with early polyarthritis, a positive HLA-B27 can help diagnose ankylosing spondylitis in patients with inflammatory back pain and normal imaging, and a positive ANCA can help diagnose ANCA-associated vasculitis in a patient with glomerulonephritis.

Reduce the likelihood of the diagnosis in question. For example, a negative antinuclear antibody test reduces the likelihood of lupus in a patient with joint pains.

Monitor the condition. For example DNA antibodies can be used to monitor the activity of lupus.

Plan the treatment strategy. For example, one might consider lifelong anticoagulation if antiphospholipid antibodies are persistently positive in a patient with thrombosis.

Prognosticate. For example, positive rheumatoid factor and anticitrullinated peptide antibody increase the risk of erosive rheumatoid arthritis.

If the test was requested in the absence of a clear indication and the result is positive, it is important to bear in mind the potential pitfalls associated with that test and not attach a diagnostic label prematurely. None of the tests can confirm or exclude a condition, so the results should always be interpreted in the context of the whole clinical picture.   

References
  1. American College of Rheumatology Ad Hoc Committee on Immunologic Testing Guidelines. Guidelines for immunologic laboratory testing in the rheumatic diseases: an introduction. Arthritis Rheum 2002; 47(4):429–433. doi:10.1002/art.10381
  2. Rang M. The Ulysses syndrome. Can Med Assoc J 1972; 106(2):122–123. pmid:5058884
  3. Ingegnoli F, Castelli R, Gualtierotti R. Rheumatoid factors: clinical applications. Dis Markers 2013; 35(6):727–734. doi:10.1155/2013/726598
  4. Nishimura K, Sugiyama D, Kogata Y, et al. Meta-analysis: diagnostic accuracy of anti-cyclic citrullinated peptide antibody and rheumatoid factor for rheumatoid arthritis. Ann Intern Med 2007; 146(11):797–808. pmid:17548411
  5. Taylor P, Gartemann J, Hsieh J, Creeden J. A systematic review of serum biomarkers anti-cyclic citrullinated Peptide and rheumatoid factor as tests for rheumatoid arthritis. Autoimmune Dis 2011; 2011:815038. doi:10.4061/2011/815038
  6. Rantapää-Dahlqvist S, de Jong BA, Berglin E, et al. Antibodies against cyclic citrullinated peptide and IgA rheumatoid factor predict the development of rheumatoid arthritis. Arthritis Rheum 2003; 48(10):2741–2749. doi:10.1002/art.11223
  7. Suresh E. Diagnosis of early rheumatoid arthritis: what the non-specialist needs to know. J R Soc Med 2004; 97(9):421–424. doi:10.1258/jrsm.97.9.421
  8. Emery P, Breedveld FC, Dougados M, Kalden JR, Schiff MH, Smolen JS. Early referral recommendation for newly diagnosed rheumatoid arthritis: evidence based development of a clinical guide. Ann Rheum Dis 2002; 61(4):290–297. pmid:11874828
  9. Combe B, Landewe R, Daien CI, et al. 2016 update of the EULAR recommendations for the management of early arthritis. Ann Rheum Dis 2017; 76(6):948–959. doi:10.1136/annrheumdis-2016-210602
  10. Egsmose C, Lund B, Borg G, et al. Patients with rheumatoid arthritis benefit from early 2nd line therapy: 5 year follow up of a prospective double blind placebo controlled study. J Rheumatol 1995; 22(12):2208–2213. pmid:8835550
  11. van der Heide A, Jacobs JW, Bijlsma JW, et al. The effectiveness of early treatment with “second-line” antirheumatic drugs. A randomized, controlled trial. Ann Intern Med 1996; 124(8):699–707. pmid:8633829
  12. Andreson JJ, Wells G, Verhoeven AC, Felson DT. Factors predicting response to treatment in rheumatoid arthritis: the importance of disease duration. Arthritis Rheum 2000; 43(1):22–29. doi:10.1002/1529-0131(200001)43:1<22::AID-ANR4>3.0.CO;2-9
  13. Aletaha D, Neogi T, Silman AJ, et al. 2010 Rheumatoid arthritis classification criteria: an American College of Rheumatology/European League Against Rheumatism collaborative initiative. Arthritis Rheum 2010; 62(9):2569–2581. doi:10.1002/art.27584
  14. Nielen MM, van Schaardenburg D, Reesink HW, et al. Specific autoantibodies precede the symptoms of rheumatoid arthritis: a study of serial measurements in blood donors. Arthritis Rheum 2004; 50(2):380–386. doi:10.1002/art.20018
  15. del Puente A, Knowler WC, Pettitt DJ, Bennett PH. The incidence of rheumatoid arthritis is predicted by rheumatoid factor titer in a longitudinal population study. Arthritis Rheum 1988; 31(10):1239–1244. pmid:3178905
  16. Deane KD, Norris JM, Holers VM. Preclinical rheumatoid arthritis: identification, evaluation, and future directions for investigation. Rheum Dis Clin North Am 2010; 36(2):213–241. doi:10.1016/j.rdc.2010.02.001
  17. Kavanaugh A, Tomar R, Reveille J, Solomon DH, Homburger HA. Guidelines for clinical use of the antinuclear antibody test and tests for specific autoantibodies to nuclear antigens. American College of Pathologists. Arch Pathol Lab Med 2000; 124(1):71–81. doi:10.1043/0003-9985(2000)124<0071:GFCUOT>2.0.CO;2
  18. Suresh E. Systemic lupus erythematosus: diagnosis for the non-specialist. Br J Hosp Med (Lond) 2007; 68(10):538–541. doi:10.12968/hmed.2007.68.10.27324
  19. Illei GG, Klippel JH. Why is the ANA result positive? Bull Rheum Dis 1999; 48(1):1–4. pmid:10028188
  20. Tan EM, Feltkamp TE, Smolen JS, et al. Range of antinuclear antibodies in “healthy” individuals. Arthritis Rheum 1997; 40(9):1601–1611. doi:10.1002/art.1780400909
  21. Langkilde H, Voss A, Heegaard N, Laustrup H. Autoantibodies persist in relatives to systemic lupus erythematosus patients during 12 years follow-up. Lupus 2017; 26(7):723–728. doi:10.1177/0961203316676378
  22. Rondeel JM. Immunofluorescence versus ELISA for the detection of antinuclear antigens. Expert Rev Mol Diagn 2002; 2(3):226–232. doi:10.1586/14737159.2.3.226
  23. Solomon DH, Kavanaugh AJ, Schur PH; American College of Rheumatology Ad Hoc Committee on Immunologic Testing Guidelines. Evidence-based guidelines for the use of immunologic tests: antinuclear antibody testing. Arthritis Rheum 2002; 47(4):434–444. doi:10.1002/art.10561
  24. Slater CA, Davis RB, Shmerling RH. Antinuclear antibody testing. A study of clinical utility. Arch Intern Med 1996; 156(13):1421–1425. pmid:8678710
  25. Maddison PJ. Is it SLE? Best Pract Res Clin Rheumatol 2002; 16(2):167–180. doi:10.1053/berh.2001.0219
  26. Price E, Walker E. Diagnostic vertigo: the journey to diagnosis in systemic lupus erythematosus. Health (London) 2014; 18(3):223–239. doi:10.1177/1363459313488008
  27. Blumenthal DE. Tired, aching, ANA-positive: does your patient have lupus or fibromyalgia? Cleve Clin J Med 2002; 69(2):143–146, 151–152. pmid:11990644
  28. Miyakis S, Lockshin MD, Atsumi T, et al. International consensus statement on an update of the classification criteria for definite antiphospholipid syndrome (APS). J Thromb Haemost 2006; 4(2):295–306. doi:10.1111/j.1538-7836.2006.01753.x
  29. Keeling D, Mackie I, Moore GW, Greer IA, Greaves M; British Committee for Standards in Haematology. Guidelines on the investigation and management of antiphospholipid syndrome. Br J Haematol 2012; 157(1):47–58. doi:10.1111/j.1365-2141.2012.09037.x
  30. Giannakopoulos B, Passam F, Iannou Y, Krillis SA. How we diagnose the antiphospholipid syndrome. Blood 2009; 113(5):985–994. doi:10.1182/blood-2007-12-129627
  31. Biggioggero M, Meroni PL. The geoepidemiology of the antiphospholipid antibody syndrome. Autoimmun Rev 2010; 9(5):A299–A304. doi:10.1016/j.autrev.2009.11.013
  32. Pengo V, Ruffatti A, Legnani C, et al. Incidence of a first thromboembolic event in asymptomatic carriers of high-risk antiphospholipid antibody profile: a multicenter prospective study. Blood 2011; 118(17):4714–4718. doi:10.1182/blood-2011-03-340232
  33. Pengo V, Ruffatti A, Legnani C, et al. Clinical course of high-risk patients diagnosed with antiphospholipid syndrome. J Thromb Haemost 2010; 8(2):237–242. doi:10.1111/j.1538-7836.2009.03674.x
  34. Galli M, Luciani D, Bertolini G, Barbui T. Lupus anticoagulants are stronger risk factors for thrombosis than anticardiolipin antibodies in the antiphospholipid syndrome: a systematic review of the literature. Blood 2003; 101(5):1827–1832. doi:10.1182/blood-2002-02-0441
  35. Garcia D, Erkan D. Diagnosis and management of the antiphospholipid syndrome. N Engl J Med 2018; 378(21):2010–2021. doi:10.1056/NEJMra1705454
  36. Garcia D, Akl EA, Carr R, Kearon C. Antiphospholipid antibodies and the risk of recurrence after a first episode of venous thromboembolism: a systematic review. Blood 2013; 122(5):817–824. doi:10.1182/blood-2013-04-496257
  37. Cervera R. Lessons from the “Euro-Phospholipid” project. Autoimmun Rev 2008; 7(3):174–178. doi:10.1016/j.autrev.2007.11.011
  38. Andreoli L, Chighizola CB, Banzato A, Pons-Estel GJ, Ramire de Jesus G, Erkan D. Estimated frequency of antiphospholipid antibodies in patients with pregnancy morbidity, stroke, myocardial infarction, and deep vein thrombosis: a critical review of the literature. Arthritis Care Res (Hoboken) 2013; 65(11):1869–1873. doi:10.1002/acr.22066
  39. Miller A, Chan M, Wiik A, Misbah SA, Luqmani RA. An approach to the diagnosis and management of systemic vasculitis. Clin Exp Immunol 2010; 160(2):143–160. doi:10.1111/j.1365-2249.2009.04078.x
  40. Cornec D, Cornec-Le-Gall E, Fervenza FC, Specks U. ANCA-associated vasculitis—clinical utility of using ANCA specificity to classify patients. Nat Rev Rheumatol 2016; 12(10):570–579. doi:10.1038/nrrheum.2016.123
  41. Edgar JD, McMillan SA, Bruce IN, Conlan SK. An audit of ANCA in routine clinical practice. Postgrad Med J 1995; 71(840):605–612. pmid:8545289
  42. McLaren JS, Stimson RH, McRorie ER, Coia JE, Luqmani RA. The diagnostic value of anti-neutrophil cytoplasmic testing in a routine clinical setting. QJM 2001; 94(11):615–621. pmid:11704691
  43. Mandl LA, Solomon DH, Smith EL, Lew RA, Katz JN, Shmerling RH. Using antineutrophil cytoplasmic antibody testing to diagnose vasculitis: can test-ordering guidelines improve diagnostic accuracy? Arch Intern Med 2002; 162(13):1509–1514. pmid:12090888
  44. Sinclair D, Saas M, Stevens JM. The effect of a symptom related “gated policy” on ANCA requests in routine clinical practice. J Clin Pathol 2004; 57(2):131–134. pmid:14747434
  45. Arnold DF, Timms A, Luqmani R, Misbah SA. Does a gating policy for ANCA overlook patients with ANCA associated vasculitis? An audit of 263 patients. J Clin Pathol 2010; 63(8):678–680. doi:10.1136/jcp.2009.072504
  46. Savige J, Gills D, Benson E, et al. International consensus statement on testing and reporting of antineutrophil cytoplasmic antibodies (ANCA). Am J Clin Pathol 1999; 111(4):507–513. pmid:10191771
  47. Robinson PC, Steele RH. Appropriateness of antineutrophil cytoplasmic antibody testing in a tertiary hospital. J Clin Pathol 2009; 62(8):743–745. doi:10.1136/jcp.2009.064485
  48. Bossuyt X, Cohen Tervaert JW, Arimura Y, et al. Position paper: revised 2017 international consensus on testing of ANCAs in granulomatosis with polyangiitis and microscopic polyangiitis. Nat Rev Rheumatol 2017; 13(11):683–692. doi:10.1038/nrrheum.2017.140
  49. Hagen EC, Daha MR, Hermans J, et al. Diagnostic value of standardized assays for anti-neutrophil cytoplasmic antibodies in idiopathic systemic vasculitis. EC/BCR Project for ANCA Assay Standardization. Kidney Int 1998; 53(3):743–753. doi:10.1046/j.1523-1755.1998.00807.x
  50. Damoiseaux J, Csemok E, Rasmussen N, et al. Detection of antineutrophil antibodies (ANCAs): a multicentre European Vasculitis Study Group (EUVAS) evaluation of the value of indirect immunofluorescence (IIF) versus antigen specific immunoassays. Ann Rheum Dis 2017; 76(4):647–653. doi:10.1136/annrheumdis-2016-209507
  51. Suresh E. Diagnostic approach to patients with suspected vasculitis. Postgrad Med J 2006; 82(970):483–488. doi:10.1136/pgmj.2005.042648
  52. Vermeersch P, Blockmans D, Bossuyt X. Use of likelihood ratios can improve the clinical usefulness of enzyme immunoassays for the diagnosis of small-vessel vasculitis. Clin Chem 2009; 55(10):1886–1888. doi:10.1373/clinchem.2009.130583
  53. Bowness P. HLA-B27. Annu Rev Immunol 2015; 33:29–48. doi:10.1146/annurev-immunol-032414-112110
  54. Sieper J, Poddubnyy D. Axial spondyloarthritis. Lancet 2017; 390(10089):73–84. doi:10.1016/S0140-6736(16)31591-4
  55. Khan MA. Thoughts concerning the early diagnosis of ankylosing spondylitis and related diseases. Clin Exp Rheumatol 2002; 20(6 suppl 28):S6–S10. pmid:12463439
  56. Braun J, Bollow M, Remlinger G, et al. Prevalence of spondyloarthropathies in HLA-B27 positive and negative blood donors. Arthritis Rheum 1998; 41(1):58–67. doi:10.1002/1529-0131(199801)41:1<58::AID-ART8>3.0.CO;2-G
  57. van der Linden SM, Valkenburg HA, de Jongh BM, Cats A. The risk of developing ankylosing spondylitis in HLA-B27 positive individuals. A comparison of relatives of spondylitis patients with the general population. Arthritis Rheum 1984; 27(3):241–249. pmid:6608352
  58. Sheehan NJ. HLA-B27: what’s new? Rheumatology (Oxford) 2010; 49(4):621–631. doi:10.1093/rheumatology/kep450
  59. Baraliakos X, Maksymmowych WP. Imaging in the diagnosis and management of axial spondyloarthritis. Best Pract Res Clin Rheumatol 2016; 30(4):608–623. doi:10.1016/j.berh.2016.09.011
  60. Mandl P, Navarro-Compan V, Terslev L, et al; European League Against Rheumatism (EULAR). EULAR recommendations for the use of imaging in the diagnosis and management of spondyloarthritis in clinical practice. Ann Rheum Dis 2015; 74(7):1327–1339. doi:10.1136/annrheumdis-2014-206971
  61. McAllister K, Goodson N, Warburton I, Rogers G. Spondyloarthritis: diagnosis and management: summary of NICE guidance. BMJ 2017; 356:j839. doi:10.1136/bmj.j839
  62. Poddubnyy D, van Tubergen A, Landewé R, Sieper J, van der Heijde D; Assessment of SpondyloArthritis international Society (ASAS). Development of an ASAS-endorsed recommendation for the early referral of patients with a suspicion of axial spondyloarthritis. Ann Rheum Dis 2015; 74(8):1483–1487. doi:10.1136/annrheumdis-2014-207151
  63. Rudwaleit M, van der Heijde D, Landewe R, et al. The development of Assessment of SpondyloArthritis International Society classification criteria for axial spondyloarthritis (part II): validation and final selection. Ann Rheum Dis 2009; 68(6):777–783. doi:10.1136/ard.2009.108233
Article PDF
Author and Disclosure Information

Ernest Suresh, MD, FRCP (London)
Senior Consultant Rheumatologist and Director of Acute and General Internal Medicine, Division of Medicine, Ng Teng Fong General Hospital, National University Health System, Singapore

Address: Ernest Suresh, MD, FRCP (London), Senior Consultant Rheumatologist, Division of Medicine, Ng Teng Fong General Hospital, 1 Jurong East Street 21, Jurong, Singapore 609606; ernest_suresh@nuhs.edu.sg

Issue
Cleveland Clinic Journal of Medicine - 86(3)
Publications
Topics
Page Number
198-210
Legacy Keywords
rheumatology, tests, rheumatoid factor, rheumatoid arthritis, polyarthritis, anticitrullinated peptide antibody, antinuclear antibody, antiphospholipid antibodies, antineutrophil cytoplasmic antibody, ANCA, ANA, human leukocyte antigen-B27, HLA-B27, ankylosing spondylitis, systemic lupus erythematosus, SLE, anticardiolipin antibodies, lupus anticoagulant, beta-2 glycoprotein I antibody, anti-beta-2GPI, Ernest Suresh
Sections
Author and Disclosure Information

Ernest Suresh, MD, FRCP (London)
Senior Consultant Rheumatologist and Director of Acute and General Internal Medicine, Division of Medicine, Ng Teng Fong General Hospital, National University Health System, Singapore

Address: Ernest Suresh, MD, FRCP (London), Senior Consultant Rheumatologist, Division of Medicine, Ng Teng Fong General Hospital, 1 Jurong East Street 21, Jurong, Singapore 609606; ernest_suresh@nuhs.edu.sg

Author and Disclosure Information

Ernest Suresh, MD, FRCP (London)
Senior Consultant Rheumatologist and Director of Acute and General Internal Medicine, Division of Medicine, Ng Teng Fong General Hospital, National University Health System, Singapore

Address: Ernest Suresh, MD, FRCP (London), Senior Consultant Rheumatologist, Division of Medicine, Ng Teng Fong General Hospital, 1 Jurong East Street 21, Jurong, Singapore 609606; ernest_suresh@nuhs.edu.sg

Article PDF
Article PDF
Related Articles

Laboratory tests are often ordered inappropriately for patients in whom a rheumatologic illness is suspected; this occurs in both primary and secondary care.1 Some tests are available both singly and as part of a battery of tests screening healthy people without symptoms.

The problem: negative test results are by no means always reassuring, and false-positive results raise the risks of unnecessary anxiety for patients and clinicians, needless referrals, and potential morbidity due to further unnecessary testing and exposure to wrong treatments.2 Clinicians should be aware of the pitfalls of these tests in order to choose them wisely and interpret the results correctly.

This article provides practical guidance on requesting and interpreting some common tests in rheumatology, with the aid of case vignettes.

RHEUMATOID FACTOR AND ANTICITRULLINATED PEPTIDE ANTIBODY

A 41-year-old woman, previously in good health, presents to her primary care practitioner with a 6-week history of pain and swelling in her hands and early morning stiffness lasting about 2 hours. She denies having any extraarticular symptoms. Physical examination reveals synovitis across her right metacarpophalangeal joints, proximal interphalangeal joint of the left middle finger, and left wrist. The primary care physician is concerned that her symptoms might be due to rheumatoid arthritis.

Would testing for rheumatoid factor and anticitrullinated peptide antibody be useful in this patient?

Rheumatoid factor is an antibody (immunoglobulin M, IgG, or IgA) targeted against the Fc fragment of IgG.3 It was so named because it was originally detected in patients with rheumatoid arthritis, but it is neither sensitive nor specific for this condition. A meta-analysis of more than 5,000 patients with rheumatoid arthritis reported that rheumatoid factor testing had a sensitivity of 69% and specificity of 85%.4

suresh_rheumatologictests_t1.jpg
Numerous other conditions can be associated with a positive test for rheumatoid factor (Table 1). Hence, a diagnosis of rheumatoid arthritis cannot be confirmed with a positive result alone, nor can it be excluded with a negative result.

Anticitrullinated peptide antibody, on the other hand, is much more specific for rheumatoid arthritis (95%), as it is seldom seen in other conditions, but its sensitivity is similar to that of rheumatoid factor (68%).4–6 A positive result would thus lend strength to the diagnosis of rheumatoid arthritis, but a negative result would not exclude it.

Approach to early arthritis

When faced with a patient with early arthritis, some key questions to ask include7,8:

Is this an inflammatory or a mechanical problem? Inflammatory arthritis is suggested by joint swelling that is not due to trauma or bony hypertrophy, early morning stiffness lasting longer than 30 minutes, and elevated inflammatory markers (erythrocyte sedimentation rate or C-reactive protein). Involvement of the small joints of the hands and feet may be suggested by pain on compression of the metacarpophalangeal and metatarsophalangeal joints, respectively.

Is there a definite identifiable underlying cause for the inflammatory arthritis? The pattern of development of joint symptoms or the presence of extraarticular symptoms may suggest an underlying problem such as gout, psoriatic arthritis, systemic lupus erythematosus, or sarcoidosis.

If the arthritis is undifferentiated (ie, there is no definite identifiable cause), is it likely to remit or persist? This is perhaps the most important question to ask in order to prognosticate. Patients with risk factors for persistent disease, ie, for development of rheumatoid arthritis, should be referred to a rheumatologist early for timely institution of disease-modifying antirheumatic drug therapy.9 Multiple studies have shown that patients in whom this therapy is started early have much better clinical, functional, and radiologic outcomes than those in whom it is delayed.10–12

The revised American College of Rheumatology and European League Against Rheumatism criteria13 include the following factors as predictors of persistence:

  • Number of involved joints (with greater weight given to involvement of small joints)
  • Duration of symptoms 6 weeks or longer
  • Elevated acute-phase response (erythrocyte sedimentation rate or C-reactive protein level)
  • A positive serologic test (either rheumatoid factor or anticitrullinated peptide antibody).

If both rheumatoid factor and anticitrullinated peptide antibody are positive in a patient with early undifferentiated arthritis, the risk of progression to rheumatoid arthritis is almost 100%, thus underscoring the importance of testing for these antibodies.5,6 Referral to a rheumatologist should, however, not be delayed in patients with negative test results (more than one-third of patients with rheumatoid arthritis may be negative for both), and should be considered in those with inflammatory joint symptoms persisting longer than 6 weeks, especially with involvement of the small joints (sparing the distal interphalangeals) and elevated acute-phase response.

Rheumatoid factor in healthy people without symptoms

In some countries, testing for rheumatoid factor is offered as part of a battery of screening tests in healthy people who have no symptoms, a practice that should be strongly discouraged.

Multiple studies, both prospective and retrospective, have demonstrated that both rheumatoid factor and anticitrullinated peptide antibody may be present several years before the clinical diagnosis of rheumatoid arthritis.6,14–16 But the risk of developing rheumatoid arthritis for asymptomatic individuals who are rheumatoid factor-positive depends on the rheumatoid factor titer, positive family history of rheumatoid arthritis in first-degree relatives, and copresence of anticitrullinated peptide antibody. The absolute risk, nevertheless, is still very small. In some, there might be an alternative explanation such as undiagnosed Sjögren syndrome or hepatitis C.

In any event, no strategy is currently available that is proven to prevent the development of rheumatoid arthritis, and there is no role for disease-modifying therapy during the preclinical phase.16

Back to our patient

Blood testing in our patient reveals normal complete blood cell counts, aminotransferase levels, and serum creatinine concentration; findings on urinalysis are normal. Her erythrocyte sedimentation rate is 56 mm/hour (reference range 0–15), and her C-reactive protein level is 26 mg/dL (normal < 3). Testing is negative for rheumatoid factor and anticitrullinated peptide antibody.

Although her rheumatoid factor and anticitrullinated peptide antibody tests are negative, she is referred to a rheumatologist because she has predictors of persistent disease, ie, symptom duration of 6 weeks, involvement of the small joints of the hands, and elevated erythrocyte sedimentation rate and C-reactive protein. The rheumatologist checks her parvovirus serology, which is negative.

The patient is given parenteral depot corticosteroid therapy, to which she responds briefly. Because her symptoms persist and continue to worsen, methotrexate treatment is started after an additional 6 weeks.

 

 

ANTINUCLEAR ANTIBODY

A 37-year-old woman presents to her primary care physician with the complaint of tiredness. She has a family history of systemic lupus erythematosus in her sister and maternal aunt. She is understandably worried about lupus because of the family history and is asking to be tested for it.

Would testing for antinuclear antibody be reasonable?

Antinuclear antibody is not a single antibody but rather a family of autoantibodies that are directed against nuclear constituents such as single- or double-stranded deoxyribonucleic acid (dsDNA), histones, centromeres, proteins complexed with ribonucleic acid (RNA), and enzymes such as topoisomerase.17,18

Protein antigens complexed with RNA and some enzymes in the nucleus are also known as extractable nuclear antigens (ENAs). They include Ro, La, Sm, Jo-1, RNP, and ScL-70 and are named after the patient in whom they were first discovered (Robert, Lavine, Smith, and John), the antigen that is targeted (ribonucleoprotein or RNP), and the disease with which they are associated (anti-ScL-70 or antitopoisomerase in diffuse cutaneous scleroderma).

Antinuclear antibody testing is commonly requested to exclude connective tissue diseases such as lupus, but the clinician needs to be aware of the following points:

Antinuclear antibody may be encountered in conditions other than lupus

These include19:

  • Other autoimmune diseases such as rheumatoid arthritis, primary Sjögren syndrome, systemic sclerosis, autoimmune thyroid disease, and myasthenia gravis
  • Infection with organisms that share the epitope with self-antigens (molecular mimicry)
  • Cancers
  • Drugs such as hydralazine, procainamide, and minocycline.

Antinuclear antibody might also be produced by the healthy immune system from time to time to clear the nuclear debris that is extruded from aging cells.

A study in healthy individuals20 reported a prevalence of positive antinuclear antibody of 32% at a titer of 1/40, 15% at a titer of 1/80, 7% at a titer of 1/160, and 3% at a titer of 1/320. Importantly, a positive result was more common among family members of patients with autoimmune connective tissue diseases.21 Hence, a positive antinuclear antibody result does not always mean lupus.

Antinuclear antibody testing is highly sensitive for lupus

With current laboratory methods, antinuclear antibody testing has a sensitivity close to 100%. Hence, a negative result virtually rules out lupus.

Two methods are commonly used to test for antinuclear antibody: indirect immunofluorescence and enzyme-linked immunosorbent assay (ELISA).22 While human epithelial (Hep2) cells are used as the source of antigen in immunofluorescence, purified nuclear antigens coated on multiple-well plates are used in ELISA.

Although ELISA is simpler to perform, immunofluorescence has a slightly better sensitivity (because the Hep2 cells express a wide range of antigens) and is still considered the gold standard. As expected, the higher sensitivity occurs at the cost of reduced specificity (about 60%), so antinuclear antibody will also be detected in all the other conditions listed above.23

To improve the specificity of antinuclear antibody testing, laboratories report titers (the highest dilution of the test serum that tested positive); a cutoff of greater than 1/80 is generally considered significant.

Do not order antinuclear antibody testing indiscriminately

suresh_rheumatologictests_t2.jpg
If the antinuclear antibody test is requested indiscriminately, the positive predictive value for the diagnosis of lupus is only 11%.24 The test should be requested only when the pretest probability of lupus or other connective tissue disease is high. The positive predictive value is much higher in patients presenting with clinical or laboratory manifestations involving 2 or more organ systems (Table 2).18,25

suresh_rheumatologictests_t3.jpg
Categorization of the specific antigen target improves disease specificity. The antinuclear antibody in patients with lupus may be targeted against single- or double-stranded DNA, histones, or 1 or more of the ENAs. Among these, the presence of anti-dsDNA or anti-Sm is highly specific for a diagnosis of lupus (close to 100%). Neither is sensitive for lupus, however, with anti-dsDNA present in only 60% of patients with lupus and anti-Sm in about 30%.17 Hence, patients with a positive antinuclear antibody and negative anti-dsDNA and anti-Sm may continue to pose a diagnostic challenge. Other examples of specific disease associations are listed in Table 3.

To sum up, the antinuclear antibody test should be requested only in patients with involvement of multiple organ systems. Although a negative result would make it extremely unlikely that the clinical presentation is due to lupus, a positive result is insufficient on its own to make a diagnosis of lupus.

Diagnosing lupus is straightforward when patients present with a specific manifestation such as inflammatory arthritis, photosensitive skin rash, hemolytic anemia, thrombocytopenia, or nephritis, or with specific antibodies such as those against dsDNA or Sm. Patients who present with nonspecific symptoms such as arthralgia or tiredness with a positive antinuclear antibody and negative anti-dsDNA and anti-Sm may present difficulties even for the specialist.25–27

Back to our patient

Our patient denies arthralgia. She has no extraarticular symptoms such as skin rashes, oral ulcers, sicca symptoms, muscle weakness, Raynaud phenomenon, pleuritic chest pain, or breathlessness. Findings on physical examination and urinalysis are unremarkable.

Her primary care physician decides to check her complete blood cell count, erythrocyte sedimentation rate, and thyroid-stimulating hormone level. Although she is reassured that her tiredness is not due to lupus, she insists on getting an antinuclear antibody test.

Her complete blood cell counts are normal. Her erythrocyte sedimentation rate is 6 mm/hour. However, her thyroid-stimulating hormone level is elevated, and subsequent testing shows low free thyroxine and positive thyroid peroxidase antibodies. The antinuclear antibody is positive in a titer of 1/80 and negative for anti-dsDNA and anti-ENA.

We explain to her that the positive antinuclear antibody is most likely related to her autoimmune thyroid disease. She is referred to an endocrinologist.

 

 

ANTIPHOSPHOLIPID ANTIBODIES

A 24-year-old woman presents to the emergency department with acute unprovoked deep vein thrombosis in her right leg, confirmed by ultrasonography. She has no history of previous thrombosis, and the relevant family history is unremarkable. She has never been pregnant. Her platelet count is 84 × 109/L (reference range 150–400), and her baseline activated partial thromboplastin time is prolonged at 62 seconds (reference range 23.0–32.4). The rest of her blood counts and her prothrombin time, liver enzyme levels, and serum creatinine level are normal.

Should this patient be tested for antiphospholipid antibodies?

Antiphospholipid antibodies are important because of their association with thrombotic risk (both venous and arterial) and pregnancy morbidity. The name is a misnomer, as these antibodies are targeted against some proteins that are bound to phospholipids and not only to the phospholipids themselves.

According to the modified Sapporo criteria for the classification of antiphospholipid syndrome,28 antiphospholipid antibodies should remain persistently positive on at least 2 separate occasions at least 12 weeks apart for the result to be considered significant because some infections and drugs may be associated with the transient presence of antiphospholipid antibodies.

Screening for antiphospholipid antibodies should include testing for IgM and IgG anticardiolipin antibodies, lupus anticoagulant, and IgM and IgG beta-2 glycoprotein I antibodies.29,30

Anticardiolipin antibodies

Anticardiolipin (aCL) antibodies may be targeted either against beta-2 glycoprotein I (beta-2GPI) that is bound to cardiolipin (a phospholipid) or against cardiolipin alone; the former is more specific. Antibodies directed against cardiolipin alone are usually transient and are associated with infections and drugs. The result is considered significant only when anticardiolipin antibodies are present in a medium to high titer (> 40 IgG phospholipid units or IgM phospholipid units, or > 99th percentile).

Lupus anticoagulant

The antibody with “lupus anticoagulant activity” is targeted against prothrombin plus phospholipid or beta-2GPI plus phospholipid. The test for it is a functional assay involving 3 steps:

Demonstrating the prolongation of a phospholipid-dependent coagulation assay like the activated partial thromboplastin time (aPTT). (This may explain the prolongation of aPTT in the patient described in the vignette.) Although the presence of lupus anticoagulant is associated with thrombosis, it is called an “anticoagulant” because of this in vitro prolongation of phospholipid-dependent coagulation assays.

Mixing study. The phospholipid-dependent coagulation assay could be prolonged because of either the deficiency of a coagulation factor or the presence of the antiphospholipid antibodies. This can be differentiated by mixing the patient’s plasma with normal plasma (which will have all the clotting factors) in a 1:1 ratio. If the coagulation assay remains prolonged after the addition of normal plasma, clotting factor deficiency can be excluded.

Addition of a phospholipid. If the prolongation of the coagulation assay is due to the presence of an antiphospholipid antibody, addition of extra phospholipid will correct this.

Beta-2 glycoprotein I antibody (anti-beta-2GPI)

The beta-2GPI that is not bound to the cardiolipin can be detected by separately testing for beta-2GPI (the anticardiolipin test only detects the beta-2GPI that is bound to the cardiolipin). The result is considered significant if beta-2GPI is present in a medium to high titer (> 99th percentile).

Studies have shown that antiphospholipid antibodies may be present in 1% to 5% of apparently healthy people in the general population.31 These are usually low-titer anticardiolipin or anti-beta-GPI IgM antibodies that are not associated with thrombosis or adverse pregnancy outcomes. Hence, the term antiphospholipid syndrome should be reserved for those who have had at least 1 episode of thrombosis or pregnancy morbidity and persistent antiphospholipid antibodies, and not those who have asymptomatic or transient antiphospholipid antibodies.

Triple positivity (positive anticardiolipin, lupus anticoagulant, and anti-beta-2GPI) seems to be associated with the highest risk of thrombosis, with a 10-year cumulative incidence of 37.1% (95% confidence interval [CI] 19.9–54.3) for a first thrombotic event,32 and 44.2% (95% CI 38.6–49.8) for recurrent thrombosis.33

The association with thrombosis is stronger for lupus anticoagulant than with the other 2 antibodies, with different studies34 finding an odds ratio ranging from 5 to 16. A positive lupus anticoagulant test with or without a moderate to high titer of anticardiolipin or anti-beta-2GPI IgM or IgG constitutes a high-risk profile, while a moderate to high titer of anticardiolipin or anti-beta-2GPI IgM or IgG constitutes a moderate-risk profile. A low titer of anticardiolipin or anti-beta-2GPI IgM or IgG constitutes a low-risk profile that may not be associated with thrombosis.35

Antiphospholipid syndrome is important to recognize because of the need for long-term anticoagulation to prevent recurrence.36 It may be primary, when it occurs on its own, or secondary, when it occurs in association with another autoimmune disease such as lupus.

Venous events in antiphospholipid syndrome most commonly manifest as lower-limb deep vein thrombosis or pulmonary embolism, while arterial events most commonly manifest as stroke or transient ischemic attack.37 Obstetric manifestations may include not only miscarriage and stillbirth, but also preterm delivery, intrauterine growth retardation, and preeclampsia, all occurring due to placental insufficiency.

The frequency of antiphospholipid antibodies has been estimated as 13.5% in patients with stroke, 11% with myocardial infarction, 9.5% with deep vein thrombosis, and 6% for those with pregnancy morbidity.38

Some noncriteria manifestations have also been recognized in antiphospholipid syndrome, such as thrombocytopenia, cardiac vegetations (Libman-Sachs endocarditis), livedo reticularis, and nephropathy.

suresh_rheumatologictests_t4.jpg
The indications for antiphospholipid antibody testing are listed in Table 4.29 For the patient described in the vignette, it would be appropriate to test for antiphospholipid antibodies because of her unprovoked thrombosis, thrombocytopenia, and prolonged aPTT. Anticoagulant treatment is known to be associated with false-positive lupus anticoagulant, so any blood samples should be drawn before such treatment is commenced.

Back to our patient

Our patient’s anticardiolipin IgG test is negative, while her lupus anticoagulant and beta-2GPI IgG are positive. She has no clinical or laboratory features suggesting lupus.

She is started on warfarin. After 3 months, the warfarin is interrupted for several days, and she is retested for all 3 antiphospholipid antibodies. Her beta-2GPI I IgG and lupus anticoagulant tests are again positive. Because of the persistent antiphospholipid antibody positivity and clinical history of deep vein thrombosis, her condition is diagnosed as primary antiphospholipid syndrome. She is advised to continue anticoagulant therapy indefinitely.

 

 

ANTINEUTROPHIL CYTOPLASMIC ANTIBODY

A 34-year-old man who is an injecting drug user presents with a 2-week history of fever, malaise, and generalized arthralgia. There are no localizing symptoms of infection. Notable findings on examination include a temperature of 38.0°C (100.4°F), needle track marks in his arms, nonblanching vasculitic rash in his legs, and a systolic murmur over the precordium.

His white blood cell count is 15.3 × 109/L (reference range 3.7–11.0), and his C-reactive protein level is 234 mg/dL (normal < 3). Otherwise, results of blood cell counts, liver enzyme tests, renal function tests, urinalysis, and chest radiography are normal.

Two sets of blood cultures are drawn. Transthoracic echocardiography and the antineutrophil cytoplasmic antibody (ANCA) test are requested, as are screening tests for human immunodeficiency virus, hepatitis B, and hepatitis C.

Was the ANCA test indicated in this patient?

ANCAs are autoantibodies against antigens located in the cytoplasmic granules of neutrophils and monocytes. They are associated with small-vessel vasculitides such as granulomatosis with polyangiitis (GPA), microscopic polyangiitis (MPA), eosinophilic granulomatosis with polyangiitis (EGPA), and isolated pauciimmune crescentic glomerulonephritis, all collectively known as ANCA-associated vasculitis (AAV).39

Laboratory methods to detect ANCA include indirect immunofluorescence and antigen-specific enzyme immunoassays. Indirect immunofluorescence only tells us whether or not an antibody that is targeting a cytoplasmic antigen is present. Based on the indirect immunofluorescent pattern, ANCA can be classified as follows:

  • Perinuclear or p-ANCA (if the targeted antigen is located just around the nucleus and extends into it)
  • Cytoplasmic or c-ANCA (if the targeted antigen is located farther away from the nucleus)
  • Atypical ANCA (if the indirect immunofluorescent pattern does not fit with either p-ANCA or c-ANCA).

Indirect immunofluorescence does not give information about the exact antigen that is targeted; this can only be obtained by performing 1 of the antigen-specific immunoassays. The target antigen for c-ANCA is usually proteinase-3 (PR3), while that for p-ANCA could be myeloperoxidase (MPO), cathepsin, lysozyme, lactoferrin, or bactericidal permeability inhibitor. Anti-PR3 is highly specific for GPA, while anti-MPO is usually associated with MPA and EGPA. Less commonly, anti-PR3 may be seen in patients with MPA and anti-MPO in those with GPA. Hence, there is an increasing trend toward classifying ANCA-associated vasculitis into PR3-associated or MPO-associated vasculitis rather than as GPA, MPA, EGPA, or renal-limited vasculitis.40

Several audits have shown that the ANCA test is widely misused and requested indiscriminately to rule out vasculitis. This results in a lower positive predictive value, possible harm to patients due to increased false-positive rates, and increased burden on the laboratory.41–43 At least 2 separate groups have demonstrated that a gating policy that refuses ANCA testing in patients without clinical evidence of systemic vasculitis can reduce the number of inappropriate requests, improve the diagnostic yield, and make it more clinically relevant and cost-effective.44,45

The clinician should bear in mind that:

suresh_rheumatologictests_t5.jpg
ANCA testing should be requested only if the pretest probability of ANCA-associated vasculitis is high. The indications proposed by the International Consensus Statement on ANCA testing46 are listed in Table 5. These criteria have been clinically validated, with 1 study even demonstrating that no cases of ANCA-associated vasculitis would be missed if these guidelines are followed.47

Current guidelines recommend using one of the antigen-specific assays for PR3 and MPO as the primary screening method.48 Until recently, indirect immunofluorescence was used to screen for ANCA-associated vasculitis, and positive results were confirmed by ELISA to detect ANCAs specific for PR3 and MPO,49 but this is no longer recommended because of recent evidence suggesting a large variability between the different indirect immunofluorescent methods and improved diagnostic performance of the antigen-specific assays.

In a large multicenter study by Damoiseaux et al, the specificity with the different antigen-specific immunoassays was 98% to 99% for PR3-ANCA and 96% to 99% for MPO-ANCA.50

ANCA-associated vasculitis should not be considered excluded if the PR3 and MPO-ANCA are negative. In the Damoiseaux study, about 11% to 15% of patients with GPA and 8% to 24% of patients with MPA tested negative for both PR3 and MPO-ANCA.50

If the ANCA result is negative and clinical suspicion for ANCA-associated vasculitis is high, the clinician may wish to consider requesting another immunoassay method or indirect immunofluorescence. Results of indirect immunofluorescent testing results may be positive in those with a negative immunoassay, and vice versa.

suresh_rheumatologictests_t6.jpg
A positive ANCA result is not diagnostic of ANCA-associated vasculitis. Numerous other conditions are associated with ANCA, usually p-ANCA or atypical ANCA (Table 6). The antigens targeted by these ANCAs are usually cathepsin, lysozyme, lactoferrin, and bactericidal permeability inhibitor.

Thus, the ANCA result should always be interpreted in the context of the whole clinical picture.51 Biopsy should still be considered the gold standard for the diagnosis of ANCA-associated vasculitis. The ANCA titer can help to improve clinical interpretation, because the likelihood of ANCA-associated vasculitis increases with higher levels of PR3 and MPO-ANCA.52

Back to our patient

Our patient’s blood cultures grow methicillin-sensitive Staphylococcus aureus in both sets after 48 hours. Transthoracic echocardiography reveals vegetations around the tricuspid valve, with no evidence of valvular regurgitation. The diagnosis is right-sided infective endocarditis. He is started on appropriate antibiotics.

Tests for human immunodeficiency virus, hepatitis B, and hepatitis C are negative. The ANCA test is positive for MPO-ANCA at 28 IU/mL (normal < 10).

The positive ANCA is thought to be related to the infective endocarditis. His vasculitis is most likely secondary to infective endocarditis and not ANCA-associated vasculitis. The ANCA test need not have been requested in the first place.

 

 

HUMAN LEUKOCYTE ANTIGEN-B27

A 22-year-old man presents to his primary care physician with a 4-month history of gradually worsening low back pain associated with early morning stiffness lasting more than 2 hours. He has no peripheral joint symptoms.

In the last 2 years, he has had 2 separate episodes of uveitis. There is a family history of ankylosing spondylitis in his father. Examination reveals global restriction of lumbar movements but is otherwise unremarkable. Magnetic resonance imaging (MRI) of the lumbar spine and sacroiliac joints is normal.

Should this patient be tested for human leukocyte antigen-B27 (HLA-B27)?

The major histocompatibility complex (MHC) is a gene complex that is present in all animals. It encodes proteins that help with immunologic tolerance. HLA simply refers to the human version of the MHC.53 The HLA gene complex, located on chromosome 6, is categorized into class I, class II, and class III. HLA-B is one of the 3 class I genes. Thus, a positive HLA-B27 result simply means that the particular gene is present in that person.

HLA-B27 is strongly associated with ankylosing spondylitis, also known as axial spondyloarthropathy.54 Other genes also contribute to the pathogenesis of ankylosing spondylitis, but HLA-B27 is present in more than 90% of patients with this disease and is by far considered the most important. The association is not as strong for peripheral spondyloarthropathy, with studies reporting a frequency of up to 75% for reactive arthritis and inflammatory bowel disease-associated arthritis, and up to 50% for psoriatic arthritis and uveitis.55

About 9% of healthy, asymptomatic individuals may have HLA-B27, so the mere presence of this gene is not evidence of disease.56 There may be up to a 20-fold increased risk of ankylosing spondylitis among those who are HLA-B27-positive.57

Some HLA genes have many different alleles, each of which is given a number (explaining the number 27 that follows the B). Closely related alleles that differ from one another by only a few amino-acid substitutions are then categorized together, thus accounting for more than 100 subtypes of HLA-B27 (designated from HLA-B*2701 to HLA-B*27106). These subtypes vary in frequency among different racial groups, and the population prevalence of ankylosing spondylitis parallels the frequency of HLA-B27.58 The most common subtype seen in white people and American Indians is B*2705. HLA-B27 is rare in blacks, explaining the rarity of ankylosing spondylitis in this population. Further examples include HLA-B*2704, which is seen in Asians, and HLA-B*2702, seen in Mediterranean populations. Not all subtypes of HLA-B27 are associated with disease, and some, like HLA-B*2706, may also be protective.

When should the clinician consider testing for HLA-B27?

suresh_rheumatologictests_t7.jpg
Not all patients with low back pain need an HLA-B27 test. First, it is important to look for clinical features of axial spondyloarthropathy (Table 7). The unifying feature of spondyloarthropathy is enthesitis (inflammation at the sites of insertion of tendons or ligaments on the skeleton). Inflammation of axial entheses causes spondylitis and sacroiliitis, manifesting as inflammatory back pain. Clinical clues to inflammatory back pain include insidious onset, aggravation with rest or inactivity, prolonged early morning stiffness, disturbed sleep during the second half of the night, relief with movement or activity, alternating gluteal pain (due to sacroiliitis), and good response to anti-inflammatory medication (although nonspecific).

Peripheral spondyloarthropathy may present with arthritis, enthesitis (eg, heel pain due to inflammation at the site of insertion of the Achilles tendon or plantar fascia), or dactylitis (“sausage” swelling of the whole finger or toe due to extension of inflammation beyond the margins of the joint). Other clues may include psoriasis, inflammatory bowel disease, history of preceding gastrointestinal or genitourinary infection, family history of similar conditions, and history of recurrent uveitis.

For the initial assessment of patients who have inflammatory back pain, plain radiography of the sacroiliac joints is considered the gold standard.59 If plain radiography does not show evidence of sacroiliitis, MRI of the sacroiliac joints should be considered. While plain radiography can reveal only structural changes such as sclerosis, erosions, and ankylosis, MRI is useful to evaluate for early inflammatory changes such as bone marrow edema. Imaging the lumbar spine is not necessary, as the sacroiliac joints are almost invariably involved in axial spondyloarthropathy, and lesions seldom occur in the lumbar spine in isolation.60

The diagnosis of ankylosing spondylitis previously relied on confirmatory imaging features, but based on the new International Society classification criteria,61–63 which can be applied to patients with more than 3 months of back pain and age of onset of symptoms before age 45, patients can be classified as having 1 of the following:

  • Radiographic axial spondyloarthropathy, if they have evidence of sacroiliitis on imaging plus 1 other feature of spondyloarthropathy
  • Nonradiographic axial spondyloarthropathy, if they have a positive HLA-B27 plus 2 other features of spondyloarthropathy (Table 7).

These new criteria have a sensitivity of 82.9% and specificity of 84.4%.62,63 The disease burden of radiographic and nonradiographic axial spondyloarthropathy has been shown to be similar, suggesting that they are part of the same disease spectrum. Thus, the HLA-B27 test is useful to make a diagnosis of axial spondyloarthropathy even in the absence of imaging features and could be requested in patients with 2 or more features of spondyloarthropathy. In the absence of imaging features and a negative HLA-B27 result, however, the patient cannot be classified as having axial spondyloarthropathy.

Back to our patient

The absence of radiographic evidence would not exclude axial spondyloarthropathy in our patient. The HLA-B27 test is requested because of the inflammatory back pain and the presence of 2 spondyloarthropathy features (uveitis and the family history) and is reported to be positive. His disease is classified as nonradiographic axial spondyloarthropathy.

He is started on regular naproxen and is referred to a physiotherapist. After 1 month, he reports significant symptomatic improvement. He asks if he can be retested for HLA-B27 to see if it has become negative. We tell him that there is no point in repeating it, as it is a gene and will not disappear.

SUMMARY: CONSIDER THE CLINICAL PICTURE

When approaching a patient suspected of having a rheumatologic disease, a clinician should first consider the clinical presentation and the intended purpose of each test. The tests, in general, might serve several purposes. They might help to:

Increase the likelihood of the diagnosis in question. For example, a positive rheumatoid factor or anticitrullinated peptide antibody can help diagnose rheumatoid arthritis in a patient with early polyarthritis, a positive HLA-B27 can help diagnose ankylosing spondylitis in patients with inflammatory back pain and normal imaging, and a positive ANCA can help diagnose ANCA-associated vasculitis in a patient with glomerulonephritis.

Reduce the likelihood of the diagnosis in question. For example, a negative antinuclear antibody test reduces the likelihood of lupus in a patient with joint pains.

Monitor the condition. For example DNA antibodies can be used to monitor the activity of lupus.

Plan the treatment strategy. For example, one might consider lifelong anticoagulation if antiphospholipid antibodies are persistently positive in a patient with thrombosis.

Prognosticate. For example, positive rheumatoid factor and anticitrullinated peptide antibody increase the risk of erosive rheumatoid arthritis.

If the test was requested in the absence of a clear indication and the result is positive, it is important to bear in mind the potential pitfalls associated with that test and not attach a diagnostic label prematurely. None of the tests can confirm or exclude a condition, so the results should always be interpreted in the context of the whole clinical picture.   

Laboratory tests are often ordered inappropriately for patients in whom a rheumatologic illness is suspected; this occurs in both primary and secondary care.1 Some tests are available both singly and as part of a battery of tests screening healthy people without symptoms.

The problem: negative test results are by no means always reassuring, and false-positive results raise the risks of unnecessary anxiety for patients and clinicians, needless referrals, and potential morbidity due to further unnecessary testing and exposure to wrong treatments.2 Clinicians should be aware of the pitfalls of these tests in order to choose them wisely and interpret the results correctly.

This article provides practical guidance on requesting and interpreting some common tests in rheumatology, with the aid of case vignettes.

RHEUMATOID FACTOR AND ANTICITRULLINATED PEPTIDE ANTIBODY

A 41-year-old woman, previously in good health, presents to her primary care practitioner with a 6-week history of pain and swelling in her hands and early morning stiffness lasting about 2 hours. She denies having any extraarticular symptoms. Physical examination reveals synovitis across her right metacarpophalangeal joints, proximal interphalangeal joint of the left middle finger, and left wrist. The primary care physician is concerned that her symptoms might be due to rheumatoid arthritis.

Would testing for rheumatoid factor and anticitrullinated peptide antibody be useful in this patient?

Rheumatoid factor is an antibody (immunoglobulin M, IgG, or IgA) targeted against the Fc fragment of IgG.3 It was so named because it was originally detected in patients with rheumatoid arthritis, but it is neither sensitive nor specific for this condition. A meta-analysis of more than 5,000 patients with rheumatoid arthritis reported that rheumatoid factor testing had a sensitivity of 69% and specificity of 85%.4

suresh_rheumatologictests_t1.jpg
Numerous other conditions can be associated with a positive test for rheumatoid factor (Table 1). Hence, a diagnosis of rheumatoid arthritis cannot be confirmed with a positive result alone, nor can it be excluded with a negative result.

Anticitrullinated peptide antibody, on the other hand, is much more specific for rheumatoid arthritis (95%), as it is seldom seen in other conditions, but its sensitivity is similar to that of rheumatoid factor (68%).4–6 A positive result would thus lend strength to the diagnosis of rheumatoid arthritis, but a negative result would not exclude it.

Approach to early arthritis

When faced with a patient with early arthritis, some key questions to ask include7,8:

Is this an inflammatory or a mechanical problem? Inflammatory arthritis is suggested by joint swelling that is not due to trauma or bony hypertrophy, early morning stiffness lasting longer than 30 minutes, and elevated inflammatory markers (erythrocyte sedimentation rate or C-reactive protein). Involvement of the small joints of the hands and feet may be suggested by pain on compression of the metacarpophalangeal and metatarsophalangeal joints, respectively.

Is there a definite identifiable underlying cause for the inflammatory arthritis? The pattern of development of joint symptoms or the presence of extraarticular symptoms may suggest an underlying problem such as gout, psoriatic arthritis, systemic lupus erythematosus, or sarcoidosis.

If the arthritis is undifferentiated (ie, there is no definite identifiable cause), is it likely to remit or persist? This is perhaps the most important question to ask in order to prognosticate. Patients with risk factors for persistent disease, ie, for development of rheumatoid arthritis, should be referred to a rheumatologist early for timely institution of disease-modifying antirheumatic drug therapy.9 Multiple studies have shown that patients in whom this therapy is started early have much better clinical, functional, and radiologic outcomes than those in whom it is delayed.10–12

The revised American College of Rheumatology and European League Against Rheumatism criteria13 include the following factors as predictors of persistence:

  • Number of involved joints (with greater weight given to involvement of small joints)
  • Duration of symptoms 6 weeks or longer
  • Elevated acute-phase response (erythrocyte sedimentation rate or C-reactive protein level)
  • A positive serologic test (either rheumatoid factor or anticitrullinated peptide antibody).

If both rheumatoid factor and anticitrullinated peptide antibody are positive in a patient with early undifferentiated arthritis, the risk of progression to rheumatoid arthritis is almost 100%, thus underscoring the importance of testing for these antibodies.5,6 Referral to a rheumatologist should, however, not be delayed in patients with negative test results (more than one-third of patients with rheumatoid arthritis may be negative for both), and should be considered in those with inflammatory joint symptoms persisting longer than 6 weeks, especially with involvement of the small joints (sparing the distal interphalangeals) and elevated acute-phase response.

Rheumatoid factor in healthy people without symptoms

In some countries, testing for rheumatoid factor is offered as part of a battery of screening tests in healthy people who have no symptoms, a practice that should be strongly discouraged.

Multiple studies, both prospective and retrospective, have demonstrated that both rheumatoid factor and anticitrullinated peptide antibody may be present several years before the clinical diagnosis of rheumatoid arthritis.6,14–16 But the risk of developing rheumatoid arthritis for asymptomatic individuals who are rheumatoid factor-positive depends on the rheumatoid factor titer, positive family history of rheumatoid arthritis in first-degree relatives, and copresence of anticitrullinated peptide antibody. The absolute risk, nevertheless, is still very small. In some, there might be an alternative explanation such as undiagnosed Sjögren syndrome or hepatitis C.

In any event, no strategy is currently available that is proven to prevent the development of rheumatoid arthritis, and there is no role for disease-modifying therapy during the preclinical phase.16

Back to our patient

Blood testing in our patient reveals normal complete blood cell counts, aminotransferase levels, and serum creatinine concentration; findings on urinalysis are normal. Her erythrocyte sedimentation rate is 56 mm/hour (reference range 0–15), and her C-reactive protein level is 26 mg/dL (normal < 3). Testing is negative for rheumatoid factor and anticitrullinated peptide antibody.

Although her rheumatoid factor and anticitrullinated peptide antibody tests are negative, she is referred to a rheumatologist because she has predictors of persistent disease, ie, symptom duration of 6 weeks, involvement of the small joints of the hands, and elevated erythrocyte sedimentation rate and C-reactive protein. The rheumatologist checks her parvovirus serology, which is negative.

The patient is given parenteral depot corticosteroid therapy, to which she responds briefly. Because her symptoms persist and continue to worsen, methotrexate treatment is started after an additional 6 weeks.

 

 

ANTINUCLEAR ANTIBODY

A 37-year-old woman presents to her primary care physician with the complaint of tiredness. She has a family history of systemic lupus erythematosus in her sister and maternal aunt. She is understandably worried about lupus because of the family history and is asking to be tested for it.

Would testing for antinuclear antibody be reasonable?

Antinuclear antibody is not a single antibody but rather a family of autoantibodies that are directed against nuclear constituents such as single- or double-stranded deoxyribonucleic acid (dsDNA), histones, centromeres, proteins complexed with ribonucleic acid (RNA), and enzymes such as topoisomerase.17,18

Protein antigens complexed with RNA and some enzymes in the nucleus are also known as extractable nuclear antigens (ENAs). They include Ro, La, Sm, Jo-1, RNP, and ScL-70 and are named after the patient in whom they were first discovered (Robert, Lavine, Smith, and John), the antigen that is targeted (ribonucleoprotein or RNP), and the disease with which they are associated (anti-ScL-70 or antitopoisomerase in diffuse cutaneous scleroderma).

Antinuclear antibody testing is commonly requested to exclude connective tissue diseases such as lupus, but the clinician needs to be aware of the following points:

Antinuclear antibody may be encountered in conditions other than lupus

These include19:

  • Other autoimmune diseases such as rheumatoid arthritis, primary Sjögren syndrome, systemic sclerosis, autoimmune thyroid disease, and myasthenia gravis
  • Infection with organisms that share the epitope with self-antigens (molecular mimicry)
  • Cancers
  • Drugs such as hydralazine, procainamide, and minocycline.

Antinuclear antibody might also be produced by the healthy immune system from time to time to clear the nuclear debris that is extruded from aging cells.

A study in healthy individuals20 reported a prevalence of positive antinuclear antibody of 32% at a titer of 1/40, 15% at a titer of 1/80, 7% at a titer of 1/160, and 3% at a titer of 1/320. Importantly, a positive result was more common among family members of patients with autoimmune connective tissue diseases.21 Hence, a positive antinuclear antibody result does not always mean lupus.

Antinuclear antibody testing is highly sensitive for lupus

With current laboratory methods, antinuclear antibody testing has a sensitivity close to 100%. Hence, a negative result virtually rules out lupus.

Two methods are commonly used to test for antinuclear antibody: indirect immunofluorescence and enzyme-linked immunosorbent assay (ELISA).22 While human epithelial (Hep2) cells are used as the source of antigen in immunofluorescence, purified nuclear antigens coated on multiple-well plates are used in ELISA.

Although ELISA is simpler to perform, immunofluorescence has a slightly better sensitivity (because the Hep2 cells express a wide range of antigens) and is still considered the gold standard. As expected, the higher sensitivity occurs at the cost of reduced specificity (about 60%), so antinuclear antibody will also be detected in all the other conditions listed above.23

To improve the specificity of antinuclear antibody testing, laboratories report titers (the highest dilution of the test serum that tested positive); a cutoff of greater than 1/80 is generally considered significant.

Do not order antinuclear antibody testing indiscriminately

suresh_rheumatologictests_t2.jpg
If the antinuclear antibody test is requested indiscriminately, the positive predictive value for the diagnosis of lupus is only 11%.24 The test should be requested only when the pretest probability of lupus or other connective tissue disease is high. The positive predictive value is much higher in patients presenting with clinical or laboratory manifestations involving 2 or more organ systems (Table 2).18,25

suresh_rheumatologictests_t3.jpg
Categorization of the specific antigen target improves disease specificity. The antinuclear antibody in patients with lupus may be targeted against single- or double-stranded DNA, histones, or 1 or more of the ENAs. Among these, the presence of anti-dsDNA or anti-Sm is highly specific for a diagnosis of lupus (close to 100%). Neither is sensitive for lupus, however, with anti-dsDNA present in only 60% of patients with lupus and anti-Sm in about 30%.17 Hence, patients with a positive antinuclear antibody and negative anti-dsDNA and anti-Sm may continue to pose a diagnostic challenge. Other examples of specific disease associations are listed in Table 3.

To sum up, the antinuclear antibody test should be requested only in patients with involvement of multiple organ systems. Although a negative result would make it extremely unlikely that the clinical presentation is due to lupus, a positive result is insufficient on its own to make a diagnosis of lupus.

Diagnosing lupus is straightforward when patients present with a specific manifestation such as inflammatory arthritis, photosensitive skin rash, hemolytic anemia, thrombocytopenia, or nephritis, or with specific antibodies such as those against dsDNA or Sm. Patients who present with nonspecific symptoms such as arthralgia or tiredness with a positive antinuclear antibody and negative anti-dsDNA and anti-Sm may present difficulties even for the specialist.25–27

Back to our patient

Our patient denies arthralgia. She has no extraarticular symptoms such as skin rashes, oral ulcers, sicca symptoms, muscle weakness, Raynaud phenomenon, pleuritic chest pain, or breathlessness. Findings on physical examination and urinalysis are unremarkable.

Her primary care physician decides to check her complete blood cell count, erythrocyte sedimentation rate, and thyroid-stimulating hormone level. Although she is reassured that her tiredness is not due to lupus, she insists on getting an antinuclear antibody test.

Her complete blood cell counts are normal. Her erythrocyte sedimentation rate is 6 mm/hour. However, her thyroid-stimulating hormone level is elevated, and subsequent testing shows low free thyroxine and positive thyroid peroxidase antibodies. The antinuclear antibody is positive in a titer of 1/80 and negative for anti-dsDNA and anti-ENA.

We explain to her that the positive antinuclear antibody is most likely related to her autoimmune thyroid disease. She is referred to an endocrinologist.

 

 

ANTIPHOSPHOLIPID ANTIBODIES

A 24-year-old woman presents to the emergency department with acute unprovoked deep vein thrombosis in her right leg, confirmed by ultrasonography. She has no history of previous thrombosis, and the relevant family history is unremarkable. She has never been pregnant. Her platelet count is 84 × 109/L (reference range 150–400), and her baseline activated partial thromboplastin time is prolonged at 62 seconds (reference range 23.0–32.4). The rest of her blood counts and her prothrombin time, liver enzyme levels, and serum creatinine level are normal.

Should this patient be tested for antiphospholipid antibodies?

Antiphospholipid antibodies are important because of their association with thrombotic risk (both venous and arterial) and pregnancy morbidity. The name is a misnomer, as these antibodies are targeted against some proteins that are bound to phospholipids and not only to the phospholipids themselves.

According to the modified Sapporo criteria for the classification of antiphospholipid syndrome,28 antiphospholipid antibodies should remain persistently positive on at least 2 separate occasions at least 12 weeks apart for the result to be considered significant because some infections and drugs may be associated with the transient presence of antiphospholipid antibodies.

Screening for antiphospholipid antibodies should include testing for IgM and IgG anticardiolipin antibodies, lupus anticoagulant, and IgM and IgG beta-2 glycoprotein I antibodies.29,30

Anticardiolipin antibodies

Anticardiolipin (aCL) antibodies may be targeted either against beta-2 glycoprotein I (beta-2GPI) that is bound to cardiolipin (a phospholipid) or against cardiolipin alone; the former is more specific. Antibodies directed against cardiolipin alone are usually transient and are associated with infections and drugs. The result is considered significant only when anticardiolipin antibodies are present in a medium to high titer (> 40 IgG phospholipid units or IgM phospholipid units, or > 99th percentile).

Lupus anticoagulant

The antibody with “lupus anticoagulant activity” is targeted against prothrombin plus phospholipid or beta-2GPI plus phospholipid. The test for it is a functional assay involving 3 steps:

Demonstrating the prolongation of a phospholipid-dependent coagulation assay like the activated partial thromboplastin time (aPTT). (This may explain the prolongation of aPTT in the patient described in the vignette.) Although the presence of lupus anticoagulant is associated with thrombosis, it is called an “anticoagulant” because of this in vitro prolongation of phospholipid-dependent coagulation assays.

Mixing study. The phospholipid-dependent coagulation assay could be prolonged because of either the deficiency of a coagulation factor or the presence of the antiphospholipid antibodies. This can be differentiated by mixing the patient’s plasma with normal plasma (which will have all the clotting factors) in a 1:1 ratio. If the coagulation assay remains prolonged after the addition of normal plasma, clotting factor deficiency can be excluded.

Addition of a phospholipid. If the prolongation of the coagulation assay is due to the presence of an antiphospholipid antibody, addition of extra phospholipid will correct this.

Beta-2 glycoprotein I antibody (anti-beta-2GPI)

The beta-2GPI that is not bound to the cardiolipin can be detected by separately testing for beta-2GPI (the anticardiolipin test only detects the beta-2GPI that is bound to the cardiolipin). The result is considered significant if beta-2GPI is present in a medium to high titer (> 99th percentile).

Studies have shown that antiphospholipid antibodies may be present in 1% to 5% of apparently healthy people in the general population.31 These are usually low-titer anticardiolipin or anti-beta-GPI IgM antibodies that are not associated with thrombosis or adverse pregnancy outcomes. Hence, the term antiphospholipid syndrome should be reserved for those who have had at least 1 episode of thrombosis or pregnancy morbidity and persistent antiphospholipid antibodies, and not those who have asymptomatic or transient antiphospholipid antibodies.

Triple positivity (positive anticardiolipin, lupus anticoagulant, and anti-beta-2GPI) seems to be associated with the highest risk of thrombosis, with a 10-year cumulative incidence of 37.1% (95% confidence interval [CI] 19.9–54.3) for a first thrombotic event,32 and 44.2% (95% CI 38.6–49.8) for recurrent thrombosis.33

The association with thrombosis is stronger for lupus anticoagulant than with the other 2 antibodies, with different studies34 finding an odds ratio ranging from 5 to 16. A positive lupus anticoagulant test with or without a moderate to high titer of anticardiolipin or anti-beta-2GPI IgM or IgG constitutes a high-risk profile, while a moderate to high titer of anticardiolipin or anti-beta-2GPI IgM or IgG constitutes a moderate-risk profile. A low titer of anticardiolipin or anti-beta-2GPI IgM or IgG constitutes a low-risk profile that may not be associated with thrombosis.35

Antiphospholipid syndrome is important to recognize because of the need for long-term anticoagulation to prevent recurrence.36 It may be primary, when it occurs on its own, or secondary, when it occurs in association with another autoimmune disease such as lupus.

Venous events in antiphospholipid syndrome most commonly manifest as lower-limb deep vein thrombosis or pulmonary embolism, while arterial events most commonly manifest as stroke or transient ischemic attack.37 Obstetric manifestations may include not only miscarriage and stillbirth, but also preterm delivery, intrauterine growth retardation, and preeclampsia, all occurring due to placental insufficiency.

The frequency of antiphospholipid antibodies has been estimated as 13.5% in patients with stroke, 11% with myocardial infarction, 9.5% with deep vein thrombosis, and 6% for those with pregnancy morbidity.38

Some noncriteria manifestations have also been recognized in antiphospholipid syndrome, such as thrombocytopenia, cardiac vegetations (Libman-Sachs endocarditis), livedo reticularis, and nephropathy.

suresh_rheumatologictests_t4.jpg
The indications for antiphospholipid antibody testing are listed in Table 4.29 For the patient described in the vignette, it would be appropriate to test for antiphospholipid antibodies because of her unprovoked thrombosis, thrombocytopenia, and prolonged aPTT. Anticoagulant treatment is known to be associated with false-positive lupus anticoagulant, so any blood samples should be drawn before such treatment is commenced.

Back to our patient

Our patient’s anticardiolipin IgG test is negative, while her lupus anticoagulant and beta-2GPI IgG are positive. She has no clinical or laboratory features suggesting lupus.

She is started on warfarin. After 3 months, the warfarin is interrupted for several days, and she is retested for all 3 antiphospholipid antibodies. Her beta-2GPI I IgG and lupus anticoagulant tests are again positive. Because of the persistent antiphospholipid antibody positivity and clinical history of deep vein thrombosis, her condition is diagnosed as primary antiphospholipid syndrome. She is advised to continue anticoagulant therapy indefinitely.

 

 

ANTINEUTROPHIL CYTOPLASMIC ANTIBODY

A 34-year-old man who is an injecting drug user presents with a 2-week history of fever, malaise, and generalized arthralgia. There are no localizing symptoms of infection. Notable findings on examination include a temperature of 38.0°C (100.4°F), needle track marks in his arms, nonblanching vasculitic rash in his legs, and a systolic murmur over the precordium.

His white blood cell count is 15.3 × 109/L (reference range 3.7–11.0), and his C-reactive protein level is 234 mg/dL (normal < 3). Otherwise, results of blood cell counts, liver enzyme tests, renal function tests, urinalysis, and chest radiography are normal.

Two sets of blood cultures are drawn. Transthoracic echocardiography and the antineutrophil cytoplasmic antibody (ANCA) test are requested, as are screening tests for human immunodeficiency virus, hepatitis B, and hepatitis C.

Was the ANCA test indicated in this patient?

ANCAs are autoantibodies against antigens located in the cytoplasmic granules of neutrophils and monocytes. They are associated with small-vessel vasculitides such as granulomatosis with polyangiitis (GPA), microscopic polyangiitis (MPA), eosinophilic granulomatosis with polyangiitis (EGPA), and isolated pauciimmune crescentic glomerulonephritis, all collectively known as ANCA-associated vasculitis (AAV).39

Laboratory methods to detect ANCA include indirect immunofluorescence and antigen-specific enzyme immunoassays. Indirect immunofluorescence only tells us whether or not an antibody that is targeting a cytoplasmic antigen is present. Based on the indirect immunofluorescent pattern, ANCA can be classified as follows:

  • Perinuclear or p-ANCA (if the targeted antigen is located just around the nucleus and extends into it)
  • Cytoplasmic or c-ANCA (if the targeted antigen is located farther away from the nucleus)
  • Atypical ANCA (if the indirect immunofluorescent pattern does not fit with either p-ANCA or c-ANCA).

Indirect immunofluorescence does not give information about the exact antigen that is targeted; this can only be obtained by performing 1 of the antigen-specific immunoassays. The target antigen for c-ANCA is usually proteinase-3 (PR3), while that for p-ANCA could be myeloperoxidase (MPO), cathepsin, lysozyme, lactoferrin, or bactericidal permeability inhibitor. Anti-PR3 is highly specific for GPA, while anti-MPO is usually associated with MPA and EGPA. Less commonly, anti-PR3 may be seen in patients with MPA and anti-MPO in those with GPA. Hence, there is an increasing trend toward classifying ANCA-associated vasculitis into PR3-associated or MPO-associated vasculitis rather than as GPA, MPA, EGPA, or renal-limited vasculitis.40

Several audits have shown that the ANCA test is widely misused and requested indiscriminately to rule out vasculitis. This results in a lower positive predictive value, possible harm to patients due to increased false-positive rates, and increased burden on the laboratory.41–43 At least 2 separate groups have demonstrated that a gating policy that refuses ANCA testing in patients without clinical evidence of systemic vasculitis can reduce the number of inappropriate requests, improve the diagnostic yield, and make it more clinically relevant and cost-effective.44,45

The clinician should bear in mind that:

suresh_rheumatologictests_t5.jpg
ANCA testing should be requested only if the pretest probability of ANCA-associated vasculitis is high. The indications proposed by the International Consensus Statement on ANCA testing46 are listed in Table 5. These criteria have been clinically validated, with 1 study even demonstrating that no cases of ANCA-associated vasculitis would be missed if these guidelines are followed.47

Current guidelines recommend using one of the antigen-specific assays for PR3 and MPO as the primary screening method.48 Until recently, indirect immunofluorescence was used to screen for ANCA-associated vasculitis, and positive results were confirmed by ELISA to detect ANCAs specific for PR3 and MPO,49 but this is no longer recommended because of recent evidence suggesting a large variability between the different indirect immunofluorescent methods and improved diagnostic performance of the antigen-specific assays.

In a large multicenter study by Damoiseaux et al, the specificity with the different antigen-specific immunoassays was 98% to 99% for PR3-ANCA and 96% to 99% for MPO-ANCA.50

ANCA-associated vasculitis should not be considered excluded if the PR3 and MPO-ANCA are negative. In the Damoiseaux study, about 11% to 15% of patients with GPA and 8% to 24% of patients with MPA tested negative for both PR3 and MPO-ANCA.50

If the ANCA result is negative and clinical suspicion for ANCA-associated vasculitis is high, the clinician may wish to consider requesting another immunoassay method or indirect immunofluorescence. Results of indirect immunofluorescent testing results may be positive in those with a negative immunoassay, and vice versa.

suresh_rheumatologictests_t6.jpg
A positive ANCA result is not diagnostic of ANCA-associated vasculitis. Numerous other conditions are associated with ANCA, usually p-ANCA or atypical ANCA (Table 6). The antigens targeted by these ANCAs are usually cathepsin, lysozyme, lactoferrin, and bactericidal permeability inhibitor.

Thus, the ANCA result should always be interpreted in the context of the whole clinical picture.51 Biopsy should still be considered the gold standard for the diagnosis of ANCA-associated vasculitis. The ANCA titer can help to improve clinical interpretation, because the likelihood of ANCA-associated vasculitis increases with higher levels of PR3 and MPO-ANCA.52

Back to our patient

Our patient’s blood cultures grow methicillin-sensitive Staphylococcus aureus in both sets after 48 hours. Transthoracic echocardiography reveals vegetations around the tricuspid valve, with no evidence of valvular regurgitation. The diagnosis is right-sided infective endocarditis. He is started on appropriate antibiotics.

Tests for human immunodeficiency virus, hepatitis B, and hepatitis C are negative. The ANCA test is positive for MPO-ANCA at 28 IU/mL (normal < 10).

The positive ANCA is thought to be related to the infective endocarditis. His vasculitis is most likely secondary to infective endocarditis and not ANCA-associated vasculitis. The ANCA test need not have been requested in the first place.

 

 

HUMAN LEUKOCYTE ANTIGEN-B27

A 22-year-old man presents to his primary care physician with a 4-month history of gradually worsening low back pain associated with early morning stiffness lasting more than 2 hours. He has no peripheral joint symptoms.

In the last 2 years, he has had 2 separate episodes of uveitis. There is a family history of ankylosing spondylitis in his father. Examination reveals global restriction of lumbar movements but is otherwise unremarkable. Magnetic resonance imaging (MRI) of the lumbar spine and sacroiliac joints is normal.

Should this patient be tested for human leukocyte antigen-B27 (HLA-B27)?

The major histocompatibility complex (MHC) is a gene complex that is present in all animals. It encodes proteins that help with immunologic tolerance. HLA simply refers to the human version of the MHC.53 The HLA gene complex, located on chromosome 6, is categorized into class I, class II, and class III. HLA-B is one of the 3 class I genes. Thus, a positive HLA-B27 result simply means that the particular gene is present in that person.

HLA-B27 is strongly associated with ankylosing spondylitis, also known as axial spondyloarthropathy.54 Other genes also contribute to the pathogenesis of ankylosing spondylitis, but HLA-B27 is present in more than 90% of patients with this disease and is by far considered the most important. The association is not as strong for peripheral spondyloarthropathy, with studies reporting a frequency of up to 75% for reactive arthritis and inflammatory bowel disease-associated arthritis, and up to 50% for psoriatic arthritis and uveitis.55

About 9% of healthy, asymptomatic individuals may have HLA-B27, so the mere presence of this gene is not evidence of disease.56 There may be up to a 20-fold increased risk of ankylosing spondylitis among those who are HLA-B27-positive.57

Some HLA genes have many different alleles, each of which is given a number (explaining the number 27 that follows the B). Closely related alleles that differ from one another by only a few amino-acid substitutions are then categorized together, thus accounting for more than 100 subtypes of HLA-B27 (designated from HLA-B*2701 to HLA-B*27106). These subtypes vary in frequency among different racial groups, and the population prevalence of ankylosing spondylitis parallels the frequency of HLA-B27.58 The most common subtype seen in white people and American Indians is B*2705. HLA-B27 is rare in blacks, explaining the rarity of ankylosing spondylitis in this population. Further examples include HLA-B*2704, which is seen in Asians, and HLA-B*2702, seen in Mediterranean populations. Not all subtypes of HLA-B27 are associated with disease, and some, like HLA-B*2706, may also be protective.

When should the clinician consider testing for HLA-B27?

suresh_rheumatologictests_t7.jpg
Not all patients with low back pain need an HLA-B27 test. First, it is important to look for clinical features of axial spondyloarthropathy (Table 7). The unifying feature of spondyloarthropathy is enthesitis (inflammation at the sites of insertion of tendons or ligaments on the skeleton). Inflammation of axial entheses causes spondylitis and sacroiliitis, manifesting as inflammatory back pain. Clinical clues to inflammatory back pain include insidious onset, aggravation with rest or inactivity, prolonged early morning stiffness, disturbed sleep during the second half of the night, relief with movement or activity, alternating gluteal pain (due to sacroiliitis), and good response to anti-inflammatory medication (although nonspecific).

Peripheral spondyloarthropathy may present with arthritis, enthesitis (eg, heel pain due to inflammation at the site of insertion of the Achilles tendon or plantar fascia), or dactylitis (“sausage” swelling of the whole finger or toe due to extension of inflammation beyond the margins of the joint). Other clues may include psoriasis, inflammatory bowel disease, history of preceding gastrointestinal or genitourinary infection, family history of similar conditions, and history of recurrent uveitis.

For the initial assessment of patients who have inflammatory back pain, plain radiography of the sacroiliac joints is considered the gold standard.59 If plain radiography does not show evidence of sacroiliitis, MRI of the sacroiliac joints should be considered. While plain radiography can reveal only structural changes such as sclerosis, erosions, and ankylosis, MRI is useful to evaluate for early inflammatory changes such as bone marrow edema. Imaging the lumbar spine is not necessary, as the sacroiliac joints are almost invariably involved in axial spondyloarthropathy, and lesions seldom occur in the lumbar spine in isolation.60

The diagnosis of ankylosing spondylitis previously relied on confirmatory imaging features, but based on the new International Society classification criteria,61–63 which can be applied to patients with more than 3 months of back pain and age of onset of symptoms before age 45, patients can be classified as having 1 of the following:

  • Radiographic axial spondyloarthropathy, if they have evidence of sacroiliitis on imaging plus 1 other feature of spondyloarthropathy
  • Nonradiographic axial spondyloarthropathy, if they have a positive HLA-B27 plus 2 other features of spondyloarthropathy (Table 7).

These new criteria have a sensitivity of 82.9% and specificity of 84.4%.62,63 The disease burden of radiographic and nonradiographic axial spondyloarthropathy has been shown to be similar, suggesting that they are part of the same disease spectrum. Thus, the HLA-B27 test is useful to make a diagnosis of axial spondyloarthropathy even in the absence of imaging features and could be requested in patients with 2 or more features of spondyloarthropathy. In the absence of imaging features and a negative HLA-B27 result, however, the patient cannot be classified as having axial spondyloarthropathy.

Back to our patient

The absence of radiographic evidence would not exclude axial spondyloarthropathy in our patient. The HLA-B27 test is requested because of the inflammatory back pain and the presence of 2 spondyloarthropathy features (uveitis and the family history) and is reported to be positive. His disease is classified as nonradiographic axial spondyloarthropathy.

He is started on regular naproxen and is referred to a physiotherapist. After 1 month, he reports significant symptomatic improvement. He asks if he can be retested for HLA-B27 to see if it has become negative. We tell him that there is no point in repeating it, as it is a gene and will not disappear.

SUMMARY: CONSIDER THE CLINICAL PICTURE

When approaching a patient suspected of having a rheumatologic disease, a clinician should first consider the clinical presentation and the intended purpose of each test. The tests, in general, might serve several purposes. They might help to:

Increase the likelihood of the diagnosis in question. For example, a positive rheumatoid factor or anticitrullinated peptide antibody can help diagnose rheumatoid arthritis in a patient with early polyarthritis, a positive HLA-B27 can help diagnose ankylosing spondylitis in patients with inflammatory back pain and normal imaging, and a positive ANCA can help diagnose ANCA-associated vasculitis in a patient with glomerulonephritis.

Reduce the likelihood of the diagnosis in question. For example, a negative antinuclear antibody test reduces the likelihood of lupus in a patient with joint pains.

Monitor the condition. For example DNA antibodies can be used to monitor the activity of lupus.

Plan the treatment strategy. For example, one might consider lifelong anticoagulation if antiphospholipid antibodies are persistently positive in a patient with thrombosis.

Prognosticate. For example, positive rheumatoid factor and anticitrullinated peptide antibody increase the risk of erosive rheumatoid arthritis.

If the test was requested in the absence of a clear indication and the result is positive, it is important to bear in mind the potential pitfalls associated with that test and not attach a diagnostic label prematurely. None of the tests can confirm or exclude a condition, so the results should always be interpreted in the context of the whole clinical picture.   

References
  1. American College of Rheumatology Ad Hoc Committee on Immunologic Testing Guidelines. Guidelines for immunologic laboratory testing in the rheumatic diseases: an introduction. Arthritis Rheum 2002; 47(4):429–433. doi:10.1002/art.10381
  2. Rang M. The Ulysses syndrome. Can Med Assoc J 1972; 106(2):122–123. pmid:5058884
  3. Ingegnoli F, Castelli R, Gualtierotti R. Rheumatoid factors: clinical applications. Dis Markers 2013; 35(6):727–734. doi:10.1155/2013/726598
  4. Nishimura K, Sugiyama D, Kogata Y, et al. Meta-analysis: diagnostic accuracy of anti-cyclic citrullinated peptide antibody and rheumatoid factor for rheumatoid arthritis. Ann Intern Med 2007; 146(11):797–808. pmid:17548411
  5. Taylor P, Gartemann J, Hsieh J, Creeden J. A systematic review of serum biomarkers anti-cyclic citrullinated Peptide and rheumatoid factor as tests for rheumatoid arthritis. Autoimmune Dis 2011; 2011:815038. doi:10.4061/2011/815038
  6. Rantapää-Dahlqvist S, de Jong BA, Berglin E, et al. Antibodies against cyclic citrullinated peptide and IgA rheumatoid factor predict the development of rheumatoid arthritis. Arthritis Rheum 2003; 48(10):2741–2749. doi:10.1002/art.11223
  7. Suresh E. Diagnosis of early rheumatoid arthritis: what the non-specialist needs to know. J R Soc Med 2004; 97(9):421–424. doi:10.1258/jrsm.97.9.421
  8. Emery P, Breedveld FC, Dougados M, Kalden JR, Schiff MH, Smolen JS. Early referral recommendation for newly diagnosed rheumatoid arthritis: evidence based development of a clinical guide. Ann Rheum Dis 2002; 61(4):290–297. pmid:11874828
  9. Combe B, Landewe R, Daien CI, et al. 2016 update of the EULAR recommendations for the management of early arthritis. Ann Rheum Dis 2017; 76(6):948–959. doi:10.1136/annrheumdis-2016-210602
  10. Egsmose C, Lund B, Borg G, et al. Patients with rheumatoid arthritis benefit from early 2nd line therapy: 5 year follow up of a prospective double blind placebo controlled study. J Rheumatol 1995; 22(12):2208–2213. pmid:8835550
  11. van der Heide A, Jacobs JW, Bijlsma JW, et al. The effectiveness of early treatment with “second-line” antirheumatic drugs. A randomized, controlled trial. Ann Intern Med 1996; 124(8):699–707. pmid:8633829
  12. Andreson JJ, Wells G, Verhoeven AC, Felson DT. Factors predicting response to treatment in rheumatoid arthritis: the importance of disease duration. Arthritis Rheum 2000; 43(1):22–29. doi:10.1002/1529-0131(200001)43:1<22::AID-ANR4>3.0.CO;2-9
  13. Aletaha D, Neogi T, Silman AJ, et al. 2010 Rheumatoid arthritis classification criteria: an American College of Rheumatology/European League Against Rheumatism collaborative initiative. Arthritis Rheum 2010; 62(9):2569–2581. doi:10.1002/art.27584
  14. Nielen MM, van Schaardenburg D, Reesink HW, et al. Specific autoantibodies precede the symptoms of rheumatoid arthritis: a study of serial measurements in blood donors. Arthritis Rheum 2004; 50(2):380–386. doi:10.1002/art.20018
  15. del Puente A, Knowler WC, Pettitt DJ, Bennett PH. The incidence of rheumatoid arthritis is predicted by rheumatoid factor titer in a longitudinal population study. Arthritis Rheum 1988; 31(10):1239–1244. pmid:3178905
  16. Deane KD, Norris JM, Holers VM. Preclinical rheumatoid arthritis: identification, evaluation, and future directions for investigation. Rheum Dis Clin North Am 2010; 36(2):213–241. doi:10.1016/j.rdc.2010.02.001
  17. Kavanaugh A, Tomar R, Reveille J, Solomon DH, Homburger HA. Guidelines for clinical use of the antinuclear antibody test and tests for specific autoantibodies to nuclear antigens. American College of Pathologists. Arch Pathol Lab Med 2000; 124(1):71–81. doi:10.1043/0003-9985(2000)124<0071:GFCUOT>2.0.CO;2
  18. Suresh E. Systemic lupus erythematosus: diagnosis for the non-specialist. Br J Hosp Med (Lond) 2007; 68(10):538–541. doi:10.12968/hmed.2007.68.10.27324
  19. Illei GG, Klippel JH. Why is the ANA result positive? Bull Rheum Dis 1999; 48(1):1–4. pmid:10028188
  20. Tan EM, Feltkamp TE, Smolen JS, et al. Range of antinuclear antibodies in “healthy” individuals. Arthritis Rheum 1997; 40(9):1601–1611. doi:10.1002/art.1780400909
  21. Langkilde H, Voss A, Heegaard N, Laustrup H. Autoantibodies persist in relatives to systemic lupus erythematosus patients during 12 years follow-up. Lupus 2017; 26(7):723–728. doi:10.1177/0961203316676378
  22. Rondeel JM. Immunofluorescence versus ELISA for the detection of antinuclear antigens. Expert Rev Mol Diagn 2002; 2(3):226–232. doi:10.1586/14737159.2.3.226
  23. Solomon DH, Kavanaugh AJ, Schur PH; American College of Rheumatology Ad Hoc Committee on Immunologic Testing Guidelines. Evidence-based guidelines for the use of immunologic tests: antinuclear antibody testing. Arthritis Rheum 2002; 47(4):434–444. doi:10.1002/art.10561
  24. Slater CA, Davis RB, Shmerling RH. Antinuclear antibody testing. A study of clinical utility. Arch Intern Med 1996; 156(13):1421–1425. pmid:8678710
  25. Maddison PJ. Is it SLE? Best Pract Res Clin Rheumatol 2002; 16(2):167–180. doi:10.1053/berh.2001.0219
  26. Price E, Walker E. Diagnostic vertigo: the journey to diagnosis in systemic lupus erythematosus. Health (London) 2014; 18(3):223–239. doi:10.1177/1363459313488008
  27. Blumenthal DE. Tired, aching, ANA-positive: does your patient have lupus or fibromyalgia? Cleve Clin J Med 2002; 69(2):143–146, 151–152. pmid:11990644
  28. Miyakis S, Lockshin MD, Atsumi T, et al. International consensus statement on an update of the classification criteria for definite antiphospholipid syndrome (APS). J Thromb Haemost 2006; 4(2):295–306. doi:10.1111/j.1538-7836.2006.01753.x
  29. Keeling D, Mackie I, Moore GW, Greer IA, Greaves M; British Committee for Standards in Haematology. Guidelines on the investigation and management of antiphospholipid syndrome. Br J Haematol 2012; 157(1):47–58. doi:10.1111/j.1365-2141.2012.09037.x
  30. Giannakopoulos B, Passam F, Iannou Y, Krillis SA. How we diagnose the antiphospholipid syndrome. Blood 2009; 113(5):985–994. doi:10.1182/blood-2007-12-129627
  31. Biggioggero M, Meroni PL. The geoepidemiology of the antiphospholipid antibody syndrome. Autoimmun Rev 2010; 9(5):A299–A304. doi:10.1016/j.autrev.2009.11.013
  32. Pengo V, Ruffatti A, Legnani C, et al. Incidence of a first thromboembolic event in asymptomatic carriers of high-risk antiphospholipid antibody profile: a multicenter prospective study. Blood 2011; 118(17):4714–4718. doi:10.1182/blood-2011-03-340232
  33. Pengo V, Ruffatti A, Legnani C, et al. Clinical course of high-risk patients diagnosed with antiphospholipid syndrome. J Thromb Haemost 2010; 8(2):237–242. doi:10.1111/j.1538-7836.2009.03674.x
  34. Galli M, Luciani D, Bertolini G, Barbui T. Lupus anticoagulants are stronger risk factors for thrombosis than anticardiolipin antibodies in the antiphospholipid syndrome: a systematic review of the literature. Blood 2003; 101(5):1827–1832. doi:10.1182/blood-2002-02-0441
  35. Garcia D, Erkan D. Diagnosis and management of the antiphospholipid syndrome. N Engl J Med 2018; 378(21):2010–2021. doi:10.1056/NEJMra1705454
  36. Garcia D, Akl EA, Carr R, Kearon C. Antiphospholipid antibodies and the risk of recurrence after a first episode of venous thromboembolism: a systematic review. Blood 2013; 122(5):817–824. doi:10.1182/blood-2013-04-496257
  37. Cervera R. Lessons from the “Euro-Phospholipid” project. Autoimmun Rev 2008; 7(3):174–178. doi:10.1016/j.autrev.2007.11.011
  38. Andreoli L, Chighizola CB, Banzato A, Pons-Estel GJ, Ramire de Jesus G, Erkan D. Estimated frequency of antiphospholipid antibodies in patients with pregnancy morbidity, stroke, myocardial infarction, and deep vein thrombosis: a critical review of the literature. Arthritis Care Res (Hoboken) 2013; 65(11):1869–1873. doi:10.1002/acr.22066
  39. Miller A, Chan M, Wiik A, Misbah SA, Luqmani RA. An approach to the diagnosis and management of systemic vasculitis. Clin Exp Immunol 2010; 160(2):143–160. doi:10.1111/j.1365-2249.2009.04078.x
  40. Cornec D, Cornec-Le-Gall E, Fervenza FC, Specks U. ANCA-associated vasculitis—clinical utility of using ANCA specificity to classify patients. Nat Rev Rheumatol 2016; 12(10):570–579. doi:10.1038/nrrheum.2016.123
  41. Edgar JD, McMillan SA, Bruce IN, Conlan SK. An audit of ANCA in routine clinical practice. Postgrad Med J 1995; 71(840):605–612. pmid:8545289
  42. McLaren JS, Stimson RH, McRorie ER, Coia JE, Luqmani RA. The diagnostic value of anti-neutrophil cytoplasmic testing in a routine clinical setting. QJM 2001; 94(11):615–621. pmid:11704691
  43. Mandl LA, Solomon DH, Smith EL, Lew RA, Katz JN, Shmerling RH. Using antineutrophil cytoplasmic antibody testing to diagnose vasculitis: can test-ordering guidelines improve diagnostic accuracy? Arch Intern Med 2002; 162(13):1509–1514. pmid:12090888
  44. Sinclair D, Saas M, Stevens JM. The effect of a symptom related “gated policy” on ANCA requests in routine clinical practice. J Clin Pathol 2004; 57(2):131–134. pmid:14747434
  45. Arnold DF, Timms A, Luqmani R, Misbah SA. Does a gating policy for ANCA overlook patients with ANCA associated vasculitis? An audit of 263 patients. J Clin Pathol 2010; 63(8):678–680. doi:10.1136/jcp.2009.072504
  46. Savige J, Gills D, Benson E, et al. International consensus statement on testing and reporting of antineutrophil cytoplasmic antibodies (ANCA). Am J Clin Pathol 1999; 111(4):507–513. pmid:10191771
  47. Robinson PC, Steele RH. Appropriateness of antineutrophil cytoplasmic antibody testing in a tertiary hospital. J Clin Pathol 2009; 62(8):743–745. doi:10.1136/jcp.2009.064485
  48. Bossuyt X, Cohen Tervaert JW, Arimura Y, et al. Position paper: revised 2017 international consensus on testing of ANCAs in granulomatosis with polyangiitis and microscopic polyangiitis. Nat Rev Rheumatol 2017; 13(11):683–692. doi:10.1038/nrrheum.2017.140
  49. Hagen EC, Daha MR, Hermans J, et al. Diagnostic value of standardized assays for anti-neutrophil cytoplasmic antibodies in idiopathic systemic vasculitis. EC/BCR Project for ANCA Assay Standardization. Kidney Int 1998; 53(3):743–753. doi:10.1046/j.1523-1755.1998.00807.x
  50. Damoiseaux J, Csemok E, Rasmussen N, et al. Detection of antineutrophil antibodies (ANCAs): a multicentre European Vasculitis Study Group (EUVAS) evaluation of the value of indirect immunofluorescence (IIF) versus antigen specific immunoassays. Ann Rheum Dis 2017; 76(4):647–653. doi:10.1136/annrheumdis-2016-209507
  51. Suresh E. Diagnostic approach to patients with suspected vasculitis. Postgrad Med J 2006; 82(970):483–488. doi:10.1136/pgmj.2005.042648
  52. Vermeersch P, Blockmans D, Bossuyt X. Use of likelihood ratios can improve the clinical usefulness of enzyme immunoassays for the diagnosis of small-vessel vasculitis. Clin Chem 2009; 55(10):1886–1888. doi:10.1373/clinchem.2009.130583
  53. Bowness P. HLA-B27. Annu Rev Immunol 2015; 33:29–48. doi:10.1146/annurev-immunol-032414-112110
  54. Sieper J, Poddubnyy D. Axial spondyloarthritis. Lancet 2017; 390(10089):73–84. doi:10.1016/S0140-6736(16)31591-4
  55. Khan MA. Thoughts concerning the early diagnosis of ankylosing spondylitis and related diseases. Clin Exp Rheumatol 2002; 20(6 suppl 28):S6–S10. pmid:12463439
  56. Braun J, Bollow M, Remlinger G, et al. Prevalence of spondyloarthropathies in HLA-B27 positive and negative blood donors. Arthritis Rheum 1998; 41(1):58–67. doi:10.1002/1529-0131(199801)41:1<58::AID-ART8>3.0.CO;2-G
  57. van der Linden SM, Valkenburg HA, de Jongh BM, Cats A. The risk of developing ankylosing spondylitis in HLA-B27 positive individuals. A comparison of relatives of spondylitis patients with the general population. Arthritis Rheum 1984; 27(3):241–249. pmid:6608352
  58. Sheehan NJ. HLA-B27: what’s new? Rheumatology (Oxford) 2010; 49(4):621–631. doi:10.1093/rheumatology/kep450
  59. Baraliakos X, Maksymmowych WP. Imaging in the diagnosis and management of axial spondyloarthritis. Best Pract Res Clin Rheumatol 2016; 30(4):608–623. doi:10.1016/j.berh.2016.09.011
  60. Mandl P, Navarro-Compan V, Terslev L, et al; European League Against Rheumatism (EULAR). EULAR recommendations for the use of imaging in the diagnosis and management of spondyloarthritis in clinical practice. Ann Rheum Dis 2015; 74(7):1327–1339. doi:10.1136/annrheumdis-2014-206971
  61. McAllister K, Goodson N, Warburton I, Rogers G. Spondyloarthritis: diagnosis and management: summary of NICE guidance. BMJ 2017; 356:j839. doi:10.1136/bmj.j839
  62. Poddubnyy D, van Tubergen A, Landewé R, Sieper J, van der Heijde D; Assessment of SpondyloArthritis international Society (ASAS). Development of an ASAS-endorsed recommendation for the early referral of patients with a suspicion of axial spondyloarthritis. Ann Rheum Dis 2015; 74(8):1483–1487. doi:10.1136/annrheumdis-2014-207151
  63. Rudwaleit M, van der Heijde D, Landewe R, et al. The development of Assessment of SpondyloArthritis International Society classification criteria for axial spondyloarthritis (part II): validation and final selection. Ann Rheum Dis 2009; 68(6):777–783. doi:10.1136/ard.2009.108233
References
  1. American College of Rheumatology Ad Hoc Committee on Immunologic Testing Guidelines. Guidelines for immunologic laboratory testing in the rheumatic diseases: an introduction. Arthritis Rheum 2002; 47(4):429–433. doi:10.1002/art.10381
  2. Rang M. The Ulysses syndrome. Can Med Assoc J 1972; 106(2):122–123. pmid:5058884
  3. Ingegnoli F, Castelli R, Gualtierotti R. Rheumatoid factors: clinical applications. Dis Markers 2013; 35(6):727–734. doi:10.1155/2013/726598
  4. Nishimura K, Sugiyama D, Kogata Y, et al. Meta-analysis: diagnostic accuracy of anti-cyclic citrullinated peptide antibody and rheumatoid factor for rheumatoid arthritis. Ann Intern Med 2007; 146(11):797–808. pmid:17548411
  5. Taylor P, Gartemann J, Hsieh J, Creeden J. A systematic review of serum biomarkers anti-cyclic citrullinated Peptide and rheumatoid factor as tests for rheumatoid arthritis. Autoimmune Dis 2011; 2011:815038. doi:10.4061/2011/815038
  6. Rantapää-Dahlqvist S, de Jong BA, Berglin E, et al. Antibodies against cyclic citrullinated peptide and IgA rheumatoid factor predict the development of rheumatoid arthritis. Arthritis Rheum 2003; 48(10):2741–2749. doi:10.1002/art.11223
  7. Suresh E. Diagnosis of early rheumatoid arthritis: what the non-specialist needs to know. J R Soc Med 2004; 97(9):421–424. doi:10.1258/jrsm.97.9.421
  8. Emery P, Breedveld FC, Dougados M, Kalden JR, Schiff MH, Smolen JS. Early referral recommendation for newly diagnosed rheumatoid arthritis: evidence based development of a clinical guide. Ann Rheum Dis 2002; 61(4):290–297. pmid:11874828
  9. Combe B, Landewe R, Daien CI, et al. 2016 update of the EULAR recommendations for the management of early arthritis. Ann Rheum Dis 2017; 76(6):948–959. doi:10.1136/annrheumdis-2016-210602
  10. Egsmose C, Lund B, Borg G, et al. Patients with rheumatoid arthritis benefit from early 2nd line therapy: 5 year follow up of a prospective double blind placebo controlled study. J Rheumatol 1995; 22(12):2208–2213. pmid:8835550
  11. van der Heide A, Jacobs JW, Bijlsma JW, et al. The effectiveness of early treatment with “second-line” antirheumatic drugs. A randomized, controlled trial. Ann Intern Med 1996; 124(8):699–707. pmid:8633829
  12. Andreson JJ, Wells G, Verhoeven AC, Felson DT. Factors predicting response to treatment in rheumatoid arthritis: the importance of disease duration. Arthritis Rheum 2000; 43(1):22–29. doi:10.1002/1529-0131(200001)43:1<22::AID-ANR4>3.0.CO;2-9
  13. Aletaha D, Neogi T, Silman AJ, et al. 2010 Rheumatoid arthritis classification criteria: an American College of Rheumatology/European League Against Rheumatism collaborative initiative. Arthritis Rheum 2010; 62(9):2569–2581. doi:10.1002/art.27584
  14. Nielen MM, van Schaardenburg D, Reesink HW, et al. Specific autoantibodies precede the symptoms of rheumatoid arthritis: a study of serial measurements in blood donors. Arthritis Rheum 2004; 50(2):380–386. doi:10.1002/art.20018
  15. del Puente A, Knowler WC, Pettitt DJ, Bennett PH. The incidence of rheumatoid arthritis is predicted by rheumatoid factor titer in a longitudinal population study. Arthritis Rheum 1988; 31(10):1239–1244. pmid:3178905
  16. Deane KD, Norris JM, Holers VM. Preclinical rheumatoid arthritis: identification, evaluation, and future directions for investigation. Rheum Dis Clin North Am 2010; 36(2):213–241. doi:10.1016/j.rdc.2010.02.001
  17. Kavanaugh A, Tomar R, Reveille J, Solomon DH, Homburger HA. Guidelines for clinical use of the antinuclear antibody test and tests for specific autoantibodies to nuclear antigens. American College of Pathologists. Arch Pathol Lab Med 2000; 124(1):71–81. doi:10.1043/0003-9985(2000)124<0071:GFCUOT>2.0.CO;2
  18. Suresh E. Systemic lupus erythematosus: diagnosis for the non-specialist. Br J Hosp Med (Lond) 2007; 68(10):538–541. doi:10.12968/hmed.2007.68.10.27324
  19. Illei GG, Klippel JH. Why is the ANA result positive? Bull Rheum Dis 1999; 48(1):1–4. pmid:10028188
  20. Tan EM, Feltkamp TE, Smolen JS, et al. Range of antinuclear antibodies in “healthy” individuals. Arthritis Rheum 1997; 40(9):1601–1611. doi:10.1002/art.1780400909
  21. Langkilde H, Voss A, Heegaard N, Laustrup H. Autoantibodies persist in relatives to systemic lupus erythematosus patients during 12 years follow-up. Lupus 2017; 26(7):723–728. doi:10.1177/0961203316676378
  22. Rondeel JM. Immunofluorescence versus ELISA for the detection of antinuclear antigens. Expert Rev Mol Diagn 2002; 2(3):226–232. doi:10.1586/14737159.2.3.226
  23. Solomon DH, Kavanaugh AJ, Schur PH; American College of Rheumatology Ad Hoc Committee on Immunologic Testing Guidelines. Evidence-based guidelines for the use of immunologic tests: antinuclear antibody testing. Arthritis Rheum 2002; 47(4):434–444. doi:10.1002/art.10561
  24. Slater CA, Davis RB, Shmerling RH. Antinuclear antibody testing. A study of clinical utility. Arch Intern Med 1996; 156(13):1421–1425. pmid:8678710
  25. Maddison PJ. Is it SLE? Best Pract Res Clin Rheumatol 2002; 16(2):167–180. doi:10.1053/berh.2001.0219
  26. Price E, Walker E. Diagnostic vertigo: the journey to diagnosis in systemic lupus erythematosus. Health (London) 2014; 18(3):223–239. doi:10.1177/1363459313488008
  27. Blumenthal DE. Tired, aching, ANA-positive: does your patient have lupus or fibromyalgia? Cleve Clin J Med 2002; 69(2):143–146, 151–152. pmid:11990644
  28. Miyakis S, Lockshin MD, Atsumi T, et al. International consensus statement on an update of the classification criteria for definite antiphospholipid syndrome (APS). J Thromb Haemost 2006; 4(2):295–306. doi:10.1111/j.1538-7836.2006.01753.x
  29. Keeling D, Mackie I, Moore GW, Greer IA, Greaves M; British Committee for Standards in Haematology. Guidelines on the investigation and management of antiphospholipid syndrome. Br J Haematol 2012; 157(1):47–58. doi:10.1111/j.1365-2141.2012.09037.x
  30. Giannakopoulos B, Passam F, Iannou Y, Krillis SA. How we diagnose the antiphospholipid syndrome. Blood 2009; 113(5):985–994. doi:10.1182/blood-2007-12-129627
  31. Biggioggero M, Meroni PL. The geoepidemiology of the antiphospholipid antibody syndrome. Autoimmun Rev 2010; 9(5):A299–A304. doi:10.1016/j.autrev.2009.11.013
  32. Pengo V, Ruffatti A, Legnani C, et al. Incidence of a first thromboembolic event in asymptomatic carriers of high-risk antiphospholipid antibody profile: a multicenter prospective study. Blood 2011; 118(17):4714–4718. doi:10.1182/blood-2011-03-340232
  33. Pengo V, Ruffatti A, Legnani C, et al. Clinical course of high-risk patients diagnosed with antiphospholipid syndrome. J Thromb Haemost 2010; 8(2):237–242. doi:10.1111/j.1538-7836.2009.03674.x
  34. Galli M, Luciani D, Bertolini G, Barbui T. Lupus anticoagulants are stronger risk factors for thrombosis than anticardiolipin antibodies in the antiphospholipid syndrome: a systematic review of the literature. Blood 2003; 101(5):1827–1832. doi:10.1182/blood-2002-02-0441
  35. Garcia D, Erkan D. Diagnosis and management of the antiphospholipid syndrome. N Engl J Med 2018; 378(21):2010–2021. doi:10.1056/NEJMra1705454
  36. Garcia D, Akl EA, Carr R, Kearon C. Antiphospholipid antibodies and the risk of recurrence after a first episode of venous thromboembolism: a systematic review. Blood 2013; 122(5):817–824. doi:10.1182/blood-2013-04-496257
  37. Cervera R. Lessons from the “Euro-Phospholipid” project. Autoimmun Rev 2008; 7(3):174–178. doi:10.1016/j.autrev.2007.11.011
  38. Andreoli L, Chighizola CB, Banzato A, Pons-Estel GJ, Ramire de Jesus G, Erkan D. Estimated frequency of antiphospholipid antibodies in patients with pregnancy morbidity, stroke, myocardial infarction, and deep vein thrombosis: a critical review of the literature. Arthritis Care Res (Hoboken) 2013; 65(11):1869–1873. doi:10.1002/acr.22066
  39. Miller A, Chan M, Wiik A, Misbah SA, Luqmani RA. An approach to the diagnosis and management of systemic vasculitis. Clin Exp Immunol 2010; 160(2):143–160. doi:10.1111/j.1365-2249.2009.04078.x
  40. Cornec D, Cornec-Le-Gall E, Fervenza FC, Specks U. ANCA-associated vasculitis—clinical utility of using ANCA specificity to classify patients. Nat Rev Rheumatol 2016; 12(10):570–579. doi:10.1038/nrrheum.2016.123
  41. Edgar JD, McMillan SA, Bruce IN, Conlan SK. An audit of ANCA in routine clinical practice. Postgrad Med J 1995; 71(840):605–612. pmid:8545289
  42. McLaren JS, Stimson RH, McRorie ER, Coia JE, Luqmani RA. The diagnostic value of anti-neutrophil cytoplasmic testing in a routine clinical setting. QJM 2001; 94(11):615–621. pmid:11704691
  43. Mandl LA, Solomon DH, Smith EL, Lew RA, Katz JN, Shmerling RH. Using antineutrophil cytoplasmic antibody testing to diagnose vasculitis: can test-ordering guidelines improve diagnostic accuracy? Arch Intern Med 2002; 162(13):1509–1514. pmid:12090888
  44. Sinclair D, Saas M, Stevens JM. The effect of a symptom related “gated policy” on ANCA requests in routine clinical practice. J Clin Pathol 2004; 57(2):131–134. pmid:14747434
  45. Arnold DF, Timms A, Luqmani R, Misbah SA. Does a gating policy for ANCA overlook patients with ANCA associated vasculitis? An audit of 263 patients. J Clin Pathol 2010; 63(8):678–680. doi:10.1136/jcp.2009.072504
  46. Savige J, Gills D, Benson E, et al. International consensus statement on testing and reporting of antineutrophil cytoplasmic antibodies (ANCA). Am J Clin Pathol 1999; 111(4):507–513. pmid:10191771
  47. Robinson PC, Steele RH. Appropriateness of antineutrophil cytoplasmic antibody testing in a tertiary hospital. J Clin Pathol 2009; 62(8):743–745. doi:10.1136/jcp.2009.064485
  48. Bossuyt X, Cohen Tervaert JW, Arimura Y, et al. Position paper: revised 2017 international consensus on testing of ANCAs in granulomatosis with polyangiitis and microscopic polyangiitis. Nat Rev Rheumatol 2017; 13(11):683–692. doi:10.1038/nrrheum.2017.140
  49. Hagen EC, Daha MR, Hermans J, et al. Diagnostic value of standardized assays for anti-neutrophil cytoplasmic antibodies in idiopathic systemic vasculitis. EC/BCR Project for ANCA Assay Standardization. Kidney Int 1998; 53(3):743–753. doi:10.1046/j.1523-1755.1998.00807.x
  50. Damoiseaux J, Csemok E, Rasmussen N, et al. Detection of antineutrophil antibodies (ANCAs): a multicentre European Vasculitis Study Group (EUVAS) evaluation of the value of indirect immunofluorescence (IIF) versus antigen specific immunoassays. Ann Rheum Dis 2017; 76(4):647–653. doi:10.1136/annrheumdis-2016-209507
  51. Suresh E. Diagnostic approach to patients with suspected vasculitis. Postgrad Med J 2006; 82(970):483–488. doi:10.1136/pgmj.2005.042648
  52. Vermeersch P, Blockmans D, Bossuyt X. Use of likelihood ratios can improve the clinical usefulness of enzyme immunoassays for the diagnosis of small-vessel vasculitis. Clin Chem 2009; 55(10):1886–1888. doi:10.1373/clinchem.2009.130583
  53. Bowness P. HLA-B27. Annu Rev Immunol 2015; 33:29–48. doi:10.1146/annurev-immunol-032414-112110
  54. Sieper J, Poddubnyy D. Axial spondyloarthritis. Lancet 2017; 390(10089):73–84. doi:10.1016/S0140-6736(16)31591-4
  55. Khan MA. Thoughts concerning the early diagnosis of ankylosing spondylitis and related diseases. Clin Exp Rheumatol 2002; 20(6 suppl 28):S6–S10. pmid:12463439
  56. Braun J, Bollow M, Remlinger G, et al. Prevalence of spondyloarthropathies in HLA-B27 positive and negative blood donors. Arthritis Rheum 1998; 41(1):58–67. doi:10.1002/1529-0131(199801)41:1<58::AID-ART8>3.0.CO;2-G
  57. van der Linden SM, Valkenburg HA, de Jongh BM, Cats A. The risk of developing ankylosing spondylitis in HLA-B27 positive individuals. A comparison of relatives of spondylitis patients with the general population. Arthritis Rheum 1984; 27(3):241–249. pmid:6608352
  58. Sheehan NJ. HLA-B27: what’s new? Rheumatology (Oxford) 2010; 49(4):621–631. doi:10.1093/rheumatology/kep450
  59. Baraliakos X, Maksymmowych WP. Imaging in the diagnosis and management of axial spondyloarthritis. Best Pract Res Clin Rheumatol 2016; 30(4):608–623. doi:10.1016/j.berh.2016.09.011
  60. Mandl P, Navarro-Compan V, Terslev L, et al; European League Against Rheumatism (EULAR). EULAR recommendations for the use of imaging in the diagnosis and management of spondyloarthritis in clinical practice. Ann Rheum Dis 2015; 74(7):1327–1339. doi:10.1136/annrheumdis-2014-206971
  61. McAllister K, Goodson N, Warburton I, Rogers G. Spondyloarthritis: diagnosis and management: summary of NICE guidance. BMJ 2017; 356:j839. doi:10.1136/bmj.j839
  62. Poddubnyy D, van Tubergen A, Landewé R, Sieper J, van der Heijde D; Assessment of SpondyloArthritis international Society (ASAS). Development of an ASAS-endorsed recommendation for the early referral of patients with a suspicion of axial spondyloarthritis. Ann Rheum Dis 2015; 74(8):1483–1487. doi:10.1136/annrheumdis-2014-207151
  63. Rudwaleit M, van der Heijde D, Landewe R, et al. The development of Assessment of SpondyloArthritis International Society classification criteria for axial spondyloarthritis (part II): validation and final selection. Ann Rheum Dis 2009; 68(6):777–783. doi:10.1136/ard.2009.108233
Issue
Cleveland Clinic Journal of Medicine - 86(3)
Issue
Cleveland Clinic Journal of Medicine - 86(3)
Page Number
198-210
Page Number
198-210
Publications
Publications
Topics
Article Type
Display Headline
Laboratory tests in rheumatology: A rational approach
Display Headline
Laboratory tests in rheumatology: A rational approach
Legacy Keywords
rheumatology, tests, rheumatoid factor, rheumatoid arthritis, polyarthritis, anticitrullinated peptide antibody, antinuclear antibody, antiphospholipid antibodies, antineutrophil cytoplasmic antibody, ANCA, ANA, human leukocyte antigen-B27, HLA-B27, ankylosing spondylitis, systemic lupus erythematosus, SLE, anticardiolipin antibodies, lupus anticoagulant, beta-2 glycoprotein I antibody, anti-beta-2GPI, Ernest Suresh
Legacy Keywords
rheumatology, tests, rheumatoid factor, rheumatoid arthritis, polyarthritis, anticitrullinated peptide antibody, antinuclear antibody, antiphospholipid antibodies, antineutrophil cytoplasmic antibody, ANCA, ANA, human leukocyte antigen-B27, HLA-B27, ankylosing spondylitis, systemic lupus erythematosus, SLE, anticardiolipin antibodies, lupus anticoagulant, beta-2 glycoprotein I antibody, anti-beta-2GPI, Ernest Suresh
Sections
Inside the Article

KEY POINTS

  • If a test was requested without a clear indication and the result is positive, it is important to bear in mind the potential pitfalls associated with that test; immunologic tests have limited specificity.
  • A positive rheumatoid factor or anticitrullinated peptide antibody test can help diagnose rheumatoid arthritis in a patient with early polyarthritis.
  • A positive HLA-B27 test can help diagnose ankylosing spondylitis in patients with inflammatory back pain and normal imaging.
  • Positive antinuclear cytoplasmic antibody (ANCA) can help diagnose ANCA-associated vasculitis in a patient with glomerulonephritis.
  • A negative antinuclear antibody test reduces the likelihood of lupus in a patient with joint pain.
Disallow All Ads
Content Gating
No Gating (article Unlocked/Free)
Alternative CME
Disqus Comments
Default
Gate On Date
Thu, 02/28/2019 - 08:30
Un-Gate On Date
Thu, 02/28/2019 - 08:30
Use ProPublica
CFC Schedule Remove Status
Thu, 02/28/2019 - 08:30
Hide sidebar & use full width
render the right sidebar.
Article PDF Media

Bicuspid aortic valve: Basics and beyond

Article Type
Changed
Tue, 10/01/2019 - 08:32
Display Headline
Bicuspid aortic valve: Basics and beyond

Bicuspid aortic valve may initially be asymptomatic, but it is associated with progressive valvular and aortic abnormalities that can lead to chronic heart failure and sudden death. Regular monitoring is required with an eye toward surgery when indicated.

See related editorial

This article reviews inheritance patterns and conditions associated with bicuspid aortic valve. We discuss diagnosis, management, and monitoring, and offer surgical recommendations. Special guidance for dental procedures, pregnancy, and athletes is also provided.

A YOUNG MAN WITH PALPITATIONS AND A MURMUR

A 34-year-old man presented to an outpatient clinic with occasional palpitations over the past several months. He reported that he had been diagnosed with a murmur as a child but had received no further testing.

Physical examination at this time revealed a faint systolic crescendo-decrescendo murmur along the right sternal border without radiation to the carotid arteries or to the apex. Transthoracic echocardiography (TTE) showed a bicuspid aortic valve with fusion of the right and left coronary cusps, with no aortic valve stenosis or insufficiency. There was mild dilation of the aortic root, but the mid-ascending aorta could not be evaluated because of limited acoustic windows.

Is further diagnostic testing needed, and if so, what? May he participate in exertional physical activity? Does his newborn son need evaluation?

ABNORMALITIES OCCUR DURING EMBRYOGENESIS

Bicuspid aortic valve develops because of abnormal valvulogenesis. Adjacent cusps fail to separate from each other, resulting in only 2 cusps, with 1 usually larger than the other. Morphology varies according to which commissures are fused.1

Bicuspid aortic valve is associated with abnormalities in the coronary artery anatomy in about 2% of patients, including anomalous origins of the coronary arteries and upwardly displaced coronary ostia.2 Such features need to be considered before surgical intervention.

Bicuspid aortic valve can be found in 1% to 2% of the general population, with a male-to-female predominance of 3:1.1,3,4 It is one of the most common congenital cardiac malformations and is the leading congenital cause of aortic valve stenosis.1,3 However, routine screening of newborns for the condition is not recommended, and most cases are diagnosed incidentally.

GENETIC FACTORS PROMINENT

Bicuspid aortic valve is thought to be primarily inherited in an autosomal-dominant pattern, but there is evidence of genetic heterogeneity, and the pattern may be variable.5,6

No single gene responsible for bicuspid aortic valve has been identified. The condition may occur as a component of different pleiotropic genetic syndromes such as Loeys-Dietz, DiGeorge, and Marfan syndromes,7,8 as well as in patients with Turner syndrome and Williams syndrome.8–11 It also commonly coexists with other congenital heart diseases, including ventricular septal defect, isolated aortic arch obstruction, and patent ductus arteriosus.9

Studies have found a 15% rate of familial clustering.6,12 In a study of 142 patients with bicuspid aortic valve, 20% of first-degree relatives had some cardiac abnormality found by screening, of whom 68% had bicuspid aortic valve. Of these, 71% were newly detected abnormalities.13

CHARACTERISTIC CLICK AND MURMUR

Physical examination findings of a functionally normal bicuspid aortic valve include a systolic ejection click followed by an early peaking systolic murmur at the apex or left lower sternal border. With progression of aortic stenosis, the ejection murmur has a harsher sound, with later peaking, and the S2 sound diminishes or becomes inaudible.14 If aortic regurgitation is present, a diastolic decrescendo murmur is heard best at the left lower sternal border.

 

 

DISEASE PROGRESSION

Although bicuspid aortic valve is typically asymp­tomatic at first, it is commonly associated with progressive valvulopathy and thoracic aortic disease.1,3,4,15 It can lead to chronic heart failure and increase the risk of acute aortic syndromes and sudden cardiac death.15

Michelena et al16 studied 212 cases of asymp­tomatic bicuspid aortic valve. Although the survival rate 20 years after diagnosis was the same as for an age-matched cohort in the general population, the frequency of adverse cardiovascular events and surgical interventions was higher.

Aortic stenosis progresses rapidly

Aortic stenosis associated with a bicuspid aortic valve tends to affect younger patients and progress more rapidly than when associated with a tricuspid valve.17

In a study of 542 patients with congenital bicuspid aortic valve undergoing aortic valve replacement,3 75% had isolated aortic stenosis, 10% had aortic stenosis with some degree of aortic insufficiency, and 13% had isolated aortic insufficiency. Given the tendency of aortic stenosis to progress rapidly, early surgery is often pursued.17,18

Aneurysmal disease is common

The thoracic aorta is at increased risk of aneurysmal disease, coarctation, and dissection in patients with a bicuspid aortic valve.1,6,15

Michelena et al16 reported that in patients without an aneurysm at the time of bicuspid aortic valve diagnosis, the 25-year risk of aneurysm formation was approximately 26%. In patients with an aneurysm at the time of diagnosis, the 15-year risk of aortic surgery after the diagnosis of aneurysm was about 46% and the risk of aortic dissection after aneurysm diagnosis was 7%.15 Compared with the general population, the age-adjusted relative risk of aortic aneurysm in patients with bicuspid aortic valve was 86.2, and that of aortic dissection was 8.4. Although the absolute incidence of dissection is low in these patients, it is markedly higher than in the general population, particularly in older patients (age > 50) and those with an aneurysm at the time of diagnosis.15

The risk of infective endocarditis

Patients with bicuspid aortic valve are highly prone to infective endocarditis for reasons that remain poorly understood. The pathogens in most cases are staphylococci or viridans streptococci.19 Patients with infective endocarditis typically require emergency surgery. Complications including valvular abscess, myocardial abscess, and overt heart failure are common.19

Lamas and Eykyn20 studied 408 cases of native valve endocarditis; in 12.3%, the patient had a bicuspid aortic valve. In this subset, all were male, the mean age was 39 at diagnosis, 82% needed surgery, and the death rate was 14%.

Patients with bicuspid aortic valve do not routinely need antibiotics before dental and surgical procedures, but if they have had endocarditis in the past, they need antibiotics to prevent a recurrence.21

REGULAR MONITORING NEEDED

Because complications may be life-threatening, early detection of progressive disease by regular screening is critical. Echocardiographic evaluation of valvular function, ventricular dimensions and function, and diameter of the aortic root and ascending aorta should be performed in every patient with bicuspid aortic valve. If initial imaging is normal and there is no aortic dilation, imaging should be repeated every 5 to 10 years. If any abnormality is found, repeat imaging is needed every year.22

shah_bicuspidaorticvalve_f1.jpg
%3Cp%3EFigure%201.%20Transthoracic%20echocardiography%2C%20parasternal%20long-axis%20view%2C%20shows%20a%20dilated%20aortic%20root%20(AR)%20measuring%204.5%20cm%20in%20a%20patient%20with%20bicuspid%20aortic%20valve%20(arrows).%20AA%2C%20ascending%20aorta%3B%20LA%2C%20left%20atrium%3B%20LV%2C%20left%20ventricle.%3C%2Fp%3E

shah_bicuspidaorticvalve_f2.jpg
%3Cp%3EFigure%202.%20Two-dimensional%20transesophageal%20echocardiography%20demonstrates%20an%20anterior-posterior%20bicuspid%20aortic%20valve%20(opening%20phase%2C%20arrows).%20RA%2C%20right%20atrium%3B%20LA%2C%20left%20atrium%3B%20RV%2C%20right%20ventricle.%3C%2Fp%3E

Magnetic resonance imaging (MRI) or computed tomographic (CT) angiography may be required to better assess the aorta for patients requiring a surgical intervention, or when aortic dimensions are not clearly visualized on TTE. MRI has 2 advantages over CT angiography: it poses no radiation risk, and it provides more information on left ventricular function and dimensions, in addition to valve assessment.23,24

No published study has compared MRI or CT angiography and transesophageal echocardiography (TEE), but in a study of 174 patients with dilated aortic root, combined TTE and TEE detected aortic valve morphology accurately in 98% of cases. As TEE is more invasive, it is not recommended for regular surveillance (Figures 1 and 2).25

FAMILY SCREENING RECOMMENDED

Close relatives should be evaluated for aortic valve and thoracic aortic disease.12,13,23,26

The American College of Cardiology (ACC) and the American Heart Association (AHA), backed by radiologic and cardiovascular associations, concur in recommending echocardiographic screening and routine screening of the thoracic aorta for aortic root dilation in first-degree relatives (ie, siblings, parents, and children) of patients with bicuspid aortic valve (class I recommendation).22,27,28

A comprehensive physical examination is recommended for family members in addition to TTE, with careful assessment of the aortic valve in short and long axes, and of the aortic root.14 If the aorta cannot be adequately evaluated with TTE, further assessment should be pursued with CT angiography or MRI.

 

 

EXERCISE RESTRICTIONS

The 2015 ACC/AHA guidelines for competitive athletes with cardiovascular abnormalities recommend annual screening with TTE or MRI angiography for athletes with bicuspid aortic valve and coexisting dilation of the ascending aorta (aortic diameter 40–42 mm in men and 36–39 mm in women) (class I recommendation, level of evidence C).29

Athletes with a bicuspid aortic valve and a normal aortic root and ascending aorta may participate in all competitive activities.29 However, those with a dilated aorta should avoid strenuous activities because of the increased risk of rupture.30 The ACC/AHA recommendations29 depend on the diameter of the ascending aorta and the nature of the sport:

  • For an aortic diameter 40 to 42 mm in men or 36 to 39 mm in women, and no features of connective tissue disease or familial thoracic ascending aortic syndrome, low- and moderate-intensity sports with a low likelihood of significant body contact may be considered; consider avoiding intense weight training (class IIb, level of evidence C)
  • For an aortic diameter 43 to 45 mm, low-intensity sports with a low likelihood of body contact may be considered (class IIb, level of evidence C)
  • For an aortic diameter greater than 43 mm in men or greater than 40 mm in women, sports involving body collision should be avoided (class III, level of evidence C)
  • For an aortic diameter greater than 45 mm, sports activities should be avoided (class III, level of evidence C).

PREGNANCY CONSIDERATIONS

Bicuspid aortic valve is associated with aortic dissection, mainly in the third trimester.31 Patients should ideally undergo echocardiographic screening before conception. The 2010 ACC/AHA guidelines for managing thoracic aortic disease recommend monthly or bimonthly echocardiography until delivery in pregnant women with a dilated thoracic aorta.22

Patients with bicuspid aortic valve and aortic root enlargement of more than 40 mm should have preconception counseling about surgery for aortic root replacement before becoming pregnant. If the diagnosis of enlarged aortic root is made during pregnancy, echocardiographic surveillance at 4- to 6-week intervals is indicated.32

SURGICAL MANAGEMENT

In the past, beta-blockers and angiotensin-converting enzyme inhibitors were recommended to minimize shear stress, with the goal of slowing progression of aortic dilation. However, evidence to support their use is inadequate.33,34

The only definitive treatment is surgery, with various procedures that lower the risk of death or dissection.24,35

The dimensions of the aortic root or ascending aorta should be examined vigilantly, according to the 2014 ACC/AHA guidelines27:

  • Repairing the aortic sinuses or replacing the ascending aorta is indicated if the diameter of the aortic sinuses or ascending aorta is greater than 5.5 cm (class I, level of evidence B)
  • Repairing the aortic sinuses or replacing the ascending aorta is reasonable if the diameter of the aortic sinuses or ascending aorta is greater than 5.0 cm and the patient has a risk factor for dissection such as a family history of aortic dissection or an increase in diameter of 0.5 cm or greater per year (class IIa, level of evidence C)
  • Replacement of the ascending aorta is reasonable if the diameter of the ascending aorta is greater than 4.5 cm and the patient is undergoing aortic valve surgery for severe aortic stenosis or regurgitation.

Valve repair or replacement

Aortic valve repair or replacement is sometimes done separately from aortic root repair.

The value of aortic valve repair is debatable, but a series of 728 patients at Cleveland Clinic showed a very low mortality rate (0.41%) and an annual reoperation rate of 2.6% during up to 15 years of follow-up.36

Aortic valve replacement is usually considered for patients with severe valve dysfunction, abnormal left ventricular dimensions, or symptoms. It is important to determine if the patient is a good surgical candidate and to refer early for surgical evaluation to avoid the higher risk of death associated with emergency surgery.36

Transcatheter aortic valve replacement has been studied in patients deemed to be at too high a risk for surgical replacement. Short- and intermediate-term outcomes have been good in these patients, but long-term data are lacking.37

Surveillance after surgery

The type of operation determines postoperative surveillance.

After isolated aortic valve repair or replacement, patients should continue with surveillance at least annually to monitor for progressive aortopathy, as they remain at increased risk of dissection or rupture after isolated valve surgery, especially if they had aortic insufficiency preoperatively.38

After definitive surgery with replacement or repair of the ascending aorta, no clear recommendations have been established for continued surveillance. However, it is reasonable to image these patients with either MRI or CT angiography 3 to 5 years after their surgery to monitor for anastomotic complications.

CASE QUESTIONS ANSWERED

Our patient should undergo repeat TTE in 1 year. He should also undergo CT angiography of the ascending aorta if it is not seen by TTE. He can participate in low-intensity sports but should avoid intense weight training. His parents, siblings, and children should be screened for bicuspid aortic valve or associated aortopathies.

References
  1. Roberts WC. The congenitally bicuspid aortic valve. A study of 85 autopsy cases. Am J Cardiol 1970; 26(1):72–83. pmid:5427836
  2. Michalowska IM, Hryniewiecki T, Kwiatek P, Stoklosa P, Swoboda-Rydz U, Szymanski P. Coronary artery variants and anomalies in patients with bicuspid aortic valve. J Thorac Imaging 2016; 31(3):156–162. doi:10.1097/RTI.0000000000000205
  3. Sabet HY, Edwards WD, Tazelaar HD, Daly RC. Congenitally bicuspid aortic valves: a surgical pathology study of 542 cases (1991 through 1996) and a literature review of 2,715 additional cases. Mayo Clin Proc 1999; 74(1):14–26. doi:10.4065/74.1.14
  4. Tutar E, Ekici F, Atalay S, Nacar N. The prevalence of bicuspid aortic valve in newborns by echocardiographic screening. Am Heart J 2005; 150(3):513–515. doi:10.1016/j.ahj.2004.10.036
  5. Benson DW. The genetics of congenital heart disease: a point in the revolution. Cardiol Clin 2002; 20(3):385–394. pmid:12371007
  6. Emanuel R, Withers R, O’Brien K, Ross P, Feizi O. Congenitally bicuspid aortic valves. Clinicogenetic study of 41 families. Br Heart J 1978; 40(12):1402–1407. pmid:737099
  7. Giusti B, Sticchi E, De Cario R, Magi A, Nistri S, Pepe G. Genetic bases of bicuspid aortic valve: the contribution of traditional and high-throughput sequencing approaches on research and diagnosis. Front Physiol 2017; 8:612. doi:10.3389/fphys.2017.00612
  8. Sachdev V, Matura LA, Sidenko S, et al. Aortic valve disease in Turner syndrome. J Am Coll Cardiol 2008; 51(19):1904–1909. doi:10.1016/j.jacc.2008.02.035
  9. Duran AC, Frescura C, Sans-Coma V, Angelini A, Basso C, Thiene G. Bicuspid aortic valves in hearts with other congenital heart disease. J Heart Valve Dis 1995; 4(6):581–590. pmid:8611973
  10. De Rubens Figueroa J, Rodríguez LM, Hach JL, Del Castillo Ruíz V, Martínez HO. Cardiovascular spectrum in Williams-Beuren syndrome: the Mexican experience in 40 patients. Tex Heart Inst J 2008; 35(3):279–285. pmid:18941598
  11. Yuan SM, Jing H. The bicuspid aortic valve and related disorders. Sao Paulo Med J 2010; 128(5):296–301. pmid:21181071
  12. Cripe L, Andelfinger G, Martin LJ, Shooner K, Benson DW. Bicuspid aortic valve is heritable. J Am Coll Cardiol 2004; 44(1):138–143. doi:10.1016/j.jacc.2004.03.050
  13. Kerstjens-Frederikse WS, Sarvaas GJ, Ruiter JS, et al. Left ventricular outflow tract obstruction: should cardiac screening be offered to first-degree relatives? Heart 2011; 97(15):1228–1232. doi:10.1136/hrt.2010.211433
  14. Siu SC, Silversides CK. Bicuspid aortic valve disease. J Am Coll Cardiol 2010; 55(25):2789–2800. doi:10.1016/j.jacc.2009.12.068
  15. Michelena HI, Khanna AD, Mahoney D, et al. Incidence of aortic complications in patients with bicuspid aortic valves. JAMA 2011; 306(10):1104–1112.
  16. Michelena HI, Desjardins VA, Avierinos JF, et al. Natural history of asymptomatic patients with normally functioning or minimally dysfunctional bicuspid aortic valve in the community. Circulation 2008; 117(21):2776–2784. doi:10.1161/CIRCULATIONAHA.107.740878
  17. Beppu S, Suzuki S, Matsuda H, Ohmori F, Nagata S, Miyatake K. Rapidity of progression of aortic stenosis in patients with congenital bicuspid aortic valves. Am J Cardiol 1993; 71(4):322–327. pmid:8427176
  18. Roberts WC, Ko JM. Frequency by decades of unicuspid, bicuspid, and tricuspid aortic valves in adults having isolated aortic valve replacement for aortic stenosis, with or without associated aortic regurgitation. Circulation 2005; 111(7):920–925. doi:10.1161/01.CIR.0000155623.48408.C5
  19. Yener N, Oktar GL, Erer D, Yardimci MM, Yener A. Bicuspid aortic valve. Ann Thorac Cardiovasc Surg 2002; 8(5):264–267. pmid:12472407
  20. Lamas CC, Eykyn SJ. Bicuspid aortic valve—a silent danger: analysis of 50 cases of infective endocarditis. Clin Infect Dis 2000; 30(2):336–341. doi:10.1086/313646
  21. Wilson W, Taubert KA, Gewitz M, et al. Prevention of infective endocarditis: guidelines from the American Heart Association. Circulation 2007; 116(15):1736–1754. doi:10.1161/CIRCULATIONAHA.106.183095
  22. Hiratzka L, Bakris G, Beckman JA, et al. 2010 ACCF/AHA/AATS/ACR/ASA/SCA/SCAI/SIR/STS/SVM guidelines for the diagnosis and management of patients with thoracic aortic disease. Circulation 2010; 121(13):e266–e369. doi:10.1161/CIR.0b013e3181d4739e
  23. Chun EJ, Choi SI, Lim C, et al. Aortic stenosis: evaluation with multidetector CT angiography and MR imaging. Korean J Radiol 2008; 9(5):439–448. doi:10.3348/kjr.2008.9.5.439
  24. Kiefer TL, Wang A, Hughes GC, Bashore TM. Management of patients with bicuspid aortic valve disease. Curr Treat Options Cardiovasc Med 2011; 13(6):489–505. doi:10.1007/s11936-011-0152-7
  25. Alegret JM, Palazon O, Duran I, Vernis JM. Aortic valve morphology definition with transthoracic combined with transesophageal echocardiography in a population with high prevalence of bicuspid aortic valve. Int J Cardiovasc Imaging 2005; 21(2-3):213–217. doi:10.1007/s10554-004-3901-9
  26. Biner S, Rafique AM, Ray I, Cuk O, Siegel RJ, Tolstrup K. Aortopathy is prevalent in relatives of bicuspid aortic valve patients. J Am Coll Cardiol 2009; 53(24):2288–2295. doi:10.1016/j.jacc.2009.03.027
  27. Nishimura RA, Otto CM, Bonow RO, et al. 2014 AHA/ACC guideline for the management of patients with valvular heart disease. J Thorac Cardiovasc Surg 2014; 148(1):e1-e132. doi:10.1016/j.jtcvs.2014.05.014
  28. Warnes CA, Williams RG, Bashore TM, et al. ACC/AHA 2008 guidelines for the management of adults with congenital heart disease. J Am Coll Cardiol 2008; 52(23):e143–e263. doi:10.1016/j.jacc.2008.10.001
  29. Braverman AC, Harris KM, Kovacs RJ, Maron BJ. Eligibility and disqualification recommendations for competitive athletes with cardiovascular abnormalities: Task Force 7: aortic diseases, including Marfan syndrome. Circulation 2015; 132(22):e303–e309. doi:10.1161/CIR.0000000000000243
  30. De Mozzi P, Longo UG, Galanti G, Maffulli N. Bicuspid aortic valve: a literature review and its impact on sport activity. Br Med Bull 2008; 85:63–85. doi:10.1093/bmb/ldn002
  31. Thorne SA. Pregnancy in heart disease. Heart 2004; 90(4):450–456. pmid:15020530
  32. Immer FF, Bansi AG, Immer-Bansi AS, et al. Aortic dissection in pregnancy: analysis of risk factors and outcome. Ann Thorac Surg 2003; 76(1):309–314. pmid:12842575
  33. Allen BD, Markl M, Barker AJ, et al. Influence of beta-blocker therapy on aortic blood flow in patients with bicuspid aortic valve. Int J Cardiovasc Imaging 2016; 32(4):621–628. doi:10.1007/s10554-015-0819-3
  34. Ohnemus D, Oster ME, Gatlin S, Jokhadar M, Mahle WT. The effect of angiotensin-converting enzyme inhibitors on the rate of ascending aorta dilation in patients with bicuspid aortic valve. Congenit Heart Dis 2015; 10(1):E1–E5. doi:10.1111/chd.12184
  35. Masri A, Kalahasti V, Alkharabsheh S, et al. Characteristics and long-term outcomes of contemporary patients with bicuspid aortic valves. J Thorac Cardiovasc Surg 2016; 151(6):1650–1659.e1. doi:10.1016/j.jtcvs.2015.12.019
  36. Svensson LG, Al Kindi AH, Vivacqua A, et al. Long-term durability of bicuspid aortic valve repair. Ann Thorac Surg 2014; 97(5):1539–1548. doi:10.1016/j.athoracsur.2013.11.036
  37. Mylotte D, Lefevre T, Sondergaard L, et al. Transcatheter aortic valve replacement in bicuspid aortic valve disease. J Am Coll Cardiol 2014; 64(22):2330–2339. doi:10.1016/j.jacc.2014.09.039
  38. Girdauskas E, Disha K, Raisin HH, Secknus MA, Borger MA, Kuntze T. Risk of late aortic events after an isolated aortic valve replacement for bicuspid aortic valve stenosis with concomitant ascending aortic dilation. Eur J Cardiothorac Surg 2012; 42(5):832–838. doi:10.1093/ejcts/ezs137
Article PDF
Author and Disclosure Information

Shailee Y. Shah, MD
Department of Internal Medicine, Cleveland Clinic

Andrew Higgins, MD
Heart and Vascular Institute, Cleveland Clinic

Milind Y. Desai, MD
Haslam Family Endowed Chair in Cardiovascular Medicine, Department of Cardiovascular Medicine, Heart and Vascular Institute, and Department of Diagnostic Radiology, Cleveland Clinic; Professor, Cleveland Clinic Lerner College of Medicine of Case Western Reserve University, Cleveland, OH

Address: Shailee Y. Shah, MD, Department of Internal Medicine, Cleveland Clinic, 9500 Euclid Avenue, Cleveland, OH 44195; shahs10@ccf.org

Issue
Cleveland Clinic Journal of Medicine - 85(10)
Publications
Topics
Page Number
779-784
Legacy Keywords
bicuspid aortic valve, BAV, aorta, aortopathy, echocardiography, stenosis, aneurysm, Loeys-Dietz syndrome, Marfan syndrome, DiGeorge syndrome, Turner syndrome, Williams syndrome, infective endocarditis, Shailee Shah, Andrew Higgins, Milind Desai
Sections
Author and Disclosure Information

Shailee Y. Shah, MD
Department of Internal Medicine, Cleveland Clinic

Andrew Higgins, MD
Heart and Vascular Institute, Cleveland Clinic

Milind Y. Desai, MD
Haslam Family Endowed Chair in Cardiovascular Medicine, Department of Cardiovascular Medicine, Heart and Vascular Institute, and Department of Diagnostic Radiology, Cleveland Clinic; Professor, Cleveland Clinic Lerner College of Medicine of Case Western Reserve University, Cleveland, OH

Address: Shailee Y. Shah, MD, Department of Internal Medicine, Cleveland Clinic, 9500 Euclid Avenue, Cleveland, OH 44195; shahs10@ccf.org

Author and Disclosure Information

Shailee Y. Shah, MD
Department of Internal Medicine, Cleveland Clinic

Andrew Higgins, MD
Heart and Vascular Institute, Cleveland Clinic

Milind Y. Desai, MD
Haslam Family Endowed Chair in Cardiovascular Medicine, Department of Cardiovascular Medicine, Heart and Vascular Institute, and Department of Diagnostic Radiology, Cleveland Clinic; Professor, Cleveland Clinic Lerner College of Medicine of Case Western Reserve University, Cleveland, OH

Address: Shailee Y. Shah, MD, Department of Internal Medicine, Cleveland Clinic, 9500 Euclid Avenue, Cleveland, OH 44195; shahs10@ccf.org

Article PDF
Article PDF
Related Articles

Bicuspid aortic valve may initially be asymptomatic, but it is associated with progressive valvular and aortic abnormalities that can lead to chronic heart failure and sudden death. Regular monitoring is required with an eye toward surgery when indicated.

See related editorial

This article reviews inheritance patterns and conditions associated with bicuspid aortic valve. We discuss diagnosis, management, and monitoring, and offer surgical recommendations. Special guidance for dental procedures, pregnancy, and athletes is also provided.

A YOUNG MAN WITH PALPITATIONS AND A MURMUR

A 34-year-old man presented to an outpatient clinic with occasional palpitations over the past several months. He reported that he had been diagnosed with a murmur as a child but had received no further testing.

Physical examination at this time revealed a faint systolic crescendo-decrescendo murmur along the right sternal border without radiation to the carotid arteries or to the apex. Transthoracic echocardiography (TTE) showed a bicuspid aortic valve with fusion of the right and left coronary cusps, with no aortic valve stenosis or insufficiency. There was mild dilation of the aortic root, but the mid-ascending aorta could not be evaluated because of limited acoustic windows.

Is further diagnostic testing needed, and if so, what? May he participate in exertional physical activity? Does his newborn son need evaluation?

ABNORMALITIES OCCUR DURING EMBRYOGENESIS

Bicuspid aortic valve develops because of abnormal valvulogenesis. Adjacent cusps fail to separate from each other, resulting in only 2 cusps, with 1 usually larger than the other. Morphology varies according to which commissures are fused.1

Bicuspid aortic valve is associated with abnormalities in the coronary artery anatomy in about 2% of patients, including anomalous origins of the coronary arteries and upwardly displaced coronary ostia.2 Such features need to be considered before surgical intervention.

Bicuspid aortic valve can be found in 1% to 2% of the general population, with a male-to-female predominance of 3:1.1,3,4 It is one of the most common congenital cardiac malformations and is the leading congenital cause of aortic valve stenosis.1,3 However, routine screening of newborns for the condition is not recommended, and most cases are diagnosed incidentally.

GENETIC FACTORS PROMINENT

Bicuspid aortic valve is thought to be primarily inherited in an autosomal-dominant pattern, but there is evidence of genetic heterogeneity, and the pattern may be variable.5,6

No single gene responsible for bicuspid aortic valve has been identified. The condition may occur as a component of different pleiotropic genetic syndromes such as Loeys-Dietz, DiGeorge, and Marfan syndromes,7,8 as well as in patients with Turner syndrome and Williams syndrome.8–11 It also commonly coexists with other congenital heart diseases, including ventricular septal defect, isolated aortic arch obstruction, and patent ductus arteriosus.9

Studies have found a 15% rate of familial clustering.6,12 In a study of 142 patients with bicuspid aortic valve, 20% of first-degree relatives had some cardiac abnormality found by screening, of whom 68% had bicuspid aortic valve. Of these, 71% were newly detected abnormalities.13

CHARACTERISTIC CLICK AND MURMUR

Physical examination findings of a functionally normal bicuspid aortic valve include a systolic ejection click followed by an early peaking systolic murmur at the apex or left lower sternal border. With progression of aortic stenosis, the ejection murmur has a harsher sound, with later peaking, and the S2 sound diminishes or becomes inaudible.14 If aortic regurgitation is present, a diastolic decrescendo murmur is heard best at the left lower sternal border.

 

 

DISEASE PROGRESSION

Although bicuspid aortic valve is typically asymp­tomatic at first, it is commonly associated with progressive valvulopathy and thoracic aortic disease.1,3,4,15 It can lead to chronic heart failure and increase the risk of acute aortic syndromes and sudden cardiac death.15

Michelena et al16 studied 212 cases of asymp­tomatic bicuspid aortic valve. Although the survival rate 20 years after diagnosis was the same as for an age-matched cohort in the general population, the frequency of adverse cardiovascular events and surgical interventions was higher.

Aortic stenosis progresses rapidly

Aortic stenosis associated with a bicuspid aortic valve tends to affect younger patients and progress more rapidly than when associated with a tricuspid valve.17

In a study of 542 patients with congenital bicuspid aortic valve undergoing aortic valve replacement,3 75% had isolated aortic stenosis, 10% had aortic stenosis with some degree of aortic insufficiency, and 13% had isolated aortic insufficiency. Given the tendency of aortic stenosis to progress rapidly, early surgery is often pursued.17,18

Aneurysmal disease is common

The thoracic aorta is at increased risk of aneurysmal disease, coarctation, and dissection in patients with a bicuspid aortic valve.1,6,15

Michelena et al16 reported that in patients without an aneurysm at the time of bicuspid aortic valve diagnosis, the 25-year risk of aneurysm formation was approximately 26%. In patients with an aneurysm at the time of diagnosis, the 15-year risk of aortic surgery after the diagnosis of aneurysm was about 46% and the risk of aortic dissection after aneurysm diagnosis was 7%.15 Compared with the general population, the age-adjusted relative risk of aortic aneurysm in patients with bicuspid aortic valve was 86.2, and that of aortic dissection was 8.4. Although the absolute incidence of dissection is low in these patients, it is markedly higher than in the general population, particularly in older patients (age > 50) and those with an aneurysm at the time of diagnosis.15

The risk of infective endocarditis

Patients with bicuspid aortic valve are highly prone to infective endocarditis for reasons that remain poorly understood. The pathogens in most cases are staphylococci or viridans streptococci.19 Patients with infective endocarditis typically require emergency surgery. Complications including valvular abscess, myocardial abscess, and overt heart failure are common.19

Lamas and Eykyn20 studied 408 cases of native valve endocarditis; in 12.3%, the patient had a bicuspid aortic valve. In this subset, all were male, the mean age was 39 at diagnosis, 82% needed surgery, and the death rate was 14%.

Patients with bicuspid aortic valve do not routinely need antibiotics before dental and surgical procedures, but if they have had endocarditis in the past, they need antibiotics to prevent a recurrence.21

REGULAR MONITORING NEEDED

Because complications may be life-threatening, early detection of progressive disease by regular screening is critical. Echocardiographic evaluation of valvular function, ventricular dimensions and function, and diameter of the aortic root and ascending aorta should be performed in every patient with bicuspid aortic valve. If initial imaging is normal and there is no aortic dilation, imaging should be repeated every 5 to 10 years. If any abnormality is found, repeat imaging is needed every year.22

shah_bicuspidaorticvalve_f1.jpg
%3Cp%3EFigure%201.%20Transthoracic%20echocardiography%2C%20parasternal%20long-axis%20view%2C%20shows%20a%20dilated%20aortic%20root%20(AR)%20measuring%204.5%20cm%20in%20a%20patient%20with%20bicuspid%20aortic%20valve%20(arrows).%20AA%2C%20ascending%20aorta%3B%20LA%2C%20left%20atrium%3B%20LV%2C%20left%20ventricle.%3C%2Fp%3E

shah_bicuspidaorticvalve_f2.jpg
%3Cp%3EFigure%202.%20Two-dimensional%20transesophageal%20echocardiography%20demonstrates%20an%20anterior-posterior%20bicuspid%20aortic%20valve%20(opening%20phase%2C%20arrows).%20RA%2C%20right%20atrium%3B%20LA%2C%20left%20atrium%3B%20RV%2C%20right%20ventricle.%3C%2Fp%3E

Magnetic resonance imaging (MRI) or computed tomographic (CT) angiography may be required to better assess the aorta for patients requiring a surgical intervention, or when aortic dimensions are not clearly visualized on TTE. MRI has 2 advantages over CT angiography: it poses no radiation risk, and it provides more information on left ventricular function and dimensions, in addition to valve assessment.23,24

No published study has compared MRI or CT angiography and transesophageal echocardiography (TEE), but in a study of 174 patients with dilated aortic root, combined TTE and TEE detected aortic valve morphology accurately in 98% of cases. As TEE is more invasive, it is not recommended for regular surveillance (Figures 1 and 2).25

FAMILY SCREENING RECOMMENDED

Close relatives should be evaluated for aortic valve and thoracic aortic disease.12,13,23,26

The American College of Cardiology (ACC) and the American Heart Association (AHA), backed by radiologic and cardiovascular associations, concur in recommending echocardiographic screening and routine screening of the thoracic aorta for aortic root dilation in first-degree relatives (ie, siblings, parents, and children) of patients with bicuspid aortic valve (class I recommendation).22,27,28

A comprehensive physical examination is recommended for family members in addition to TTE, with careful assessment of the aortic valve in short and long axes, and of the aortic root.14 If the aorta cannot be adequately evaluated with TTE, further assessment should be pursued with CT angiography or MRI.

 

 

EXERCISE RESTRICTIONS

The 2015 ACC/AHA guidelines for competitive athletes with cardiovascular abnormalities recommend annual screening with TTE or MRI angiography for athletes with bicuspid aortic valve and coexisting dilation of the ascending aorta (aortic diameter 40–42 mm in men and 36–39 mm in women) (class I recommendation, level of evidence C).29

Athletes with a bicuspid aortic valve and a normal aortic root and ascending aorta may participate in all competitive activities.29 However, those with a dilated aorta should avoid strenuous activities because of the increased risk of rupture.30 The ACC/AHA recommendations29 depend on the diameter of the ascending aorta and the nature of the sport:

  • For an aortic diameter 40 to 42 mm in men or 36 to 39 mm in women, and no features of connective tissue disease or familial thoracic ascending aortic syndrome, low- and moderate-intensity sports with a low likelihood of significant body contact may be considered; consider avoiding intense weight training (class IIb, level of evidence C)
  • For an aortic diameter 43 to 45 mm, low-intensity sports with a low likelihood of body contact may be considered (class IIb, level of evidence C)
  • For an aortic diameter greater than 43 mm in men or greater than 40 mm in women, sports involving body collision should be avoided (class III, level of evidence C)
  • For an aortic diameter greater than 45 mm, sports activities should be avoided (class III, level of evidence C).

PREGNANCY CONSIDERATIONS

Bicuspid aortic valve is associated with aortic dissection, mainly in the third trimester.31 Patients should ideally undergo echocardiographic screening before conception. The 2010 ACC/AHA guidelines for managing thoracic aortic disease recommend monthly or bimonthly echocardiography until delivery in pregnant women with a dilated thoracic aorta.22

Patients with bicuspid aortic valve and aortic root enlargement of more than 40 mm should have preconception counseling about surgery for aortic root replacement before becoming pregnant. If the diagnosis of enlarged aortic root is made during pregnancy, echocardiographic surveillance at 4- to 6-week intervals is indicated.32

SURGICAL MANAGEMENT

In the past, beta-blockers and angiotensin-converting enzyme inhibitors were recommended to minimize shear stress, with the goal of slowing progression of aortic dilation. However, evidence to support their use is inadequate.33,34

The only definitive treatment is surgery, with various procedures that lower the risk of death or dissection.24,35

The dimensions of the aortic root or ascending aorta should be examined vigilantly, according to the 2014 ACC/AHA guidelines27:

  • Repairing the aortic sinuses or replacing the ascending aorta is indicated if the diameter of the aortic sinuses or ascending aorta is greater than 5.5 cm (class I, level of evidence B)
  • Repairing the aortic sinuses or replacing the ascending aorta is reasonable if the diameter of the aortic sinuses or ascending aorta is greater than 5.0 cm and the patient has a risk factor for dissection such as a family history of aortic dissection or an increase in diameter of 0.5 cm or greater per year (class IIa, level of evidence C)
  • Replacement of the ascending aorta is reasonable if the diameter of the ascending aorta is greater than 4.5 cm and the patient is undergoing aortic valve surgery for severe aortic stenosis or regurgitation.

Valve repair or replacement

Aortic valve repair or replacement is sometimes done separately from aortic root repair.

The value of aortic valve repair is debatable, but a series of 728 patients at Cleveland Clinic showed a very low mortality rate (0.41%) and an annual reoperation rate of 2.6% during up to 15 years of follow-up.36

Aortic valve replacement is usually considered for patients with severe valve dysfunction, abnormal left ventricular dimensions, or symptoms. It is important to determine if the patient is a good surgical candidate and to refer early for surgical evaluation to avoid the higher risk of death associated with emergency surgery.36

Transcatheter aortic valve replacement has been studied in patients deemed to be at too high a risk for surgical replacement. Short- and intermediate-term outcomes have been good in these patients, but long-term data are lacking.37

Surveillance after surgery

The type of operation determines postoperative surveillance.

After isolated aortic valve repair or replacement, patients should continue with surveillance at least annually to monitor for progressive aortopathy, as they remain at increased risk of dissection or rupture after isolated valve surgery, especially if they had aortic insufficiency preoperatively.38

After definitive surgery with replacement or repair of the ascending aorta, no clear recommendations have been established for continued surveillance. However, it is reasonable to image these patients with either MRI or CT angiography 3 to 5 years after their surgery to monitor for anastomotic complications.

CASE QUESTIONS ANSWERED

Our patient should undergo repeat TTE in 1 year. He should also undergo CT angiography of the ascending aorta if it is not seen by TTE. He can participate in low-intensity sports but should avoid intense weight training. His parents, siblings, and children should be screened for bicuspid aortic valve or associated aortopathies.

Bicuspid aortic valve may initially be asymptomatic, but it is associated with progressive valvular and aortic abnormalities that can lead to chronic heart failure and sudden death. Regular monitoring is required with an eye toward surgery when indicated.

See related editorial

This article reviews inheritance patterns and conditions associated with bicuspid aortic valve. We discuss diagnosis, management, and monitoring, and offer surgical recommendations. Special guidance for dental procedures, pregnancy, and athletes is also provided.

A YOUNG MAN WITH PALPITATIONS AND A MURMUR

A 34-year-old man presented to an outpatient clinic with occasional palpitations over the past several months. He reported that he had been diagnosed with a murmur as a child but had received no further testing.

Physical examination at this time revealed a faint systolic crescendo-decrescendo murmur along the right sternal border without radiation to the carotid arteries or to the apex. Transthoracic echocardiography (TTE) showed a bicuspid aortic valve with fusion of the right and left coronary cusps, with no aortic valve stenosis or insufficiency. There was mild dilation of the aortic root, but the mid-ascending aorta could not be evaluated because of limited acoustic windows.

Is further diagnostic testing needed, and if so, what? May he participate in exertional physical activity? Does his newborn son need evaluation?

ABNORMALITIES OCCUR DURING EMBRYOGENESIS

Bicuspid aortic valve develops because of abnormal valvulogenesis. Adjacent cusps fail to separate from each other, resulting in only 2 cusps, with 1 usually larger than the other. Morphology varies according to which commissures are fused.1

Bicuspid aortic valve is associated with abnormalities in the coronary artery anatomy in about 2% of patients, including anomalous origins of the coronary arteries and upwardly displaced coronary ostia.2 Such features need to be considered before surgical intervention.

Bicuspid aortic valve can be found in 1% to 2% of the general population, with a male-to-female predominance of 3:1.1,3,4 It is one of the most common congenital cardiac malformations and is the leading congenital cause of aortic valve stenosis.1,3 However, routine screening of newborns for the condition is not recommended, and most cases are diagnosed incidentally.

GENETIC FACTORS PROMINENT

Bicuspid aortic valve is thought to be primarily inherited in an autosomal-dominant pattern, but there is evidence of genetic heterogeneity, and the pattern may be variable.5,6

No single gene responsible for bicuspid aortic valve has been identified. The condition may occur as a component of different pleiotropic genetic syndromes such as Loeys-Dietz, DiGeorge, and Marfan syndromes,7,8 as well as in patients with Turner syndrome and Williams syndrome.8–11 It also commonly coexists with other congenital heart diseases, including ventricular septal defect, isolated aortic arch obstruction, and patent ductus arteriosus.9

Studies have found a 15% rate of familial clustering.6,12 In a study of 142 patients with bicuspid aortic valve, 20% of first-degree relatives had some cardiac abnormality found by screening, of whom 68% had bicuspid aortic valve. Of these, 71% were newly detected abnormalities.13

CHARACTERISTIC CLICK AND MURMUR

Physical examination findings of a functionally normal bicuspid aortic valve include a systolic ejection click followed by an early peaking systolic murmur at the apex or left lower sternal border. With progression of aortic stenosis, the ejection murmur has a harsher sound, with later peaking, and the S2 sound diminishes or becomes inaudible.14 If aortic regurgitation is present, a diastolic decrescendo murmur is heard best at the left lower sternal border.

 

 

DISEASE PROGRESSION

Although bicuspid aortic valve is typically asymp­tomatic at first, it is commonly associated with progressive valvulopathy and thoracic aortic disease.1,3,4,15 It can lead to chronic heart failure and increase the risk of acute aortic syndromes and sudden cardiac death.15

Michelena et al16 studied 212 cases of asymp­tomatic bicuspid aortic valve. Although the survival rate 20 years after diagnosis was the same as for an age-matched cohort in the general population, the frequency of adverse cardiovascular events and surgical interventions was higher.

Aortic stenosis progresses rapidly

Aortic stenosis associated with a bicuspid aortic valve tends to affect younger patients and progress more rapidly than when associated with a tricuspid valve.17

In a study of 542 patients with congenital bicuspid aortic valve undergoing aortic valve replacement,3 75% had isolated aortic stenosis, 10% had aortic stenosis with some degree of aortic insufficiency, and 13% had isolated aortic insufficiency. Given the tendency of aortic stenosis to progress rapidly, early surgery is often pursued.17,18

Aneurysmal disease is common

The thoracic aorta is at increased risk of aneurysmal disease, coarctation, and dissection in patients with a bicuspid aortic valve.1,6,15

Michelena et al16 reported that in patients without an aneurysm at the time of bicuspid aortic valve diagnosis, the 25-year risk of aneurysm formation was approximately 26%. In patients with an aneurysm at the time of diagnosis, the 15-year risk of aortic surgery after the diagnosis of aneurysm was about 46% and the risk of aortic dissection after aneurysm diagnosis was 7%.15 Compared with the general population, the age-adjusted relative risk of aortic aneurysm in patients with bicuspid aortic valve was 86.2, and that of aortic dissection was 8.4. Although the absolute incidence of dissection is low in these patients, it is markedly higher than in the general population, particularly in older patients (age > 50) and those with an aneurysm at the time of diagnosis.15

The risk of infective endocarditis

Patients with bicuspid aortic valve are highly prone to infective endocarditis for reasons that remain poorly understood. The pathogens in most cases are staphylococci or viridans streptococci.19 Patients with infective endocarditis typically require emergency surgery. Complications including valvular abscess, myocardial abscess, and overt heart failure are common.19

Lamas and Eykyn20 studied 408 cases of native valve endocarditis; in 12.3%, the patient had a bicuspid aortic valve. In this subset, all were male, the mean age was 39 at diagnosis, 82% needed surgery, and the death rate was 14%.

Patients with bicuspid aortic valve do not routinely need antibiotics before dental and surgical procedures, but if they have had endocarditis in the past, they need antibiotics to prevent a recurrence.21

REGULAR MONITORING NEEDED

Because complications may be life-threatening, early detection of progressive disease by regular screening is critical. Echocardiographic evaluation of valvular function, ventricular dimensions and function, and diameter of the aortic root and ascending aorta should be performed in every patient with bicuspid aortic valve. If initial imaging is normal and there is no aortic dilation, imaging should be repeated every 5 to 10 years. If any abnormality is found, repeat imaging is needed every year.22

shah_bicuspidaorticvalve_f1.jpg
%3Cp%3EFigure%201.%20Transthoracic%20echocardiography%2C%20parasternal%20long-axis%20view%2C%20shows%20a%20dilated%20aortic%20root%20(AR)%20measuring%204.5%20cm%20in%20a%20patient%20with%20bicuspid%20aortic%20valve%20(arrows).%20AA%2C%20ascending%20aorta%3B%20LA%2C%20left%20atrium%3B%20LV%2C%20left%20ventricle.%3C%2Fp%3E

shah_bicuspidaorticvalve_f2.jpg
%3Cp%3EFigure%202.%20Two-dimensional%20transesophageal%20echocardiography%20demonstrates%20an%20anterior-posterior%20bicuspid%20aortic%20valve%20(opening%20phase%2C%20arrows).%20RA%2C%20right%20atrium%3B%20LA%2C%20left%20atrium%3B%20RV%2C%20right%20ventricle.%3C%2Fp%3E

Magnetic resonance imaging (MRI) or computed tomographic (CT) angiography may be required to better assess the aorta for patients requiring a surgical intervention, or when aortic dimensions are not clearly visualized on TTE. MRI has 2 advantages over CT angiography: it poses no radiation risk, and it provides more information on left ventricular function and dimensions, in addition to valve assessment.23,24

No published study has compared MRI or CT angiography and transesophageal echocardiography (TEE), but in a study of 174 patients with dilated aortic root, combined TTE and TEE detected aortic valve morphology accurately in 98% of cases. As TEE is more invasive, it is not recommended for regular surveillance (Figures 1 and 2).25

FAMILY SCREENING RECOMMENDED

Close relatives should be evaluated for aortic valve and thoracic aortic disease.12,13,23,26

The American College of Cardiology (ACC) and the American Heart Association (AHA), backed by radiologic and cardiovascular associations, concur in recommending echocardiographic screening and routine screening of the thoracic aorta for aortic root dilation in first-degree relatives (ie, siblings, parents, and children) of patients with bicuspid aortic valve (class I recommendation).22,27,28

A comprehensive physical examination is recommended for family members in addition to TTE, with careful assessment of the aortic valve in short and long axes, and of the aortic root.14 If the aorta cannot be adequately evaluated with TTE, further assessment should be pursued with CT angiography or MRI.

 

 

EXERCISE RESTRICTIONS

The 2015 ACC/AHA guidelines for competitive athletes with cardiovascular abnormalities recommend annual screening with TTE or MRI angiography for athletes with bicuspid aortic valve and coexisting dilation of the ascending aorta (aortic diameter 40–42 mm in men and 36–39 mm in women) (class I recommendation, level of evidence C).29

Athletes with a bicuspid aortic valve and a normal aortic root and ascending aorta may participate in all competitive activities.29 However, those with a dilated aorta should avoid strenuous activities because of the increased risk of rupture.30 The ACC/AHA recommendations29 depend on the diameter of the ascending aorta and the nature of the sport:

  • For an aortic diameter 40 to 42 mm in men or 36 to 39 mm in women, and no features of connective tissue disease or familial thoracic ascending aortic syndrome, low- and moderate-intensity sports with a low likelihood of significant body contact may be considered; consider avoiding intense weight training (class IIb, level of evidence C)
  • For an aortic diameter 43 to 45 mm, low-intensity sports with a low likelihood of body contact may be considered (class IIb, level of evidence C)
  • For an aortic diameter greater than 43 mm in men or greater than 40 mm in women, sports involving body collision should be avoided (class III, level of evidence C)
  • For an aortic diameter greater than 45 mm, sports activities should be avoided (class III, level of evidence C).

PREGNANCY CONSIDERATIONS

Bicuspid aortic valve is associated with aortic dissection, mainly in the third trimester.31 Patients should ideally undergo echocardiographic screening before conception. The 2010 ACC/AHA guidelines for managing thoracic aortic disease recommend monthly or bimonthly echocardiography until delivery in pregnant women with a dilated thoracic aorta.22

Patients with bicuspid aortic valve and aortic root enlargement of more than 40 mm should have preconception counseling about surgery for aortic root replacement before becoming pregnant. If the diagnosis of enlarged aortic root is made during pregnancy, echocardiographic surveillance at 4- to 6-week intervals is indicated.32

SURGICAL MANAGEMENT

In the past, beta-blockers and angiotensin-converting enzyme inhibitors were recommended to minimize shear stress, with the goal of slowing progression of aortic dilation. However, evidence to support their use is inadequate.33,34

The only definitive treatment is surgery, with various procedures that lower the risk of death or dissection.24,35

The dimensions of the aortic root or ascending aorta should be examined vigilantly, according to the 2014 ACC/AHA guidelines27:

  • Repairing the aortic sinuses or replacing the ascending aorta is indicated if the diameter of the aortic sinuses or ascending aorta is greater than 5.5 cm (class I, level of evidence B)
  • Repairing the aortic sinuses or replacing the ascending aorta is reasonable if the diameter of the aortic sinuses or ascending aorta is greater than 5.0 cm and the patient has a risk factor for dissection such as a family history of aortic dissection or an increase in diameter of 0.5 cm or greater per year (class IIa, level of evidence C)
  • Replacement of the ascending aorta is reasonable if the diameter of the ascending aorta is greater than 4.5 cm and the patient is undergoing aortic valve surgery for severe aortic stenosis or regurgitation.

Valve repair or replacement

Aortic valve repair or replacement is sometimes done separately from aortic root repair.

The value of aortic valve repair is debatable, but a series of 728 patients at Cleveland Clinic showed a very low mortality rate (0.41%) and an annual reoperation rate of 2.6% during up to 15 years of follow-up.36

Aortic valve replacement is usually considered for patients with severe valve dysfunction, abnormal left ventricular dimensions, or symptoms. It is important to determine if the patient is a good surgical candidate and to refer early for surgical evaluation to avoid the higher risk of death associated with emergency surgery.36

Transcatheter aortic valve replacement has been studied in patients deemed to be at too high a risk for surgical replacement. Short- and intermediate-term outcomes have been good in these patients, but long-term data are lacking.37

Surveillance after surgery

The type of operation determines postoperative surveillance.

After isolated aortic valve repair or replacement, patients should continue with surveillance at least annually to monitor for progressive aortopathy, as they remain at increased risk of dissection or rupture after isolated valve surgery, especially if they had aortic insufficiency preoperatively.38

After definitive surgery with replacement or repair of the ascending aorta, no clear recommendations have been established for continued surveillance. However, it is reasonable to image these patients with either MRI or CT angiography 3 to 5 years after their surgery to monitor for anastomotic complications.

CASE QUESTIONS ANSWERED

Our patient should undergo repeat TTE in 1 year. He should also undergo CT angiography of the ascending aorta if it is not seen by TTE. He can participate in low-intensity sports but should avoid intense weight training. His parents, siblings, and children should be screened for bicuspid aortic valve or associated aortopathies.

References
  1. Roberts WC. The congenitally bicuspid aortic valve. A study of 85 autopsy cases. Am J Cardiol 1970; 26(1):72–83. pmid:5427836
  2. Michalowska IM, Hryniewiecki T, Kwiatek P, Stoklosa P, Swoboda-Rydz U, Szymanski P. Coronary artery variants and anomalies in patients with bicuspid aortic valve. J Thorac Imaging 2016; 31(3):156–162. doi:10.1097/RTI.0000000000000205
  3. Sabet HY, Edwards WD, Tazelaar HD, Daly RC. Congenitally bicuspid aortic valves: a surgical pathology study of 542 cases (1991 through 1996) and a literature review of 2,715 additional cases. Mayo Clin Proc 1999; 74(1):14–26. doi:10.4065/74.1.14
  4. Tutar E, Ekici F, Atalay S, Nacar N. The prevalence of bicuspid aortic valve in newborns by echocardiographic screening. Am Heart J 2005; 150(3):513–515. doi:10.1016/j.ahj.2004.10.036
  5. Benson DW. The genetics of congenital heart disease: a point in the revolution. Cardiol Clin 2002; 20(3):385–394. pmid:12371007
  6. Emanuel R, Withers R, O’Brien K, Ross P, Feizi O. Congenitally bicuspid aortic valves. Clinicogenetic study of 41 families. Br Heart J 1978; 40(12):1402–1407. pmid:737099
  7. Giusti B, Sticchi E, De Cario R, Magi A, Nistri S, Pepe G. Genetic bases of bicuspid aortic valve: the contribution of traditional and high-throughput sequencing approaches on research and diagnosis. Front Physiol 2017; 8:612. doi:10.3389/fphys.2017.00612
  8. Sachdev V, Matura LA, Sidenko S, et al. Aortic valve disease in Turner syndrome. J Am Coll Cardiol 2008; 51(19):1904–1909. doi:10.1016/j.jacc.2008.02.035
  9. Duran AC, Frescura C, Sans-Coma V, Angelini A, Basso C, Thiene G. Bicuspid aortic valves in hearts with other congenital heart disease. J Heart Valve Dis 1995; 4(6):581–590. pmid:8611973
  10. De Rubens Figueroa J, Rodríguez LM, Hach JL, Del Castillo Ruíz V, Martínez HO. Cardiovascular spectrum in Williams-Beuren syndrome: the Mexican experience in 40 patients. Tex Heart Inst J 2008; 35(3):279–285. pmid:18941598
  11. Yuan SM, Jing H. The bicuspid aortic valve and related disorders. Sao Paulo Med J 2010; 128(5):296–301. pmid:21181071
  12. Cripe L, Andelfinger G, Martin LJ, Shooner K, Benson DW. Bicuspid aortic valve is heritable. J Am Coll Cardiol 2004; 44(1):138–143. doi:10.1016/j.jacc.2004.03.050
  13. Kerstjens-Frederikse WS, Sarvaas GJ, Ruiter JS, et al. Left ventricular outflow tract obstruction: should cardiac screening be offered to first-degree relatives? Heart 2011; 97(15):1228–1232. doi:10.1136/hrt.2010.211433
  14. Siu SC, Silversides CK. Bicuspid aortic valve disease. J Am Coll Cardiol 2010; 55(25):2789–2800. doi:10.1016/j.jacc.2009.12.068
  15. Michelena HI, Khanna AD, Mahoney D, et al. Incidence of aortic complications in patients with bicuspid aortic valves. JAMA 2011; 306(10):1104–1112.
  16. Michelena HI, Desjardins VA, Avierinos JF, et al. Natural history of asymptomatic patients with normally functioning or minimally dysfunctional bicuspid aortic valve in the community. Circulation 2008; 117(21):2776–2784. doi:10.1161/CIRCULATIONAHA.107.740878
  17. Beppu S, Suzuki S, Matsuda H, Ohmori F, Nagata S, Miyatake K. Rapidity of progression of aortic stenosis in patients with congenital bicuspid aortic valves. Am J Cardiol 1993; 71(4):322–327. pmid:8427176
  18. Roberts WC, Ko JM. Frequency by decades of unicuspid, bicuspid, and tricuspid aortic valves in adults having isolated aortic valve replacement for aortic stenosis, with or without associated aortic regurgitation. Circulation 2005; 111(7):920–925. doi:10.1161/01.CIR.0000155623.48408.C5
  19. Yener N, Oktar GL, Erer D, Yardimci MM, Yener A. Bicuspid aortic valve. Ann Thorac Cardiovasc Surg 2002; 8(5):264–267. pmid:12472407
  20. Lamas CC, Eykyn SJ. Bicuspid aortic valve—a silent danger: analysis of 50 cases of infective endocarditis. Clin Infect Dis 2000; 30(2):336–341. doi:10.1086/313646
  21. Wilson W, Taubert KA, Gewitz M, et al. Prevention of infective endocarditis: guidelines from the American Heart Association. Circulation 2007; 116(15):1736–1754. doi:10.1161/CIRCULATIONAHA.106.183095
  22. Hiratzka L, Bakris G, Beckman JA, et al. 2010 ACCF/AHA/AATS/ACR/ASA/SCA/SCAI/SIR/STS/SVM guidelines for the diagnosis and management of patients with thoracic aortic disease. Circulation 2010; 121(13):e266–e369. doi:10.1161/CIR.0b013e3181d4739e
  23. Chun EJ, Choi SI, Lim C, et al. Aortic stenosis: evaluation with multidetector CT angiography and MR imaging. Korean J Radiol 2008; 9(5):439–448. doi:10.3348/kjr.2008.9.5.439
  24. Kiefer TL, Wang A, Hughes GC, Bashore TM. Management of patients with bicuspid aortic valve disease. Curr Treat Options Cardiovasc Med 2011; 13(6):489–505. doi:10.1007/s11936-011-0152-7
  25. Alegret JM, Palazon O, Duran I, Vernis JM. Aortic valve morphology definition with transthoracic combined with transesophageal echocardiography in a population with high prevalence of bicuspid aortic valve. Int J Cardiovasc Imaging 2005; 21(2-3):213–217. doi:10.1007/s10554-004-3901-9
  26. Biner S, Rafique AM, Ray I, Cuk O, Siegel RJ, Tolstrup K. Aortopathy is prevalent in relatives of bicuspid aortic valve patients. J Am Coll Cardiol 2009; 53(24):2288–2295. doi:10.1016/j.jacc.2009.03.027
  27. Nishimura RA, Otto CM, Bonow RO, et al. 2014 AHA/ACC guideline for the management of patients with valvular heart disease. J Thorac Cardiovasc Surg 2014; 148(1):e1-e132. doi:10.1016/j.jtcvs.2014.05.014
  28. Warnes CA, Williams RG, Bashore TM, et al. ACC/AHA 2008 guidelines for the management of adults with congenital heart disease. J Am Coll Cardiol 2008; 52(23):e143–e263. doi:10.1016/j.jacc.2008.10.001
  29. Braverman AC, Harris KM, Kovacs RJ, Maron BJ. Eligibility and disqualification recommendations for competitive athletes with cardiovascular abnormalities: Task Force 7: aortic diseases, including Marfan syndrome. Circulation 2015; 132(22):e303–e309. doi:10.1161/CIR.0000000000000243
  30. De Mozzi P, Longo UG, Galanti G, Maffulli N. Bicuspid aortic valve: a literature review and its impact on sport activity. Br Med Bull 2008; 85:63–85. doi:10.1093/bmb/ldn002
  31. Thorne SA. Pregnancy in heart disease. Heart 2004; 90(4):450–456. pmid:15020530
  32. Immer FF, Bansi AG, Immer-Bansi AS, et al. Aortic dissection in pregnancy: analysis of risk factors and outcome. Ann Thorac Surg 2003; 76(1):309–314. pmid:12842575
  33. Allen BD, Markl M, Barker AJ, et al. Influence of beta-blocker therapy on aortic blood flow in patients with bicuspid aortic valve. Int J Cardiovasc Imaging 2016; 32(4):621–628. doi:10.1007/s10554-015-0819-3
  34. Ohnemus D, Oster ME, Gatlin S, Jokhadar M, Mahle WT. The effect of angiotensin-converting enzyme inhibitors on the rate of ascending aorta dilation in patients with bicuspid aortic valve. Congenit Heart Dis 2015; 10(1):E1–E5. doi:10.1111/chd.12184
  35. Masri A, Kalahasti V, Alkharabsheh S, et al. Characteristics and long-term outcomes of contemporary patients with bicuspid aortic valves. J Thorac Cardiovasc Surg 2016; 151(6):1650–1659.e1. doi:10.1016/j.jtcvs.2015.12.019
  36. Svensson LG, Al Kindi AH, Vivacqua A, et al. Long-term durability of bicuspid aortic valve repair. Ann Thorac Surg 2014; 97(5):1539–1548. doi:10.1016/j.athoracsur.2013.11.036
  37. Mylotte D, Lefevre T, Sondergaard L, et al. Transcatheter aortic valve replacement in bicuspid aortic valve disease. J Am Coll Cardiol 2014; 64(22):2330–2339. doi:10.1016/j.jacc.2014.09.039
  38. Girdauskas E, Disha K, Raisin HH, Secknus MA, Borger MA, Kuntze T. Risk of late aortic events after an isolated aortic valve replacement for bicuspid aortic valve stenosis with concomitant ascending aortic dilation. Eur J Cardiothorac Surg 2012; 42(5):832–838. doi:10.1093/ejcts/ezs137
References
  1. Roberts WC. The congenitally bicuspid aortic valve. A study of 85 autopsy cases. Am J Cardiol 1970; 26(1):72–83. pmid:5427836
  2. Michalowska IM, Hryniewiecki T, Kwiatek P, Stoklosa P, Swoboda-Rydz U, Szymanski P. Coronary artery variants and anomalies in patients with bicuspid aortic valve. J Thorac Imaging 2016; 31(3):156–162. doi:10.1097/RTI.0000000000000205
  3. Sabet HY, Edwards WD, Tazelaar HD, Daly RC. Congenitally bicuspid aortic valves: a surgical pathology study of 542 cases (1991 through 1996) and a literature review of 2,715 additional cases. Mayo Clin Proc 1999; 74(1):14–26. doi:10.4065/74.1.14
  4. Tutar E, Ekici F, Atalay S, Nacar N. The prevalence of bicuspid aortic valve in newborns by echocardiographic screening. Am Heart J 2005; 150(3):513–515. doi:10.1016/j.ahj.2004.10.036
  5. Benson DW. The genetics of congenital heart disease: a point in the revolution. Cardiol Clin 2002; 20(3):385–394. pmid:12371007
  6. Emanuel R, Withers R, O’Brien K, Ross P, Feizi O. Congenitally bicuspid aortic valves. Clinicogenetic study of 41 families. Br Heart J 1978; 40(12):1402–1407. pmid:737099
  7. Giusti B, Sticchi E, De Cario R, Magi A, Nistri S, Pepe G. Genetic bases of bicuspid aortic valve: the contribution of traditional and high-throughput sequencing approaches on research and diagnosis. Front Physiol 2017; 8:612. doi:10.3389/fphys.2017.00612
  8. Sachdev V, Matura LA, Sidenko S, et al. Aortic valve disease in Turner syndrome. J Am Coll Cardiol 2008; 51(19):1904–1909. doi:10.1016/j.jacc.2008.02.035
  9. Duran AC, Frescura C, Sans-Coma V, Angelini A, Basso C, Thiene G. Bicuspid aortic valves in hearts with other congenital heart disease. J Heart Valve Dis 1995; 4(6):581–590. pmid:8611973
  10. De Rubens Figueroa J, Rodríguez LM, Hach JL, Del Castillo Ruíz V, Martínez HO. Cardiovascular spectrum in Williams-Beuren syndrome: the Mexican experience in 40 patients. Tex Heart Inst J 2008; 35(3):279–285. pmid:18941598
  11. Yuan SM, Jing H. The bicuspid aortic valve and related disorders. Sao Paulo Med J 2010; 128(5):296–301. pmid:21181071
  12. Cripe L, Andelfinger G, Martin LJ, Shooner K, Benson DW. Bicuspid aortic valve is heritable. J Am Coll Cardiol 2004; 44(1):138–143. doi:10.1016/j.jacc.2004.03.050
  13. Kerstjens-Frederikse WS, Sarvaas GJ, Ruiter JS, et al. Left ventricular outflow tract obstruction: should cardiac screening be offered to first-degree relatives? Heart 2011; 97(15):1228–1232. doi:10.1136/hrt.2010.211433
  14. Siu SC, Silversides CK. Bicuspid aortic valve disease. J Am Coll Cardiol 2010; 55(25):2789–2800. doi:10.1016/j.jacc.2009.12.068
  15. Michelena HI, Khanna AD, Mahoney D, et al. Incidence of aortic complications in patients with bicuspid aortic valves. JAMA 2011; 306(10):1104–1112.
  16. Michelena HI, Desjardins VA, Avierinos JF, et al. Natural history of asymptomatic patients with normally functioning or minimally dysfunctional bicuspid aortic valve in the community. Circulation 2008; 117(21):2776–2784. doi:10.1161/CIRCULATIONAHA.107.740878
  17. Beppu S, Suzuki S, Matsuda H, Ohmori F, Nagata S, Miyatake K. Rapidity of progression of aortic stenosis in patients with congenital bicuspid aortic valves. Am J Cardiol 1993; 71(4):322–327. pmid:8427176
  18. Roberts WC, Ko JM. Frequency by decades of unicuspid, bicuspid, and tricuspid aortic valves in adults having isolated aortic valve replacement for aortic stenosis, with or without associated aortic regurgitation. Circulation 2005; 111(7):920–925. doi:10.1161/01.CIR.0000155623.48408.C5
  19. Yener N, Oktar GL, Erer D, Yardimci MM, Yener A. Bicuspid aortic valve. Ann Thorac Cardiovasc Surg 2002; 8(5):264–267. pmid:12472407
  20. Lamas CC, Eykyn SJ. Bicuspid aortic valve—a silent danger: analysis of 50 cases of infective endocarditis. Clin Infect Dis 2000; 30(2):336–341. doi:10.1086/313646
  21. Wilson W, Taubert KA, Gewitz M, et al. Prevention of infective endocarditis: guidelines from the American Heart Association. Circulation 2007; 116(15):1736–1754. doi:10.1161/CIRCULATIONAHA.106.183095
  22. Hiratzka L, Bakris G, Beckman JA, et al. 2010 ACCF/AHA/AATS/ACR/ASA/SCA/SCAI/SIR/STS/SVM guidelines for the diagnosis and management of patients with thoracic aortic disease. Circulation 2010; 121(13):e266–e369. doi:10.1161/CIR.0b013e3181d4739e
  23. Chun EJ, Choi SI, Lim C, et al. Aortic stenosis: evaluation with multidetector CT angiography and MR imaging. Korean J Radiol 2008; 9(5):439–448. doi:10.3348/kjr.2008.9.5.439
  24. Kiefer TL, Wang A, Hughes GC, Bashore TM. Management of patients with bicuspid aortic valve disease. Curr Treat Options Cardiovasc Med 2011; 13(6):489–505. doi:10.1007/s11936-011-0152-7
  25. Alegret JM, Palazon O, Duran I, Vernis JM. Aortic valve morphology definition with transthoracic combined with transesophageal echocardiography in a population with high prevalence of bicuspid aortic valve. Int J Cardiovasc Imaging 2005; 21(2-3):213–217. doi:10.1007/s10554-004-3901-9
  26. Biner S, Rafique AM, Ray I, Cuk O, Siegel RJ, Tolstrup K. Aortopathy is prevalent in relatives of bicuspid aortic valve patients. J Am Coll Cardiol 2009; 53(24):2288–2295. doi:10.1016/j.jacc.2009.03.027
  27. Nishimura RA, Otto CM, Bonow RO, et al. 2014 AHA/ACC guideline for the management of patients with valvular heart disease. J Thorac Cardiovasc Surg 2014; 148(1):e1-e132. doi:10.1016/j.jtcvs.2014.05.014
  28. Warnes CA, Williams RG, Bashore TM, et al. ACC/AHA 2008 guidelines for the management of adults with congenital heart disease. J Am Coll Cardiol 2008; 52(23):e143–e263. doi:10.1016/j.jacc.2008.10.001
  29. Braverman AC, Harris KM, Kovacs RJ, Maron BJ. Eligibility and disqualification recommendations for competitive athletes with cardiovascular abnormalities: Task Force 7: aortic diseases, including Marfan syndrome. Circulation 2015; 132(22):e303–e309. doi:10.1161/CIR.0000000000000243
  30. De Mozzi P, Longo UG, Galanti G, Maffulli N. Bicuspid aortic valve: a literature review and its impact on sport activity. Br Med Bull 2008; 85:63–85. doi:10.1093/bmb/ldn002
  31. Thorne SA. Pregnancy in heart disease. Heart 2004; 90(4):450–456. pmid:15020530
  32. Immer FF, Bansi AG, Immer-Bansi AS, et al. Aortic dissection in pregnancy: analysis of risk factors and outcome. Ann Thorac Surg 2003; 76(1):309–314. pmid:12842575
  33. Allen BD, Markl M, Barker AJ, et al. Influence of beta-blocker therapy on aortic blood flow in patients with bicuspid aortic valve. Int J Cardiovasc Imaging 2016; 32(4):621–628. doi:10.1007/s10554-015-0819-3
  34. Ohnemus D, Oster ME, Gatlin S, Jokhadar M, Mahle WT. The effect of angiotensin-converting enzyme inhibitors on the rate of ascending aorta dilation in patients with bicuspid aortic valve. Congenit Heart Dis 2015; 10(1):E1–E5. doi:10.1111/chd.12184
  35. Masri A, Kalahasti V, Alkharabsheh S, et al. Characteristics and long-term outcomes of contemporary patients with bicuspid aortic valves. J Thorac Cardiovasc Surg 2016; 151(6):1650–1659.e1. doi:10.1016/j.jtcvs.2015.12.019
  36. Svensson LG, Al Kindi AH, Vivacqua A, et al. Long-term durability of bicuspid aortic valve repair. Ann Thorac Surg 2014; 97(5):1539–1548. doi:10.1016/j.athoracsur.2013.11.036
  37. Mylotte D, Lefevre T, Sondergaard L, et al. Transcatheter aortic valve replacement in bicuspid aortic valve disease. J Am Coll Cardiol 2014; 64(22):2330–2339. doi:10.1016/j.jacc.2014.09.039
  38. Girdauskas E, Disha K, Raisin HH, Secknus MA, Borger MA, Kuntze T. Risk of late aortic events after an isolated aortic valve replacement for bicuspid aortic valve stenosis with concomitant ascending aortic dilation. Eur J Cardiothorac Surg 2012; 42(5):832–838. doi:10.1093/ejcts/ezs137
Issue
Cleveland Clinic Journal of Medicine - 85(10)
Issue
Cleveland Clinic Journal of Medicine - 85(10)
Page Number
779-784
Page Number
779-784
Publications
Publications
Topics
Article Type
Display Headline
Bicuspid aortic valve: Basics and beyond
Display Headline
Bicuspid aortic valve: Basics and beyond
Legacy Keywords
bicuspid aortic valve, BAV, aorta, aortopathy, echocardiography, stenosis, aneurysm, Loeys-Dietz syndrome, Marfan syndrome, DiGeorge syndrome, Turner syndrome, Williams syndrome, infective endocarditis, Shailee Shah, Andrew Higgins, Milind Desai
Legacy Keywords
bicuspid aortic valve, BAV, aorta, aortopathy, echocardiography, stenosis, aneurysm, Loeys-Dietz syndrome, Marfan syndrome, DiGeorge syndrome, Turner syndrome, Williams syndrome, infective endocarditis, Shailee Shah, Andrew Higgins, Milind Desai
Sections
Inside the Article

KEY POINTS

  • Associated aortopathies such as aortic root dilation, aneurysm, dissection, and coarctation may initially be asymptomatic.
  • Regular surveillance with transthoracic echocardiography (TTE) is required.
  • Transesophageal echocardiography should be performed if TTE does not clearly show the aorta and aortic root. Magnetic resonance imaging or computed tomographic angiography may also be needed to measure the aortic root and ascending thoracic aorta.
  • If initial imaging is normal and there is no aortic dilation, repeat imaging should be done every 5 to 10 years. If any abnormality is found, annual surveillance is needed.
  • Women with a bicuspid aortic valve who are contemplating pregnancy should undergo echocardiography first, and some may need to undergo surgery.
Disallow All Ads
Content Gating
No Gating (article Unlocked/Free)
Alternative CME
Disqus Comments
Default
Gate On Date
Wed, 09/26/2018 - 11:30
Un-Gate On Date
Wed, 09/26/2018 - 11:30
Use ProPublica
CFC Schedule Remove Status
Wed, 09/26/2018 - 11:30
Hide sidebar & use full width
render the right sidebar.
Article PDF Media

When stroke runs in the family

Article Type
Changed
Mon, 10/01/2018 - 06:55
Display Headline
When stroke runs in the family

A 54-year-old man presented to our hospital with acute-onset left-sided weakness and right facial droop. Three days earlier he had also had migraine-like headaches, which he had never experienced before. He also reported a change in behavior during the past week, which his family had described as inappropriate laughter.

He had no history of hypertension, diabetes, or dyslipidemia. He did not smoke or drink alcohol. However, he had an extensive family history of stroke. His mother had a stroke at age 50, his brother a stroke at age 57, and his sister had been admitted for a stroke 1 month earlier at the age of 52.

On examination, he had weakness of the left arm and leg, right facial droop, and hyperactive reflexes on the left side. He had no sensory or cerebellar deficits. He had episodes of laughter during the examination.

candelario_stroke_f1.jpg
%3Cp%3EFigure%201.%20Computed%20tomography%20of%20the%20brain%20without%20contrast%20showed%20patchy%20confluent%20areas%20of%20low%20attenuation%20in%20both%20cerebral%20hemispheres%20(arrows).%3C%2Fp%3E
Behavioral changes such as inappropriate laughter are nonspecific and can be associated with any subcortical frontoparietal and temporal infarction.

candelario_stroke_f2.jpg
%3Cp%3EFigure%202.%20Magnetic%20resonance%20imaging%20with%20axial%20T2%20fluid-attenuated%20inversion%20recovery%20showed%20confluent%20and%20symmetric%20white%20matter%20hyperintensities%20in%20the%20frontoparietotemporal%20region%20(arrows).%20These%20changes%2C%20especially%20in%20the%20temporal%20lobe%2C%20were%20atypical%20of%20microvascular%20disease%20given%20the%20patient%E2%80%99s%20age.%3C%2Fp%3E
Computed tomography of the brain without contrast (Figure 1) showed patchy confluent areas of low attenuation in both cerebral hemispheres. Magnetic resonance imaging (MRI) showed an 8-mm focus of mild restricted diffusion in the right lateral thalamus, indicating acute infarction, and axial T2 fluid-attenuated inversion recovery images (Figure 2) showed confluent and symmetric white matter hyperintensities in the frontoparietotemporal region. The extent of these white matter changes, especially in the temporal lobe, were not typical of the microvascular disease changes seen in elderly individuals with risk factors for stroke. Microvascular disease may present the same way, but not as extensively and this was not consistent with the patient’s relatively young age. The imaging findings suggested cerebral autosomal dominant arteriopathy with subcortical infarcts and leukoencephalopathy (CADASIL).

We learned that the patient’s sister had undergone a workup showing mutations in the NOTCH3 gene and a skin biopsy study consistent with CADASIL.

Our patient was started on antiplatelet and high-intensity statin therapy. His symptoms improved, and he was discharged to an acute inpatient rehabilitation facility. He was referred to a CADASIL registry.

STROKE AND HEREDITY

CADASIL is a rare hereditary vascular disorder inherited in an autosomal dominant manner. It is the most common inherited form of small-vessel disease and results from a mutation in the NOTCH3 gene that leads to degeneration of smooth muscle in cerebral blood vessels. It can manifest as migraine with aura, vascular dementia, cognitive impairment, or ischemic stroke.

The diagnosis is based on a clinical picture that typically includes stroke at a young age (age 40 to 50) in the absence of stroke risk factors, or frequent lacunar infarction episodes that can manifest as migraine, lacunar infarct, or dementia.1 Some patients, such as ours, may have subtle nonspecific behavioral changes such as inappropriate laughter, which may herald the development of an infarct.

Characteristic findings on MRI are white matter hyperintensities that tend to be bilateral and symmetrical in the periventricular areas. Symmetrical involvement in the temporal lobes has high sensitivity and specificity for CADASIL.2 Biopsy study of the skin, muscle, or sural nerve shows small-vessel changes that include thickening of the media, granular material positive on periodic acid-Schiff staining, and narrowing of the lumen. However, the gold standard for diagnosis is confirmation of the NOTCH3 mutation on chromosome 19.1,2

There is no known treatment for CADASIL.

References
  1. Davous P. CADASIL: a review with proposed diagnostic criteria. Eur J Neurol 1998; 5(3):219–233. pmid:10210836
  2. Stojanov D, Vojinovic S, Aracki-Trenkic A, et al. Imaging characteristics of cerebral autosomal dominant arteriopathy with subcortical infarcts and leucoencephalopathy (CADASIL). Bosn J Basic Med Sci 2015; 15(1):1–8. doi:10.17305/bjbms.2015.247
Article PDF
Author and Disclosure Information

Nellowe Candelario, MD
Department of Internal Medicine, Einstein Medical Center, Philadelphia, PA

Kevin Bryan Lo, MD
Department of Internal Medicine, Einstein Medical Center, Philadelphia, PA

Address: Nellowe Candelario, MD, Department of Internal Medicine, Einstein Medical Center, 5501 Old York Road, Philadelphia, PA 19141; candelan@einstein.edu

Issue
Cleveland Clinic Journal of Medicine - 85(10)
Publications
Topics
Page Number
743-744
Legacy Keywords
stroke, CADASIL, cerebral autosomal dominant arteriopathy with subcortical infarcts and leukoencephalopathy, NOTCH3 gene, lacunar infarction, migraine, dementia, laughter, CT, computed tomography, magnetic resonance imaging, MRI, Nellowe Candelario, Kevin Bryan
Sections
Author and Disclosure Information

Nellowe Candelario, MD
Department of Internal Medicine, Einstein Medical Center, Philadelphia, PA

Kevin Bryan Lo, MD
Department of Internal Medicine, Einstein Medical Center, Philadelphia, PA

Address: Nellowe Candelario, MD, Department of Internal Medicine, Einstein Medical Center, 5501 Old York Road, Philadelphia, PA 19141; candelan@einstein.edu

Author and Disclosure Information

Nellowe Candelario, MD
Department of Internal Medicine, Einstein Medical Center, Philadelphia, PA

Kevin Bryan Lo, MD
Department of Internal Medicine, Einstein Medical Center, Philadelphia, PA

Address: Nellowe Candelario, MD, Department of Internal Medicine, Einstein Medical Center, 5501 Old York Road, Philadelphia, PA 19141; candelan@einstein.edu

Article PDF
Article PDF
Related Articles

A 54-year-old man presented to our hospital with acute-onset left-sided weakness and right facial droop. Three days earlier he had also had migraine-like headaches, which he had never experienced before. He also reported a change in behavior during the past week, which his family had described as inappropriate laughter.

He had no history of hypertension, diabetes, or dyslipidemia. He did not smoke or drink alcohol. However, he had an extensive family history of stroke. His mother had a stroke at age 50, his brother a stroke at age 57, and his sister had been admitted for a stroke 1 month earlier at the age of 52.

On examination, he had weakness of the left arm and leg, right facial droop, and hyperactive reflexes on the left side. He had no sensory or cerebellar deficits. He had episodes of laughter during the examination.

candelario_stroke_f1.jpg
%3Cp%3EFigure%201.%20Computed%20tomography%20of%20the%20brain%20without%20contrast%20showed%20patchy%20confluent%20areas%20of%20low%20attenuation%20in%20both%20cerebral%20hemispheres%20(arrows).%3C%2Fp%3E
Behavioral changes such as inappropriate laughter are nonspecific and can be associated with any subcortical frontoparietal and temporal infarction.

candelario_stroke_f2.jpg
%3Cp%3EFigure%202.%20Magnetic%20resonance%20imaging%20with%20axial%20T2%20fluid-attenuated%20inversion%20recovery%20showed%20confluent%20and%20symmetric%20white%20matter%20hyperintensities%20in%20the%20frontoparietotemporal%20region%20(arrows).%20These%20changes%2C%20especially%20in%20the%20temporal%20lobe%2C%20were%20atypical%20of%20microvascular%20disease%20given%20the%20patient%E2%80%99s%20age.%3C%2Fp%3E
Computed tomography of the brain without contrast (Figure 1) showed patchy confluent areas of low attenuation in both cerebral hemispheres. Magnetic resonance imaging (MRI) showed an 8-mm focus of mild restricted diffusion in the right lateral thalamus, indicating acute infarction, and axial T2 fluid-attenuated inversion recovery images (Figure 2) showed confluent and symmetric white matter hyperintensities in the frontoparietotemporal region. The extent of these white matter changes, especially in the temporal lobe, were not typical of the microvascular disease changes seen in elderly individuals with risk factors for stroke. Microvascular disease may present the same way, but not as extensively and this was not consistent with the patient’s relatively young age. The imaging findings suggested cerebral autosomal dominant arteriopathy with subcortical infarcts and leukoencephalopathy (CADASIL).

We learned that the patient’s sister had undergone a workup showing mutations in the NOTCH3 gene and a skin biopsy study consistent with CADASIL.

Our patient was started on antiplatelet and high-intensity statin therapy. His symptoms improved, and he was discharged to an acute inpatient rehabilitation facility. He was referred to a CADASIL registry.

STROKE AND HEREDITY

CADASIL is a rare hereditary vascular disorder inherited in an autosomal dominant manner. It is the most common inherited form of small-vessel disease and results from a mutation in the NOTCH3 gene that leads to degeneration of smooth muscle in cerebral blood vessels. It can manifest as migraine with aura, vascular dementia, cognitive impairment, or ischemic stroke.

The diagnosis is based on a clinical picture that typically includes stroke at a young age (age 40 to 50) in the absence of stroke risk factors, or frequent lacunar infarction episodes that can manifest as migraine, lacunar infarct, or dementia.1 Some patients, such as ours, may have subtle nonspecific behavioral changes such as inappropriate laughter, which may herald the development of an infarct.

Characteristic findings on MRI are white matter hyperintensities that tend to be bilateral and symmetrical in the periventricular areas. Symmetrical involvement in the temporal lobes has high sensitivity and specificity for CADASIL.2 Biopsy study of the skin, muscle, or sural nerve shows small-vessel changes that include thickening of the media, granular material positive on periodic acid-Schiff staining, and narrowing of the lumen. However, the gold standard for diagnosis is confirmation of the NOTCH3 mutation on chromosome 19.1,2

There is no known treatment for CADASIL.

A 54-year-old man presented to our hospital with acute-onset left-sided weakness and right facial droop. Three days earlier he had also had migraine-like headaches, which he had never experienced before. He also reported a change in behavior during the past week, which his family had described as inappropriate laughter.

He had no history of hypertension, diabetes, or dyslipidemia. He did not smoke or drink alcohol. However, he had an extensive family history of stroke. His mother had a stroke at age 50, his brother a stroke at age 57, and his sister had been admitted for a stroke 1 month earlier at the age of 52.

On examination, he had weakness of the left arm and leg, right facial droop, and hyperactive reflexes on the left side. He had no sensory or cerebellar deficits. He had episodes of laughter during the examination.

candelario_stroke_f1.jpg
%3Cp%3EFigure%201.%20Computed%20tomography%20of%20the%20brain%20without%20contrast%20showed%20patchy%20confluent%20areas%20of%20low%20attenuation%20in%20both%20cerebral%20hemispheres%20(arrows).%3C%2Fp%3E
Behavioral changes such as inappropriate laughter are nonspecific and can be associated with any subcortical frontoparietal and temporal infarction.

candelario_stroke_f2.jpg
%3Cp%3EFigure%202.%20Magnetic%20resonance%20imaging%20with%20axial%20T2%20fluid-attenuated%20inversion%20recovery%20showed%20confluent%20and%20symmetric%20white%20matter%20hyperintensities%20in%20the%20frontoparietotemporal%20region%20(arrows).%20These%20changes%2C%20especially%20in%20the%20temporal%20lobe%2C%20were%20atypical%20of%20microvascular%20disease%20given%20the%20patient%E2%80%99s%20age.%3C%2Fp%3E
Computed tomography of the brain without contrast (Figure 1) showed patchy confluent areas of low attenuation in both cerebral hemispheres. Magnetic resonance imaging (MRI) showed an 8-mm focus of mild restricted diffusion in the right lateral thalamus, indicating acute infarction, and axial T2 fluid-attenuated inversion recovery images (Figure 2) showed confluent and symmetric white matter hyperintensities in the frontoparietotemporal region. The extent of these white matter changes, especially in the temporal lobe, were not typical of the microvascular disease changes seen in elderly individuals with risk factors for stroke. Microvascular disease may present the same way, but not as extensively and this was not consistent with the patient’s relatively young age. The imaging findings suggested cerebral autosomal dominant arteriopathy with subcortical infarcts and leukoencephalopathy (CADASIL).

We learned that the patient’s sister had undergone a workup showing mutations in the NOTCH3 gene and a skin biopsy study consistent with CADASIL.

Our patient was started on antiplatelet and high-intensity statin therapy. His symptoms improved, and he was discharged to an acute inpatient rehabilitation facility. He was referred to a CADASIL registry.

STROKE AND HEREDITY

CADASIL is a rare hereditary vascular disorder inherited in an autosomal dominant manner. It is the most common inherited form of small-vessel disease and results from a mutation in the NOTCH3 gene that leads to degeneration of smooth muscle in cerebral blood vessels. It can manifest as migraine with aura, vascular dementia, cognitive impairment, or ischemic stroke.

The diagnosis is based on a clinical picture that typically includes stroke at a young age (age 40 to 50) in the absence of stroke risk factors, or frequent lacunar infarction episodes that can manifest as migraine, lacunar infarct, or dementia.1 Some patients, such as ours, may have subtle nonspecific behavioral changes such as inappropriate laughter, which may herald the development of an infarct.

Characteristic findings on MRI are white matter hyperintensities that tend to be bilateral and symmetrical in the periventricular areas. Symmetrical involvement in the temporal lobes has high sensitivity and specificity for CADASIL.2 Biopsy study of the skin, muscle, or sural nerve shows small-vessel changes that include thickening of the media, granular material positive on periodic acid-Schiff staining, and narrowing of the lumen. However, the gold standard for diagnosis is confirmation of the NOTCH3 mutation on chromosome 19.1,2

There is no known treatment for CADASIL.

References
  1. Davous P. CADASIL: a review with proposed diagnostic criteria. Eur J Neurol 1998; 5(3):219–233. pmid:10210836
  2. Stojanov D, Vojinovic S, Aracki-Trenkic A, et al. Imaging characteristics of cerebral autosomal dominant arteriopathy with subcortical infarcts and leucoencephalopathy (CADASIL). Bosn J Basic Med Sci 2015; 15(1):1–8. doi:10.17305/bjbms.2015.247
References
  1. Davous P. CADASIL: a review with proposed diagnostic criteria. Eur J Neurol 1998; 5(3):219–233. pmid:10210836
  2. Stojanov D, Vojinovic S, Aracki-Trenkic A, et al. Imaging characteristics of cerebral autosomal dominant arteriopathy with subcortical infarcts and leucoencephalopathy (CADASIL). Bosn J Basic Med Sci 2015; 15(1):1–8. doi:10.17305/bjbms.2015.247
Issue
Cleveland Clinic Journal of Medicine - 85(10)
Issue
Cleveland Clinic Journal of Medicine - 85(10)
Page Number
743-744
Page Number
743-744
Publications
Publications
Topics
Article Type
Display Headline
When stroke runs in the family
Display Headline
When stroke runs in the family
Legacy Keywords
stroke, CADASIL, cerebral autosomal dominant arteriopathy with subcortical infarcts and leukoencephalopathy, NOTCH3 gene, lacunar infarction, migraine, dementia, laughter, CT, computed tomography, magnetic resonance imaging, MRI, Nellowe Candelario, Kevin Bryan
Legacy Keywords
stroke, CADASIL, cerebral autosomal dominant arteriopathy with subcortical infarcts and leukoencephalopathy, NOTCH3 gene, lacunar infarction, migraine, dementia, laughter, CT, computed tomography, magnetic resonance imaging, MRI, Nellowe Candelario, Kevin Bryan
Sections
Disallow All Ads
Content Gating
No Gating (article Unlocked/Free)
Alternative CME
Disqus Comments
Default
Gate On Date
Wed, 09/26/2018 - 11:30
Un-Gate On Date
Wed, 09/26/2018 - 11:30
Use ProPublica
CFC Schedule Remove Status
Wed, 09/26/2018 - 11:30
Article PDF Media

Aortic aneurysm: Fluoroquinolones, genetic counseling

Article Type
Changed
Tue, 09/04/2018 - 09:16
Display Headline
Aortic aneurysm: Fluoroquinolones, genetic counseling

To the Editor: The review of thoracic aortic aneurysm by Cikach et al1 was excellent. However, we noted that referral for clinical genetic counseling and testing is suggested only if 1 or more first-degree relatives have aneurysmal disease.

Absence of a family history does not rule out syndromic aortopathy, which can occur de novo. In addition, a clinical diagnosis of syndromic aortopathy can be made on the basis of physical features that can be very subtle, such as pectus deformities, scoliosis, dolichostenomelia, joint hypermobility or contractures, craniofacial features, or skin fragility.2

Genetic counseling is paramount even if molecular testing is negative or inconclusive, which can occur in more than 50% of patients referred.3 Clinical genetic evaluation would also facilitate testing for other family members who may be affected, and would help to coordinate care for nonvascular conditions that may be associated with the syndrome.

References
  1. Cikach F, Desai MY, Roselli EE, Kalahasti V. Thoracic aortic aneurysm: how to counsel, when to refer. Cleve Clin J Med 2018; 85(6):481–492. doi:10.3949/ccjm.85a.17039
  2. McKusick-Nathans Institute of Genetic Medicine, Johns Hopkins University. OMIM. Online mendelian inheritance in man. https://omim.org. Accessed July 31, 2018.
  3. Mazine A, Moryousef-Abitbol JH, Faghfoury H, Meza JM, Morel C, Ouzounian M. Yield of genetic testing in patients with thoracic aortic disease. J Am Coll Cardiol 2017; 69(11):2005. doi:10.1016/S0735-1097(17)35394-9
Article PDF
Author and Disclosure Information

Houriya Ayoubieh, MD
University of New Mexico, Albuquerque, NM

Gretchen Maccarrick, MS, CGC
Institute of Genetic Medicine, Johns Hopkins University, Baltimore, MD

Issue
Cleveland Clinic Journal of Medicine - 85(9)
Publications
Topics
Page Number
661-662
Legacy Keywords
thoracic aortic aneurysm, genetic counseling, Houriya Ayoubieh, Gretchen MacCarrick
Sections
Author and Disclosure Information

Houriya Ayoubieh, MD
University of New Mexico, Albuquerque, NM

Gretchen Maccarrick, MS, CGC
Institute of Genetic Medicine, Johns Hopkins University, Baltimore, MD

Author and Disclosure Information

Houriya Ayoubieh, MD
University of New Mexico, Albuquerque, NM

Gretchen Maccarrick, MS, CGC
Institute of Genetic Medicine, Johns Hopkins University, Baltimore, MD

Article PDF
Article PDF
Related Articles

To the Editor: The review of thoracic aortic aneurysm by Cikach et al1 was excellent. However, we noted that referral for clinical genetic counseling and testing is suggested only if 1 or more first-degree relatives have aneurysmal disease.

Absence of a family history does not rule out syndromic aortopathy, which can occur de novo. In addition, a clinical diagnosis of syndromic aortopathy can be made on the basis of physical features that can be very subtle, such as pectus deformities, scoliosis, dolichostenomelia, joint hypermobility or contractures, craniofacial features, or skin fragility.2

Genetic counseling is paramount even if molecular testing is negative or inconclusive, which can occur in more than 50% of patients referred.3 Clinical genetic evaluation would also facilitate testing for other family members who may be affected, and would help to coordinate care for nonvascular conditions that may be associated with the syndrome.

To the Editor: The review of thoracic aortic aneurysm by Cikach et al1 was excellent. However, we noted that referral for clinical genetic counseling and testing is suggested only if 1 or more first-degree relatives have aneurysmal disease.

Absence of a family history does not rule out syndromic aortopathy, which can occur de novo. In addition, a clinical diagnosis of syndromic aortopathy can be made on the basis of physical features that can be very subtle, such as pectus deformities, scoliosis, dolichostenomelia, joint hypermobility or contractures, craniofacial features, or skin fragility.2

Genetic counseling is paramount even if molecular testing is negative or inconclusive, which can occur in more than 50% of patients referred.3 Clinical genetic evaluation would also facilitate testing for other family members who may be affected, and would help to coordinate care for nonvascular conditions that may be associated with the syndrome.

References
  1. Cikach F, Desai MY, Roselli EE, Kalahasti V. Thoracic aortic aneurysm: how to counsel, when to refer. Cleve Clin J Med 2018; 85(6):481–492. doi:10.3949/ccjm.85a.17039
  2. McKusick-Nathans Institute of Genetic Medicine, Johns Hopkins University. OMIM. Online mendelian inheritance in man. https://omim.org. Accessed July 31, 2018.
  3. Mazine A, Moryousef-Abitbol JH, Faghfoury H, Meza JM, Morel C, Ouzounian M. Yield of genetic testing in patients with thoracic aortic disease. J Am Coll Cardiol 2017; 69(11):2005. doi:10.1016/S0735-1097(17)35394-9
References
  1. Cikach F, Desai MY, Roselli EE, Kalahasti V. Thoracic aortic aneurysm: how to counsel, when to refer. Cleve Clin J Med 2018; 85(6):481–492. doi:10.3949/ccjm.85a.17039
  2. McKusick-Nathans Institute of Genetic Medicine, Johns Hopkins University. OMIM. Online mendelian inheritance in man. https://omim.org. Accessed July 31, 2018.
  3. Mazine A, Moryousef-Abitbol JH, Faghfoury H, Meza JM, Morel C, Ouzounian M. Yield of genetic testing in patients with thoracic aortic disease. J Am Coll Cardiol 2017; 69(11):2005. doi:10.1016/S0735-1097(17)35394-9
Issue
Cleveland Clinic Journal of Medicine - 85(9)
Issue
Cleveland Clinic Journal of Medicine - 85(9)
Page Number
661-662
Page Number
661-662
Publications
Publications
Topics
Article Type
Display Headline
Aortic aneurysm: Fluoroquinolones, genetic counseling
Display Headline
Aortic aneurysm: Fluoroquinolones, genetic counseling
Legacy Keywords
thoracic aortic aneurysm, genetic counseling, Houriya Ayoubieh, Gretchen MacCarrick
Legacy Keywords
thoracic aortic aneurysm, genetic counseling, Houriya Ayoubieh, Gretchen MacCarrick
Sections
Disallow All Ads
Content Gating
No Gating (article Unlocked/Free)
Alternative CME
Disqus Comments
Default
Gate On Date
Tue, 08/28/2018 - 15:00
Un-Gate On Date
Tue, 08/28/2018 - 15:00
Use ProPublica
CFC Schedule Remove Status
Tue, 08/28/2018 - 15:00
Article PDF Media

In reply: Aortic aneurysm: Fluoroquinolones, genetic counseling

Article Type
Changed
Tue, 09/04/2018 - 09:17
Display Headline
In reply: Aortic aneurysm: Fluoroquinolones, genetic counseling

In Reply: We thank Drs. Goldstein and Mascitelli for their comments regarding fluoroquinolones and thoracic aortic aneurysms. We acknowledge that fluoroquinolones (particularly ciprofloxacin) have been associated with a risk of aortic aneurysm and dissection based on large observational studies from Taiwan, Canada, and Sweden. Although all of the studies have shown an association between ciprofloxacin and aortic aneurysm, the causative role is not well established. In addition, the numbers of events were very small in these large cohorts of patients. In our large tertiary care practice at Cleveland Clinic, we have very few patients with aortic aneurysm or dissection who have used fluoroquinolones.

We recognize the association; however, our paper was intended to emphasize the more common causes and treatment options that primary care physicians are likely to encounter in routine practice.

We also thank Drs. Ayoubieh and MacCarrick for their comments about genetic counseling. We agree that genetic counseling is important, as is a detailed physical examination for subtle features of genetically mediated aortic aneurysm. In fact, we incorporate the physical examination when patients are seen at our aortic center so as to recognize the physical features. We do routinely recommend screening of first-degree relatives even without significant family history on an individual basis and make appropriate referrals for other conditions that can be seen in these patients. Our article, however, is primarily intended to emphasize the importance of referring these patients for more-focused care at a specialized center, where we incorporate all of the suggestions that were made.

Article PDF
Author and Disclosure Information

Vidyasagar Kalahasti, MD
Cleveland Clinic

Frank Cikach, MD
Cleveland Clinic

Milind Y. Desai, MD, FACC, FAHA, FESC
Cleveland Clinic

Eric E. Roselli, MD, FACS
Cleveland Clinic

Issue
Cleveland Clinic Journal of Medicine - 85(9)
Publications
Topics
Page Number
661-662
Legacy Keywords
thoracic aortic aneurysm, fluroquinolones, genetic counseling, Vidyasagar Kalahasti, Frank Cikach, Milind Desai, Eric Roselli
Sections
Author and Disclosure Information

Vidyasagar Kalahasti, MD
Cleveland Clinic

Frank Cikach, MD
Cleveland Clinic

Milind Y. Desai, MD, FACC, FAHA, FESC
Cleveland Clinic

Eric E. Roselli, MD, FACS
Cleveland Clinic

Author and Disclosure Information

Vidyasagar Kalahasti, MD
Cleveland Clinic

Frank Cikach, MD
Cleveland Clinic

Milind Y. Desai, MD, FACC, FAHA, FESC
Cleveland Clinic

Eric E. Roselli, MD, FACS
Cleveland Clinic

Article PDF
Article PDF
Related Articles

In Reply: We thank Drs. Goldstein and Mascitelli for their comments regarding fluoroquinolones and thoracic aortic aneurysms. We acknowledge that fluoroquinolones (particularly ciprofloxacin) have been associated with a risk of aortic aneurysm and dissection based on large observational studies from Taiwan, Canada, and Sweden. Although all of the studies have shown an association between ciprofloxacin and aortic aneurysm, the causative role is not well established. In addition, the numbers of events were very small in these large cohorts of patients. In our large tertiary care practice at Cleveland Clinic, we have very few patients with aortic aneurysm or dissection who have used fluoroquinolones.

We recognize the association; however, our paper was intended to emphasize the more common causes and treatment options that primary care physicians are likely to encounter in routine practice.

We also thank Drs. Ayoubieh and MacCarrick for their comments about genetic counseling. We agree that genetic counseling is important, as is a detailed physical examination for subtle features of genetically mediated aortic aneurysm. In fact, we incorporate the physical examination when patients are seen at our aortic center so as to recognize the physical features. We do routinely recommend screening of first-degree relatives even without significant family history on an individual basis and make appropriate referrals for other conditions that can be seen in these patients. Our article, however, is primarily intended to emphasize the importance of referring these patients for more-focused care at a specialized center, where we incorporate all of the suggestions that were made.

In Reply: We thank Drs. Goldstein and Mascitelli for their comments regarding fluoroquinolones and thoracic aortic aneurysms. We acknowledge that fluoroquinolones (particularly ciprofloxacin) have been associated with a risk of aortic aneurysm and dissection based on large observational studies from Taiwan, Canada, and Sweden. Although all of the studies have shown an association between ciprofloxacin and aortic aneurysm, the causative role is not well established. In addition, the numbers of events were very small in these large cohorts of patients. In our large tertiary care practice at Cleveland Clinic, we have very few patients with aortic aneurysm or dissection who have used fluoroquinolones.

We recognize the association; however, our paper was intended to emphasize the more common causes and treatment options that primary care physicians are likely to encounter in routine practice.

We also thank Drs. Ayoubieh and MacCarrick for their comments about genetic counseling. We agree that genetic counseling is important, as is a detailed physical examination for subtle features of genetically mediated aortic aneurysm. In fact, we incorporate the physical examination when patients are seen at our aortic center so as to recognize the physical features. We do routinely recommend screening of first-degree relatives even without significant family history on an individual basis and make appropriate referrals for other conditions that can be seen in these patients. Our article, however, is primarily intended to emphasize the importance of referring these patients for more-focused care at a specialized center, where we incorporate all of the suggestions that were made.

Issue
Cleveland Clinic Journal of Medicine - 85(9)
Issue
Cleveland Clinic Journal of Medicine - 85(9)
Page Number
661-662
Page Number
661-662
Publications
Publications
Topics
Article Type
Display Headline
In reply: Aortic aneurysm: Fluoroquinolones, genetic counseling
Display Headline
In reply: Aortic aneurysm: Fluoroquinolones, genetic counseling
Legacy Keywords
thoracic aortic aneurysm, fluroquinolones, genetic counseling, Vidyasagar Kalahasti, Frank Cikach, Milind Desai, Eric Roselli
Legacy Keywords
thoracic aortic aneurysm, fluroquinolones, genetic counseling, Vidyasagar Kalahasti, Frank Cikach, Milind Desai, Eric Roselli
Sections
Disallow All Ads
Content Gating
No Gating (article Unlocked/Free)
Alternative CME
Disqus Comments
Default
Gate On Date
Tue, 08/28/2018 - 15:15
Un-Gate On Date
Tue, 08/28/2018 - 15:15
Use ProPublica
CFC Schedule Remove Status
Tue, 08/28/2018 - 15:15
Article PDF Media

Thoracic aortic aneurysm: How to counsel, when to refer

Article Type
Changed
Mon, 06/03/2019 - 08:47
Display Headline
Thoracic aortic aneurysm: How to counsel, when to refer

Thoracic aortic aneurysm (TAA) needs to be detected, monitored, and managed in a timely manner to prevent a serious consequence such as acute dissection or rupture. But only about 5% of patients experience symptoms before an acute event occurs, and for the other 95% the first “symptom” is often death.1 Most cases are detected either incidentally with echocardiography, computed tomography (CT), or magnetic resonance imaging (MRI) during workup for another condition. Patients may also be diagnosed during workup of a murmur or after a family member is found to have an aneurysm. Therefore, its true incidence is difficult to determine.2

With these facts in mind, how would you manage the following 2 cases?

Case 1: Bicuspid aortic valve, ascending aortic aneurysm

A 45-year-old man with stage 1 hypertension presents for evaluation of a bicuspid aortic valve and ascending aortic aneurysm. He has several first-degree relatives with similar conditions, and his brother recently underwent elective aortic repair. At the urging of his primary care physician, he underwent screening echocardiography, which demonstrated a “dilated root and ascending aorta” 4.6 cm in diameter. He presents today to discuss management options and how the aneurysm could affect his everyday life.

Case 2: Marfan syndrome in a young woman

A 24-year-old woman with Marfan syndrome diagnosed in adolescence presents for annual follow-up. She has many family members with the same condition, and several have undergone prophylactic aortic root repair. Her aortic root has been monitored annually for progression of dilation, and today it is 4.6 cm in diameter, a 3-mm increase from the last measurement. She has grade 2+ aortic insufficiency (on a scale of 1+ to 4+) based on echocardiography, but she has no symptoms. She is curious about what size her aortic root will need to reach for surgery to be considered.

LIKELY UNDERDETECTED

TAA is being detected more often than in the past thanks to better detection methods and heightened awareness among physicians and patients. While an incidence rate of 10.4 per 100,000 patient-years is often cited,3 this figure likely underestimates the true incidence of this clinically silent condition. The most robust data come from studies based on in-hospital diagnostic codes coupled with data from autopsies for out-of-hospital deaths.

Olsson et al,4 in a 2016 study in Sweden, found the incidence of TAA and aortic dissection to be 16.3 per 100,000 per year for men and 9.1 per 100,000 per year for women.

Clouse et al5 reported the incidence of thoracic aortic dissection as 3.5 per 100,000 patient-years, and the same figure for thoracic aortic rupture. 

Aneurysmal disease accounts for 52,000 deaths per year in the United States, making it the 19th most common cause of death.6 These figures are likely lower than the true mortality rate for this condition, given that aortic dissection is often mistaken for acute myocardial infarction or other acute event if an autopsy is not done to confirm the cause of death.7

RISK FACTORS FOR THORACIC AORTIC ANEURYSM

Risk factors for TAA include genetic conditions that lead to aortic medial weakness or destruction such as Loeys-Dietz syndrome and Marfan syndrome.2 In addition, family history is important even in the absence of known genetic mutations. Other risk factors include conditions that increase aortic wall stress, such as hypertension, cocaine abuse, extreme weightlifting, trauma, and aortic coarctation.2

DIAMETER INCREASES WITH AGE, BODY SURFACE AREA

cikach_thoracicaorticaneurysm_f1.jpg
Figure%201.
The thoracic aorta consists of the root and the ascending, arch, and descending segments (Figure 1); the abdominal aorta consists of the suprarenal and infrarenal segments.8,9 These divisions are useful, as aneurysmal disease can be confined to specific locations along the length of the vessel, and the location can affect the clinical presentation and management decisions and lend insight into the pathogenesis.

Normal dimensions for the aortic segments differ depending on age, sex, and body surface area.8,44,45 The size of the aortic root may also vary depending on how it is measured, due to the root’s trefoil shape. Measured sinus to sinus, the root is larger than when measured sinus to commissure on CT angiography or cardiac MRI. It is also larger when measured leading edge to leading edge than inner edge to inner edge on echocardiography.10

TAA is defined as an aortic diameter at least 50% greater than the upper limit of normal.8 

cikach_thoracicaorticaneurysm_t1.jpg
The aorta increases in diameter by 0.7 to 1.9 mm per year if not dilated, and larger-diameter aortas grow faster.11 In addition, men have a larger aortic diameter than women.10 Size-based criteria and indices are useful for defining and monitoring aneurysmal progression, since larger patients tend to have a larger aorta.10  Table 1 lists upper limits of normal values for the ascending and descending aorta by age, sex, and body surface area obtained by Wolak et al in a study using noncontrast CT.10

Geometric changes in the curvature of the ascending aorta, aortic arch, and descending thoracic aorta can occur as the result of hypertension, atherosclerosis, or connective tissue disease. 

 

 

HOW IS TAA DIAGNOSED?

cikach_thoracicaorticaneurysm_t2.jpg
TAA is asymptomatic in most patients and is usually detected on imaging. However, it should be actively looked for in patients who have a family history of Marfan, Loeys-Dietz, or Ehlers-Danlos syndrome or a family history of aortic aneurysm or dissection (not necessarily in a first-degree relative, but more significant in a first-degree relative or in multiple family members across generations), and in patients with a bicuspid aortic valve or autoimmune disease such as Takayasu or giant cell arteritis (Table 2). Table 3 lists the common genetic disorders with their associated mutations and clinical features.

cikach_thoracicaorticaneurysm_t3.jpg
Some patients present with chest pain that may be related to local compression due to the aorta’s large size. Hoarseness, dysphagia, or chronic cough may be a presenting symptom, particularly in patients with descending aortic aneurysm or congenital aortic anomaly.

cikach_thoracicaorticaneurysm_t4.jpg
An abnormal chest radiograph with a prominent aortic shadow or mediastinal widening should prompt further evaluation for TAA. In addition, patients with known abdominal aortic aneurysm should have the rest of the aorta imaged as well to rule out associated TAA.

Imaging tests

cikach_thoracicaorticaneurysm_f2.jpg
Figure%202.%20Echocardiographic%20image%20of%20an%20aneurysmal%20aortic%20root%20(white%20arrow)%20that%20tapers%20to%20normal%20dimensions%20at%20the%20sinotubular%20junction%20(yellow%20arrow)%20and%20ascending%20aorta.%20
TAA can be diagnosed with several imaging tests, each with advantages and disadvantages (Table 4).12 Most commonly used in its diagnosis and follow-up are transthoracic echocardiography (Figure 2 [video 1, video 2]), cardiac-gated computed tomographic angiography (CTA), and MRI (Figure 3 [video 3, video 4, video 5]). Transesophageal echocardiography is not routinely used in diagnosing TAA but is invaluable during surgery to assess aortic valve function and immediate results of aortic repair.

cikach_thoracicaorticaneurysm_f3.jpg
Figure%203.%20An%20aortic%20root%20aneurysm%20in%20a%20patient%20with%20Marfan%20syndrome%20using%20magnetic%20resonance%20angiography.
Which imaging test to use depends on the clinical context as well as the availability and expertise at each institution. For example, screening of first-degree relatives of a patient with thoracic aortic disease typically begins with transthoracic echocardiography and can be escalated to CTA or MRI if an abnormality is detected. Alternatively, patients with connective tissue disease with a particularly severe vascular phenotype such as Loeys-Dietz syndrome should undergo screening with dedicated aortic imaging such as CTA, since this disease can affect the entire aorta and its branch vessels.

cikach_thoracicaorticaneurysm_f4.jpg
Figure%204.%20Top%2C%203D%20reconstruction%20of%20an%20ascending%20aortic%20aneurysm%20in%20a%20patient%20with%20a%20bicuspid%20aortic%20valve.%20Bottom%2C%203D%20reconstruction%20of%20a%20young%20patient%20with%20an%20aortic%20root%20aneurysm.%20Note%20the%20tapering%20from%20the%20aneurysmal%20aortic%20root%20to%20the%20normal-sized%20ascending%20aorta.%20LMCA%20%3D%20left%20main%20coronary%20artery%3B%20RCA%20%3D%20right%20coronary%20artery.
The aortic diameter should be measured perpendicularly to the centerline of flow, which is now easier with dedicated aortic imaging technology and widely available, user-friendly, 3-dimensional reconstruction software (Figure 4 [video 6]).2

It is particularly important to obtain a gated CTA image in patients with aortic root aneurysm to avoid motion artifact and possible erroneous measurements. Gated CTA is done with electrocardiographic synchronization and allows for image processing to correct for cardiac motion.

 

 

HOW IS TAA CLASSIFIED?

TAA can be caused by a variety of inherited and sporadic conditions. These differences in pathogenesis lend themselves to classification of aneurysms into groups. Table 3 highlights the most common conditions associated with TAA.13

Bicuspid aortic valve aortopathy

From 1% to 2% of people have a bicuspid aortic valve, with a 3-to-1 male predominance.14,15 Aortic dilation occurs in 35% to 80% of people who have a bicuspid aortic valve, conferring a risk of dissection 8 times higher than in the general population.16–18

The pathogenic mechanisms that lead to this condition are widely debated, although a combination of genetic defects leading to intrinsic weakening of the aortic wall and hemodynamic effects likely contribute.19 Evidence of hemodynamic contributions to aortic dilation comes from findings that particular patterns of cusp fusion of the bicuspid aortic valve result in changes in transvalvular flow, placing more stress on specific regions of the ascending aorta.20,21 These hemodynamic alterations result in patterns of aortic dilation that depend on cusp fusion and the presence of valvular disease.

Multiple small studies found that replacing bicuspid aortic valves reduced the rate of aortic dilation, suggesting that hemodynamic factors may play a larger role than intrinsic wall properties in genetically susceptible individuals.22,23 However, larger studies are needed before any definitive conclusions can be made.

HOW IS ANEURYSM MANAGED ON AN OUTPATIENT BASIS?

Patients with a new diagnosis of TAA should be referred to a cardiologist with expertise in managing aortic disease or to a cardiac surgeon specializing in aortic surgery, depending on the initial size of the aneurysm.

Control blood pressure with beta-blockers

Medical management for patients with TAA has historically been limited to strict blood pressure control aimed at reducing aortic wall stress, mainly with beta-blockers.

Are angiotensin II receptor blockers (ARBs) beneficial? Studies in a mouse model of Marfan syndrome revealed that the ARB losartan attenuated aortic root growth.24 The results of early, small studies in humans were promising,25–27 but larger randomized trials have shown no advantage of losartan over beta-blockers in slowing aortic root growth.28 These negative results led many to question the effectiveness of losartan, although some point out that no studies have shown even beta-blockers to be beneficial in reducing the clinical end points of death or dissection.29 On the other hand, patients with certain FBN1 mutations respond more readily than others to losartan.30 Additional clinical trials of ARBs in Marfan syndrome are ongoing.

Current guidelines recommend stringent blood pressure control and smoking cessation for patients with a small aneurysm not requiring surgery and for those who are considered unsuitable for surgical or percutaneous intervention (level of evidence C, the lowest).2 For patients with TAA, it is considered reasonable to give beta-blockers. Angiotensin-converting enzyme inhibitors or ARBs may be used in combination with beta-blockers, titrated to the lowest tolerable blood pressure without adverse effects (level of evidence B).2

The recommended target blood pressure is less than 140/90 mm Hg, or 130/80 mm Hg in those with diabetes or chronic kidney disease (level of evidence B).2 However, we recommend more stringent blood pressure control: ie, less than 130/80 mm Hg for all patients with aortic aneurysm and a heart rate goal of 70 beats per minute or less, as tolerated.

Activity restriction

Activity restrictions for patients with TAA are largely based on theory, and certain activities may require more modification than others. For example, heavy lifting should be discouraged, as it may increase blood pressure significantly for short periods of time.2,31 The increased wall stress, in theory, could initiate dissection or rupture. However, moderate-intensity aerobic activity is rarely associated with significant elevations in blood pressure and should be encouraged. Stressful emotional states have been anecdotally associated with aortic dissection; thus, measures to reduce stress may offer some benefit.31

Our recommendations. While there are no published guidelines regarding activity restrictions in patients with TAA, we use a graded approach based on aortic diameter:

  • 4.0 to 4.4 cm—lift no more than 75 pounds
  • 4.5 to 5 cm—lift no more than 50 pounds
  • 5 cm—lift no more than 25 pounds.

We also recommend not lifting anything heavier than half of one’s body weight and to avoid breath-holding or performing the Valsalva maneuver while lifting. Although these recommendations are somewhat arbitrary, based on theory and a large clinical experience at our aortic center, they seem reasonable and practical.

Activity restrictions should be stringent and individualized in patients with Marfan, Loeys-Dietz, or Ehlers-Danlos syndrome due to increased risk of dissection or rupture even if the aorta is normal in size.

We sometimes recommend exercise stress testing to assess the heart rate and blood pressure response to exercise, and we are developing research protocols to help tailor activity recommendations.

 

 

WHEN SHOULD A PATIENT BE REFERRED?

To a cardiologist at the time of diagnosis

As soon as TAA is diagnosed, the patient should be referred to a cardiologist who has special interest in aortic disease. This will allow for appropriate and timely decisions about medical management, imaging, follow-up, and referral to surgery. Additional recommendations for screening of family members and referral to clinical geneticists can be discussed at this juncture. Activity restrictions should be reviewed at the initial evaluation.

To a surgeon relatively early

Size thresholds for surgical intervention are discussed below, but one should not wait until these thresholds are reached to send the patient for surgical consultation. It is beneficial to the state of mind of a potential surgical candidate to have early discussions pertaining to the types of operations available, their outcomes, and associated risks and benefits. If a patient’s aortic size remains stable over time, he or she may be followed by the cardiologist until significant size or growth has been documented, at which time the patient and surgeon can reconvene to discuss options for definitive treatment.

To a clinical geneticist

If 1 or more first-degree relatives of a patient with TAA or dissection are found to have aneurysmal disease, referral to a clinical geneticist is very important for genetic testing of multiple genes that have been implicated in thoracic aortic aneurysm and dissection.

WHEN SHOULD TAA BE REPAIRED?

Surgery to prevent rupture or dissection remains the definitive treatment of TAA when size thresholds are reached, and symptomatic aneurysm should be operated on regardless of the size. However, rarely are thoracic aneurysms symptomatic unless they rupture or dissect. The size criteria are based on underlying genetic etiology if known and on the behavior and natural course of TAA.

Size and other factors

Treatment should be tailored to the patient’s clinical scenario, family history, and estimated risk of rupture or dissection, balanced against the individual center’s outcomes of elective aortic replacement.32 For example, young and otherwise healthy patients with TAA and a family history of aortic dissection (who may be more likely to have connective tissue disorders such as Marfan syndrome, Loeys-Dietz syndrome, or vascular Ehler-Danlos syndrome) may elect to undergo repair when the aneurysm reaches or nearly reaches the diameter of that of the family member’s aorta when dissection occurred.2 On the other hand, TAA of degenerative etiology (eg, related to smoking or hypertension) measuring less than 5.5 cm in an older patient with comorbidities poses a lower risk of a catastrophic event such as dissection or rupture than the risk of surgery.11

Thresholds for surgery. Once the diameter of the ascending aorta reaches 6 cm, the likelihood of an acute dissection is 31%.11 A similar threshold is reached for the descending aorta at a size of 7 cm.11 Therefore, to avoid high-risk emergency surgery on an acutely dissected aorta, surgery on an ascending aortic aneurysm of degenerative etiology is usually suggested when the aneurysm reaches 5.5 cm or a documented growth rate greater than 0.5 cm/year.2,33

Additionally, in patients already undergoing surgery for valvular or coronary disease, prophylactic aortic replacement is recommended if the ascending aorta is larger than 4.5 cm. The threshold for intervention is lower in patients with connective tissue disease (> 5.0 cm for Marfan syndrome, 4.4–4.6 cm for Loeys-Dietz syndrome).2,33

Observational studies suggest that the risk of aortic complications in patients with bicuspid aortic valve aortopathy is low overall, though significantly greater than in the general population.18,34,35 These findings led to changes in the 2014 American College of Cardiology/American Heart Association guidelines on valvular heart disease,36 suggesting a surgical threshold of 5.5 cm in the absence of significant valve disease or family history of dissection of an aorta of smaller diameter.

A 2015 study of dissection risk in patients with bicuspid aortic valve aortopathy by our group found a dramatic increase in risk of aortic dissection for ascending aortic diameters greater than 5.3 cm, and a gradual increase in risk for aortic root diameters greater than 5.0 cm.37 In addition, a near-constant 3% to 4% risk of dissection was present for aortic diameters ranging from 4.7 cm to 5.0 cm, revealing that watchful waiting carries its own inherent risks.37 In our surgical experience with this population, the hospital mortality rate and risk of stroke from aortic surgery were 0.25% and 0.75%, respectively.37 Thus, the decision to operate for aortic aneurysm in the setting of a bicuspid aortic valve should take into account patient-specific factors and institutional outcomes.

A statement of clarification in the American College of Cardiology/American Heart Association guidelines was published in 2015, recommending surgery for patients with an aortic diameter of 5.0 cm or greater if the patient is at low risk and the surgery is performed by an experienced surgical team at a center with established surgical expertise in this condition.38 However, current recommendations are for surgery at 5.5 cm if the above conditions are not met.

Ratio of aortic cross-sectional area to height

Although size alone has long been used to guide surgical intervention, a recent review from the International Registry of Aortic Dissection revealed that 59% of patients suffered aortic dissection at diameters less than 5.5 cm, and that patients with certain connective tissue diseases such as Loeys-Dietz syndrome or familial thoracic aneurysm and dissection had a documented propensity for dissection at smaller diameters.39–41

Size indices such as the aortic cross-sectional area indexed to height have been implemented in guidelines for certain patient populations (eg, 10 cm2/m in Marfan syndrome) and provide better risk stratification than size cutoffs alone.2,42

The ratio of aortic cross-sectional area to the patient’s height has also been applied to patients with bicuspid aortic valve-associated aortopathy and to those with a dilated aorta and a tricuspid aortic valve.43,44 Notably, a ratio greater than 10 cm2/m has been associated with aortic dissection in these groups, and this cutoff provides better stratification for prediction of death than traditional size metrics.27,28

 

 

HOW SHOULD PATIENTS BE SCREENED? WHAT FOLLOW-UP IS NECESSARY?

Initial screening and follow-up

Follow-up of TAA depends on the initial aortic size or rate of growth, or both. For patients presenting for the first time with TAA, it is reasonable to obtain definitive aortic imaging with CT or magnetic resonance angiography (MRA), then to repeat imaging at 6 months to document stability. If the aortic dimensions remain stable, then annual follow-up with CT or MRA is reasonable.2

cikach_thoracicaorticaneurysm_f5.jpg
Figure%205.%20Initial%20screening%20and%20follow-up%20of%20thoracic%20aortic%20aneurysm.
MRA may be preferable to CT over the long term to limit radiation exposure.2 Echocardiography should be used if the aortic root or ascending aorta is well visualized, but in most patients the view of the mid to distal ascending aorta is limited. Echocardiography also offers evaluation of left ventricular size and function and allows for follow-up of aortic valve disease.

Our flow chart of initial screening and follow-up is shown in Figure 5.

Screening of family members

In our center, we routinely recommend screening of all first-degree relatives of patients with TAA. Aortic imaging with echocardiography plus CT or MRI should be considered to detect asymptomatic disease.2 In patients with a strong family history (ie, multiple relatives affected with aortic aneurysm, dissection, or sudden cardiac death), genetic screening and testing for known mutations are recommended for the patient as well as for the family members.

If a mutation is identified in a family, then first-degree relatives should undergo genetic screening for the mutation and aortic imaging.2 Imaging in second-degree relatives may also be considered if one or more first-degree relatives are found to have aortic dilation.2

We recommend similar screening of first-degree family members of patients with bicuspid aortic valve aortopathy. In patients with young children, we recommend obtaining an echocardiogram of the child to look for a bicuspid aortic valve or aortic dilation. If an abnormality is detected or suspected, dedicated imaging with MRA to assess aortic dimensions is warranted.

BACK TO OUR PATIENT WITH A BICUSPID AORTIC VALVE

Our patient with a bicuspid aortic valve had a 4.6-cm root, an ascending aortic aneurysm, and several affected family members.

We would obtain dedicated aortic imaging at this patient’s initial visit with either gated CT with contrast or MRA, and we would obtain a cardioaortic surgery consult. We would repeat these studies at a follow-up visit 6 months later to detect any aortic growth compared with initial studies, and follow up annually thereafter. Echocardiography can also be done at the initial visit to determine if valvular disease is present that may influence clinical decisions.

Surgery would likely be recommended once the root reached a maximum area-to-height ratio greater than 10 cm2/m, or if the valve became severely dysfunctional during follow-up.

BACK TO OUR PATIENT WITH MARFAN SYNDROME

The young woman with Marfan syndrome has a 4.6-cm aortic root aneurysm and 2+ aortic insufficiency. Her question pertains to the threshold at which an operation would be considered. This question is complicated and is influenced by several concurrent clinical features in her presentation.

Starting with size criteria, patients with Marfan syndrome should be considered for elective aortic root repair at a diameter greater than 5 cm. However, an aortic cross-sectional area-to-height ratio greater than 10 cm2/m may provide a more robust metric for clinical decision-making than aortic diameter alone. Additional factors such as degree of aortic insufficiency and deleterious left ventricular remodeling may urge one to consider aortic root repair at a diameter of 4.5 cm.

These factors, including rate of growth and the surgeon’s assessment about his or her ability to preserve the aortic valve during repair, should be considered collectively in this scenario.

References
  1. Elefteriades JA, Farkas EA. Thoracic aortic aneurysm clinically pertinent controversies and uncertainties. J Am Coll Cardiol 2010; 55(9):841–857. doi:10.1016/j.jacc.2009.08.084
  2. Hiratzka LF, Bakris GL, Beckman JA, et al. 2010 ACCF/AHA/AATS/ACR/ASA/SCA/SCAI/SIR/STS/SVM guidelines for the diagnosis and management of patients with thoracic aortic disease: executive summary. Anesth Analg 2010; 111(2):279–315. doi:10.1213/ANE.0b013e3181dd869b
  3. Clouse WD, Hallett JW Jr, Schaff HV, Gayari MM, Ilstrup DM, Melton LJ 3rd. Improved prognosis of thoracic aortic aneurysms: a population-based study. JAMA 1998; 280(22):1926–1929. pmid:9851478
  4. Olsson C, Thelin S, Ståhle E, Ekbom A, Granath F. Thoracic aortic aneurysm and dissection: increasing prevalence and improved outcomes reported in a nationwide population-based study of more than 14,000 cases from 1987 to 2002. Circulation 2006; 114(24):2611–2618. doi:10.1161/CIRCULATIONAHA.106.630400
  5. Clouse WD, Hallett JW Jr, Schaff HV, et al. Acute aortic dissection: population-based incidence compared with degenerative aortic aneurysm rupture. Mayo Clin Proc 2004; 79(2):176–180. pmid:14959911
  6. US Centers for Disease Control and Prevention (CDC). National Center for Injury Prevention and Control. WISQARS leading causes of death reports, 1999 – 2007. https://webappa.cdc.gov/sasweb/ncipc/leadcaus10.html. Accessed May 21, 2018.
  7. Hansen MS, Nogareda GJ, Hutchison SJ. Frequency of and inappropriate treatment of misdiagnosis of acute aortic dissection. Am J Cardiol 2007; 99(6):852–856. doi:10.1016/j.amjcard.2006.10.055
  8. Goldfinger JZ, Halperin JL, Marin ML, Stewart AS, Eagle KA, Fuster V. Thoracic aortic aneurysm and dissection. J Am Coll Cardiol 2014; 64(16):1725–1739. doi:10.1016/j.jacc.2014.08.025
  9. Kumar V, Abbas A, Aster J. Robbins and Cotran Pathologic Basis of Disease. 9th ed. Philadelphia, PA: Elsevier/Saunders; 2015.
  10. Wolak A, Gransar H, Thomson LE, et al. Aortic size assessment by noncontrast cardiac computed tomography: normal limits by age, gender, and body surface area. JACC Cardiovasc Imaging 2008; 1(2):200–209. doi:10.1016/j.jcmg.2007.11.005
  11. Elefteriades JA. Natural history of thoracic aortic aneurysms: indications for surgery, and surgical versus nonsurgical risks. Ann Thorac Surg 2002; 74(5):S1877–S1880; discussion S1892–S1898. pmid:12440685
  12. Smith AD, Schoenhagen P. CT imaging for acute aortic syndrome. Cleve Clin J Med 2008; 75(1):7–17. pmid:18236724
  13. Cury M, Zeidan F, Lobato AC. Aortic disease in the young: genetic aneurysm syndromes, connective tissue disorders, and familial aortic aneurysms and dissections. Int J Vasc Med 2013(2013); 2013:267215. doi:10.1155/2013/267215
  14. Hoffman JI, Kaplan S. The incidence of congenital heart disease. J Am Coll Cardiol 2002; 39(12):1890–1900. doi:10.1016/S0735-1097(02)01886-7
  15. Fedak PW, Verma S, David TE, Leask RL, Weisel RD, Butany J. Clinical and pathophysiological implications of a bicuspid aortic valve. Circulation 2002; 106(8):900–904. pmid:12186790
  16. Della Corte A, Bancone C, Quarto C, et al. Predictors of ascending aortic dilatation with bicuspid aortic valve: a wide spectrum of disease expression. Eur J Cardiothorac Surg 2007; 31(3):397–405. doi:10.1016/j.ejcts.2006.12.006
  17. Jackson V, Petrini J, Caidahl K, et al. Bicuspid aortic valve leaflet morphology in relation to aortic root morphology: a study of 300 patients undergoing open-heart surgery. Eur J Cardiothorac Surg 2011; 40(3):e118–e124. doi:10.1016/j.ejcts.2011.04.014
  18. Michelena HI, Khanna AD, Mahoney D, et al. Incidence of aortic complications in patients with bicuspid aortic valves. JAMA 2011; 306(10):1104–1112. doi:10.1001/jama.2011.1286
  19. Verma S, Siu SC. Aortic dilatation in patients with bicuspid aortic valve. N Engl J Med 2014; 370(20):1920–1929. doi:10.1056/NEJMra1207059
  20. Barker AJ, Markl M, Bürk J, et al. Bicuspid aortic valve is associated with altered wall shear stress in the ascending aorta. Circ Cardiovasc Imaging 2012; 5(4):457–466. doi:10.1161/CIRCIMAGING.112.973370
  21. Hope MD, Hope TA, Meadows AK, et al. Bicuspid aortic valve: four-dimensional MR evaluation of ascending aortic systolic flow patterns. Radiology 2010; 255(1):53–61. doi:10.1148/radiol.09091437
  22. Abdulkareem N, Soppa G, Jones S, Valencia O, Smelt J, Jahangiri M. Dilatation of the remaining aorta after aortic valve or aortic root replacement in patients with bicuspid aortic valve: a 5-year follow-up. Ann Thorac Surg 2013; 96(1):43–49. doi:10.1016/j.athoracsur.2013.03.086
  23. Regeer MV, Versteegh MI, Klautz RJ, et al. Effect of aortic valve replacement on aortic root dilatation rate in patients with bicuspid and tricuspid aortic valves. Ann Thorac Surg 2016; 102(6):1981–1987. doi:10.1016/j.athoracsur.2016.05.038
  24. Habashi JP, Judge DP, Holm TM, et al. Losartan, an AT1 antagonist, prevents aortic aneurysm in a mouse model of Marfan syndrome. Science 2006; 312(5770):117–121. doi:10.1126/science.1124287
  25. Brooke BS, Habashi JP, Judge DP, Patel N, Loeys B, Dietz HC 3rd. Angiotensin II blockade and aortic-root dilation in Marfan’s syndrome. N Engl J Med 2008; 358(26):2787–2795. doi:10.1056/NEJMoa0706585
  26. Chiu HH, Wu MH, Wang JK, et al. Losartan added to ß-blockade therapy for aortic root dilation in Marfan syndrome: a randomized, open-label pilot study. Mayo Clin Proc 2013; 88(3):271–276. doi:10.1016/j.mayocp.2012.11.005
  27. Groenink M, den Hartog AW, Franken R, et al. Losartan reduces aortic dilatation rate in adults with Marfan syndrome: a randomized controlled trial. Eur Heart J 2013; 34(45):3491–3500. doi:10.1093/eurheartj/eht334
  28. Lacro RV, Dietz HC, Sleeper LA, et al; Pediatric Heart Network Investigators. Atenolol versus losartan in children and young adults with Marfan’s syndrome. N Engl J Med 2014; 371(22):2061–2071. doi:10.1056/NEJMoa1404731
  29. Ziganshin BA, Mukherjee SK, Elefteriades JA, et al. Atenolol versus losartan in Marfan’s syndrome (letters). N Engl J Med 2015; 372(10):977–981. doi:10.1056/NEJMc1500128
  30. Franken R, den Hartog AW, Radonic T, et al. Beneficial outcome of losartan therapy depends on type of FBN1 mutation in Marfan syndrome. Circ Cardiovasc Genet 2015; 8(2):383–388. doi:10.1161/CIRCGENETICS.114.000950
  31. Elefteriades JA. Thoracic aortic aneurysm: reading the enemy’s playbook. Curr Probl Cardiol 2008; 33(5):203–277. doi:10.1016/j.cpcardiol.2008.01.004
  32. Idrees JJ, Roselli EE, Lowry AM, et al. Outcomes after elective proximal aortic replacement: a matched comparison of isolated versus multicomponent operations. Ann Thorac Surg 2016; 101(6):2185–2192. doi:10.1016/j.athoracsur.2015.12.026
  33. Hiratzka LF, Creager MA, Isselbacher EM, et al. Surgery for aortic dilatation in patients with bicuspid aortic valves: a statement of clarification from the American College of Cardiology/American Heart Association Task Force on Clinical Practice Guidelines. J Thorac Cardiovasc Surg 2016; 151(4):959–966. doi:10.1016/j.jtcvs.2015.12.001
  34. Tzemos N, Therrien J, Yip J, et al. Outcomes in adults with bicuspid aortic valves. JAMA 2008; 300(11):1317–1325. doi:10.1001/jama.300.11.1317
  35. Davies RR, Goldstein LJ, Coady MA, et al. Yearly rupture or dissection rates for thoracic aortic aneurysms: simple prediction based on size. Ann Thorac Surg 2002; 73(1):17–28. pmid:11834007
  36. Nishimura RA, Otto CM, Bono RO, et al. 2014 AHA/ACC guideline for the management of patients with valvular heart disease: executive summary: a report of the American College of Cardiology/American heart Association Task Force on Practice Guidelines. Circulation 2014; 129(23):2440–2492. doi:10.1161/CIR.0000000000000029
  37. Wojnarski CM, Svensson LG, Roselli EE, et al. Aortic dissection in patients with bicuspid aortic valve–associated aneurysms. Ann Thorac Surg 2015; 100(5):1666–1674. doi:10.1016/j.athoracsur.2015.04.126
  38. Hiratzka LF, Creager MA, Isselbacher EM, et al. Surgery for aortic dilatation in patients with bicuspid aortic valves: a statement of clarification from the American College of Cardiology/American Heart Association Task Force on Clinical Practice Guidelines. Circulation 2016; 133(7):680–686. doi:10.1161/CIR.0000000000000331
  39. Pape LA, Tsai TT, Isselbacher EM, et al; International Registry of Acute Aortic Dissection (IRAD) Investigators. Aortic diameter > or = 5.5 cm is not a good predictor of type A aortic dissection: observations from the International Registry of Acute Aortic Dissection (IRAD). Circulation 2007; 116(10):1120–1127. doi:10.1161/CIRCULATIONAHA.107.702720
  40. Loeys BL, Schwarze U, Holm T, et al. Aneurysm syndromes caused by mutations in the TGF-beta receptor. N Engl J Med 2006; 355(8):788–798. doi:10.1056/NEJMoa055695
  41. Guo DC, Pannu H, Tran-Fadulu V, et al. Mutations in smooth muscle alpha-actin (ACTA2) lead to thoracic aortic aneurysms and dissections. Nat Genet 2007; 39(12):1488–1493. doi:10.1038/ng.2007.6
  42. Svensson LG, Khitin L. Aortic cross-sectional area/height ratio timing of aortic surgery in asymptomatic patients with Marfan syndrome. J Thorac Cardiovasc Surg 2002; 123(2):360–361. pmid:11828302
  43. Svensson LG, Kim KH, Lytle BW, Cosgrove DM. Relationship of aortic cross-sectional area to height ratio and the risk of aortic dissection in patients with bicuspid aortic valves. J Thorac Cardiovasc Surg 2003; 126(3):892–893. pmid:14502185
  44. Masri A, Kalahasti V, Svensson LG, et al. Aortic cross-sectional area/height ratio and outcomes in patients with a trileaflet aortic valve and a dilated aorta. Circulation 2016; 134(22):1724–1737. doi:10.1161/CIRCULATIONAHA.116.022995
Article PDF
Author and Disclosure Information

Frank Cikach, MD
Cleveland Clinic Lerner College of Medicine of Case Western Reserve University, Cleveland, OH

Milind Y. Desai, MD, FACC, FAHA, FESC
Haslam Family Endowed Chair in Cardiovascular Medicine, Department of Cardiovascular Medicine, Medical Director, Aorta Center, Heart and Vascular Institute, Cleveland Clinic; Professor, Cleveland Clinic Lerner College of Medicine of Case Western Reserve University, Cleveland, OH

Eric E. Roselli, MD, FACS
Chief, Adult Cardiac Surgery, Surgical Director, Aorta Center, Director, Heart and Vascular Condition Centers, Heart and Vascular Institute, Cleveland Clinic; Clinical Assistant Professor, Cleveland Clinic Lerner College of Medicine of Case Western Reserve University, Cleveland, OH

Vidyasagar Kalahasti, MD
Director, Marfan and Other Connective Tissue Disorders Clinic, Aorta Center, Heart and Vascular Institute, Cleveland Clinic; Assistant Professor, Cleveland Clinic Lerner College of Medicine of Case Western Reserve University, Cleveland, OH

Address: Vidyasagar Kalahasti, MD, Heart and Vascular Institute, J1-5, Cleveland Clinic, 9500 Euclid Avenue, Cleveland, OH 44195; kalahav@ccf.org

Dr. Roselli has disclosed consulting for Bolton Medical, Medtronic, Sorin Group, and W.L. Gore & Associates and teaching and speaking for Cook Medical, Edwards Lifesciences, Sorin Group, St. Jude Medical, and Terumo.  

Issue
Cleveland Clinic Journal of Medicine - 85(6)
Publications
Topics
Page Number
481-492
Legacy Keywords
thoracic aortic aneurysm, aorta, bicuspid aortic valve, Marfan syndrome, Loeys-Dietz syndrome, Ehlers-Danlos syndrome, dilation, mutations, beta-blockers, dissection, rupture, Frank Cikach, Milind Desai, Eric Roselli, Vidyasagar Kalahasti
Sections
Author and Disclosure Information

Frank Cikach, MD
Cleveland Clinic Lerner College of Medicine of Case Western Reserve University, Cleveland, OH

Milind Y. Desai, MD, FACC, FAHA, FESC
Haslam Family Endowed Chair in Cardiovascular Medicine, Department of Cardiovascular Medicine, Medical Director, Aorta Center, Heart and Vascular Institute, Cleveland Clinic; Professor, Cleveland Clinic Lerner College of Medicine of Case Western Reserve University, Cleveland, OH

Eric E. Roselli, MD, FACS
Chief, Adult Cardiac Surgery, Surgical Director, Aorta Center, Director, Heart and Vascular Condition Centers, Heart and Vascular Institute, Cleveland Clinic; Clinical Assistant Professor, Cleveland Clinic Lerner College of Medicine of Case Western Reserve University, Cleveland, OH

Vidyasagar Kalahasti, MD
Director, Marfan and Other Connective Tissue Disorders Clinic, Aorta Center, Heart and Vascular Institute, Cleveland Clinic; Assistant Professor, Cleveland Clinic Lerner College of Medicine of Case Western Reserve University, Cleveland, OH

Address: Vidyasagar Kalahasti, MD, Heart and Vascular Institute, J1-5, Cleveland Clinic, 9500 Euclid Avenue, Cleveland, OH 44195; kalahav@ccf.org

Dr. Roselli has disclosed consulting for Bolton Medical, Medtronic, Sorin Group, and W.L. Gore & Associates and teaching and speaking for Cook Medical, Edwards Lifesciences, Sorin Group, St. Jude Medical, and Terumo.  

Author and Disclosure Information

Frank Cikach, MD
Cleveland Clinic Lerner College of Medicine of Case Western Reserve University, Cleveland, OH

Milind Y. Desai, MD, FACC, FAHA, FESC
Haslam Family Endowed Chair in Cardiovascular Medicine, Department of Cardiovascular Medicine, Medical Director, Aorta Center, Heart and Vascular Institute, Cleveland Clinic; Professor, Cleveland Clinic Lerner College of Medicine of Case Western Reserve University, Cleveland, OH

Eric E. Roselli, MD, FACS
Chief, Adult Cardiac Surgery, Surgical Director, Aorta Center, Director, Heart and Vascular Condition Centers, Heart and Vascular Institute, Cleveland Clinic; Clinical Assistant Professor, Cleveland Clinic Lerner College of Medicine of Case Western Reserve University, Cleveland, OH

Vidyasagar Kalahasti, MD
Director, Marfan and Other Connective Tissue Disorders Clinic, Aorta Center, Heart and Vascular Institute, Cleveland Clinic; Assistant Professor, Cleveland Clinic Lerner College of Medicine of Case Western Reserve University, Cleveland, OH

Address: Vidyasagar Kalahasti, MD, Heart and Vascular Institute, J1-5, Cleveland Clinic, 9500 Euclid Avenue, Cleveland, OH 44195; kalahav@ccf.org

Dr. Roselli has disclosed consulting for Bolton Medical, Medtronic, Sorin Group, and W.L. Gore & Associates and teaching and speaking for Cook Medical, Edwards Lifesciences, Sorin Group, St. Jude Medical, and Terumo.  

Article PDF
Article PDF
Related Articles

Thoracic aortic aneurysm (TAA) needs to be detected, monitored, and managed in a timely manner to prevent a serious consequence such as acute dissection or rupture. But only about 5% of patients experience symptoms before an acute event occurs, and for the other 95% the first “symptom” is often death.1 Most cases are detected either incidentally with echocardiography, computed tomography (CT), or magnetic resonance imaging (MRI) during workup for another condition. Patients may also be diagnosed during workup of a murmur or after a family member is found to have an aneurysm. Therefore, its true incidence is difficult to determine.2

With these facts in mind, how would you manage the following 2 cases?

Case 1: Bicuspid aortic valve, ascending aortic aneurysm

A 45-year-old man with stage 1 hypertension presents for evaluation of a bicuspid aortic valve and ascending aortic aneurysm. He has several first-degree relatives with similar conditions, and his brother recently underwent elective aortic repair. At the urging of his primary care physician, he underwent screening echocardiography, which demonstrated a “dilated root and ascending aorta” 4.6 cm in diameter. He presents today to discuss management options and how the aneurysm could affect his everyday life.

Case 2: Marfan syndrome in a young woman

A 24-year-old woman with Marfan syndrome diagnosed in adolescence presents for annual follow-up. She has many family members with the same condition, and several have undergone prophylactic aortic root repair. Her aortic root has been monitored annually for progression of dilation, and today it is 4.6 cm in diameter, a 3-mm increase from the last measurement. She has grade 2+ aortic insufficiency (on a scale of 1+ to 4+) based on echocardiography, but she has no symptoms. She is curious about what size her aortic root will need to reach for surgery to be considered.

LIKELY UNDERDETECTED

TAA is being detected more often than in the past thanks to better detection methods and heightened awareness among physicians and patients. While an incidence rate of 10.4 per 100,000 patient-years is often cited,3 this figure likely underestimates the true incidence of this clinically silent condition. The most robust data come from studies based on in-hospital diagnostic codes coupled with data from autopsies for out-of-hospital deaths.

Olsson et al,4 in a 2016 study in Sweden, found the incidence of TAA and aortic dissection to be 16.3 per 100,000 per year for men and 9.1 per 100,000 per year for women.

Clouse et al5 reported the incidence of thoracic aortic dissection as 3.5 per 100,000 patient-years, and the same figure for thoracic aortic rupture. 

Aneurysmal disease accounts for 52,000 deaths per year in the United States, making it the 19th most common cause of death.6 These figures are likely lower than the true mortality rate for this condition, given that aortic dissection is often mistaken for acute myocardial infarction or other acute event if an autopsy is not done to confirm the cause of death.7

RISK FACTORS FOR THORACIC AORTIC ANEURYSM

Risk factors for TAA include genetic conditions that lead to aortic medial weakness or destruction such as Loeys-Dietz syndrome and Marfan syndrome.2 In addition, family history is important even in the absence of known genetic mutations. Other risk factors include conditions that increase aortic wall stress, such as hypertension, cocaine abuse, extreme weightlifting, trauma, and aortic coarctation.2

DIAMETER INCREASES WITH AGE, BODY SURFACE AREA

cikach_thoracicaorticaneurysm_f1.jpg
Figure%201.
The thoracic aorta consists of the root and the ascending, arch, and descending segments (Figure 1); the abdominal aorta consists of the suprarenal and infrarenal segments.8,9 These divisions are useful, as aneurysmal disease can be confined to specific locations along the length of the vessel, and the location can affect the clinical presentation and management decisions and lend insight into the pathogenesis.

Normal dimensions for the aortic segments differ depending on age, sex, and body surface area.8,44,45 The size of the aortic root may also vary depending on how it is measured, due to the root’s trefoil shape. Measured sinus to sinus, the root is larger than when measured sinus to commissure on CT angiography or cardiac MRI. It is also larger when measured leading edge to leading edge than inner edge to inner edge on echocardiography.10

TAA is defined as an aortic diameter at least 50% greater than the upper limit of normal.8 

cikach_thoracicaorticaneurysm_t1.jpg
The aorta increases in diameter by 0.7 to 1.9 mm per year if not dilated, and larger-diameter aortas grow faster.11 In addition, men have a larger aortic diameter than women.10 Size-based criteria and indices are useful for defining and monitoring aneurysmal progression, since larger patients tend to have a larger aorta.10  Table 1 lists upper limits of normal values for the ascending and descending aorta by age, sex, and body surface area obtained by Wolak et al in a study using noncontrast CT.10

Geometric changes in the curvature of the ascending aorta, aortic arch, and descending thoracic aorta can occur as the result of hypertension, atherosclerosis, or connective tissue disease. 

 

 

HOW IS TAA DIAGNOSED?

cikach_thoracicaorticaneurysm_t2.jpg
TAA is asymptomatic in most patients and is usually detected on imaging. However, it should be actively looked for in patients who have a family history of Marfan, Loeys-Dietz, or Ehlers-Danlos syndrome or a family history of aortic aneurysm or dissection (not necessarily in a first-degree relative, but more significant in a first-degree relative or in multiple family members across generations), and in patients with a bicuspid aortic valve or autoimmune disease such as Takayasu or giant cell arteritis (Table 2). Table 3 lists the common genetic disorders with their associated mutations and clinical features.

cikach_thoracicaorticaneurysm_t3.jpg
Some patients present with chest pain that may be related to local compression due to the aorta’s large size. Hoarseness, dysphagia, or chronic cough may be a presenting symptom, particularly in patients with descending aortic aneurysm or congenital aortic anomaly.

cikach_thoracicaorticaneurysm_t4.jpg
An abnormal chest radiograph with a prominent aortic shadow or mediastinal widening should prompt further evaluation for TAA. In addition, patients with known abdominal aortic aneurysm should have the rest of the aorta imaged as well to rule out associated TAA.

Imaging tests

cikach_thoracicaorticaneurysm_f2.jpg
Figure%202.%20Echocardiographic%20image%20of%20an%20aneurysmal%20aortic%20root%20(white%20arrow)%20that%20tapers%20to%20normal%20dimensions%20at%20the%20sinotubular%20junction%20(yellow%20arrow)%20and%20ascending%20aorta.%20
TAA can be diagnosed with several imaging tests, each with advantages and disadvantages (Table 4).12 Most commonly used in its diagnosis and follow-up are transthoracic echocardiography (Figure 2 [video 1, video 2]), cardiac-gated computed tomographic angiography (CTA), and MRI (Figure 3 [video 3, video 4, video 5]). Transesophageal echocardiography is not routinely used in diagnosing TAA but is invaluable during surgery to assess aortic valve function and immediate results of aortic repair.

cikach_thoracicaorticaneurysm_f3.jpg
Figure%203.%20An%20aortic%20root%20aneurysm%20in%20a%20patient%20with%20Marfan%20syndrome%20using%20magnetic%20resonance%20angiography.
Which imaging test to use depends on the clinical context as well as the availability and expertise at each institution. For example, screening of first-degree relatives of a patient with thoracic aortic disease typically begins with transthoracic echocardiography and can be escalated to CTA or MRI if an abnormality is detected. Alternatively, patients with connective tissue disease with a particularly severe vascular phenotype such as Loeys-Dietz syndrome should undergo screening with dedicated aortic imaging such as CTA, since this disease can affect the entire aorta and its branch vessels.

cikach_thoracicaorticaneurysm_f4.jpg
Figure%204.%20Top%2C%203D%20reconstruction%20of%20an%20ascending%20aortic%20aneurysm%20in%20a%20patient%20with%20a%20bicuspid%20aortic%20valve.%20Bottom%2C%203D%20reconstruction%20of%20a%20young%20patient%20with%20an%20aortic%20root%20aneurysm.%20Note%20the%20tapering%20from%20the%20aneurysmal%20aortic%20root%20to%20the%20normal-sized%20ascending%20aorta.%20LMCA%20%3D%20left%20main%20coronary%20artery%3B%20RCA%20%3D%20right%20coronary%20artery.
The aortic diameter should be measured perpendicularly to the centerline of flow, which is now easier with dedicated aortic imaging technology and widely available, user-friendly, 3-dimensional reconstruction software (Figure 4 [video 6]).2

It is particularly important to obtain a gated CTA image in patients with aortic root aneurysm to avoid motion artifact and possible erroneous measurements. Gated CTA is done with electrocardiographic synchronization and allows for image processing to correct for cardiac motion.

 

 

HOW IS TAA CLASSIFIED?

TAA can be caused by a variety of inherited and sporadic conditions. These differences in pathogenesis lend themselves to classification of aneurysms into groups. Table 3 highlights the most common conditions associated with TAA.13

Bicuspid aortic valve aortopathy

From 1% to 2% of people have a bicuspid aortic valve, with a 3-to-1 male predominance.14,15 Aortic dilation occurs in 35% to 80% of people who have a bicuspid aortic valve, conferring a risk of dissection 8 times higher than in the general population.16–18

The pathogenic mechanisms that lead to this condition are widely debated, although a combination of genetic defects leading to intrinsic weakening of the aortic wall and hemodynamic effects likely contribute.19 Evidence of hemodynamic contributions to aortic dilation comes from findings that particular patterns of cusp fusion of the bicuspid aortic valve result in changes in transvalvular flow, placing more stress on specific regions of the ascending aorta.20,21 These hemodynamic alterations result in patterns of aortic dilation that depend on cusp fusion and the presence of valvular disease.

Multiple small studies found that replacing bicuspid aortic valves reduced the rate of aortic dilation, suggesting that hemodynamic factors may play a larger role than intrinsic wall properties in genetically susceptible individuals.22,23 However, larger studies are needed before any definitive conclusions can be made.

HOW IS ANEURYSM MANAGED ON AN OUTPATIENT BASIS?

Patients with a new diagnosis of TAA should be referred to a cardiologist with expertise in managing aortic disease or to a cardiac surgeon specializing in aortic surgery, depending on the initial size of the aneurysm.

Control blood pressure with beta-blockers

Medical management for patients with TAA has historically been limited to strict blood pressure control aimed at reducing aortic wall stress, mainly with beta-blockers.

Are angiotensin II receptor blockers (ARBs) beneficial? Studies in a mouse model of Marfan syndrome revealed that the ARB losartan attenuated aortic root growth.24 The results of early, small studies in humans were promising,25–27 but larger randomized trials have shown no advantage of losartan over beta-blockers in slowing aortic root growth.28 These negative results led many to question the effectiveness of losartan, although some point out that no studies have shown even beta-blockers to be beneficial in reducing the clinical end points of death or dissection.29 On the other hand, patients with certain FBN1 mutations respond more readily than others to losartan.30 Additional clinical trials of ARBs in Marfan syndrome are ongoing.

Current guidelines recommend stringent blood pressure control and smoking cessation for patients with a small aneurysm not requiring surgery and for those who are considered unsuitable for surgical or percutaneous intervention (level of evidence C, the lowest).2 For patients with TAA, it is considered reasonable to give beta-blockers. Angiotensin-converting enzyme inhibitors or ARBs may be used in combination with beta-blockers, titrated to the lowest tolerable blood pressure without adverse effects (level of evidence B).2

The recommended target blood pressure is less than 140/90 mm Hg, or 130/80 mm Hg in those with diabetes or chronic kidney disease (level of evidence B).2 However, we recommend more stringent blood pressure control: ie, less than 130/80 mm Hg for all patients with aortic aneurysm and a heart rate goal of 70 beats per minute or less, as tolerated.

Activity restriction

Activity restrictions for patients with TAA are largely based on theory, and certain activities may require more modification than others. For example, heavy lifting should be discouraged, as it may increase blood pressure significantly for short periods of time.2,31 The increased wall stress, in theory, could initiate dissection or rupture. However, moderate-intensity aerobic activity is rarely associated with significant elevations in blood pressure and should be encouraged. Stressful emotional states have been anecdotally associated with aortic dissection; thus, measures to reduce stress may offer some benefit.31

Our recommendations. While there are no published guidelines regarding activity restrictions in patients with TAA, we use a graded approach based on aortic diameter:

  • 4.0 to 4.4 cm—lift no more than 75 pounds
  • 4.5 to 5 cm—lift no more than 50 pounds
  • 5 cm—lift no more than 25 pounds.

We also recommend not lifting anything heavier than half of one’s body weight and to avoid breath-holding or performing the Valsalva maneuver while lifting. Although these recommendations are somewhat arbitrary, based on theory and a large clinical experience at our aortic center, they seem reasonable and practical.

Activity restrictions should be stringent and individualized in patients with Marfan, Loeys-Dietz, or Ehlers-Danlos syndrome due to increased risk of dissection or rupture even if the aorta is normal in size.

We sometimes recommend exercise stress testing to assess the heart rate and blood pressure response to exercise, and we are developing research protocols to help tailor activity recommendations.

 

 

WHEN SHOULD A PATIENT BE REFERRED?

To a cardiologist at the time of diagnosis

As soon as TAA is diagnosed, the patient should be referred to a cardiologist who has special interest in aortic disease. This will allow for appropriate and timely decisions about medical management, imaging, follow-up, and referral to surgery. Additional recommendations for screening of family members and referral to clinical geneticists can be discussed at this juncture. Activity restrictions should be reviewed at the initial evaluation.

To a surgeon relatively early

Size thresholds for surgical intervention are discussed below, but one should not wait until these thresholds are reached to send the patient for surgical consultation. It is beneficial to the state of mind of a potential surgical candidate to have early discussions pertaining to the types of operations available, their outcomes, and associated risks and benefits. If a patient’s aortic size remains stable over time, he or she may be followed by the cardiologist until significant size or growth has been documented, at which time the patient and surgeon can reconvene to discuss options for definitive treatment.

To a clinical geneticist

If 1 or more first-degree relatives of a patient with TAA or dissection are found to have aneurysmal disease, referral to a clinical geneticist is very important for genetic testing of multiple genes that have been implicated in thoracic aortic aneurysm and dissection.

WHEN SHOULD TAA BE REPAIRED?

Surgery to prevent rupture or dissection remains the definitive treatment of TAA when size thresholds are reached, and symptomatic aneurysm should be operated on regardless of the size. However, rarely are thoracic aneurysms symptomatic unless they rupture or dissect. The size criteria are based on underlying genetic etiology if known and on the behavior and natural course of TAA.

Size and other factors

Treatment should be tailored to the patient’s clinical scenario, family history, and estimated risk of rupture or dissection, balanced against the individual center’s outcomes of elective aortic replacement.32 For example, young and otherwise healthy patients with TAA and a family history of aortic dissection (who may be more likely to have connective tissue disorders such as Marfan syndrome, Loeys-Dietz syndrome, or vascular Ehler-Danlos syndrome) may elect to undergo repair when the aneurysm reaches or nearly reaches the diameter of that of the family member’s aorta when dissection occurred.2 On the other hand, TAA of degenerative etiology (eg, related to smoking or hypertension) measuring less than 5.5 cm in an older patient with comorbidities poses a lower risk of a catastrophic event such as dissection or rupture than the risk of surgery.11

Thresholds for surgery. Once the diameter of the ascending aorta reaches 6 cm, the likelihood of an acute dissection is 31%.11 A similar threshold is reached for the descending aorta at a size of 7 cm.11 Therefore, to avoid high-risk emergency surgery on an acutely dissected aorta, surgery on an ascending aortic aneurysm of degenerative etiology is usually suggested when the aneurysm reaches 5.5 cm or a documented growth rate greater than 0.5 cm/year.2,33

Additionally, in patients already undergoing surgery for valvular or coronary disease, prophylactic aortic replacement is recommended if the ascending aorta is larger than 4.5 cm. The threshold for intervention is lower in patients with connective tissue disease (> 5.0 cm for Marfan syndrome, 4.4–4.6 cm for Loeys-Dietz syndrome).2,33

Observational studies suggest that the risk of aortic complications in patients with bicuspid aortic valve aortopathy is low overall, though significantly greater than in the general population.18,34,35 These findings led to changes in the 2014 American College of Cardiology/American Heart Association guidelines on valvular heart disease,36 suggesting a surgical threshold of 5.5 cm in the absence of significant valve disease or family history of dissection of an aorta of smaller diameter.

A 2015 study of dissection risk in patients with bicuspid aortic valve aortopathy by our group found a dramatic increase in risk of aortic dissection for ascending aortic diameters greater than 5.3 cm, and a gradual increase in risk for aortic root diameters greater than 5.0 cm.37 In addition, a near-constant 3% to 4% risk of dissection was present for aortic diameters ranging from 4.7 cm to 5.0 cm, revealing that watchful waiting carries its own inherent risks.37 In our surgical experience with this population, the hospital mortality rate and risk of stroke from aortic surgery were 0.25% and 0.75%, respectively.37 Thus, the decision to operate for aortic aneurysm in the setting of a bicuspid aortic valve should take into account patient-specific factors and institutional outcomes.

A statement of clarification in the American College of Cardiology/American Heart Association guidelines was published in 2015, recommending surgery for patients with an aortic diameter of 5.0 cm or greater if the patient is at low risk and the surgery is performed by an experienced surgical team at a center with established surgical expertise in this condition.38 However, current recommendations are for surgery at 5.5 cm if the above conditions are not met.

Ratio of aortic cross-sectional area to height

Although size alone has long been used to guide surgical intervention, a recent review from the International Registry of Aortic Dissection revealed that 59% of patients suffered aortic dissection at diameters less than 5.5 cm, and that patients with certain connective tissue diseases such as Loeys-Dietz syndrome or familial thoracic aneurysm and dissection had a documented propensity for dissection at smaller diameters.39–41

Size indices such as the aortic cross-sectional area indexed to height have been implemented in guidelines for certain patient populations (eg, 10 cm2/m in Marfan syndrome) and provide better risk stratification than size cutoffs alone.2,42

The ratio of aortic cross-sectional area to the patient’s height has also been applied to patients with bicuspid aortic valve-associated aortopathy and to those with a dilated aorta and a tricuspid aortic valve.43,44 Notably, a ratio greater than 10 cm2/m has been associated with aortic dissection in these groups, and this cutoff provides better stratification for prediction of death than traditional size metrics.27,28

 

 

HOW SHOULD PATIENTS BE SCREENED? WHAT FOLLOW-UP IS NECESSARY?

Initial screening and follow-up

Follow-up of TAA depends on the initial aortic size or rate of growth, or both. For patients presenting for the first time with TAA, it is reasonable to obtain definitive aortic imaging with CT or magnetic resonance angiography (MRA), then to repeat imaging at 6 months to document stability. If the aortic dimensions remain stable, then annual follow-up with CT or MRA is reasonable.2

cikach_thoracicaorticaneurysm_f5.jpg
Figure%205.%20Initial%20screening%20and%20follow-up%20of%20thoracic%20aortic%20aneurysm.
MRA may be preferable to CT over the long term to limit radiation exposure.2 Echocardiography should be used if the aortic root or ascending aorta is well visualized, but in most patients the view of the mid to distal ascending aorta is limited. Echocardiography also offers evaluation of left ventricular size and function and allows for follow-up of aortic valve disease.

Our flow chart of initial screening and follow-up is shown in Figure 5.

Screening of family members

In our center, we routinely recommend screening of all first-degree relatives of patients with TAA. Aortic imaging with echocardiography plus CT or MRI should be considered to detect asymptomatic disease.2 In patients with a strong family history (ie, multiple relatives affected with aortic aneurysm, dissection, or sudden cardiac death), genetic screening and testing for known mutations are recommended for the patient as well as for the family members.

If a mutation is identified in a family, then first-degree relatives should undergo genetic screening for the mutation and aortic imaging.2 Imaging in second-degree relatives may also be considered if one or more first-degree relatives are found to have aortic dilation.2

We recommend similar screening of first-degree family members of patients with bicuspid aortic valve aortopathy. In patients with young children, we recommend obtaining an echocardiogram of the child to look for a bicuspid aortic valve or aortic dilation. If an abnormality is detected or suspected, dedicated imaging with MRA to assess aortic dimensions is warranted.

BACK TO OUR PATIENT WITH A BICUSPID AORTIC VALVE

Our patient with a bicuspid aortic valve had a 4.6-cm root, an ascending aortic aneurysm, and several affected family members.

We would obtain dedicated aortic imaging at this patient’s initial visit with either gated CT with contrast or MRA, and we would obtain a cardioaortic surgery consult. We would repeat these studies at a follow-up visit 6 months later to detect any aortic growth compared with initial studies, and follow up annually thereafter. Echocardiography can also be done at the initial visit to determine if valvular disease is present that may influence clinical decisions.

Surgery would likely be recommended once the root reached a maximum area-to-height ratio greater than 10 cm2/m, or if the valve became severely dysfunctional during follow-up.

BACK TO OUR PATIENT WITH MARFAN SYNDROME

The young woman with Marfan syndrome has a 4.6-cm aortic root aneurysm and 2+ aortic insufficiency. Her question pertains to the threshold at which an operation would be considered. This question is complicated and is influenced by several concurrent clinical features in her presentation.

Starting with size criteria, patients with Marfan syndrome should be considered for elective aortic root repair at a diameter greater than 5 cm. However, an aortic cross-sectional area-to-height ratio greater than 10 cm2/m may provide a more robust metric for clinical decision-making than aortic diameter alone. Additional factors such as degree of aortic insufficiency and deleterious left ventricular remodeling may urge one to consider aortic root repair at a diameter of 4.5 cm.

These factors, including rate of growth and the surgeon’s assessment about his or her ability to preserve the aortic valve during repair, should be considered collectively in this scenario.

Thoracic aortic aneurysm (TAA) needs to be detected, monitored, and managed in a timely manner to prevent a serious consequence such as acute dissection or rupture. But only about 5% of patients experience symptoms before an acute event occurs, and for the other 95% the first “symptom” is often death.1 Most cases are detected either incidentally with echocardiography, computed tomography (CT), or magnetic resonance imaging (MRI) during workup for another condition. Patients may also be diagnosed during workup of a murmur or after a family member is found to have an aneurysm. Therefore, its true incidence is difficult to determine.2

With these facts in mind, how would you manage the following 2 cases?

Case 1: Bicuspid aortic valve, ascending aortic aneurysm

A 45-year-old man with stage 1 hypertension presents for evaluation of a bicuspid aortic valve and ascending aortic aneurysm. He has several first-degree relatives with similar conditions, and his brother recently underwent elective aortic repair. At the urging of his primary care physician, he underwent screening echocardiography, which demonstrated a “dilated root and ascending aorta” 4.6 cm in diameter. He presents today to discuss management options and how the aneurysm could affect his everyday life.

Case 2: Marfan syndrome in a young woman

A 24-year-old woman with Marfan syndrome diagnosed in adolescence presents for annual follow-up. She has many family members with the same condition, and several have undergone prophylactic aortic root repair. Her aortic root has been monitored annually for progression of dilation, and today it is 4.6 cm in diameter, a 3-mm increase from the last measurement. She has grade 2+ aortic insufficiency (on a scale of 1+ to 4+) based on echocardiography, but she has no symptoms. She is curious about what size her aortic root will need to reach for surgery to be considered.

LIKELY UNDERDETECTED

TAA is being detected more often than in the past thanks to better detection methods and heightened awareness among physicians and patients. While an incidence rate of 10.4 per 100,000 patient-years is often cited,3 this figure likely underestimates the true incidence of this clinically silent condition. The most robust data come from studies based on in-hospital diagnostic codes coupled with data from autopsies for out-of-hospital deaths.

Olsson et al,4 in a 2016 study in Sweden, found the incidence of TAA and aortic dissection to be 16.3 per 100,000 per year for men and 9.1 per 100,000 per year for women.

Clouse et al5 reported the incidence of thoracic aortic dissection as 3.5 per 100,000 patient-years, and the same figure for thoracic aortic rupture. 

Aneurysmal disease accounts for 52,000 deaths per year in the United States, making it the 19th most common cause of death.6 These figures are likely lower than the true mortality rate for this condition, given that aortic dissection is often mistaken for acute myocardial infarction or other acute event if an autopsy is not done to confirm the cause of death.7

RISK FACTORS FOR THORACIC AORTIC ANEURYSM

Risk factors for TAA include genetic conditions that lead to aortic medial weakness or destruction such as Loeys-Dietz syndrome and Marfan syndrome.2 In addition, family history is important even in the absence of known genetic mutations. Other risk factors include conditions that increase aortic wall stress, such as hypertension, cocaine abuse, extreme weightlifting, trauma, and aortic coarctation.2

DIAMETER INCREASES WITH AGE, BODY SURFACE AREA

cikach_thoracicaorticaneurysm_f1.jpg
Figure%201.
The thoracic aorta consists of the root and the ascending, arch, and descending segments (Figure 1); the abdominal aorta consists of the suprarenal and infrarenal segments.8,9 These divisions are useful, as aneurysmal disease can be confined to specific locations along the length of the vessel, and the location can affect the clinical presentation and management decisions and lend insight into the pathogenesis.

Normal dimensions for the aortic segments differ depending on age, sex, and body surface area.8,44,45 The size of the aortic root may also vary depending on how it is measured, due to the root’s trefoil shape. Measured sinus to sinus, the root is larger than when measured sinus to commissure on CT angiography or cardiac MRI. It is also larger when measured leading edge to leading edge than inner edge to inner edge on echocardiography.10

TAA is defined as an aortic diameter at least 50% greater than the upper limit of normal.8 

cikach_thoracicaorticaneurysm_t1.jpg
The aorta increases in diameter by 0.7 to 1.9 mm per year if not dilated, and larger-diameter aortas grow faster.11 In addition, men have a larger aortic diameter than women.10 Size-based criteria and indices are useful for defining and monitoring aneurysmal progression, since larger patients tend to have a larger aorta.10  Table 1 lists upper limits of normal values for the ascending and descending aorta by age, sex, and body surface area obtained by Wolak et al in a study using noncontrast CT.10

Geometric changes in the curvature of the ascending aorta, aortic arch, and descending thoracic aorta can occur as the result of hypertension, atherosclerosis, or connective tissue disease. 

 

 

HOW IS TAA DIAGNOSED?

cikach_thoracicaorticaneurysm_t2.jpg
TAA is asymptomatic in most patients and is usually detected on imaging. However, it should be actively looked for in patients who have a family history of Marfan, Loeys-Dietz, or Ehlers-Danlos syndrome or a family history of aortic aneurysm or dissection (not necessarily in a first-degree relative, but more significant in a first-degree relative or in multiple family members across generations), and in patients with a bicuspid aortic valve or autoimmune disease such as Takayasu or giant cell arteritis (Table 2). Table 3 lists the common genetic disorders with their associated mutations and clinical features.

cikach_thoracicaorticaneurysm_t3.jpg
Some patients present with chest pain that may be related to local compression due to the aorta’s large size. Hoarseness, dysphagia, or chronic cough may be a presenting symptom, particularly in patients with descending aortic aneurysm or congenital aortic anomaly.

cikach_thoracicaorticaneurysm_t4.jpg
An abnormal chest radiograph with a prominent aortic shadow or mediastinal widening should prompt further evaluation for TAA. In addition, patients with known abdominal aortic aneurysm should have the rest of the aorta imaged as well to rule out associated TAA.

Imaging tests

cikach_thoracicaorticaneurysm_f2.jpg
Figure%202.%20Echocardiographic%20image%20of%20an%20aneurysmal%20aortic%20root%20(white%20arrow)%20that%20tapers%20to%20normal%20dimensions%20at%20the%20sinotubular%20junction%20(yellow%20arrow)%20and%20ascending%20aorta.%20
TAA can be diagnosed with several imaging tests, each with advantages and disadvantages (Table 4).12 Most commonly used in its diagnosis and follow-up are transthoracic echocardiography (Figure 2 [video 1, video 2]), cardiac-gated computed tomographic angiography (CTA), and MRI (Figure 3 [video 3, video 4, video 5]). Transesophageal echocardiography is not routinely used in diagnosing TAA but is invaluable during surgery to assess aortic valve function and immediate results of aortic repair.

cikach_thoracicaorticaneurysm_f3.jpg
Figure%203.%20An%20aortic%20root%20aneurysm%20in%20a%20patient%20with%20Marfan%20syndrome%20using%20magnetic%20resonance%20angiography.
Which imaging test to use depends on the clinical context as well as the availability and expertise at each institution. For example, screening of first-degree relatives of a patient with thoracic aortic disease typically begins with transthoracic echocardiography and can be escalated to CTA or MRI if an abnormality is detected. Alternatively, patients with connective tissue disease with a particularly severe vascular phenotype such as Loeys-Dietz syndrome should undergo screening with dedicated aortic imaging such as CTA, since this disease can affect the entire aorta and its branch vessels.

cikach_thoracicaorticaneurysm_f4.jpg
Figure%204.%20Top%2C%203D%20reconstruction%20of%20an%20ascending%20aortic%20aneurysm%20in%20a%20patient%20with%20a%20bicuspid%20aortic%20valve.%20Bottom%2C%203D%20reconstruction%20of%20a%20young%20patient%20with%20an%20aortic%20root%20aneurysm.%20Note%20the%20tapering%20from%20the%20aneurysmal%20aortic%20root%20to%20the%20normal-sized%20ascending%20aorta.%20LMCA%20%3D%20left%20main%20coronary%20artery%3B%20RCA%20%3D%20right%20coronary%20artery.
The aortic diameter should be measured perpendicularly to the centerline of flow, which is now easier with dedicated aortic imaging technology and widely available, user-friendly, 3-dimensional reconstruction software (Figure 4 [video 6]).2

It is particularly important to obtain a gated CTA image in patients with aortic root aneurysm to avoid motion artifact and possible erroneous measurements. Gated CTA is done with electrocardiographic synchronization and allows for image processing to correct for cardiac motion.

 

 

HOW IS TAA CLASSIFIED?

TAA can be caused by a variety of inherited and sporadic conditions. These differences in pathogenesis lend themselves to classification of aneurysms into groups. Table 3 highlights the most common conditions associated with TAA.13

Bicuspid aortic valve aortopathy

From 1% to 2% of people have a bicuspid aortic valve, with a 3-to-1 male predominance.14,15 Aortic dilation occurs in 35% to 80% of people who have a bicuspid aortic valve, conferring a risk of dissection 8 times higher than in the general population.16–18

The pathogenic mechanisms that lead to this condition are widely debated, although a combination of genetic defects leading to intrinsic weakening of the aortic wall and hemodynamic effects likely contribute.19 Evidence of hemodynamic contributions to aortic dilation comes from findings that particular patterns of cusp fusion of the bicuspid aortic valve result in changes in transvalvular flow, placing more stress on specific regions of the ascending aorta.20,21 These hemodynamic alterations result in patterns of aortic dilation that depend on cusp fusion and the presence of valvular disease.

Multiple small studies found that replacing bicuspid aortic valves reduced the rate of aortic dilation, suggesting that hemodynamic factors may play a larger role than intrinsic wall properties in genetically susceptible individuals.22,23 However, larger studies are needed before any definitive conclusions can be made.

HOW IS ANEURYSM MANAGED ON AN OUTPATIENT BASIS?

Patients with a new diagnosis of TAA should be referred to a cardiologist with expertise in managing aortic disease or to a cardiac surgeon specializing in aortic surgery, depending on the initial size of the aneurysm.

Control blood pressure with beta-blockers

Medical management for patients with TAA has historically been limited to strict blood pressure control aimed at reducing aortic wall stress, mainly with beta-blockers.

Are angiotensin II receptor blockers (ARBs) beneficial? Studies in a mouse model of Marfan syndrome revealed that the ARB losartan attenuated aortic root growth.24 The results of early, small studies in humans were promising,25–27 but larger randomized trials have shown no advantage of losartan over beta-blockers in slowing aortic root growth.28 These negative results led many to question the effectiveness of losartan, although some point out that no studies have shown even beta-blockers to be beneficial in reducing the clinical end points of death or dissection.29 On the other hand, patients with certain FBN1 mutations respond more readily than others to losartan.30 Additional clinical trials of ARBs in Marfan syndrome are ongoing.

Current guidelines recommend stringent blood pressure control and smoking cessation for patients with a small aneurysm not requiring surgery and for those who are considered unsuitable for surgical or percutaneous intervention (level of evidence C, the lowest).2 For patients with TAA, it is considered reasonable to give beta-blockers. Angiotensin-converting enzyme inhibitors or ARBs may be used in combination with beta-blockers, titrated to the lowest tolerable blood pressure without adverse effects (level of evidence B).2

The recommended target blood pressure is less than 140/90 mm Hg, or 130/80 mm Hg in those with diabetes or chronic kidney disease (level of evidence B).2 However, we recommend more stringent blood pressure control: ie, less than 130/80 mm Hg for all patients with aortic aneurysm and a heart rate goal of 70 beats per minute or less, as tolerated.

Activity restriction

Activity restrictions for patients with TAA are largely based on theory, and certain activities may require more modification than others. For example, heavy lifting should be discouraged, as it may increase blood pressure significantly for short periods of time.2,31 The increased wall stress, in theory, could initiate dissection or rupture. However, moderate-intensity aerobic activity is rarely associated with significant elevations in blood pressure and should be encouraged. Stressful emotional states have been anecdotally associated with aortic dissection; thus, measures to reduce stress may offer some benefit.31

Our recommendations. While there are no published guidelines regarding activity restrictions in patients with TAA, we use a graded approach based on aortic diameter:

  • 4.0 to 4.4 cm—lift no more than 75 pounds
  • 4.5 to 5 cm—lift no more than 50 pounds
  • 5 cm—lift no more than 25 pounds.

We also recommend not lifting anything heavier than half of one’s body weight and to avoid breath-holding or performing the Valsalva maneuver while lifting. Although these recommendations are somewhat arbitrary, based on theory and a large clinical experience at our aortic center, they seem reasonable and practical.

Activity restrictions should be stringent and individualized in patients with Marfan, Loeys-Dietz, or Ehlers-Danlos syndrome due to increased risk of dissection or rupture even if the aorta is normal in size.

We sometimes recommend exercise stress testing to assess the heart rate and blood pressure response to exercise, and we are developing research protocols to help tailor activity recommendations.

 

 

WHEN SHOULD A PATIENT BE REFERRED?

To a cardiologist at the time of diagnosis

As soon as TAA is diagnosed, the patient should be referred to a cardiologist who has special interest in aortic disease. This will allow for appropriate and timely decisions about medical management, imaging, follow-up, and referral to surgery. Additional recommendations for screening of family members and referral to clinical geneticists can be discussed at this juncture. Activity restrictions should be reviewed at the initial evaluation.

To a surgeon relatively early

Size thresholds for surgical intervention are discussed below, but one should not wait until these thresholds are reached to send the patient for surgical consultation. It is beneficial to the state of mind of a potential surgical candidate to have early discussions pertaining to the types of operations available, their outcomes, and associated risks and benefits. If a patient’s aortic size remains stable over time, he or she may be followed by the cardiologist until significant size or growth has been documented, at which time the patient and surgeon can reconvene to discuss options for definitive treatment.

To a clinical geneticist

If 1 or more first-degree relatives of a patient with TAA or dissection are found to have aneurysmal disease, referral to a clinical geneticist is very important for genetic testing of multiple genes that have been implicated in thoracic aortic aneurysm and dissection.

WHEN SHOULD TAA BE REPAIRED?

Surgery to prevent rupture or dissection remains the definitive treatment of TAA when size thresholds are reached, and symptomatic aneurysm should be operated on regardless of the size. However, rarely are thoracic aneurysms symptomatic unless they rupture or dissect. The size criteria are based on underlying genetic etiology if known and on the behavior and natural course of TAA.

Size and other factors

Treatment should be tailored to the patient’s clinical scenario, family history, and estimated risk of rupture or dissection, balanced against the individual center’s outcomes of elective aortic replacement.32 For example, young and otherwise healthy patients with TAA and a family history of aortic dissection (who may be more likely to have connective tissue disorders such as Marfan syndrome, Loeys-Dietz syndrome, or vascular Ehler-Danlos syndrome) may elect to undergo repair when the aneurysm reaches or nearly reaches the diameter of that of the family member’s aorta when dissection occurred.2 On the other hand, TAA of degenerative etiology (eg, related to smoking or hypertension) measuring less than 5.5 cm in an older patient with comorbidities poses a lower risk of a catastrophic event such as dissection or rupture than the risk of surgery.11

Thresholds for surgery. Once the diameter of the ascending aorta reaches 6 cm, the likelihood of an acute dissection is 31%.11 A similar threshold is reached for the descending aorta at a size of 7 cm.11 Therefore, to avoid high-risk emergency surgery on an acutely dissected aorta, surgery on an ascending aortic aneurysm of degenerative etiology is usually suggested when the aneurysm reaches 5.5 cm or a documented growth rate greater than 0.5 cm/year.2,33

Additionally, in patients already undergoing surgery for valvular or coronary disease, prophylactic aortic replacement is recommended if the ascending aorta is larger than 4.5 cm. The threshold for intervention is lower in patients with connective tissue disease (> 5.0 cm for Marfan syndrome, 4.4–4.6 cm for Loeys-Dietz syndrome).2,33

Observational studies suggest that the risk of aortic complications in patients with bicuspid aortic valve aortopathy is low overall, though significantly greater than in the general population.18,34,35 These findings led to changes in the 2014 American College of Cardiology/American Heart Association guidelines on valvular heart disease,36 suggesting a surgical threshold of 5.5 cm in the absence of significant valve disease or family history of dissection of an aorta of smaller diameter.

A 2015 study of dissection risk in patients with bicuspid aortic valve aortopathy by our group found a dramatic increase in risk of aortic dissection for ascending aortic diameters greater than 5.3 cm, and a gradual increase in risk for aortic root diameters greater than 5.0 cm.37 In addition, a near-constant 3% to 4% risk of dissection was present for aortic diameters ranging from 4.7 cm to 5.0 cm, revealing that watchful waiting carries its own inherent risks.37 In our surgical experience with this population, the hospital mortality rate and risk of stroke from aortic surgery were 0.25% and 0.75%, respectively.37 Thus, the decision to operate for aortic aneurysm in the setting of a bicuspid aortic valve should take into account patient-specific factors and institutional outcomes.

A statement of clarification in the American College of Cardiology/American Heart Association guidelines was published in 2015, recommending surgery for patients with an aortic diameter of 5.0 cm or greater if the patient is at low risk and the surgery is performed by an experienced surgical team at a center with established surgical expertise in this condition.38 However, current recommendations are for surgery at 5.5 cm if the above conditions are not met.

Ratio of aortic cross-sectional area to height

Although size alone has long been used to guide surgical intervention, a recent review from the International Registry of Aortic Dissection revealed that 59% of patients suffered aortic dissection at diameters less than 5.5 cm, and that patients with certain connective tissue diseases such as Loeys-Dietz syndrome or familial thoracic aneurysm and dissection had a documented propensity for dissection at smaller diameters.39–41

Size indices such as the aortic cross-sectional area indexed to height have been implemented in guidelines for certain patient populations (eg, 10 cm2/m in Marfan syndrome) and provide better risk stratification than size cutoffs alone.2,42

The ratio of aortic cross-sectional area to the patient’s height has also been applied to patients with bicuspid aortic valve-associated aortopathy and to those with a dilated aorta and a tricuspid aortic valve.43,44 Notably, a ratio greater than 10 cm2/m has been associated with aortic dissection in these groups, and this cutoff provides better stratification for prediction of death than traditional size metrics.27,28

 

 

HOW SHOULD PATIENTS BE SCREENED? WHAT FOLLOW-UP IS NECESSARY?

Initial screening and follow-up

Follow-up of TAA depends on the initial aortic size or rate of growth, or both. For patients presenting for the first time with TAA, it is reasonable to obtain definitive aortic imaging with CT or magnetic resonance angiography (MRA), then to repeat imaging at 6 months to document stability. If the aortic dimensions remain stable, then annual follow-up with CT or MRA is reasonable.2

cikach_thoracicaorticaneurysm_f5.jpg
Figure%205.%20Initial%20screening%20and%20follow-up%20of%20thoracic%20aortic%20aneurysm.
MRA may be preferable to CT over the long term to limit radiation exposure.2 Echocardiography should be used if the aortic root or ascending aorta is well visualized, but in most patients the view of the mid to distal ascending aorta is limited. Echocardiography also offers evaluation of left ventricular size and function and allows for follow-up of aortic valve disease.

Our flow chart of initial screening and follow-up is shown in Figure 5.

Screening of family members

In our center, we routinely recommend screening of all first-degree relatives of patients with TAA. Aortic imaging with echocardiography plus CT or MRI should be considered to detect asymptomatic disease.2 In patients with a strong family history (ie, multiple relatives affected with aortic aneurysm, dissection, or sudden cardiac death), genetic screening and testing for known mutations are recommended for the patient as well as for the family members.

If a mutation is identified in a family, then first-degree relatives should undergo genetic screening for the mutation and aortic imaging.2 Imaging in second-degree relatives may also be considered if one or more first-degree relatives are found to have aortic dilation.2

We recommend similar screening of first-degree family members of patients with bicuspid aortic valve aortopathy. In patients with young children, we recommend obtaining an echocardiogram of the child to look for a bicuspid aortic valve or aortic dilation. If an abnormality is detected or suspected, dedicated imaging with MRA to assess aortic dimensions is warranted.

BACK TO OUR PATIENT WITH A BICUSPID AORTIC VALVE

Our patient with a bicuspid aortic valve had a 4.6-cm root, an ascending aortic aneurysm, and several affected family members.

We would obtain dedicated aortic imaging at this patient’s initial visit with either gated CT with contrast or MRA, and we would obtain a cardioaortic surgery consult. We would repeat these studies at a follow-up visit 6 months later to detect any aortic growth compared with initial studies, and follow up annually thereafter. Echocardiography can also be done at the initial visit to determine if valvular disease is present that may influence clinical decisions.

Surgery would likely be recommended once the root reached a maximum area-to-height ratio greater than 10 cm2/m, or if the valve became severely dysfunctional during follow-up.

BACK TO OUR PATIENT WITH MARFAN SYNDROME

The young woman with Marfan syndrome has a 4.6-cm aortic root aneurysm and 2+ aortic insufficiency. Her question pertains to the threshold at which an operation would be considered. This question is complicated and is influenced by several concurrent clinical features in her presentation.

Starting with size criteria, patients with Marfan syndrome should be considered for elective aortic root repair at a diameter greater than 5 cm. However, an aortic cross-sectional area-to-height ratio greater than 10 cm2/m may provide a more robust metric for clinical decision-making than aortic diameter alone. Additional factors such as degree of aortic insufficiency and deleterious left ventricular remodeling may urge one to consider aortic root repair at a diameter of 4.5 cm.

These factors, including rate of growth and the surgeon’s assessment about his or her ability to preserve the aortic valve during repair, should be considered collectively in this scenario.

References
  1. Elefteriades JA, Farkas EA. Thoracic aortic aneurysm clinically pertinent controversies and uncertainties. J Am Coll Cardiol 2010; 55(9):841–857. doi:10.1016/j.jacc.2009.08.084
  2. Hiratzka LF, Bakris GL, Beckman JA, et al. 2010 ACCF/AHA/AATS/ACR/ASA/SCA/SCAI/SIR/STS/SVM guidelines for the diagnosis and management of patients with thoracic aortic disease: executive summary. Anesth Analg 2010; 111(2):279–315. doi:10.1213/ANE.0b013e3181dd869b
  3. Clouse WD, Hallett JW Jr, Schaff HV, Gayari MM, Ilstrup DM, Melton LJ 3rd. Improved prognosis of thoracic aortic aneurysms: a population-based study. JAMA 1998; 280(22):1926–1929. pmid:9851478
  4. Olsson C, Thelin S, Ståhle E, Ekbom A, Granath F. Thoracic aortic aneurysm and dissection: increasing prevalence and improved outcomes reported in a nationwide population-based study of more than 14,000 cases from 1987 to 2002. Circulation 2006; 114(24):2611–2618. doi:10.1161/CIRCULATIONAHA.106.630400
  5. Clouse WD, Hallett JW Jr, Schaff HV, et al. Acute aortic dissection: population-based incidence compared with degenerative aortic aneurysm rupture. Mayo Clin Proc 2004; 79(2):176–180. pmid:14959911
  6. US Centers for Disease Control and Prevention (CDC). National Center for Injury Prevention and Control. WISQARS leading causes of death reports, 1999 – 2007. https://webappa.cdc.gov/sasweb/ncipc/leadcaus10.html. Accessed May 21, 2018.
  7. Hansen MS, Nogareda GJ, Hutchison SJ. Frequency of and inappropriate treatment of misdiagnosis of acute aortic dissection. Am J Cardiol 2007; 99(6):852–856. doi:10.1016/j.amjcard.2006.10.055
  8. Goldfinger JZ, Halperin JL, Marin ML, Stewart AS, Eagle KA, Fuster V. Thoracic aortic aneurysm and dissection. J Am Coll Cardiol 2014; 64(16):1725–1739. doi:10.1016/j.jacc.2014.08.025
  9. Kumar V, Abbas A, Aster J. Robbins and Cotran Pathologic Basis of Disease. 9th ed. Philadelphia, PA: Elsevier/Saunders; 2015.
  10. Wolak A, Gransar H, Thomson LE, et al. Aortic size assessment by noncontrast cardiac computed tomography: normal limits by age, gender, and body surface area. JACC Cardiovasc Imaging 2008; 1(2):200–209. doi:10.1016/j.jcmg.2007.11.005
  11. Elefteriades JA. Natural history of thoracic aortic aneurysms: indications for surgery, and surgical versus nonsurgical risks. Ann Thorac Surg 2002; 74(5):S1877–S1880; discussion S1892–S1898. pmid:12440685
  12. Smith AD, Schoenhagen P. CT imaging for acute aortic syndrome. Cleve Clin J Med 2008; 75(1):7–17. pmid:18236724
  13. Cury M, Zeidan F, Lobato AC. Aortic disease in the young: genetic aneurysm syndromes, connective tissue disorders, and familial aortic aneurysms and dissections. Int J Vasc Med 2013(2013); 2013:267215. doi:10.1155/2013/267215
  14. Hoffman JI, Kaplan S. The incidence of congenital heart disease. J Am Coll Cardiol 2002; 39(12):1890–1900. doi:10.1016/S0735-1097(02)01886-7
  15. Fedak PW, Verma S, David TE, Leask RL, Weisel RD, Butany J. Clinical and pathophysiological implications of a bicuspid aortic valve. Circulation 2002; 106(8):900–904. pmid:12186790
  16. Della Corte A, Bancone C, Quarto C, et al. Predictors of ascending aortic dilatation with bicuspid aortic valve: a wide spectrum of disease expression. Eur J Cardiothorac Surg 2007; 31(3):397–405. doi:10.1016/j.ejcts.2006.12.006
  17. Jackson V, Petrini J, Caidahl K, et al. Bicuspid aortic valve leaflet morphology in relation to aortic root morphology: a study of 300 patients undergoing open-heart surgery. Eur J Cardiothorac Surg 2011; 40(3):e118–e124. doi:10.1016/j.ejcts.2011.04.014
  18. Michelena HI, Khanna AD, Mahoney D, et al. Incidence of aortic complications in patients with bicuspid aortic valves. JAMA 2011; 306(10):1104–1112. doi:10.1001/jama.2011.1286
  19. Verma S, Siu SC. Aortic dilatation in patients with bicuspid aortic valve. N Engl J Med 2014; 370(20):1920–1929. doi:10.1056/NEJMra1207059
  20. Barker AJ, Markl M, Bürk J, et al. Bicuspid aortic valve is associated with altered wall shear stress in the ascending aorta. Circ Cardiovasc Imaging 2012; 5(4):457–466. doi:10.1161/CIRCIMAGING.112.973370
  21. Hope MD, Hope TA, Meadows AK, et al. Bicuspid aortic valve: four-dimensional MR evaluation of ascending aortic systolic flow patterns. Radiology 2010; 255(1):53–61. doi:10.1148/radiol.09091437
  22. Abdulkareem N, Soppa G, Jones S, Valencia O, Smelt J, Jahangiri M. Dilatation of the remaining aorta after aortic valve or aortic root replacement in patients with bicuspid aortic valve: a 5-year follow-up. Ann Thorac Surg 2013; 96(1):43–49. doi:10.1016/j.athoracsur.2013.03.086
  23. Regeer MV, Versteegh MI, Klautz RJ, et al. Effect of aortic valve replacement on aortic root dilatation rate in patients with bicuspid and tricuspid aortic valves. Ann Thorac Surg 2016; 102(6):1981–1987. doi:10.1016/j.athoracsur.2016.05.038
  24. Habashi JP, Judge DP, Holm TM, et al. Losartan, an AT1 antagonist, prevents aortic aneurysm in a mouse model of Marfan syndrome. Science 2006; 312(5770):117–121. doi:10.1126/science.1124287
  25. Brooke BS, Habashi JP, Judge DP, Patel N, Loeys B, Dietz HC 3rd. Angiotensin II blockade and aortic-root dilation in Marfan’s syndrome. N Engl J Med 2008; 358(26):2787–2795. doi:10.1056/NEJMoa0706585
  26. Chiu HH, Wu MH, Wang JK, et al. Losartan added to ß-blockade therapy for aortic root dilation in Marfan syndrome: a randomized, open-label pilot study. Mayo Clin Proc 2013; 88(3):271–276. doi:10.1016/j.mayocp.2012.11.005
  27. Groenink M, den Hartog AW, Franken R, et al. Losartan reduces aortic dilatation rate in adults with Marfan syndrome: a randomized controlled trial. Eur Heart J 2013; 34(45):3491–3500. doi:10.1093/eurheartj/eht334
  28. Lacro RV, Dietz HC, Sleeper LA, et al; Pediatric Heart Network Investigators. Atenolol versus losartan in children and young adults with Marfan’s syndrome. N Engl J Med 2014; 371(22):2061–2071. doi:10.1056/NEJMoa1404731
  29. Ziganshin BA, Mukherjee SK, Elefteriades JA, et al. Atenolol versus losartan in Marfan’s syndrome (letters). N Engl J Med 2015; 372(10):977–981. doi:10.1056/NEJMc1500128
  30. Franken R, den Hartog AW, Radonic T, et al. Beneficial outcome of losartan therapy depends on type of FBN1 mutation in Marfan syndrome. Circ Cardiovasc Genet 2015; 8(2):383–388. doi:10.1161/CIRCGENETICS.114.000950
  31. Elefteriades JA. Thoracic aortic aneurysm: reading the enemy’s playbook. Curr Probl Cardiol 2008; 33(5):203–277. doi:10.1016/j.cpcardiol.2008.01.004
  32. Idrees JJ, Roselli EE, Lowry AM, et al. Outcomes after elective proximal aortic replacement: a matched comparison of isolated versus multicomponent operations. Ann Thorac Surg 2016; 101(6):2185–2192. doi:10.1016/j.athoracsur.2015.12.026
  33. Hiratzka LF, Creager MA, Isselbacher EM, et al. Surgery for aortic dilatation in patients with bicuspid aortic valves: a statement of clarification from the American College of Cardiology/American Heart Association Task Force on Clinical Practice Guidelines. J Thorac Cardiovasc Surg 2016; 151(4):959–966. doi:10.1016/j.jtcvs.2015.12.001
  34. Tzemos N, Therrien J, Yip J, et al. Outcomes in adults with bicuspid aortic valves. JAMA 2008; 300(11):1317–1325. doi:10.1001/jama.300.11.1317
  35. Davies RR, Goldstein LJ, Coady MA, et al. Yearly rupture or dissection rates for thoracic aortic aneurysms: simple prediction based on size. Ann Thorac Surg 2002; 73(1):17–28. pmid:11834007
  36. Nishimura RA, Otto CM, Bono RO, et al. 2014 AHA/ACC guideline for the management of patients with valvular heart disease: executive summary: a report of the American College of Cardiology/American heart Association Task Force on Practice Guidelines. Circulation 2014; 129(23):2440–2492. doi:10.1161/CIR.0000000000000029
  37. Wojnarski CM, Svensson LG, Roselli EE, et al. Aortic dissection in patients with bicuspid aortic valve–associated aneurysms. Ann Thorac Surg 2015; 100(5):1666–1674. doi:10.1016/j.athoracsur.2015.04.126
  38. Hiratzka LF, Creager MA, Isselbacher EM, et al. Surgery for aortic dilatation in patients with bicuspid aortic valves: a statement of clarification from the American College of Cardiology/American Heart Association Task Force on Clinical Practice Guidelines. Circulation 2016; 133(7):680–686. doi:10.1161/CIR.0000000000000331
  39. Pape LA, Tsai TT, Isselbacher EM, et al; International Registry of Acute Aortic Dissection (IRAD) Investigators. Aortic diameter > or = 5.5 cm is not a good predictor of type A aortic dissection: observations from the International Registry of Acute Aortic Dissection (IRAD). Circulation 2007; 116(10):1120–1127. doi:10.1161/CIRCULATIONAHA.107.702720
  40. Loeys BL, Schwarze U, Holm T, et al. Aneurysm syndromes caused by mutations in the TGF-beta receptor. N Engl J Med 2006; 355(8):788–798. doi:10.1056/NEJMoa055695
  41. Guo DC, Pannu H, Tran-Fadulu V, et al. Mutations in smooth muscle alpha-actin (ACTA2) lead to thoracic aortic aneurysms and dissections. Nat Genet 2007; 39(12):1488–1493. doi:10.1038/ng.2007.6
  42. Svensson LG, Khitin L. Aortic cross-sectional area/height ratio timing of aortic surgery in asymptomatic patients with Marfan syndrome. J Thorac Cardiovasc Surg 2002; 123(2):360–361. pmid:11828302
  43. Svensson LG, Kim KH, Lytle BW, Cosgrove DM. Relationship of aortic cross-sectional area to height ratio and the risk of aortic dissection in patients with bicuspid aortic valves. J Thorac Cardiovasc Surg 2003; 126(3):892–893. pmid:14502185
  44. Masri A, Kalahasti V, Svensson LG, et al. Aortic cross-sectional area/height ratio and outcomes in patients with a trileaflet aortic valve and a dilated aorta. Circulation 2016; 134(22):1724–1737. doi:10.1161/CIRCULATIONAHA.116.022995
References
  1. Elefteriades JA, Farkas EA. Thoracic aortic aneurysm clinically pertinent controversies and uncertainties. J Am Coll Cardiol 2010; 55(9):841–857. doi:10.1016/j.jacc.2009.08.084
  2. Hiratzka LF, Bakris GL, Beckman JA, et al. 2010 ACCF/AHA/AATS/ACR/ASA/SCA/SCAI/SIR/STS/SVM guidelines for the diagnosis and management of patients with thoracic aortic disease: executive summary. Anesth Analg 2010; 111(2):279–315. doi:10.1213/ANE.0b013e3181dd869b
  3. Clouse WD, Hallett JW Jr, Schaff HV, Gayari MM, Ilstrup DM, Melton LJ 3rd. Improved prognosis of thoracic aortic aneurysms: a population-based study. JAMA 1998; 280(22):1926–1929. pmid:9851478
  4. Olsson C, Thelin S, Ståhle E, Ekbom A, Granath F. Thoracic aortic aneurysm and dissection: increasing prevalence and improved outcomes reported in a nationwide population-based study of more than 14,000 cases from 1987 to 2002. Circulation 2006; 114(24):2611–2618. doi:10.1161/CIRCULATIONAHA.106.630400
  5. Clouse WD, Hallett JW Jr, Schaff HV, et al. Acute aortic dissection: population-based incidence compared with degenerative aortic aneurysm rupture. Mayo Clin Proc 2004; 79(2):176–180. pmid:14959911
  6. US Centers for Disease Control and Prevention (CDC). National Center for Injury Prevention and Control. WISQARS leading causes of death reports, 1999 – 2007. https://webappa.cdc.gov/sasweb/ncipc/leadcaus10.html. Accessed May 21, 2018.
  7. Hansen MS, Nogareda GJ, Hutchison SJ. Frequency of and inappropriate treatment of misdiagnosis of acute aortic dissection. Am J Cardiol 2007; 99(6):852–856. doi:10.1016/j.amjcard.2006.10.055
  8. Goldfinger JZ, Halperin JL, Marin ML, Stewart AS, Eagle KA, Fuster V. Thoracic aortic aneurysm and dissection. J Am Coll Cardiol 2014; 64(16):1725–1739. doi:10.1016/j.jacc.2014.08.025
  9. Kumar V, Abbas A, Aster J. Robbins and Cotran Pathologic Basis of Disease. 9th ed. Philadelphia, PA: Elsevier/Saunders; 2015.
  10. Wolak A, Gransar H, Thomson LE, et al. Aortic size assessment by noncontrast cardiac computed tomography: normal limits by age, gender, and body surface area. JACC Cardiovasc Imaging 2008; 1(2):200–209. doi:10.1016/j.jcmg.2007.11.005
  11. Elefteriades JA. Natural history of thoracic aortic aneurysms: indications for surgery, and surgical versus nonsurgical risks. Ann Thorac Surg 2002; 74(5):S1877–S1880; discussion S1892–S1898. pmid:12440685
  12. Smith AD, Schoenhagen P. CT imaging for acute aortic syndrome. Cleve Clin J Med 2008; 75(1):7–17. pmid:18236724
  13. Cury M, Zeidan F, Lobato AC. Aortic disease in the young: genetic aneurysm syndromes, connective tissue disorders, and familial aortic aneurysms and dissections. Int J Vasc Med 2013(2013); 2013:267215. doi:10.1155/2013/267215
  14. Hoffman JI, Kaplan S. The incidence of congenital heart disease. J Am Coll Cardiol 2002; 39(12):1890–1900. doi:10.1016/S0735-1097(02)01886-7
  15. Fedak PW, Verma S, David TE, Leask RL, Weisel RD, Butany J. Clinical and pathophysiological implications of a bicuspid aortic valve. Circulation 2002; 106(8):900–904. pmid:12186790
  16. Della Corte A, Bancone C, Quarto C, et al. Predictors of ascending aortic dilatation with bicuspid aortic valve: a wide spectrum of disease expression. Eur J Cardiothorac Surg 2007; 31(3):397–405. doi:10.1016/j.ejcts.2006.12.006
  17. Jackson V, Petrini J, Caidahl K, et al. Bicuspid aortic valve leaflet morphology in relation to aortic root morphology: a study of 300 patients undergoing open-heart surgery. Eur J Cardiothorac Surg 2011; 40(3):e118–e124. doi:10.1016/j.ejcts.2011.04.014
  18. Michelena HI, Khanna AD, Mahoney D, et al. Incidence of aortic complications in patients with bicuspid aortic valves. JAMA 2011; 306(10):1104–1112. doi:10.1001/jama.2011.1286
  19. Verma S, Siu SC. Aortic dilatation in patients with bicuspid aortic valve. N Engl J Med 2014; 370(20):1920–1929. doi:10.1056/NEJMra1207059
  20. Barker AJ, Markl M, Bürk J, et al. Bicuspid aortic valve is associated with altered wall shear stress in the ascending aorta. Circ Cardiovasc Imaging 2012; 5(4):457–466. doi:10.1161/CIRCIMAGING.112.973370
  21. Hope MD, Hope TA, Meadows AK, et al. Bicuspid aortic valve: four-dimensional MR evaluation of ascending aortic systolic flow patterns. Radiology 2010; 255(1):53–61. doi:10.1148/radiol.09091437
  22. Abdulkareem N, Soppa G, Jones S, Valencia O, Smelt J, Jahangiri M. Dilatation of the remaining aorta after aortic valve or aortic root replacement in patients with bicuspid aortic valve: a 5-year follow-up. Ann Thorac Surg 2013; 96(1):43–49. doi:10.1016/j.athoracsur.2013.03.086
  23. Regeer MV, Versteegh MI, Klautz RJ, et al. Effect of aortic valve replacement on aortic root dilatation rate in patients with bicuspid and tricuspid aortic valves. Ann Thorac Surg 2016; 102(6):1981–1987. doi:10.1016/j.athoracsur.2016.05.038
  24. Habashi JP, Judge DP, Holm TM, et al. Losartan, an AT1 antagonist, prevents aortic aneurysm in a mouse model of Marfan syndrome. Science 2006; 312(5770):117–121. doi:10.1126/science.1124287
  25. Brooke BS, Habashi JP, Judge DP, Patel N, Loeys B, Dietz HC 3rd. Angiotensin II blockade and aortic-root dilation in Marfan’s syndrome. N Engl J Med 2008; 358(26):2787–2795. doi:10.1056/NEJMoa0706585
  26. Chiu HH, Wu MH, Wang JK, et al. Losartan added to ß-blockade therapy for aortic root dilation in Marfan syndrome: a randomized, open-label pilot study. Mayo Clin Proc 2013; 88(3):271–276. doi:10.1016/j.mayocp.2012.11.005
  27. Groenink M, den Hartog AW, Franken R, et al. Losartan reduces aortic dilatation rate in adults with Marfan syndrome: a randomized controlled trial. Eur Heart J 2013; 34(45):3491–3500. doi:10.1093/eurheartj/eht334
  28. Lacro RV, Dietz HC, Sleeper LA, et al; Pediatric Heart Network Investigators. Atenolol versus losartan in children and young adults with Marfan’s syndrome. N Engl J Med 2014; 371(22):2061–2071. doi:10.1056/NEJMoa1404731
  29. Ziganshin BA, Mukherjee SK, Elefteriades JA, et al. Atenolol versus losartan in Marfan’s syndrome (letters). N Engl J Med 2015; 372(10):977–981. doi:10.1056/NEJMc1500128
  30. Franken R, den Hartog AW, Radonic T, et al. Beneficial outcome of losartan therapy depends on type of FBN1 mutation in Marfan syndrome. Circ Cardiovasc Genet 2015; 8(2):383–388. doi:10.1161/CIRCGENETICS.114.000950
  31. Elefteriades JA. Thoracic aortic aneurysm: reading the enemy’s playbook. Curr Probl Cardiol 2008; 33(5):203–277. doi:10.1016/j.cpcardiol.2008.01.004
  32. Idrees JJ, Roselli EE, Lowry AM, et al. Outcomes after elective proximal aortic replacement: a matched comparison of isolated versus multicomponent operations. Ann Thorac Surg 2016; 101(6):2185–2192. doi:10.1016/j.athoracsur.2015.12.026
  33. Hiratzka LF, Creager MA, Isselbacher EM, et al. Surgery for aortic dilatation in patients with bicuspid aortic valves: a statement of clarification from the American College of Cardiology/American Heart Association Task Force on Clinical Practice Guidelines. J Thorac Cardiovasc Surg 2016; 151(4):959–966. doi:10.1016/j.jtcvs.2015.12.001
  34. Tzemos N, Therrien J, Yip J, et al. Outcomes in adults with bicuspid aortic valves. JAMA 2008; 300(11):1317–1325. doi:10.1001/jama.300.11.1317
  35. Davies RR, Goldstein LJ, Coady MA, et al. Yearly rupture or dissection rates for thoracic aortic aneurysms: simple prediction based on size. Ann Thorac Surg 2002; 73(1):17–28. pmid:11834007
  36. Nishimura RA, Otto CM, Bono RO, et al. 2014 AHA/ACC guideline for the management of patients with valvular heart disease: executive summary: a report of the American College of Cardiology/American heart Association Task Force on Practice Guidelines. Circulation 2014; 129(23):2440–2492. doi:10.1161/CIR.0000000000000029
  37. Wojnarski CM, Svensson LG, Roselli EE, et al. Aortic dissection in patients with bicuspid aortic valve–associated aneurysms. Ann Thorac Surg 2015; 100(5):1666–1674. doi:10.1016/j.athoracsur.2015.04.126
  38. Hiratzka LF, Creager MA, Isselbacher EM, et al. Surgery for aortic dilatation in patients with bicuspid aortic valves: a statement of clarification from the American College of Cardiology/American Heart Association Task Force on Clinical Practice Guidelines. Circulation 2016; 133(7):680–686. doi:10.1161/CIR.0000000000000331
  39. Pape LA, Tsai TT, Isselbacher EM, et al; International Registry of Acute Aortic Dissection (IRAD) Investigators. Aortic diameter > or = 5.5 cm is not a good predictor of type A aortic dissection: observations from the International Registry of Acute Aortic Dissection (IRAD). Circulation 2007; 116(10):1120–1127. doi:10.1161/CIRCULATIONAHA.107.702720
  40. Loeys BL, Schwarze U, Holm T, et al. Aneurysm syndromes caused by mutations in the TGF-beta receptor. N Engl J Med 2006; 355(8):788–798. doi:10.1056/NEJMoa055695
  41. Guo DC, Pannu H, Tran-Fadulu V, et al. Mutations in smooth muscle alpha-actin (ACTA2) lead to thoracic aortic aneurysms and dissections. Nat Genet 2007; 39(12):1488–1493. doi:10.1038/ng.2007.6
  42. Svensson LG, Khitin L. Aortic cross-sectional area/height ratio timing of aortic surgery in asymptomatic patients with Marfan syndrome. J Thorac Cardiovasc Surg 2002; 123(2):360–361. pmid:11828302
  43. Svensson LG, Kim KH, Lytle BW, Cosgrove DM. Relationship of aortic cross-sectional area to height ratio and the risk of aortic dissection in patients with bicuspid aortic valves. J Thorac Cardiovasc Surg 2003; 126(3):892–893. pmid:14502185
  44. Masri A, Kalahasti V, Svensson LG, et al. Aortic cross-sectional area/height ratio and outcomes in patients with a trileaflet aortic valve and a dilated aorta. Circulation 2016; 134(22):1724–1737. doi:10.1161/CIRCULATIONAHA.116.022995
Issue
Cleveland Clinic Journal of Medicine - 85(6)
Issue
Cleveland Clinic Journal of Medicine - 85(6)
Page Number
481-492
Page Number
481-492
Publications
Publications
Topics
Article Type
Display Headline
Thoracic aortic aneurysm: How to counsel, when to refer
Display Headline
Thoracic aortic aneurysm: How to counsel, when to refer
Legacy Keywords
thoracic aortic aneurysm, aorta, bicuspid aortic valve, Marfan syndrome, Loeys-Dietz syndrome, Ehlers-Danlos syndrome, dilation, mutations, beta-blockers, dissection, rupture, Frank Cikach, Milind Desai, Eric Roselli, Vidyasagar Kalahasti
Legacy Keywords
thoracic aortic aneurysm, aorta, bicuspid aortic valve, Marfan syndrome, Loeys-Dietz syndrome, Ehlers-Danlos syndrome, dilation, mutations, beta-blockers, dissection, rupture, Frank Cikach, Milind Desai, Eric Roselli, Vidyasagar Kalahasti
Sections
Inside the Article

KEY POINTS

  • Screening and referral depend on clinical context. A size-based model to determine screening, referral, follow-up, and management serves most cases but should be modified in the context of connective tissue disease or family history of aneurysm and dissection.
  • Medical management involves strict blood pressure and heart rate control with beta-blockers and angiotensin-converting enzyme inhibitors or angiotensin II receptor blockers. Activity modifications should be tailored to the individual, although extreme isometric exercises and heavy lifting should be discouraged.
  • Patients with TAA should be followed up annually, unless the patient is presenting for initial evaluation or significant changes are seen with dedicated imaging.
Disallow All Ads
Content Gating
No Gating (article Unlocked/Free)
Alternative CME
Disqus Comments
Default
Gate On Date
Tue, 05/29/2018 - 11:45
Un-Gate On Date
Tue, 05/29/2018 - 11:45
Use ProPublica
CFC Schedule Remove Status
Tue, 05/29/2018 - 11:45
Hide sidebar & use full width
render the right sidebar.
Article PDF Media

Hypertrophic cardiomyopathy: A complex disease

Article Type
Changed
Wed, 05/01/2019 - 07:37
Display Headline
Hypertrophic cardiomyopathy: A complex disease

Hypertrophic cardiomyopathy (HCM) is a complex disease. Most people who carry the mutations that cause it are never affected at any point in their life, but some are affected at a young age. And in rare but tragic cases, some die suddenly while competing in sports. With such a wide range of phenotypic expressions, a single therapy does not fit all.

HCM is more common than once thought. Since the discovery of its genetic predisposition in 1960, it has come to be recognized as the most common heritable cardiovascular disease.1 Although earlier epidemiologic studies had estimated a prevalence of 1 in 500 (0.2%) of the general population, genetic testing and cardiac magnetic resonance imaging (MRI) now show that up to 1 in 200 (0.5%) of all people may be affected.1,2 Its prevalence is significant in all ethnic groups.

This review outlines our expanding knowledge of the pathophysiology, diagnosis, and clinical management of HCM.

A PLETHORA OF MUTATIONS IN CARDIAC SARCOMERIC GENES

young_hypertrophiccardiomyopathy_f1.jpg
Figure%201.%20A%2C%20echocardiography%2C%20apical%204-chamber%20view%2C%20demonstrates%20septal%20hypertrophy%20(arrow).%20B%2C%20cardiac%20magnetic%20resonance%20imaging%20of%20the%20left%20ventricular%20outflow%20tract%20also%20demonstrates%20septal%20hypertrophy%20(arrow).%20C%2C%20echocardiography%20with%20continuous-wave%20Doppler%20across%20the%20left%20ventricular%20outflow%20tract%20demonstrates%20a%20gradient%20of%2070%20mm%20Hg%2C%20consistent%20with%20obstruction.%20D%2C%20electrocardiography%20reveals%20signs%20of%20left%20ventricular%20hypertrophy%20by%20Sokolov-Lynon%20criteria%20with%20S%20wave%20depth%20in%20V%3Csub%3E1%3C%2Fsub%3E%20plus%20R%20wave%20height%20in%20V%3Csub%3E5%3C%2Fsub%3E%20%3E%2035%20mm%20(arrows).
The genetic basis of HCM is much more complex than was originally thought, with more than 1,400 mutations in 11 sarcomeric protein genes now known to be associated with the disease. Most of these mutations are autosomal dominant.3

The genetic differences within HCM result in varying degrees and locations of left ventricular hypertrophy. Any segment of the ventricle can be involved, although HCM is classically asymmetric and mainly involves the septum (Figure 1). A variant form of HCM involves the apex of the heart (Figure 2).

LEFT VENTRICULAR OUTFLOW TRACT OBSTRUCTION

young_hypertrophiccardiomyopathy_f2.jpg
Figure%202.%20A%2C%20echocardiography%2C%20apical%204-chamber%20view%2C%20shows%20apical%20hypertrophy%20(arrows).%20B%2C%20cardiac%20magnetic%20resonance%20imaging%20(4-chamber%20view)%20shows%20apical%20hypertrophy%20(red%20arrows)%2C%20as%20well%20as%20an%20apical%20aneurysm%20(blue%20arrow).%20C%2C%20electrocardiography%20demonstrates%20giant%20T-wave%20inversions%20in%20the%20left%20precordial%20leads%2C%20characteristic%20of%20apical%20hypertrophic%20cardiomyopathy%20(arrows).
Obstruction of the left ventricular outflow tract is thought to be the pivotal pathophysiologic process of HCM. Other abnormalities may include myocardial ischemia and diastolic dysfunction, believed to be related to narrowing of the intramural coronary arteries.4 Histopathologic study of heart muscle in HCM demonstrates disarray of the hypertrophied myocyte architecture with variable patterns of interstitial fibrosis.

Only in the last decade has the significance of left ventricular outflow tract obstruction in HCM been truly appreciated. The degree of obstruction in HCM is dynamic, as opposed to the fixed obstruction in patients with aortic stenosis or congenital subvalvular membranes. Therefore, in HCM, exercise or drugs (eg, dobutamine) that increase cardiac contractility increase the obstruction, as do maneuvers or drugs (the Valsalva maneuver, nitrates) that reduce filling of the left ventricle.

young_hypertrophiccardiomyopathy_f3.jpg
Figure%203.%20Left%20ventricular%20outflow%20tract%20obstruction%20due%20to%20ventricular%20septal%20hypertrophy.%20The%20obstruction%20is%20dynamic%2C%20as%20the%20blood%20flow%20sweeps%20the%20mitral%20valve%20toward%20the%20septum.
The obstruction is usually due to a combination of systolic anterior motion of the mitral valve and accelerated blood flow around the hypertrophied septum, resulting in a pushing force that sweeps the mitral valve toward the septum (Figure 3).5,6

A less common source of dynamic obstruction is the papillary muscles (Figure 4). Hypertrophy of the papillary muscles can result in obstruction by these muscles themselves, which is visible on echocardiography. Anatomic variations include anteroapical displacement or bifid papillary muscles, and these variants can be associated with dynamic left ventricular outflow tract obstruction, even with no evidence of septal thickening (Figure 5).7,8 Recognizing this patient subset has important implications for management, as discussed below.

DIAGNOSTIC EVALUATION

The clinical presentation varies

young_hypertrophiccardiomyopathy_f4.jpg
Figure%204.%20A%2C%20echocardiography%2C%20apical%204-chamber%20view%2C%20demonstrates%20a%20bifid%20papillary%20muscle%20resulting%20in%20left%20ventricular%20outflow%20tract%20obstruction%20(arrows).%20B%2C%20cardiac%20magnetic%20resonance%20imaging%20(left%20ventricular%20outflow%20tract%20view)%20demonstrates%20a%20bifid%20papillary%20muscle%20(arrows).%20C%2C%20an%20electrocardiogram%20of%20a%20patient%20with%20obstruction%20related%20to%20abnormal%20papillary%20muscle%20morphology%20demonstrates%20a%20lack%20of%20significant%20left%20ventricular%20hypertrophy.%20D%2C%20continuous-wave%20Doppler%20through%20the%20left%20ventricular%20outflow%20tract%20demonstrates%20a%20peak%20gradient%20of%2099%20mm%20Hg%2C%20consistent%20with%20obstruction%2C%20which%20increases%20with%20the%20Valsalva%20maneuver%20to%20119%20mm%20Hg%20(E).
HCM is a clinical diagnosis: currently, there is no test that can definitively confirm it. It is defined as left ventricular hypertrophy without dilated ventricular chambers that cannot be explained by another disease state, with hypertrophy defined as wall thickness of 15 mm or greater in adults.9 The differential diagnosis of HCM is summarized in Table 1.

Even if patients harbor the same genetic variant, the clinical presentation can differ widely. Although the most feared presentation is sudden cardiac death, particularly in young athletes, most patients have no symptoms and can anticipate a normal life expectancy. The annual incidence of sudden cardiac death in all HCM patients is estimated at less than 1%.10 Sudden cardiac death in HCM patients is most often due to ventricular tachyarrhythmias and most often occurs in asymptomatic patients under age 35.

young_hypertrophiccardiomyopathy_f5.jpg
Figure%205.%20Left%20ventricular%20outflow%20tract%20(LVOT)%20obstruction%20without%20significant%20left%20ventricular%20hypertrophy.%20The%20prominent%20bifid%20papillary%20muscles%20lead%20to%20systolic%20anterior%20motion%20of%20the%20mitral%20valve%2C%20causing%20LVOT%20obstruction%20and%20simultaneous%20mitral%20regurgitation.
Patients with symptoms may present with progressive exertional dyspnea, chest pain, or syncope that may be related to left ventricular outflow tract obstruction, myocardial ischemia, arrhythmia, or heart failure. Left ventricular outflow tract obstruction, defined as a resting peak gradient of 30 mm Hg or higher, affects one-third of HCM patients. Another third have a dynamic, provoked gradient of 30 mm Hg or higher during the Valsalva maneuver, aerobic exercise, or pharmacologic provocation with amyl nitrate.11 Identifying these patients at the time of diagnosis is important for prognostication, as discussed below.

Physical findings are nonspecific

young_hypertrophiccardiomyopathy_t1.jpg
Physical findings may be unremarkable, especially in patients without resting left ventricular outflow tract obstruction. When present, the physical findings are nonspecific and include systolic murmurs, bifid carotid pulse, a fourth heart sound, and a hyperdynamic precordium.

It can be difficult to distinguish the murmur of left ventricular outflow tract obstruction in HCM from a murmur related to aortic stenosis by auscultation alone. The simplest clinical method for telling them apart involves the Valsalva maneuver: bearing down creates a positive intrathoracic pressure and limits venous return, thus decreasing intracardiac filling pressure. This in turn results in less separation between the mitral valve and the ventricular septum in HCM, which increases obstruction and therefore makes the murmur louder. In contrast, in patients with fixed obstruction due to aortic stenosis, the murmur will decrease in intensity owing to the reduced flow associated with reduced preload.

 

 

Laboratory testing for phenocopies of HCM

young_hypertrophiccardiomyopathy_t2.jpg
Laboratory testing should be done at index encounters for all patients suspected of having HCM, as testing can help identify patients with HCM phenocopies, ie, a group of rare but clinically important diseases that cause pathologic left ventricular hypertrophy that is not due to sarcomeric gene defects. Identifying these conditions early is pivotal, as their natural history, management, and prognosis are significantly different (Table 2).

A metabolic panel will show derangements in liver function and glucose levels in patients with glycogen storage disorders such as Pompe disease. 

Serum creatinine. Renal dysfunction will be seen in patients with Fabry disease or amyloidosis.

Creatine kinase may be elevated in patients with Danon disease.

Electrocardiographic findings are common

More than 90% of HCM patients have electrocardiographic abnormalities. Although the findings can vary widely, common manifestations include:

  • Left ventricular hypertrophy
  • A pseudoinfarct pattern with Q waves in the anterolateral leads
  • Repolarization changes such as T-wave inversions and horizontal or down-sloping ST segments.

Apical HCM, seen mainly in Asian populations, often presents with giant T-wave inversion (> 10 mm) in the anterolateral leads, most prominent in V4, V5, and V6.

Notably, the degree of electrocardiographic abnormalities does not correlate with the severity or pattern of hypertrophy.9 Electrocardiography lacks specificity for definitive diagnosis, and further diagnostic testing should therefore be pursued.

Echocardiography: Initial imaging test

Transthoracic echocardiography is the initial imaging test in patients with suspected HCM, allowing for cost-effective quantitative and qualitative assessment of left ventricular morphology and function. Left ventricular hypertrophy is considered pathologic if wall thickness is 15 mm or greater without a known cause. Transthoracic echocardiography also allows for evaluation of left atrial volume and mitral valve anatomy and function.

Speckle tracking imaging is an advanced echocardiographic technique that measures strain. Its major advantage is in identifying early abnormalities in genotype-positive, phenotype-negative HCM patients, ie, people who harbor mutations but who have no clinical symptoms or signs of HCM, potentially allowing for modification of the natural history of HCM.12 Strain imaging can also differentiate between physiologic hypertrophy (“athlete’s heart”) and hypertension and HCM.13,14

The utility of echocardiography in HCM is heavily influenced by the sonographer’s experience in obtaining adequate acoustic windows. This may be more difficult in obese patients, patients with advanced obstructive lung disease or pleural effusions, and women with breast implants.

Magnetic resonance imaging

MRI has an emerging role in both diagnosing and predicting risk in HCM, and is routinely done as an adjunct to transthoracic echocardiography on initial diagnosis in our tertiary referral center. It is particularly useful in patients suspected of having apical hypertrophy (Figure 2), in whom the diagnosis may be missed in up to 10% on transthoracic echocardiography alone.15 MRI can also enhance the assessment of left ventricular hypertrophy and has been shown to improve the diagnostic classification of HCM.16 It is the best way to assess myocardial tissue abnormalities, and late gadolinium enhancement to detect interstitial fibrosis can be used for further prognostication. While historically the primary role of MRI in HCM has been in phenotype classification, there is currently much interest in its role in risk stratification of HCM patients for ICD implantation.

MRI with late gadolinium enhancement provides insight into the location, pattern, and extent of myocardial fibrosis; the extent of fibrosis has been shown to be a strong independent predictor of poor outcomes, including sudden cardiac death.17–20 However, late gadolinium enhancement can be technically challenging, as variations in the timing of postcontrast imaging, sequences for measuring late gadolinium enhancement, or detection thresholds can result in widely variable image quality. Cardiac MRI should therefore be performed at an experienced center with standardized imaging protocols in place.

Current guidelines recommend considering cardiac MRI if a patient’s risk of sudden cardiac death remains inconclusive after conventional risk stratification, as discussed below.9,21

Stress testing for risk stratification

Exercise stress electrocardiography. Treadmill exercise stress testing with electrocardiography and hemodynamic monitoring was one of the first tools used for risk stratification in HCM.

Although systolic blood pressure normally increases by at least 20 mm Hg with exercise, one-quarter of HCM patients have either a blunted response (failure of systolic blood pressure to increase by at least 20 mm Hg) or a hypotensive response (a drop in systolic blood pressure of 20 mm Hg or more, either continuously or after an initial increase). Studies have shown that HCM patients who have abnormal blood pressure responses during exercise have a higher risk of sudden cardiac death.22–24

Exercise stress echocardiography can be useful to evaluate for provoked increases in the left ventricular outflow tract gradient, which may contribute to a patient’s symptoms even if the resting left ventricular outflow tract gradient is normal. Exercise testing is preferred over pharmacologic stimulation because it can provide functional assessment of whether a patient’s clinical symptoms are truly related to hemodynamic changes due to the hypertrophied ventricle, or whether alternative mechanisms should be explored.

Cardiopulmonary stress testing can readily add prognostic value with additional measurements of functional capacity. HCM patients who cannot achieve their predicted maximal exercise value such as peak rate of oxygen consumption, ventilation efficiency, or anaerobic threshold have higher rates of morbidity and mortality.25,26 Stress testing can also be useful for risk stratification in asymptomatic patients, with one study showing that those who achieve more than 100% of their age- and sex-predicted metabolic equivalents have a low event rate.27

 

 

Ambulatory electrocardiographic monitoring in all patients at diagnosis

Ambulatory electrocardiographic monitoring for 24 to 48 hours is recommended for all HCM patients at the time of diagnosis, even if they have no symptoms. Any evidence of nonsustained ventricular tachycardia suggests a substantially higher risk of sudden cardiac death.28,29

In patients with no symptoms or history of arrhythmia, current guidelines suggest ambulatory electrocardiographic monitoring every 1 to 2 years.9,21

Two risk-stratification models

young_hypertrophiccardiomyopathy_t3.jpg
Two models are widely available for risk stratification in HCM (Table 3). While the consensus is to implant a cardioverter-defibrillator for secondary prevention if a patient has a history of ventricular arrhythmia or cardiac arrest, the approach to primary prevention differs between these 2 models.

The North American model was the first risk-stratification tool and considers 5 risk factors.9 However, if this algorithm were strictly followed, up to 60% of HCM patients would be candidates for cardioverter-defibrillator implantation.

The European model. This concern led to the development of the HCM Risk-SCD (sudden cardiac death), a risk-stratification tool introduced in the 2014 European Society of Cardiology HCM guidelines.30 This web-based calculator estimates a patient’s 5-year risk of sudden cardiac death using a complex calculation based on 7 clinical risk factors. If a patient’s calculated 5-year risk of sudden cardiac death is 6% or higher, cardioverter-defibrillator implantation is recommended for primary prevention.

The HCM Risk-SCD calculator was validated and compared with classic risk factors alone in a retrospective cohort study in 48 HCM patients.30 Compared with the North American model, the European model results in a lower rate of cardioverter-defibrillator implantation (20% to 26%).31,32

Despite the better specificity of the European model, a large retrospective cohort analysis showed that a significant number of patients stratified as being at low risk for sudden cardiac death were ultimately found to be at high risk in clinical practice.31 Further research is needed to find the optimal risk-stratification approach in HCM patients at low to intermediate risk.

GENETIC TESTING, COUNSELING, AND FAMILY SCREENING

Genetic testing is becoming more widely available and has rapidly expanded in clinical practice. Genetic counseling must be performed alongside genetic testing and requires professionals trained to handle the clinical and social implications of genetic testing. With this in mind, genetic testing can provide a definitive means of identifying family members at risk of HCM.

Given the autosomal dominant nature of HCM, screening for HCM is recommended in all first-degree relatives of an affected patient. Genetic testing may be a means to achieve this if a pathogenic mutation has been identified in the affected patient. However, serial electrocardiographic and transthoracic echocardiographic monitoring is an acceptable alternative in those without a clear genetic mutation association or in those who do not want to undergo genetic testing. If these first-degree relatives who do not undergo genetic testing are adult athletes or adolescents, they should undergo surveillance monitoring, with echocardiography and electrocardiography, whereas adults not participating in athletics should be monitored every 5 years.9,21

As genetic counseling and testing become more widely available, more patients are being found who harbor a mutation but have no phenotypic manifestations of HCM on initial presentation. Clinical expression varies, so continued monitoring of these patients is important. Expert guidelines again recommend serial electrocardiography, transthoracic echocardiography, and clinical assessment every 5 years for adults.9

Recent data suggest that up to 40% of HCM cases are nonfamilial, ie, their inheritance is sporadic with no known family history and no sarcomeric gene mutation evident on screening.33,34 The clinical course in this subgroup seems to be more benign, with later clinical presentations (age > 40) and lower risk of major adverse cardiovascular events.

MANAGEMENT

Conservative management

Asymptomatic HCM can usually be managed with lifestyle modifications.

Avoiding high-risk physical activities is the most important modification. All HCM patients should be counseled on the risk of sudden cardiac death and advised against participating in competitive sports or intense physical activity.35 Aerobic exercise is preferable to isometric exercises such as weightlifting, which may prompt the Valsalva maneuver with worsening of left ventricular outflow tract obstruction leading to syncope. A recent study showed that moderate-intensity aerobic exercise can safely improve exercise capacity, which may ultimately improve functional status and quality of life.36

Avoiding dehydration and excessive alcohol intake are also important in maintaining adequate preload to prevent an increasing left ventricular outflow tract gradient, given the dynamic nature of the left ventricular outflow tract obstruction in HCM.

 

 

Medical management: Beta-blockers, then calcium channel blockers

Beta-blockers are the first-line therapy for symptomatic HCM related to left ventricular outflow tract obstruction. Their negative inotropic effect reduces the contractile force of the ventricle, effectively reducing the pressure gradient across the outflow tract. Reduced contractility also means that the overall myocardial workload is less, which ultimately translates to a reduced oxygen demand. With their negative chronotropic effect, beta-blockers lower the heart rate and thereby lengthen the diastolic filling phase, allowing for optimization of preload conditions to help prevent increasing the left ventricular outflow tract gradient.37,38

Beta-blockers can be titrated according to the patient’s symptoms and tolerance. Fatigue and loss of libido are among the most common side effects.

Nondihydropyridine calcium channel blockers can be a second-line therapy in patients who cannot tolerate beta-blockers. Several studies have shown improvement in surrogate outcomes such as estimated left ventricular mass and QRS amplitude on electrocardiography, but currently no available data show that these drugs improve symptoms.28,39,40 They should be avoided in those with severe left ventricular outflow tract obstruction (gradient 100 mm Hg), as they can lead to critical outflow tract obstruction owing to their peripheral vasodilatory effect.

Dihydropyridine calcium channel blockers should be avoided altogether, as they produce even more peripheral vasodilation and afterload reduction than nondihydropyridine calcium channel blockers.

Disopyramide, a class IA antiarrhythmic, has been shown to effectively reduce outflow gradients and relieve symptoms. However, in view of its adverse effects, it is a third-line therapy, given to those for whom beta-blockers and calcium channel blockers have failed. Its most worrisome adverse effect is QT prolongation, and the QT interval should therefore be closely monitored at the start of treatment. Anticholinergic effects are common and include dry eyes and mouth, urinary retention, and drowsiness.

Disopyramide is usually used in combination with beta-blockers for symptom control as a bridge to a planned invasive intervention.41

Use with caution

Any medication that causes afterload reduction, peripheral vasodilation, intravascular volume depletion, or positive inotropy can worsen the dynamic left ventricular outflow tract obstruction in a patient with HCM and should be avoided.

Angiotensin-converting enzyme (ACE) inhibitors, angiotensin II receptor blockers (ARBs), and nitrates must be used with extreme caution in these patients.

Diuretics. Even restrained use of diuretics can cause significant hemodynamic compromise in patients with obstructive physiology. Therefore, diuretics should be used sparingly in these patients.

Digoxin should not be used for managing atrial fibrillation in these patients, as its positive inotropic effect increases contractility and increases the left ventricular outflow tract gradient.

Norepinephrine and inotropic agents such as dobutamine and dopamine should be avoided for the same reason as digoxin. In patients with circulatory shock requiring vasopressor support, pure alpha-agonists such as phenylephrine are preferred, as they increase peripheral resistance without an inotropic effect.

Anticoagulation for atrial tachyarrhythmias

The risk of systemic thromboembolic events is significantly increased in HCM patients with atrial fibrillation or flutter, regardless of their estimated risk using conventional risk-stratification tools such as the CHADS2 score.42–44 In accordance with current American Heart Association and American College of Cardiology guidelines, we recommend anticoagulation therapy for all HCM patients with a history of atrial fibrillation or flutter. Warfarin is the preferred anticoagulant; direct oral anticoagulants can be considered, but there are currently no data on their use in HCM.9

Standard heart failure treatments

End-stage systolic heart failure is a consequence of HCM but affects only 3% to 4% of patients.45 While most randomized controlled trials of heart failure treatment have excluded HCM patients, current guidelines recommend the same evidence-based medical therapies used in other patients who have heart failure with reduced ejection fraction. This includes ACE inhibitors, ARBs, beta-blockers, and aldosterone antagonists if indicated.9,21

Heart transplant should be considered in patients with class III or IV New York Heart Association functional status despite optimization of their HCM treatment regimen. Heart transplant outcomes for HCM patients are comparable to outcomes for patients who receive a transplant for non-HCM cardiovascular disease.45,46

 

 

Septal reduction therapy

If medical therapy fails or is not tolerated in patients with severe symptoms, surgery can be considered for obstructive HCM.

Ventricular septal myectomy has been the long-standing gold standard of invasive therapy. Multiple studies have demonstrated long-term survival after myectomy to be equivalent to that in the general population and better than that of HCM patients who do not undergo this surgery.47–50 Factors that may be associated with better surgical outcomes include age younger than 50, left atrial size less than 46 mm, and resolution of atrial fibrillation during follow-up.51

Septal reduction therapy may also be considered in patients at high risk of sudden cardiac death based on a history of recurrent ventricular tachycardia or risk-stratification models as described above. Retrospective analyses have shown that surgical myectomy can markedly reduce the incidence of appropriate implantable cardioverter-defibrillator discharges and the risk of sudden cardiac death.52

Alcohol septal ablation is an alternative. This percutaneous procedure, first described in the mid-1990s, consists of injecting a small amount of alcohol into the artery supplying the septum to induce myocardial necrosis, ultimately leading to scarring and widening of the left ventricular outflow tract.53

Up to 50% of patients develop right bundle branch block after alcohol septal ablation, and the risk of complete heart block is highest in those with preexisting left bundle branch block. Nevertheless, studies have shown significant symptomatic improvement after alcohol septal ablation, with long-term survival comparable to that in the general population.53–56

Several meta-analyses compared alcohol septal ablation and septal myectomy and found that the rates of functional improvement and long-term mortality were similar.57–59 However, the less-invasive approach with alcohol septal ablation comes at the cost of a higher incidence of conduction abnormalities and higher left ventricular outflow tract gradients afterward. One meta-analysis found that alcohol septal ablation patients may have 5 times the risk of needing additional septal reduction therapy compared with their myectomy counterparts.

Current US guidelines recommend septal myectomy, performed at an experienced center, as the first-line interventional treatment, leaving alcohol septal ablation to be considered in those who have contraindications to myectomy.9 The treatment strategy should ultimately be individualized based on a patient’s comorbidities and personal preferences following informed consent.

A nationwide database study recently suggested that postmyectomy mortality rates may be as high as 5.9%,60 although earlier studies at high-volume centers showed much lower mortality rates (< 1%).50–52,61 This discrepancy highlights the critical role of expert centers in optimizing surgical management of these patients. Regardless of the approach, interventional therapies for HCM should be performed by a multidisciplinary team at a medical center able to handle the complexity of these cases.

Additional surgical procedures

A handful of other procedures may benefit specific patient subgroups.

young_hypertrophiccardiomyopathy_f6.jpg
Figure%206.%20Reorientation%20surgery%20reduces%20mobility%20of%20bifid%20hypermobile%20papillary%20muscles%2C%20reducing%20left%20ventricular%20outflow%20tract%20(LVOT)%20obstruction.
Papillary muscle reorientation surgery (Figure 6) has been shown in retrospective studies to reduce mobility of bifid hypermobile papillary muscles and alleviate left ventricular outflow tract obstruction.62 It should be considered in patients who have this problem, even if they have no left ventricular hypertrophy.

Apical myectomy has been shown to improve functional status in patients with isolated apical hypertrophy by reducing left ventricular end-diastolic pressure and thereby allowing for improved diastolic filling.63

Mitral valve surgery may need to be considered at the time of myectomy in patients with degenerative valve disease. As in the general population, mitral valve repair is preferred to replacement if possible. 

References
  1. Maron BJ. Hypertrophic cardiomyopathy: an important global disease. Am J Med 2004; 116(1):63–65. pmid:14706671
  2. Semsarian C, Ingles J, Maron MS, Maron BJ. New perspectives on the prevalence of hypertrophic cardiomyopathy. J Am Coll Cardiol 2015; 65(12):1249–1254. doi:10.1016/j.jacc.2015.01.019
  3. Maron BJ, Maron MS, Semsarian C. Genetics of hypertrophic cardiomyopathy after 20 years: clinical perspectives. J Am Coll Cardiol 2012; 60(8):705–715. doi:10.1016/j.jacc.2012.02.068
  4. Shirani J, Pick R, Roberts WC, Maron BJ. Morphology and significance of the left ventricular collagen network in young patients with hypertrophic cardiomyopathy and sudden cardiac death. J Am Coll Cardiol 2000; 35(1):36–44. pmid:10636256
  5. Sherrid MV, Chu CK, Delia E, Mogtader A, Dwyer EM Jr. An echocardiographic study of the fluid mechanics of obstruction in hypertrophic cardiomyopathy. J Am Coll Cardiol 1993; 22(3):816–825. pmid:8354817
  6. Ro R, Halpern D, Sahn DJ, et al. Vector flow mapping in obstructive hypertrophic cardiomyopathy to assess the relationship of early systolic left ventricular flow and the mitral valve. J Am Coll Cardiol 2014; 64(19):1984–1995. doi:10.1016/j.jacc.2014.04.090
  7. Kwon DH, Setser RM, Thamilarasan M, et al. Abnormal papillary muscle morphology is independently associated with increased left ventricular outflow tract obstruction in hypertrophic cardiomyopathy. Heart 2008; 94(10):1295–1301. doi:10.1136/hrt.2007.118018
  8. Patel P, Dhillon A, Popovic ZB, et al. Left ventricular outflow tract obstruction in hypertrophic cardiomyopathy patients without severe septal hypertrophy: implications of mitral valve and papillary muscle abnormalities assessed using cardiac magnetic resonance and echocardiography. Circ Cardiovasc Imaging 2015; 8(7):e003132. doi:10.1161/CIRCIMAGING.115.003132
  9. Gersh BJ, Maron BJ, Bonow RO, et al. 2011 ACCF/AHA guideline for the diagnosis and treatment of hypertrophic cardiomyopathy: a report of the American College of Cardiology Foundation/American Heart Association Task Force on Practice Guidelines. J Thorac Cardiovasc Surg 2011; 142(6):e153-e203. doi:10.1016/j.jtcvs.2011.10.020
  10. Elliott PM, Gimeno JR, Thaman R, et al. Historical trends in reported survival rates in patients with hypertrophic cardiomyopathy. Heart 2006; 92(6):785–791. doi:10.1136/hrt.2005.068577
  11. Maron MS, Olivotto I, Zenovich AG, et al. Hypertrophic cardiomyopathy is predominantly a disease of left ventricular outflow tract obstruction. Circulation 2006; 114(21):2232–2239. doi:10.1161/CIRCULATIONAHA.106.644682
  12. Ho CY, Carlsen C, Thune JJ, et al. Echocardiographic strain imaging to assess early and late consequences of sarcomere mutations in hypertrophic cardiomyopathy. Circ Cardiovasc Genet 2009; 2(4):314–321. doi:10.1161/CIRCGENETICS.109.862128
  13. Wasfy MM, Weiner RB. Differentiating the athlete’s heart from hypertrophic cardiomyopathy. Curr Opin Cardiol 2015; 30(5):500–505. doi:10.1097/HCO.0000000000000203
  14. Palka P, Lange A, Fleming AD, et al. Differences in myocardial velocity gradient measured throughout the cardiac cycle in patients with hypertrophic cardiomyopathy, athletes and patients with left ventricular hypertrophy due to hypertension. J Am Coll Cardiol 1997; 30(3):760–768. pmid:9283537
  15. Eriksson MJ, Sonnenberg B, Woo A, et al. Long-term outcome in patients with apical hypertrophic cardiomyopathy. J Am Coll Cardiol 2002; 39(4):638–645. pmid:11849863
  16. Rickers C, Wilke NM, Jerosch-Herold M, et al. Utility of cardiac magnetic resonance imaging in the diagnosis of hypertrophic cardiomyopathy. Circulation 2005; 112(6):855–861. doi:10.1161/CIRCULATIONAHA.104.507723
  17. Kwon DH, Setser RM, Popovic ZB, et al. Association of myocardial fibrosis, electrocardiography and ventricular tachyarrhythmia in hypertrophic cardiomyopathy: a delayed contrast enhanced MRI study. Int J Cardiovasc Imaging 2008; 24(6):617–625. doi:10.1007/s10554-008-9292-6
  18. Rubinshtein R, Glockner JF, Ommen SR, et al. Characteristics and clinical significance of late gadolinium enhancement by contrast-enhanced magnetic resonance imaging in patients with hypertrophic cardiomyopathy. Circ Heart Fail 2010; 3(1):51–58. doi:10.1161/CIRCHEARTFAILURE.109.854026
  19. O’Hanlon R, Grasso A, Roughton M, et al. Prognostic significance of myocardial fibrosis in hypertrophic cardiomyopathy. J Am Coll Cardiol 2010; 56(11):867–874. doi:10.1016/j.jacc.2010.05.010
  20. Bruder O, Wagner A, Jensen CJ, et al. Myocardial scar visualized by cardiovascular magnetic resonance imaging predicts major adverse events in patients with hypertrophic cardiomyopathy. J Am Coll Cardiol 2010; 56(11):875–887. doi:10.1016/j.jacc.2010.05.007
  21. Authors/Task Force members, Elliott PM, Anastasakis A, Borger MA, et al. 2014 ESC guidelines on diagnosis and management of hypertrophic cardiomyopathy: the Task Force for the Diagnosis and Management of Hypertrophic Cardiomyopathy of the European Society of Cardiology (ESC). Eur Heart J 2014; 35(39):2733–2779. doi:10.1093/eurheartj/ehu284
  22. Olivotto I, Maron BJ, Montereggi A, Mazzuoli F, Dolara A, Cecchi F. Prognostic value of systemic blood pressure response during exercise in a community-based patient population with hypertrophic cardiomyopathy. J Am Coll Cardiol 1999; 33(7):2044–2051. pmid:10362212
  23. Sadoul N, Prasad K, Elliott PM, Bannerjee S, Frenneaux MP, McKenna WJ. Prospective prognostic assessment of blood pressure response during exercise in patients with hypertrophic cardiomyopathy. Circulation 1997; 96(9):2987–2991. pmid:9386166
  24. Elliott PM, Poloniecki J, Dickie S, et al. Sudden death in hypertrophic cardiomyopathy: identification of high risk patients. J Am Coll Cardiol 2000; 36(7):2212–2218. pmid:11127463
  25. Masri A, Pierson LM, Smedira NG, et al. Predictors of long-term outcomes in patients with hypertrophic cardiomyopathy undergoing cardiopulmonary stress testing and echocardiography. Am Heart J 2015; 169(5):684–692.e1. doi:10.1016/j.ahj.2015.02.006
  26. Coats CJ, Rantell K, Bartnik A, et al. Cardiopulmonary exercise testing and prognosis in hypertrophic cardiomyopathy. Circ Heart Fail 2015; 8(6):1022–1031. doi:10.1161/CIRCHEARTFAILURE.114.002248
  27. Desai MY, Bhonsale A, Patel P, et al. Exercise echocardiography in asymptomatic HCM: exercise capacity, and not LV outflow tract gradient predicts long-term outcomes. JACC Cardiovasc Imaging 2014; 7(1):26–36. doi:10.1016/j.jcmg.2013.08.010
  28. Spirito P, Seidman CE, McKenna WJ, Maron BJ. The management of hypertrophic cardiomyopathy. N Engl J Med 1997; 336(11):775–785. doi:10.1056/NEJM199703133361107
  29. Wang W, Lian Z, Rowin EJ, Maron BJ, Maron MS, Link MS. Prognostic implications of nonsustained ventricular tachycardia in high-risk patients with hypertrophic cardiomyopathy. Circ Arrhythm Electrophysiol 2017; 10(3)e004604. doi:10.1161/CIRCEP.116.004604
  30. O’Mahony C, Jichi F, Pavlou M, et al. A novel clinical risk prediction model for sudden cardiac death in hypertrophic cardiomyopathy (HCM risk-SCD). Eur Heart J 2014; 35(30):2010–2020. doi:10.1093/eurheartj/eht439
  31. Maron BJ, Casey SA, Chan RH, Garberich RF, Rowin EJ, Maron MS. Independent assessment of the European Society of Cardiology sudden death risk model for hypertrophic cardiomyopathy. Am J Cardiol 2015; 116(5):757–764. doi:10.1016/j.amjcard.2015.05.047
  32. Jahnlová D, Tomašov P, Zemánek D, Veselka J. Transatlantic differences in assessment of risk of sudden cardiac death in patients with hypertrophic cardiomyopathy. Int J Cardiol 2015; 186:3–4. doi:10.1016/j.ijcard.2015.03.207
  33. Ingles J, Burns C, Bagnall RD, et al. Nonfamilial hypertrophic cardiomyopathy: prevalence, natural history, and clinical implications. Circ Cardiovasc Genet 2017; 10(2)e001620. doi:10.1161/CIRCGENETICS.116.001620
  34. Ko C, Arscott P, Concannon M, et al. Genetic testing impacts the utility of prospective familial screening in hypertrophic cardiomyopathy through identification of a nonfamilial subgroup. Genet Med 2017; 20(1):69–75. doi:10.1038/gim.2017.79
  35. Maron BJ, Chaitman BR, Ackerman MJ, et al. Recommendations for physical activity and recreational sports participation for young patients with genetic cardiovascular diseases. Circulation 2004; 109(22):2807–2816. doi:10.1161/01.CIR.0000128363.85581.E1
  36. Saberi S, Wheeler M, Bragg-Gresham J, et al. Effect of moderate-intensity exercise training on peak oxygen consumption in patients with hypertrophic cardiomyopathy: a randomized clinical trial. JAMA 2017; 317(13):1349–1357. doi:10.1001/jama.2017.2503
  37. Bourmayan C, Razavi A, Fournier C, et al. Effect of propranolol on left ventricular relaxation in hypertrophic cardiomyopathy: an echographic study. Am Heart J 1985; 109(6):1311–1316. pmid:4039882
  38. Spoladore R, Maron MS, D’Amato R, Camici PG, Olivotto I. Pharmacological treatment options for hypertrophic cardiomyopathy: high time for evidence. Eur Heart J 2012; 33(14):1724–1733. doi:10.1093/eurheartj/ehs150
  39. Choudhury L, Elliott P, Rimoldi O, et al. Transmural myocardial blood flow distribution in hypertrophic cardiomyopathy and effect of treatment. Basic Res Cardiol 1999; 94(1):49–59. pmid:10097830
  40. Kaltenbach M, Hopf R, Kober G, Bussmann WD, Keller M, Petersen Y. Treatment of hypertrophic obstructive cardiomyopathy with verapamil. Br Heart J 1979; 42(1):35–42. doi:10.1136/hrt.42.1.35
  41. Sherrid MV, Shetty A, Winson G, et al. Treatment of obstructive hypertrophic cardiomyopathy symptoms and gradient resistant to first-line therapy with beta-blockade or verapamil. Circ Heart Fail 2013; 6(4):694–702. doi:10.1161/CIRCHEARTFAILURE.112.000122
  42. Guttmann OP, Rahman MS, O’Mahony C, Anastasakis A, Elliott PM. Atrial fibrillation and thromboembolism in patients with hypertrophic cardiomyopathy: systematic review. Heart 2014; 100(6):465–472. doi:10.1136/heartjnl-2013-304276
  43. Olivotto I, Cecchi F, Casey SA, Dolara A, Traverse JH, Maron BJ. Impact of atrial fibrillation on the clinical course of hypertrophic cardiomyopathy. Circulation 2001; 104(21):2517–2524. pmid:11714644
  44. Maron BJ, Olivotto I, Spirito P, et al. Epidemiology of hypertrophic cardiomyopathy-related death: revisited in a large non-referral-based patient population. Circulation 2000; 102(8):858–864. pmid:10952953
  45. Harris KM, Spirito P, Maron MS, et al. Prevalence, clinical profile, and significance of left ventricular remodeling in the end-stage phase of hypertrophic cardiomyopathy. Circulation 2006; 114(3):216-225. doi:10.1161/CIRCULATIONAHA.105.583500
  46. Maron MS, Kalsmith BM, Udelson JE, Li W, DeNofrio D. Survival after cardiac transplantation in patients with hypertrophic cardiomyopathy. Circ Heart Fail 2010; 3(5):574–579. doi:10.1161/CIRCHEARTFAILURE.109.922872
  47. Smedira NG, Lytle BW, Lever HM, et al. Current effectiveness and risks of isolated septal myectomy for hypertrophic obstructive cardiomyopathy. Ann Thorac Surg 2008; 85(1):127–133. doi:10.1016/j.athoracsur.2007.07.063
  48. Robbins RC, Stinson EB. Long-term results of left ventricular myotomy and myectomy for obstructive hypertrophic cardiomyopathy. J Thorac Cardiovasc Surg 1996; 111(3):586–594. pmid:8601973
  49. Heric B, Lytle BW, Miller DP, Rosenkranz ER, Lever HM, Cosgrove DM. Surgical management of hypertrophic obstructive cardiomyopathy. Early and late results. J Thorac Cardiovasc Surg 1995; 110(1):195–208. pmid:7609544
  50. Ommen SR, Maron BJ, Olivotto I, et al. Long-term effects of surgical septal myectomy on survival in patients with obstructive hypertrophic cardiomyopathy. J Am Coll Cardiol 2005; 46(3):470–476. doi:10.1016/j.jacc.2005.02.090
  51. Desai MY, Bhonsale A, Smedira NG, et al. Predictors of long-term outcomes in symptomatic hypertrophic obstructive cardiomyopathy patients undergoing surgical relief of left ventricular outflow tract obstruction. Circulation 2013; 128(3):209–216. doi:10.1161/CIRCULATIONAHA.112.000849
  52. McLeod CJ, Ommen SR, Ackerman MJ, et al. Surgical septal myectomy decreases the risk for appropriate implantable cardioverter defibrillator discharge in obstructive hypertrophic cardiomyopathy. Eur Heart J 2007; 28(21):2583–2588. doi:10.1093/eurheartj/ehm117
  53. Veselka J, Tomasov P, Zemanek D. Long-term effects of varying alcohol dosing in percutaneous septal ablation for obstructive hypertrophic cardiomyopathy: a randomized study with a follow-up up to 11 years. Can J Cardiol 2011; 27(6):763–767. doi:10.1016/j.cjca.2011.09.001
  54. Veselka J, Jensen MK, Liebregts M, et al. Low procedure-related mortality achieved with alcohol septal ablation in European patients. Int J Cardiol 2016; 209:194–195. doi:10.1016/j.ijcard.2016.02.077
  55. Veselka J, Krejci J, Tomašov P, Zemánek D. Long-term survival after alcohol septal ablation for hypertrophic obstructive cardiomyopathy: a comparison with general population. Eur Heart J 2014; 35(30):2040–2045. doi:10.1093/eurheartj/eht495
  56. Sorajja P, Ommen SR, Holmes DR Jr, et al. Survival after alcohol septal ablation for obstructive hypertrophic cardiomyopathy. Circulation 2012; 126(20):2374–2380. doi:10.1161/CIRCULATIONAHA.111.076257
  57. Agarwal S, Tuzcu EM, Desai MY, et al. Updated meta-analysis of septal alcohol ablation versus myectomy for hypertrophic cardiomyopathy. J Am Coll Cardiol 2010; 55(8):823–834. doi:10.1016/j.jacc.2009.09.047
  58. Leonardi RA, Kransdorf EP, Simel DL, Wang A. Meta-analyses of septal reduction therapies for obstructive hypertrophic cardiomyopathy: comparative rates of overall mortality and sudden cardiac death after treatment. Circ Cardiovasc Interv 2010; 3(2):97–104. doi:10.1161/CIRCINTERVENTIONS.109.916676
  59. Liebregts M, Vriesendorp PA, Mahmoodi BK, Schinkel AF, Michels M, ten Berg JM. A systematic review and meta-analysis of long-term outcomes after septal reduction therapy in patients with hypertrophic cardiomyopathy. JACC Heart Fail 2015; 3(11):896–905. doi:10.1016/j.jchf.2015.06.011
  60. Panaich SS, Badheka AO, Chothani A, et al. Results of ventricular septal myectomy and hypertrophic cardiomyopathy (from Nationwide Inpatient Sample [1998-2010]). Am J Cardiol 2014; 114(9):1390–1395. doi:10.1016/j.amjcard.2014.07.075
  61. Maron BJ, Dearani JA, Ommen SR, et al. Low operative mortality achieved with surgical septal myectomy: role of dedicated hypertrophic cardiomyopathy centers in the management of dynamic subaortic obstruction. J Am Coll Cardiol 2015; 66(11):1307–1308. doi:10.1016/j.jacc.2015.06.1333
  62. Kwon DH, Smedira NG, Thamilarasan M, Lytle BW, Lever H, Desai MY. Characteristics and surgical outcomes of symptomatic patients with hypertrophic cardiomyopathy with abnormal papillary muscle morphology undergoing papillary muscle reorientation. J Thorac Cardiovasc Surg 2010; 140(2):317–324. doi:10.1016/j.jtcvs.2009.10.045
  63. Schaff HV, Brown ML, Dearani JA, et al. Apical myectomy: a new surgical technique for management of severely symptomatic patients with apical hypertrophic cardiomyopathy. J Thorac Cardiovasc Surg 2010; 139(3):634–640. doi:10.1016/j.jtcvs.2009.07.079
Article PDF
Author and Disclosure Information

Laura Young, MD
Robert and Suzanne Tomsich Department of Cardiovascular Medicine, Heart and Vascular Institute, Cleveland Clinic

Nicholas G. Smedira, MD
Department of Cardiothoracic Surgery, Heart and Vascular Institute, and Transplantation Center, Cleveland Clinic; Professor, Cleveland Clinic Lerner College of Medicine of Case Western Reserve University, Cleveland, OH

Albree Tower-Rader, MD
Robert and Suzanne Tomsich Department of Cardiovascular Medicine, Heart and Vascular Institute, Cleveland Clinic; Clinical Instructor, Cleveland Clinic Lerner College of Medicine of Case Western Reserve University, Cleveland, OH

Harry Lever, MD
Robert and Suzanne Tomsich Department of Cardiovascular Medicine, Heart and Vascular Institute, Cleveland Clinic; Clinical Assistant Professor, Cleveland Clinic Lerner College of Medicine of Case Western Reserve University, Cleveland, OH

Milind Y. Desai, MD
Robert and Suzanne Tomsich Department of Cardiovascular Medicine, Heart and Vascular Institute, and Department of Diagnostic Radiology, Cleveland Clinic; Professor, Cleveland Clinic Lerner College of Medicine of Case Western Reserve University, Cleveland, OH

Address: Milind Y. Desai, MD, Department of Cardiovascular Medicine, Cleveland Clinic, 9500 Euclid Avenue, J1-5, Cleveland, OH 44195; desaim2@ccf.org

Issue
Cleveland Clinic Journal of Medicine - 85(5)
Publications
Topics
Page Number
399-411
Legacy Keywords
hypertrophic cardiomyopathy, HCM, sarcomere, sudden cardiac death, left ventricular hypertrophy, LVH, left ventricular outflow tract obstruction, LVOT, mitral valve, papillary muscle, Laura Young, Nicholas Smedira, Albree Tower-Rader, Harry Lever, and Milind Desai
Sections
Author and Disclosure Information

Laura Young, MD
Robert and Suzanne Tomsich Department of Cardiovascular Medicine, Heart and Vascular Institute, Cleveland Clinic

Nicholas G. Smedira, MD
Department of Cardiothoracic Surgery, Heart and Vascular Institute, and Transplantation Center, Cleveland Clinic; Professor, Cleveland Clinic Lerner College of Medicine of Case Western Reserve University, Cleveland, OH

Albree Tower-Rader, MD
Robert and Suzanne Tomsich Department of Cardiovascular Medicine, Heart and Vascular Institute, Cleveland Clinic; Clinical Instructor, Cleveland Clinic Lerner College of Medicine of Case Western Reserve University, Cleveland, OH

Harry Lever, MD
Robert and Suzanne Tomsich Department of Cardiovascular Medicine, Heart and Vascular Institute, Cleveland Clinic; Clinical Assistant Professor, Cleveland Clinic Lerner College of Medicine of Case Western Reserve University, Cleveland, OH

Milind Y. Desai, MD
Robert and Suzanne Tomsich Department of Cardiovascular Medicine, Heart and Vascular Institute, and Department of Diagnostic Radiology, Cleveland Clinic; Professor, Cleveland Clinic Lerner College of Medicine of Case Western Reserve University, Cleveland, OH

Address: Milind Y. Desai, MD, Department of Cardiovascular Medicine, Cleveland Clinic, 9500 Euclid Avenue, J1-5, Cleveland, OH 44195; desaim2@ccf.org

Author and Disclosure Information

Laura Young, MD
Robert and Suzanne Tomsich Department of Cardiovascular Medicine, Heart and Vascular Institute, Cleveland Clinic

Nicholas G. Smedira, MD
Department of Cardiothoracic Surgery, Heart and Vascular Institute, and Transplantation Center, Cleveland Clinic; Professor, Cleveland Clinic Lerner College of Medicine of Case Western Reserve University, Cleveland, OH

Albree Tower-Rader, MD
Robert and Suzanne Tomsich Department of Cardiovascular Medicine, Heart and Vascular Institute, Cleveland Clinic; Clinical Instructor, Cleveland Clinic Lerner College of Medicine of Case Western Reserve University, Cleveland, OH

Harry Lever, MD
Robert and Suzanne Tomsich Department of Cardiovascular Medicine, Heart and Vascular Institute, Cleveland Clinic; Clinical Assistant Professor, Cleveland Clinic Lerner College of Medicine of Case Western Reserve University, Cleveland, OH

Milind Y. Desai, MD
Robert and Suzanne Tomsich Department of Cardiovascular Medicine, Heart and Vascular Institute, and Department of Diagnostic Radiology, Cleveland Clinic; Professor, Cleveland Clinic Lerner College of Medicine of Case Western Reserve University, Cleveland, OH

Address: Milind Y. Desai, MD, Department of Cardiovascular Medicine, Cleveland Clinic, 9500 Euclid Avenue, J1-5, Cleveland, OH 44195; desaim2@ccf.org

Article PDF
Article PDF
Related Articles

Hypertrophic cardiomyopathy (HCM) is a complex disease. Most people who carry the mutations that cause it are never affected at any point in their life, but some are affected at a young age. And in rare but tragic cases, some die suddenly while competing in sports. With such a wide range of phenotypic expressions, a single therapy does not fit all.

HCM is more common than once thought. Since the discovery of its genetic predisposition in 1960, it has come to be recognized as the most common heritable cardiovascular disease.1 Although earlier epidemiologic studies had estimated a prevalence of 1 in 500 (0.2%) of the general population, genetic testing and cardiac magnetic resonance imaging (MRI) now show that up to 1 in 200 (0.5%) of all people may be affected.1,2 Its prevalence is significant in all ethnic groups.

This review outlines our expanding knowledge of the pathophysiology, diagnosis, and clinical management of HCM.

A PLETHORA OF MUTATIONS IN CARDIAC SARCOMERIC GENES

young_hypertrophiccardiomyopathy_f1.jpg
Figure%201.%20A%2C%20echocardiography%2C%20apical%204-chamber%20view%2C%20demonstrates%20septal%20hypertrophy%20(arrow).%20B%2C%20cardiac%20magnetic%20resonance%20imaging%20of%20the%20left%20ventricular%20outflow%20tract%20also%20demonstrates%20septal%20hypertrophy%20(arrow).%20C%2C%20echocardiography%20with%20continuous-wave%20Doppler%20across%20the%20left%20ventricular%20outflow%20tract%20demonstrates%20a%20gradient%20of%2070%20mm%20Hg%2C%20consistent%20with%20obstruction.%20D%2C%20electrocardiography%20reveals%20signs%20of%20left%20ventricular%20hypertrophy%20by%20Sokolov-Lynon%20criteria%20with%20S%20wave%20depth%20in%20V%3Csub%3E1%3C%2Fsub%3E%20plus%20R%20wave%20height%20in%20V%3Csub%3E5%3C%2Fsub%3E%20%3E%2035%20mm%20(arrows).
The genetic basis of HCM is much more complex than was originally thought, with more than 1,400 mutations in 11 sarcomeric protein genes now known to be associated with the disease. Most of these mutations are autosomal dominant.3

The genetic differences within HCM result in varying degrees and locations of left ventricular hypertrophy. Any segment of the ventricle can be involved, although HCM is classically asymmetric and mainly involves the septum (Figure 1). A variant form of HCM involves the apex of the heart (Figure 2).

LEFT VENTRICULAR OUTFLOW TRACT OBSTRUCTION

young_hypertrophiccardiomyopathy_f2.jpg
Figure%202.%20A%2C%20echocardiography%2C%20apical%204-chamber%20view%2C%20shows%20apical%20hypertrophy%20(arrows).%20B%2C%20cardiac%20magnetic%20resonance%20imaging%20(4-chamber%20view)%20shows%20apical%20hypertrophy%20(red%20arrows)%2C%20as%20well%20as%20an%20apical%20aneurysm%20(blue%20arrow).%20C%2C%20electrocardiography%20demonstrates%20giant%20T-wave%20inversions%20in%20the%20left%20precordial%20leads%2C%20characteristic%20of%20apical%20hypertrophic%20cardiomyopathy%20(arrows).
Obstruction of the left ventricular outflow tract is thought to be the pivotal pathophysiologic process of HCM. Other abnormalities may include myocardial ischemia and diastolic dysfunction, believed to be related to narrowing of the intramural coronary arteries.4 Histopathologic study of heart muscle in HCM demonstrates disarray of the hypertrophied myocyte architecture with variable patterns of interstitial fibrosis.

Only in the last decade has the significance of left ventricular outflow tract obstruction in HCM been truly appreciated. The degree of obstruction in HCM is dynamic, as opposed to the fixed obstruction in patients with aortic stenosis or congenital subvalvular membranes. Therefore, in HCM, exercise or drugs (eg, dobutamine) that increase cardiac contractility increase the obstruction, as do maneuvers or drugs (the Valsalva maneuver, nitrates) that reduce filling of the left ventricle.

young_hypertrophiccardiomyopathy_f3.jpg
Figure%203.%20Left%20ventricular%20outflow%20tract%20obstruction%20due%20to%20ventricular%20septal%20hypertrophy.%20The%20obstruction%20is%20dynamic%2C%20as%20the%20blood%20flow%20sweeps%20the%20mitral%20valve%20toward%20the%20septum.
The obstruction is usually due to a combination of systolic anterior motion of the mitral valve and accelerated blood flow around the hypertrophied septum, resulting in a pushing force that sweeps the mitral valve toward the septum (Figure 3).5,6

A less common source of dynamic obstruction is the papillary muscles (Figure 4). Hypertrophy of the papillary muscles can result in obstruction by these muscles themselves, which is visible on echocardiography. Anatomic variations include anteroapical displacement or bifid papillary muscles, and these variants can be associated with dynamic left ventricular outflow tract obstruction, even with no evidence of septal thickening (Figure 5).7,8 Recognizing this patient subset has important implications for management, as discussed below.

DIAGNOSTIC EVALUATION

The clinical presentation varies

young_hypertrophiccardiomyopathy_f4.jpg
Figure%204.%20A%2C%20echocardiography%2C%20apical%204-chamber%20view%2C%20demonstrates%20a%20bifid%20papillary%20muscle%20resulting%20in%20left%20ventricular%20outflow%20tract%20obstruction%20(arrows).%20B%2C%20cardiac%20magnetic%20resonance%20imaging%20(left%20ventricular%20outflow%20tract%20view)%20demonstrates%20a%20bifid%20papillary%20muscle%20(arrows).%20C%2C%20an%20electrocardiogram%20of%20a%20patient%20with%20obstruction%20related%20to%20abnormal%20papillary%20muscle%20morphology%20demonstrates%20a%20lack%20of%20significant%20left%20ventricular%20hypertrophy.%20D%2C%20continuous-wave%20Doppler%20through%20the%20left%20ventricular%20outflow%20tract%20demonstrates%20a%20peak%20gradient%20of%2099%20mm%20Hg%2C%20consistent%20with%20obstruction%2C%20which%20increases%20with%20the%20Valsalva%20maneuver%20to%20119%20mm%20Hg%20(E).
HCM is a clinical diagnosis: currently, there is no test that can definitively confirm it. It is defined as left ventricular hypertrophy without dilated ventricular chambers that cannot be explained by another disease state, with hypertrophy defined as wall thickness of 15 mm or greater in adults.9 The differential diagnosis of HCM is summarized in Table 1.

Even if patients harbor the same genetic variant, the clinical presentation can differ widely. Although the most feared presentation is sudden cardiac death, particularly in young athletes, most patients have no symptoms and can anticipate a normal life expectancy. The annual incidence of sudden cardiac death in all HCM patients is estimated at less than 1%.10 Sudden cardiac death in HCM patients is most often due to ventricular tachyarrhythmias and most often occurs in asymptomatic patients under age 35.

young_hypertrophiccardiomyopathy_f5.jpg
Figure%205.%20Left%20ventricular%20outflow%20tract%20(LVOT)%20obstruction%20without%20significant%20left%20ventricular%20hypertrophy.%20The%20prominent%20bifid%20papillary%20muscles%20lead%20to%20systolic%20anterior%20motion%20of%20the%20mitral%20valve%2C%20causing%20LVOT%20obstruction%20and%20simultaneous%20mitral%20regurgitation.
Patients with symptoms may present with progressive exertional dyspnea, chest pain, or syncope that may be related to left ventricular outflow tract obstruction, myocardial ischemia, arrhythmia, or heart failure. Left ventricular outflow tract obstruction, defined as a resting peak gradient of 30 mm Hg or higher, affects one-third of HCM patients. Another third have a dynamic, provoked gradient of 30 mm Hg or higher during the Valsalva maneuver, aerobic exercise, or pharmacologic provocation with amyl nitrate.11 Identifying these patients at the time of diagnosis is important for prognostication, as discussed below.

Physical findings are nonspecific

young_hypertrophiccardiomyopathy_t1.jpg
Physical findings may be unremarkable, especially in patients without resting left ventricular outflow tract obstruction. When present, the physical findings are nonspecific and include systolic murmurs, bifid carotid pulse, a fourth heart sound, and a hyperdynamic precordium.

It can be difficult to distinguish the murmur of left ventricular outflow tract obstruction in HCM from a murmur related to aortic stenosis by auscultation alone. The simplest clinical method for telling them apart involves the Valsalva maneuver: bearing down creates a positive intrathoracic pressure and limits venous return, thus decreasing intracardiac filling pressure. This in turn results in less separation between the mitral valve and the ventricular septum in HCM, which increases obstruction and therefore makes the murmur louder. In contrast, in patients with fixed obstruction due to aortic stenosis, the murmur will decrease in intensity owing to the reduced flow associated with reduced preload.

 

 

Laboratory testing for phenocopies of HCM

young_hypertrophiccardiomyopathy_t2.jpg
Laboratory testing should be done at index encounters for all patients suspected of having HCM, as testing can help identify patients with HCM phenocopies, ie, a group of rare but clinically important diseases that cause pathologic left ventricular hypertrophy that is not due to sarcomeric gene defects. Identifying these conditions early is pivotal, as their natural history, management, and prognosis are significantly different (Table 2).

A metabolic panel will show derangements in liver function and glucose levels in patients with glycogen storage disorders such as Pompe disease. 

Serum creatinine. Renal dysfunction will be seen in patients with Fabry disease or amyloidosis.

Creatine kinase may be elevated in patients with Danon disease.

Electrocardiographic findings are common

More than 90% of HCM patients have electrocardiographic abnormalities. Although the findings can vary widely, common manifestations include:

  • Left ventricular hypertrophy
  • A pseudoinfarct pattern with Q waves in the anterolateral leads
  • Repolarization changes such as T-wave inversions and horizontal or down-sloping ST segments.

Apical HCM, seen mainly in Asian populations, often presents with giant T-wave inversion (> 10 mm) in the anterolateral leads, most prominent in V4, V5, and V6.

Notably, the degree of electrocardiographic abnormalities does not correlate with the severity or pattern of hypertrophy.9 Electrocardiography lacks specificity for definitive diagnosis, and further diagnostic testing should therefore be pursued.

Echocardiography: Initial imaging test

Transthoracic echocardiography is the initial imaging test in patients with suspected HCM, allowing for cost-effective quantitative and qualitative assessment of left ventricular morphology and function. Left ventricular hypertrophy is considered pathologic if wall thickness is 15 mm or greater without a known cause. Transthoracic echocardiography also allows for evaluation of left atrial volume and mitral valve anatomy and function.

Speckle tracking imaging is an advanced echocardiographic technique that measures strain. Its major advantage is in identifying early abnormalities in genotype-positive, phenotype-negative HCM patients, ie, people who harbor mutations but who have no clinical symptoms or signs of HCM, potentially allowing for modification of the natural history of HCM.12 Strain imaging can also differentiate between physiologic hypertrophy (“athlete’s heart”) and hypertension and HCM.13,14

The utility of echocardiography in HCM is heavily influenced by the sonographer’s experience in obtaining adequate acoustic windows. This may be more difficult in obese patients, patients with advanced obstructive lung disease or pleural effusions, and women with breast implants.

Magnetic resonance imaging

MRI has an emerging role in both diagnosing and predicting risk in HCM, and is routinely done as an adjunct to transthoracic echocardiography on initial diagnosis in our tertiary referral center. It is particularly useful in patients suspected of having apical hypertrophy (Figure 2), in whom the diagnosis may be missed in up to 10% on transthoracic echocardiography alone.15 MRI can also enhance the assessment of left ventricular hypertrophy and has been shown to improve the diagnostic classification of HCM.16 It is the best way to assess myocardial tissue abnormalities, and late gadolinium enhancement to detect interstitial fibrosis can be used for further prognostication. While historically the primary role of MRI in HCM has been in phenotype classification, there is currently much interest in its role in risk stratification of HCM patients for ICD implantation.

MRI with late gadolinium enhancement provides insight into the location, pattern, and extent of myocardial fibrosis; the extent of fibrosis has been shown to be a strong independent predictor of poor outcomes, including sudden cardiac death.17–20 However, late gadolinium enhancement can be technically challenging, as variations in the timing of postcontrast imaging, sequences for measuring late gadolinium enhancement, or detection thresholds can result in widely variable image quality. Cardiac MRI should therefore be performed at an experienced center with standardized imaging protocols in place.

Current guidelines recommend considering cardiac MRI if a patient’s risk of sudden cardiac death remains inconclusive after conventional risk stratification, as discussed below.9,21

Stress testing for risk stratification

Exercise stress electrocardiography. Treadmill exercise stress testing with electrocardiography and hemodynamic monitoring was one of the first tools used for risk stratification in HCM.

Although systolic blood pressure normally increases by at least 20 mm Hg with exercise, one-quarter of HCM patients have either a blunted response (failure of systolic blood pressure to increase by at least 20 mm Hg) or a hypotensive response (a drop in systolic blood pressure of 20 mm Hg or more, either continuously or after an initial increase). Studies have shown that HCM patients who have abnormal blood pressure responses during exercise have a higher risk of sudden cardiac death.22–24

Exercise stress echocardiography can be useful to evaluate for provoked increases in the left ventricular outflow tract gradient, which may contribute to a patient’s symptoms even if the resting left ventricular outflow tract gradient is normal. Exercise testing is preferred over pharmacologic stimulation because it can provide functional assessment of whether a patient’s clinical symptoms are truly related to hemodynamic changes due to the hypertrophied ventricle, or whether alternative mechanisms should be explored.

Cardiopulmonary stress testing can readily add prognostic value with additional measurements of functional capacity. HCM patients who cannot achieve their predicted maximal exercise value such as peak rate of oxygen consumption, ventilation efficiency, or anaerobic threshold have higher rates of morbidity and mortality.25,26 Stress testing can also be useful for risk stratification in asymptomatic patients, with one study showing that those who achieve more than 100% of their age- and sex-predicted metabolic equivalents have a low event rate.27

 

 

Ambulatory electrocardiographic monitoring in all patients at diagnosis

Ambulatory electrocardiographic monitoring for 24 to 48 hours is recommended for all HCM patients at the time of diagnosis, even if they have no symptoms. Any evidence of nonsustained ventricular tachycardia suggests a substantially higher risk of sudden cardiac death.28,29

In patients with no symptoms or history of arrhythmia, current guidelines suggest ambulatory electrocardiographic monitoring every 1 to 2 years.9,21

Two risk-stratification models

young_hypertrophiccardiomyopathy_t3.jpg
Two models are widely available for risk stratification in HCM (Table 3). While the consensus is to implant a cardioverter-defibrillator for secondary prevention if a patient has a history of ventricular arrhythmia or cardiac arrest, the approach to primary prevention differs between these 2 models.

The North American model was the first risk-stratification tool and considers 5 risk factors.9 However, if this algorithm were strictly followed, up to 60% of HCM patients would be candidates for cardioverter-defibrillator implantation.

The European model. This concern led to the development of the HCM Risk-SCD (sudden cardiac death), a risk-stratification tool introduced in the 2014 European Society of Cardiology HCM guidelines.30 This web-based calculator estimates a patient’s 5-year risk of sudden cardiac death using a complex calculation based on 7 clinical risk factors. If a patient’s calculated 5-year risk of sudden cardiac death is 6% or higher, cardioverter-defibrillator implantation is recommended for primary prevention.

The HCM Risk-SCD calculator was validated and compared with classic risk factors alone in a retrospective cohort study in 48 HCM patients.30 Compared with the North American model, the European model results in a lower rate of cardioverter-defibrillator implantation (20% to 26%).31,32

Despite the better specificity of the European model, a large retrospective cohort analysis showed that a significant number of patients stratified as being at low risk for sudden cardiac death were ultimately found to be at high risk in clinical practice.31 Further research is needed to find the optimal risk-stratification approach in HCM patients at low to intermediate risk.

GENETIC TESTING, COUNSELING, AND FAMILY SCREENING

Genetic testing is becoming more widely available and has rapidly expanded in clinical practice. Genetic counseling must be performed alongside genetic testing and requires professionals trained to handle the clinical and social implications of genetic testing. With this in mind, genetic testing can provide a definitive means of identifying family members at risk of HCM.

Given the autosomal dominant nature of HCM, screening for HCM is recommended in all first-degree relatives of an affected patient. Genetic testing may be a means to achieve this if a pathogenic mutation has been identified in the affected patient. However, serial electrocardiographic and transthoracic echocardiographic monitoring is an acceptable alternative in those without a clear genetic mutation association or in those who do not want to undergo genetic testing. If these first-degree relatives who do not undergo genetic testing are adult athletes or adolescents, they should undergo surveillance monitoring, with echocardiography and electrocardiography, whereas adults not participating in athletics should be monitored every 5 years.9,21

As genetic counseling and testing become more widely available, more patients are being found who harbor a mutation but have no phenotypic manifestations of HCM on initial presentation. Clinical expression varies, so continued monitoring of these patients is important. Expert guidelines again recommend serial electrocardiography, transthoracic echocardiography, and clinical assessment every 5 years for adults.9

Recent data suggest that up to 40% of HCM cases are nonfamilial, ie, their inheritance is sporadic with no known family history and no sarcomeric gene mutation evident on screening.33,34 The clinical course in this subgroup seems to be more benign, with later clinical presentations (age > 40) and lower risk of major adverse cardiovascular events.

MANAGEMENT

Conservative management

Asymptomatic HCM can usually be managed with lifestyle modifications.

Avoiding high-risk physical activities is the most important modification. All HCM patients should be counseled on the risk of sudden cardiac death and advised against participating in competitive sports or intense physical activity.35 Aerobic exercise is preferable to isometric exercises such as weightlifting, which may prompt the Valsalva maneuver with worsening of left ventricular outflow tract obstruction leading to syncope. A recent study showed that moderate-intensity aerobic exercise can safely improve exercise capacity, which may ultimately improve functional status and quality of life.36

Avoiding dehydration and excessive alcohol intake are also important in maintaining adequate preload to prevent an increasing left ventricular outflow tract gradient, given the dynamic nature of the left ventricular outflow tract obstruction in HCM.

 

 

Medical management: Beta-blockers, then calcium channel blockers

Beta-blockers are the first-line therapy for symptomatic HCM related to left ventricular outflow tract obstruction. Their negative inotropic effect reduces the contractile force of the ventricle, effectively reducing the pressure gradient across the outflow tract. Reduced contractility also means that the overall myocardial workload is less, which ultimately translates to a reduced oxygen demand. With their negative chronotropic effect, beta-blockers lower the heart rate and thereby lengthen the diastolic filling phase, allowing for optimization of preload conditions to help prevent increasing the left ventricular outflow tract gradient.37,38

Beta-blockers can be titrated according to the patient’s symptoms and tolerance. Fatigue and loss of libido are among the most common side effects.

Nondihydropyridine calcium channel blockers can be a second-line therapy in patients who cannot tolerate beta-blockers. Several studies have shown improvement in surrogate outcomes such as estimated left ventricular mass and QRS amplitude on electrocardiography, but currently no available data show that these drugs improve symptoms.28,39,40 They should be avoided in those with severe left ventricular outflow tract obstruction (gradient 100 mm Hg), as they can lead to critical outflow tract obstruction owing to their peripheral vasodilatory effect.

Dihydropyridine calcium channel blockers should be avoided altogether, as they produce even more peripheral vasodilation and afterload reduction than nondihydropyridine calcium channel blockers.

Disopyramide, a class IA antiarrhythmic, has been shown to effectively reduce outflow gradients and relieve symptoms. However, in view of its adverse effects, it is a third-line therapy, given to those for whom beta-blockers and calcium channel blockers have failed. Its most worrisome adverse effect is QT prolongation, and the QT interval should therefore be closely monitored at the start of treatment. Anticholinergic effects are common and include dry eyes and mouth, urinary retention, and drowsiness.

Disopyramide is usually used in combination with beta-blockers for symptom control as a bridge to a planned invasive intervention.41

Use with caution

Any medication that causes afterload reduction, peripheral vasodilation, intravascular volume depletion, or positive inotropy can worsen the dynamic left ventricular outflow tract obstruction in a patient with HCM and should be avoided.

Angiotensin-converting enzyme (ACE) inhibitors, angiotensin II receptor blockers (ARBs), and nitrates must be used with extreme caution in these patients.

Diuretics. Even restrained use of diuretics can cause significant hemodynamic compromise in patients with obstructive physiology. Therefore, diuretics should be used sparingly in these patients.

Digoxin should not be used for managing atrial fibrillation in these patients, as its positive inotropic effect increases contractility and increases the left ventricular outflow tract gradient.

Norepinephrine and inotropic agents such as dobutamine and dopamine should be avoided for the same reason as digoxin. In patients with circulatory shock requiring vasopressor support, pure alpha-agonists such as phenylephrine are preferred, as they increase peripheral resistance without an inotropic effect.

Anticoagulation for atrial tachyarrhythmias

The risk of systemic thromboembolic events is significantly increased in HCM patients with atrial fibrillation or flutter, regardless of their estimated risk using conventional risk-stratification tools such as the CHADS2 score.42–44 In accordance with current American Heart Association and American College of Cardiology guidelines, we recommend anticoagulation therapy for all HCM patients with a history of atrial fibrillation or flutter. Warfarin is the preferred anticoagulant; direct oral anticoagulants can be considered, but there are currently no data on their use in HCM.9

Standard heart failure treatments

End-stage systolic heart failure is a consequence of HCM but affects only 3% to 4% of patients.45 While most randomized controlled trials of heart failure treatment have excluded HCM patients, current guidelines recommend the same evidence-based medical therapies used in other patients who have heart failure with reduced ejection fraction. This includes ACE inhibitors, ARBs, beta-blockers, and aldosterone antagonists if indicated.9,21

Heart transplant should be considered in patients with class III or IV New York Heart Association functional status despite optimization of their HCM treatment regimen. Heart transplant outcomes for HCM patients are comparable to outcomes for patients who receive a transplant for non-HCM cardiovascular disease.45,46

 

 

Septal reduction therapy

If medical therapy fails or is not tolerated in patients with severe symptoms, surgery can be considered for obstructive HCM.

Ventricular septal myectomy has been the long-standing gold standard of invasive therapy. Multiple studies have demonstrated long-term survival after myectomy to be equivalent to that in the general population and better than that of HCM patients who do not undergo this surgery.47–50 Factors that may be associated with better surgical outcomes include age younger than 50, left atrial size less than 46 mm, and resolution of atrial fibrillation during follow-up.51

Septal reduction therapy may also be considered in patients at high risk of sudden cardiac death based on a history of recurrent ventricular tachycardia or risk-stratification models as described above. Retrospective analyses have shown that surgical myectomy can markedly reduce the incidence of appropriate implantable cardioverter-defibrillator discharges and the risk of sudden cardiac death.52

Alcohol septal ablation is an alternative. This percutaneous procedure, first described in the mid-1990s, consists of injecting a small amount of alcohol into the artery supplying the septum to induce myocardial necrosis, ultimately leading to scarring and widening of the left ventricular outflow tract.53

Up to 50% of patients develop right bundle branch block after alcohol septal ablation, and the risk of complete heart block is highest in those with preexisting left bundle branch block. Nevertheless, studies have shown significant symptomatic improvement after alcohol septal ablation, with long-term survival comparable to that in the general population.53–56

Several meta-analyses compared alcohol septal ablation and septal myectomy and found that the rates of functional improvement and long-term mortality were similar.57–59 However, the less-invasive approach with alcohol septal ablation comes at the cost of a higher incidence of conduction abnormalities and higher left ventricular outflow tract gradients afterward. One meta-analysis found that alcohol septal ablation patients may have 5 times the risk of needing additional septal reduction therapy compared with their myectomy counterparts.

Current US guidelines recommend septal myectomy, performed at an experienced center, as the first-line interventional treatment, leaving alcohol septal ablation to be considered in those who have contraindications to myectomy.9 The treatment strategy should ultimately be individualized based on a patient’s comorbidities and personal preferences following informed consent.

A nationwide database study recently suggested that postmyectomy mortality rates may be as high as 5.9%,60 although earlier studies at high-volume centers showed much lower mortality rates (< 1%).50–52,61 This discrepancy highlights the critical role of expert centers in optimizing surgical management of these patients. Regardless of the approach, interventional therapies for HCM should be performed by a multidisciplinary team at a medical center able to handle the complexity of these cases.

Additional surgical procedures

A handful of other procedures may benefit specific patient subgroups.

young_hypertrophiccardiomyopathy_f6.jpg
Figure%206.%20Reorientation%20surgery%20reduces%20mobility%20of%20bifid%20hypermobile%20papillary%20muscles%2C%20reducing%20left%20ventricular%20outflow%20tract%20(LVOT)%20obstruction.
Papillary muscle reorientation surgery (Figure 6) has been shown in retrospective studies to reduce mobility of bifid hypermobile papillary muscles and alleviate left ventricular outflow tract obstruction.62 It should be considered in patients who have this problem, even if they have no left ventricular hypertrophy.

Apical myectomy has been shown to improve functional status in patients with isolated apical hypertrophy by reducing left ventricular end-diastolic pressure and thereby allowing for improved diastolic filling.63

Mitral valve surgery may need to be considered at the time of myectomy in patients with degenerative valve disease. As in the general population, mitral valve repair is preferred to replacement if possible. 

Hypertrophic cardiomyopathy (HCM) is a complex disease. Most people who carry the mutations that cause it are never affected at any point in their life, but some are affected at a young age. And in rare but tragic cases, some die suddenly while competing in sports. With such a wide range of phenotypic expressions, a single therapy does not fit all.

HCM is more common than once thought. Since the discovery of its genetic predisposition in 1960, it has come to be recognized as the most common heritable cardiovascular disease.1 Although earlier epidemiologic studies had estimated a prevalence of 1 in 500 (0.2%) of the general population, genetic testing and cardiac magnetic resonance imaging (MRI) now show that up to 1 in 200 (0.5%) of all people may be affected.1,2 Its prevalence is significant in all ethnic groups.

This review outlines our expanding knowledge of the pathophysiology, diagnosis, and clinical management of HCM.

A PLETHORA OF MUTATIONS IN CARDIAC SARCOMERIC GENES

young_hypertrophiccardiomyopathy_f1.jpg
Figure%201.%20A%2C%20echocardiography%2C%20apical%204-chamber%20view%2C%20demonstrates%20septal%20hypertrophy%20(arrow).%20B%2C%20cardiac%20magnetic%20resonance%20imaging%20of%20the%20left%20ventricular%20outflow%20tract%20also%20demonstrates%20septal%20hypertrophy%20(arrow).%20C%2C%20echocardiography%20with%20continuous-wave%20Doppler%20across%20the%20left%20ventricular%20outflow%20tract%20demonstrates%20a%20gradient%20of%2070%20mm%20Hg%2C%20consistent%20with%20obstruction.%20D%2C%20electrocardiography%20reveals%20signs%20of%20left%20ventricular%20hypertrophy%20by%20Sokolov-Lynon%20criteria%20with%20S%20wave%20depth%20in%20V%3Csub%3E1%3C%2Fsub%3E%20plus%20R%20wave%20height%20in%20V%3Csub%3E5%3C%2Fsub%3E%20%3E%2035%20mm%20(arrows).
The genetic basis of HCM is much more complex than was originally thought, with more than 1,400 mutations in 11 sarcomeric protein genes now known to be associated with the disease. Most of these mutations are autosomal dominant.3

The genetic differences within HCM result in varying degrees and locations of left ventricular hypertrophy. Any segment of the ventricle can be involved, although HCM is classically asymmetric and mainly involves the septum (Figure 1). A variant form of HCM involves the apex of the heart (Figure 2).

LEFT VENTRICULAR OUTFLOW TRACT OBSTRUCTION

young_hypertrophiccardiomyopathy_f2.jpg
Figure%202.%20A%2C%20echocardiography%2C%20apical%204-chamber%20view%2C%20shows%20apical%20hypertrophy%20(arrows).%20B%2C%20cardiac%20magnetic%20resonance%20imaging%20(4-chamber%20view)%20shows%20apical%20hypertrophy%20(red%20arrows)%2C%20as%20well%20as%20an%20apical%20aneurysm%20(blue%20arrow).%20C%2C%20electrocardiography%20demonstrates%20giant%20T-wave%20inversions%20in%20the%20left%20precordial%20leads%2C%20characteristic%20of%20apical%20hypertrophic%20cardiomyopathy%20(arrows).
Obstruction of the left ventricular outflow tract is thought to be the pivotal pathophysiologic process of HCM. Other abnormalities may include myocardial ischemia and diastolic dysfunction, believed to be related to narrowing of the intramural coronary arteries.4 Histopathologic study of heart muscle in HCM demonstrates disarray of the hypertrophied myocyte architecture with variable patterns of interstitial fibrosis.

Only in the last decade has the significance of left ventricular outflow tract obstruction in HCM been truly appreciated. The degree of obstruction in HCM is dynamic, as opposed to the fixed obstruction in patients with aortic stenosis or congenital subvalvular membranes. Therefore, in HCM, exercise or drugs (eg, dobutamine) that increase cardiac contractility increase the obstruction, as do maneuvers or drugs (the Valsalva maneuver, nitrates) that reduce filling of the left ventricle.

young_hypertrophiccardiomyopathy_f3.jpg
Figure%203.%20Left%20ventricular%20outflow%20tract%20obstruction%20due%20to%20ventricular%20septal%20hypertrophy.%20The%20obstruction%20is%20dynamic%2C%20as%20the%20blood%20flow%20sweeps%20the%20mitral%20valve%20toward%20the%20septum.
The obstruction is usually due to a combination of systolic anterior motion of the mitral valve and accelerated blood flow around the hypertrophied septum, resulting in a pushing force that sweeps the mitral valve toward the septum (Figure 3).5,6

A less common source of dynamic obstruction is the papillary muscles (Figure 4). Hypertrophy of the papillary muscles can result in obstruction by these muscles themselves, which is visible on echocardiography. Anatomic variations include anteroapical displacement or bifid papillary muscles, and these variants can be associated with dynamic left ventricular outflow tract obstruction, even with no evidence of septal thickening (Figure 5).7,8 Recognizing this patient subset has important implications for management, as discussed below.

DIAGNOSTIC EVALUATION

The clinical presentation varies

young_hypertrophiccardiomyopathy_f4.jpg
Figure%204.%20A%2C%20echocardiography%2C%20apical%204-chamber%20view%2C%20demonstrates%20a%20bifid%20papillary%20muscle%20resulting%20in%20left%20ventricular%20outflow%20tract%20obstruction%20(arrows).%20B%2C%20cardiac%20magnetic%20resonance%20imaging%20(left%20ventricular%20outflow%20tract%20view)%20demonstrates%20a%20bifid%20papillary%20muscle%20(arrows).%20C%2C%20an%20electrocardiogram%20of%20a%20patient%20with%20obstruction%20related%20to%20abnormal%20papillary%20muscle%20morphology%20demonstrates%20a%20lack%20of%20significant%20left%20ventricular%20hypertrophy.%20D%2C%20continuous-wave%20Doppler%20through%20the%20left%20ventricular%20outflow%20tract%20demonstrates%20a%20peak%20gradient%20of%2099%20mm%20Hg%2C%20consistent%20with%20obstruction%2C%20which%20increases%20with%20the%20Valsalva%20maneuver%20to%20119%20mm%20Hg%20(E).
HCM is a clinical diagnosis: currently, there is no test that can definitively confirm it. It is defined as left ventricular hypertrophy without dilated ventricular chambers that cannot be explained by another disease state, with hypertrophy defined as wall thickness of 15 mm or greater in adults.9 The differential diagnosis of HCM is summarized in Table 1.

Even if patients harbor the same genetic variant, the clinical presentation can differ widely. Although the most feared presentation is sudden cardiac death, particularly in young athletes, most patients have no symptoms and can anticipate a normal life expectancy. The annual incidence of sudden cardiac death in all HCM patients is estimated at less than 1%.10 Sudden cardiac death in HCM patients is most often due to ventricular tachyarrhythmias and most often occurs in asymptomatic patients under age 35.

young_hypertrophiccardiomyopathy_f5.jpg
Figure%205.%20Left%20ventricular%20outflow%20tract%20(LVOT)%20obstruction%20without%20significant%20left%20ventricular%20hypertrophy.%20The%20prominent%20bifid%20papillary%20muscles%20lead%20to%20systolic%20anterior%20motion%20of%20the%20mitral%20valve%2C%20causing%20LVOT%20obstruction%20and%20simultaneous%20mitral%20regurgitation.
Patients with symptoms may present with progressive exertional dyspnea, chest pain, or syncope that may be related to left ventricular outflow tract obstruction, myocardial ischemia, arrhythmia, or heart failure. Left ventricular outflow tract obstruction, defined as a resting peak gradient of 30 mm Hg or higher, affects one-third of HCM patients. Another third have a dynamic, provoked gradient of 30 mm Hg or higher during the Valsalva maneuver, aerobic exercise, or pharmacologic provocation with amyl nitrate.11 Identifying these patients at the time of diagnosis is important for prognostication, as discussed below.

Physical findings are nonspecific

young_hypertrophiccardiomyopathy_t1.jpg
Physical findings may be unremarkable, especially in patients without resting left ventricular outflow tract obstruction. When present, the physical findings are nonspecific and include systolic murmurs, bifid carotid pulse, a fourth heart sound, and a hyperdynamic precordium.

It can be difficult to distinguish the murmur of left ventricular outflow tract obstruction in HCM from a murmur related to aortic stenosis by auscultation alone. The simplest clinical method for telling them apart involves the Valsalva maneuver: bearing down creates a positive intrathoracic pressure and limits venous return, thus decreasing intracardiac filling pressure. This in turn results in less separation between the mitral valve and the ventricular septum in HCM, which increases obstruction and therefore makes the murmur louder. In contrast, in patients with fixed obstruction due to aortic stenosis, the murmur will decrease in intensity owing to the reduced flow associated with reduced preload.

 

 

Laboratory testing for phenocopies of HCM

young_hypertrophiccardiomyopathy_t2.jpg
Laboratory testing should be done at index encounters for all patients suspected of having HCM, as testing can help identify patients with HCM phenocopies, ie, a group of rare but clinically important diseases that cause pathologic left ventricular hypertrophy that is not due to sarcomeric gene defects. Identifying these conditions early is pivotal, as their natural history, management, and prognosis are significantly different (Table 2).

A metabolic panel will show derangements in liver function and glucose levels in patients with glycogen storage disorders such as Pompe disease. 

Serum creatinine. Renal dysfunction will be seen in patients with Fabry disease or amyloidosis.

Creatine kinase may be elevated in patients with Danon disease.

Electrocardiographic findings are common

More than 90% of HCM patients have electrocardiographic abnormalities. Although the findings can vary widely, common manifestations include:

  • Left ventricular hypertrophy
  • A pseudoinfarct pattern with Q waves in the anterolateral leads
  • Repolarization changes such as T-wave inversions and horizontal or down-sloping ST segments.

Apical HCM, seen mainly in Asian populations, often presents with giant T-wave inversion (> 10 mm) in the anterolateral leads, most prominent in V4, V5, and V6.

Notably, the degree of electrocardiographic abnormalities does not correlate with the severity or pattern of hypertrophy.9 Electrocardiography lacks specificity for definitive diagnosis, and further diagnostic testing should therefore be pursued.

Echocardiography: Initial imaging test

Transthoracic echocardiography is the initial imaging test in patients with suspected HCM, allowing for cost-effective quantitative and qualitative assessment of left ventricular morphology and function. Left ventricular hypertrophy is considered pathologic if wall thickness is 15 mm or greater without a known cause. Transthoracic echocardiography also allows for evaluation of left atrial volume and mitral valve anatomy and function.

Speckle tracking imaging is an advanced echocardiographic technique that measures strain. Its major advantage is in identifying early abnormalities in genotype-positive, phenotype-negative HCM patients, ie, people who harbor mutations but who have no clinical symptoms or signs of HCM, potentially allowing for modification of the natural history of HCM.12 Strain imaging can also differentiate between physiologic hypertrophy (“athlete’s heart”) and hypertension and HCM.13,14

The utility of echocardiography in HCM is heavily influenced by the sonographer’s experience in obtaining adequate acoustic windows. This may be more difficult in obese patients, patients with advanced obstructive lung disease or pleural effusions, and women with breast implants.

Magnetic resonance imaging

MRI has an emerging role in both diagnosing and predicting risk in HCM, and is routinely done as an adjunct to transthoracic echocardiography on initial diagnosis in our tertiary referral center. It is particularly useful in patients suspected of having apical hypertrophy (Figure 2), in whom the diagnosis may be missed in up to 10% on transthoracic echocardiography alone.15 MRI can also enhance the assessment of left ventricular hypertrophy and has been shown to improve the diagnostic classification of HCM.16 It is the best way to assess myocardial tissue abnormalities, and late gadolinium enhancement to detect interstitial fibrosis can be used for further prognostication. While historically the primary role of MRI in HCM has been in phenotype classification, there is currently much interest in its role in risk stratification of HCM patients for ICD implantation.

MRI with late gadolinium enhancement provides insight into the location, pattern, and extent of myocardial fibrosis; the extent of fibrosis has been shown to be a strong independent predictor of poor outcomes, including sudden cardiac death.17–20 However, late gadolinium enhancement can be technically challenging, as variations in the timing of postcontrast imaging, sequences for measuring late gadolinium enhancement, or detection thresholds can result in widely variable image quality. Cardiac MRI should therefore be performed at an experienced center with standardized imaging protocols in place.

Current guidelines recommend considering cardiac MRI if a patient’s risk of sudden cardiac death remains inconclusive after conventional risk stratification, as discussed below.9,21

Stress testing for risk stratification

Exercise stress electrocardiography. Treadmill exercise stress testing with electrocardiography and hemodynamic monitoring was one of the first tools used for risk stratification in HCM.

Although systolic blood pressure normally increases by at least 20 mm Hg with exercise, one-quarter of HCM patients have either a blunted response (failure of systolic blood pressure to increase by at least 20 mm Hg) or a hypotensive response (a drop in systolic blood pressure of 20 mm Hg or more, either continuously or after an initial increase). Studies have shown that HCM patients who have abnormal blood pressure responses during exercise have a higher risk of sudden cardiac death.22–24

Exercise stress echocardiography can be useful to evaluate for provoked increases in the left ventricular outflow tract gradient, which may contribute to a patient’s symptoms even if the resting left ventricular outflow tract gradient is normal. Exercise testing is preferred over pharmacologic stimulation because it can provide functional assessment of whether a patient’s clinical symptoms are truly related to hemodynamic changes due to the hypertrophied ventricle, or whether alternative mechanisms should be explored.

Cardiopulmonary stress testing can readily add prognostic value with additional measurements of functional capacity. HCM patients who cannot achieve their predicted maximal exercise value such as peak rate of oxygen consumption, ventilation efficiency, or anaerobic threshold have higher rates of morbidity and mortality.25,26 Stress testing can also be useful for risk stratification in asymptomatic patients, with one study showing that those who achieve more than 100% of their age- and sex-predicted metabolic equivalents have a low event rate.27

 

 

Ambulatory electrocardiographic monitoring in all patients at diagnosis

Ambulatory electrocardiographic monitoring for 24 to 48 hours is recommended for all HCM patients at the time of diagnosis, even if they have no symptoms. Any evidence of nonsustained ventricular tachycardia suggests a substantially higher risk of sudden cardiac death.28,29

In patients with no symptoms or history of arrhythmia, current guidelines suggest ambulatory electrocardiographic monitoring every 1 to 2 years.9,21

Two risk-stratification models

young_hypertrophiccardiomyopathy_t3.jpg
Two models are widely available for risk stratification in HCM (Table 3). While the consensus is to implant a cardioverter-defibrillator for secondary prevention if a patient has a history of ventricular arrhythmia or cardiac arrest, the approach to primary prevention differs between these 2 models.

The North American model was the first risk-stratification tool and considers 5 risk factors.9 However, if this algorithm were strictly followed, up to 60% of HCM patients would be candidates for cardioverter-defibrillator implantation.

The European model. This concern led to the development of the HCM Risk-SCD (sudden cardiac death), a risk-stratification tool introduced in the 2014 European Society of Cardiology HCM guidelines.30 This web-based calculator estimates a patient’s 5-year risk of sudden cardiac death using a complex calculation based on 7 clinical risk factors. If a patient’s calculated 5-year risk of sudden cardiac death is 6% or higher, cardioverter-defibrillator implantation is recommended for primary prevention.

The HCM Risk-SCD calculator was validated and compared with classic risk factors alone in a retrospective cohort study in 48 HCM patients.30 Compared with the North American model, the European model results in a lower rate of cardioverter-defibrillator implantation (20% to 26%).31,32

Despite the better specificity of the European model, a large retrospective cohort analysis showed that a significant number of patients stratified as being at low risk for sudden cardiac death were ultimately found to be at high risk in clinical practice.31 Further research is needed to find the optimal risk-stratification approach in HCM patients at low to intermediate risk.

GENETIC TESTING, COUNSELING, AND FAMILY SCREENING

Genetic testing is becoming more widely available and has rapidly expanded in clinical practice. Genetic counseling must be performed alongside genetic testing and requires professionals trained to handle the clinical and social implications of genetic testing. With this in mind, genetic testing can provide a definitive means of identifying family members at risk of HCM.

Given the autosomal dominant nature of HCM, screening for HCM is recommended in all first-degree relatives of an affected patient. Genetic testing may be a means to achieve this if a pathogenic mutation has been identified in the affected patient. However, serial electrocardiographic and transthoracic echocardiographic monitoring is an acceptable alternative in those without a clear genetic mutation association or in those who do not want to undergo genetic testing. If these first-degree relatives who do not undergo genetic testing are adult athletes or adolescents, they should undergo surveillance monitoring, with echocardiography and electrocardiography, whereas adults not participating in athletics should be monitored every 5 years.9,21

As genetic counseling and testing become more widely available, more patients are being found who harbor a mutation but have no phenotypic manifestations of HCM on initial presentation. Clinical expression varies, so continued monitoring of these patients is important. Expert guidelines again recommend serial electrocardiography, transthoracic echocardiography, and clinical assessment every 5 years for adults.9

Recent data suggest that up to 40% of HCM cases are nonfamilial, ie, their inheritance is sporadic with no known family history and no sarcomeric gene mutation evident on screening.33,34 The clinical course in this subgroup seems to be more benign, with later clinical presentations (age > 40) and lower risk of major adverse cardiovascular events.

MANAGEMENT

Conservative management

Asymptomatic HCM can usually be managed with lifestyle modifications.

Avoiding high-risk physical activities is the most important modification. All HCM patients should be counseled on the risk of sudden cardiac death and advised against participating in competitive sports or intense physical activity.35 Aerobic exercise is preferable to isometric exercises such as weightlifting, which may prompt the Valsalva maneuver with worsening of left ventricular outflow tract obstruction leading to syncope. A recent study showed that moderate-intensity aerobic exercise can safely improve exercise capacity, which may ultimately improve functional status and quality of life.36

Avoiding dehydration and excessive alcohol intake are also important in maintaining adequate preload to prevent an increasing left ventricular outflow tract gradient, given the dynamic nature of the left ventricular outflow tract obstruction in HCM.

 

 

Medical management: Beta-blockers, then calcium channel blockers

Beta-blockers are the first-line therapy for symptomatic HCM related to left ventricular outflow tract obstruction. Their negative inotropic effect reduces the contractile force of the ventricle, effectively reducing the pressure gradient across the outflow tract. Reduced contractility also means that the overall myocardial workload is less, which ultimately translates to a reduced oxygen demand. With their negative chronotropic effect, beta-blockers lower the heart rate and thereby lengthen the diastolic filling phase, allowing for optimization of preload conditions to help prevent increasing the left ventricular outflow tract gradient.37,38

Beta-blockers can be titrated according to the patient’s symptoms and tolerance. Fatigue and loss of libido are among the most common side effects.

Nondihydropyridine calcium channel blockers can be a second-line therapy in patients who cannot tolerate beta-blockers. Several studies have shown improvement in surrogate outcomes such as estimated left ventricular mass and QRS amplitude on electrocardiography, but currently no available data show that these drugs improve symptoms.28,39,40 They should be avoided in those with severe left ventricular outflow tract obstruction (gradient 100 mm Hg), as they can lead to critical outflow tract obstruction owing to their peripheral vasodilatory effect.

Dihydropyridine calcium channel blockers should be avoided altogether, as they produce even more peripheral vasodilation and afterload reduction than nondihydropyridine calcium channel blockers.

Disopyramide, a class IA antiarrhythmic, has been shown to effectively reduce outflow gradients and relieve symptoms. However, in view of its adverse effects, it is a third-line therapy, given to those for whom beta-blockers and calcium channel blockers have failed. Its most worrisome adverse effect is QT prolongation, and the QT interval should therefore be closely monitored at the start of treatment. Anticholinergic effects are common and include dry eyes and mouth, urinary retention, and drowsiness.

Disopyramide is usually used in combination with beta-blockers for symptom control as a bridge to a planned invasive intervention.41

Use with caution

Any medication that causes afterload reduction, peripheral vasodilation, intravascular volume depletion, or positive inotropy can worsen the dynamic left ventricular outflow tract obstruction in a patient with HCM and should be avoided.

Angiotensin-converting enzyme (ACE) inhibitors, angiotensin II receptor blockers (ARBs), and nitrates must be used with extreme caution in these patients.

Diuretics. Even restrained use of diuretics can cause significant hemodynamic compromise in patients with obstructive physiology. Therefore, diuretics should be used sparingly in these patients.

Digoxin should not be used for managing atrial fibrillation in these patients, as its positive inotropic effect increases contractility and increases the left ventricular outflow tract gradient.

Norepinephrine and inotropic agents such as dobutamine and dopamine should be avoided for the same reason as digoxin. In patients with circulatory shock requiring vasopressor support, pure alpha-agonists such as phenylephrine are preferred, as they increase peripheral resistance without an inotropic effect.

Anticoagulation for atrial tachyarrhythmias

The risk of systemic thromboembolic events is significantly increased in HCM patients with atrial fibrillation or flutter, regardless of their estimated risk using conventional risk-stratification tools such as the CHADS2 score.42–44 In accordance with current American Heart Association and American College of Cardiology guidelines, we recommend anticoagulation therapy for all HCM patients with a history of atrial fibrillation or flutter. Warfarin is the preferred anticoagulant; direct oral anticoagulants can be considered, but there are currently no data on their use in HCM.9

Standard heart failure treatments

End-stage systolic heart failure is a consequence of HCM but affects only 3% to 4% of patients.45 While most randomized controlled trials of heart failure treatment have excluded HCM patients, current guidelines recommend the same evidence-based medical therapies used in other patients who have heart failure with reduced ejection fraction. This includes ACE inhibitors, ARBs, beta-blockers, and aldosterone antagonists if indicated.9,21

Heart transplant should be considered in patients with class III or IV New York Heart Association functional status despite optimization of their HCM treatment regimen. Heart transplant outcomes for HCM patients are comparable to outcomes for patients who receive a transplant for non-HCM cardiovascular disease.45,46

 

 

Septal reduction therapy

If medical therapy fails or is not tolerated in patients with severe symptoms, surgery can be considered for obstructive HCM.

Ventricular septal myectomy has been the long-standing gold standard of invasive therapy. Multiple studies have demonstrated long-term survival after myectomy to be equivalent to that in the general population and better than that of HCM patients who do not undergo this surgery.47–50 Factors that may be associated with better surgical outcomes include age younger than 50, left atrial size less than 46 mm, and resolution of atrial fibrillation during follow-up.51

Septal reduction therapy may also be considered in patients at high risk of sudden cardiac death based on a history of recurrent ventricular tachycardia or risk-stratification models as described above. Retrospective analyses have shown that surgical myectomy can markedly reduce the incidence of appropriate implantable cardioverter-defibrillator discharges and the risk of sudden cardiac death.52

Alcohol septal ablation is an alternative. This percutaneous procedure, first described in the mid-1990s, consists of injecting a small amount of alcohol into the artery supplying the septum to induce myocardial necrosis, ultimately leading to scarring and widening of the left ventricular outflow tract.53

Up to 50% of patients develop right bundle branch block after alcohol septal ablation, and the risk of complete heart block is highest in those with preexisting left bundle branch block. Nevertheless, studies have shown significant symptomatic improvement after alcohol septal ablation, with long-term survival comparable to that in the general population.53–56

Several meta-analyses compared alcohol septal ablation and septal myectomy and found that the rates of functional improvement and long-term mortality were similar.57–59 However, the less-invasive approach with alcohol septal ablation comes at the cost of a higher incidence of conduction abnormalities and higher left ventricular outflow tract gradients afterward. One meta-analysis found that alcohol septal ablation patients may have 5 times the risk of needing additional septal reduction therapy compared with their myectomy counterparts.

Current US guidelines recommend septal myectomy, performed at an experienced center, as the first-line interventional treatment, leaving alcohol septal ablation to be considered in those who have contraindications to myectomy.9 The treatment strategy should ultimately be individualized based on a patient’s comorbidities and personal preferences following informed consent.

A nationwide database study recently suggested that postmyectomy mortality rates may be as high as 5.9%,60 although earlier studies at high-volume centers showed much lower mortality rates (< 1%).50–52,61 This discrepancy highlights the critical role of expert centers in optimizing surgical management of these patients. Regardless of the approach, interventional therapies for HCM should be performed by a multidisciplinary team at a medical center able to handle the complexity of these cases.

Additional surgical procedures

A handful of other procedures may benefit specific patient subgroups.

young_hypertrophiccardiomyopathy_f6.jpg
Figure%206.%20Reorientation%20surgery%20reduces%20mobility%20of%20bifid%20hypermobile%20papillary%20muscles%2C%20reducing%20left%20ventricular%20outflow%20tract%20(LVOT)%20obstruction.
Papillary muscle reorientation surgery (Figure 6) has been shown in retrospective studies to reduce mobility of bifid hypermobile papillary muscles and alleviate left ventricular outflow tract obstruction.62 It should be considered in patients who have this problem, even if they have no left ventricular hypertrophy.

Apical myectomy has been shown to improve functional status in patients with isolated apical hypertrophy by reducing left ventricular end-diastolic pressure and thereby allowing for improved diastolic filling.63

Mitral valve surgery may need to be considered at the time of myectomy in patients with degenerative valve disease. As in the general population, mitral valve repair is preferred to replacement if possible. 

References
  1. Maron BJ. Hypertrophic cardiomyopathy: an important global disease. Am J Med 2004; 116(1):63–65. pmid:14706671
  2. Semsarian C, Ingles J, Maron MS, Maron BJ. New perspectives on the prevalence of hypertrophic cardiomyopathy. J Am Coll Cardiol 2015; 65(12):1249–1254. doi:10.1016/j.jacc.2015.01.019
  3. Maron BJ, Maron MS, Semsarian C. Genetics of hypertrophic cardiomyopathy after 20 years: clinical perspectives. J Am Coll Cardiol 2012; 60(8):705–715. doi:10.1016/j.jacc.2012.02.068
  4. Shirani J, Pick R, Roberts WC, Maron BJ. Morphology and significance of the left ventricular collagen network in young patients with hypertrophic cardiomyopathy and sudden cardiac death. J Am Coll Cardiol 2000; 35(1):36–44. pmid:10636256
  5. Sherrid MV, Chu CK, Delia E, Mogtader A, Dwyer EM Jr. An echocardiographic study of the fluid mechanics of obstruction in hypertrophic cardiomyopathy. J Am Coll Cardiol 1993; 22(3):816–825. pmid:8354817
  6. Ro R, Halpern D, Sahn DJ, et al. Vector flow mapping in obstructive hypertrophic cardiomyopathy to assess the relationship of early systolic left ventricular flow and the mitral valve. J Am Coll Cardiol 2014; 64(19):1984–1995. doi:10.1016/j.jacc.2014.04.090
  7. Kwon DH, Setser RM, Thamilarasan M, et al. Abnormal papillary muscle morphology is independently associated with increased left ventricular outflow tract obstruction in hypertrophic cardiomyopathy. Heart 2008; 94(10):1295–1301. doi:10.1136/hrt.2007.118018
  8. Patel P, Dhillon A, Popovic ZB, et al. Left ventricular outflow tract obstruction in hypertrophic cardiomyopathy patients without severe septal hypertrophy: implications of mitral valve and papillary muscle abnormalities assessed using cardiac magnetic resonance and echocardiography. Circ Cardiovasc Imaging 2015; 8(7):e003132. doi:10.1161/CIRCIMAGING.115.003132
  9. Gersh BJ, Maron BJ, Bonow RO, et al. 2011 ACCF/AHA guideline for the diagnosis and treatment of hypertrophic cardiomyopathy: a report of the American College of Cardiology Foundation/American Heart Association Task Force on Practice Guidelines. J Thorac Cardiovasc Surg 2011; 142(6):e153-e203. doi:10.1016/j.jtcvs.2011.10.020
  10. Elliott PM, Gimeno JR, Thaman R, et al. Historical trends in reported survival rates in patients with hypertrophic cardiomyopathy. Heart 2006; 92(6):785–791. doi:10.1136/hrt.2005.068577
  11. Maron MS, Olivotto I, Zenovich AG, et al. Hypertrophic cardiomyopathy is predominantly a disease of left ventricular outflow tract obstruction. Circulation 2006; 114(21):2232–2239. doi:10.1161/CIRCULATIONAHA.106.644682
  12. Ho CY, Carlsen C, Thune JJ, et al. Echocardiographic strain imaging to assess early and late consequences of sarcomere mutations in hypertrophic cardiomyopathy. Circ Cardiovasc Genet 2009; 2(4):314–321. doi:10.1161/CIRCGENETICS.109.862128
  13. Wasfy MM, Weiner RB. Differentiating the athlete’s heart from hypertrophic cardiomyopathy. Curr Opin Cardiol 2015; 30(5):500–505. doi:10.1097/HCO.0000000000000203
  14. Palka P, Lange A, Fleming AD, et al. Differences in myocardial velocity gradient measured throughout the cardiac cycle in patients with hypertrophic cardiomyopathy, athletes and patients with left ventricular hypertrophy due to hypertension. J Am Coll Cardiol 1997; 30(3):760–768. pmid:9283537
  15. Eriksson MJ, Sonnenberg B, Woo A, et al. Long-term outcome in patients with apical hypertrophic cardiomyopathy. J Am Coll Cardiol 2002; 39(4):638–645. pmid:11849863
  16. Rickers C, Wilke NM, Jerosch-Herold M, et al. Utility of cardiac magnetic resonance imaging in the diagnosis of hypertrophic cardiomyopathy. Circulation 2005; 112(6):855–861. doi:10.1161/CIRCULATIONAHA.104.507723
  17. Kwon DH, Setser RM, Popovic ZB, et al. Association of myocardial fibrosis, electrocardiography and ventricular tachyarrhythmia in hypertrophic cardiomyopathy: a delayed contrast enhanced MRI study. Int J Cardiovasc Imaging 2008; 24(6):617–625. doi:10.1007/s10554-008-9292-6
  18. Rubinshtein R, Glockner JF, Ommen SR, et al. Characteristics and clinical significance of late gadolinium enhancement by contrast-enhanced magnetic resonance imaging in patients with hypertrophic cardiomyopathy. Circ Heart Fail 2010; 3(1):51–58. doi:10.1161/CIRCHEARTFAILURE.109.854026
  19. O’Hanlon R, Grasso A, Roughton M, et al. Prognostic significance of myocardial fibrosis in hypertrophic cardiomyopathy. J Am Coll Cardiol 2010; 56(11):867–874. doi:10.1016/j.jacc.2010.05.010
  20. Bruder O, Wagner A, Jensen CJ, et al. Myocardial scar visualized by cardiovascular magnetic resonance imaging predicts major adverse events in patients with hypertrophic cardiomyopathy. J Am Coll Cardiol 2010; 56(11):875–887. doi:10.1016/j.jacc.2010.05.007
  21. Authors/Task Force members, Elliott PM, Anastasakis A, Borger MA, et al. 2014 ESC guidelines on diagnosis and management of hypertrophic cardiomyopathy: the Task Force for the Diagnosis and Management of Hypertrophic Cardiomyopathy of the European Society of Cardiology (ESC). Eur Heart J 2014; 35(39):2733–2779. doi:10.1093/eurheartj/ehu284
  22. Olivotto I, Maron BJ, Montereggi A, Mazzuoli F, Dolara A, Cecchi F. Prognostic value of systemic blood pressure response during exercise in a community-based patient population with hypertrophic cardiomyopathy. J Am Coll Cardiol 1999; 33(7):2044–2051. pmid:10362212
  23. Sadoul N, Prasad K, Elliott PM, Bannerjee S, Frenneaux MP, McKenna WJ. Prospective prognostic assessment of blood pressure response during exercise in patients with hypertrophic cardiomyopathy. Circulation 1997; 96(9):2987–2991. pmid:9386166
  24. Elliott PM, Poloniecki J, Dickie S, et al. Sudden death in hypertrophic cardiomyopathy: identification of high risk patients. J Am Coll Cardiol 2000; 36(7):2212–2218. pmid:11127463
  25. Masri A, Pierson LM, Smedira NG, et al. Predictors of long-term outcomes in patients with hypertrophic cardiomyopathy undergoing cardiopulmonary stress testing and echocardiography. Am Heart J 2015; 169(5):684–692.e1. doi:10.1016/j.ahj.2015.02.006
  26. Coats CJ, Rantell K, Bartnik A, et al. Cardiopulmonary exercise testing and prognosis in hypertrophic cardiomyopathy. Circ Heart Fail 2015; 8(6):1022–1031. doi:10.1161/CIRCHEARTFAILURE.114.002248
  27. Desai MY, Bhonsale A, Patel P, et al. Exercise echocardiography in asymptomatic HCM: exercise capacity, and not LV outflow tract gradient predicts long-term outcomes. JACC Cardiovasc Imaging 2014; 7(1):26–36. doi:10.1016/j.jcmg.2013.08.010
  28. Spirito P, Seidman CE, McKenna WJ, Maron BJ. The management of hypertrophic cardiomyopathy. N Engl J Med 1997; 336(11):775–785. doi:10.1056/NEJM199703133361107
  29. Wang W, Lian Z, Rowin EJ, Maron BJ, Maron MS, Link MS. Prognostic implications of nonsustained ventricular tachycardia in high-risk patients with hypertrophic cardiomyopathy. Circ Arrhythm Electrophysiol 2017; 10(3)e004604. doi:10.1161/CIRCEP.116.004604
  30. O’Mahony C, Jichi F, Pavlou M, et al. A novel clinical risk prediction model for sudden cardiac death in hypertrophic cardiomyopathy (HCM risk-SCD). Eur Heart J 2014; 35(30):2010–2020. doi:10.1093/eurheartj/eht439
  31. Maron BJ, Casey SA, Chan RH, Garberich RF, Rowin EJ, Maron MS. Independent assessment of the European Society of Cardiology sudden death risk model for hypertrophic cardiomyopathy. Am J Cardiol 2015; 116(5):757–764. doi:10.1016/j.amjcard.2015.05.047
  32. Jahnlová D, Tomašov P, Zemánek D, Veselka J. Transatlantic differences in assessment of risk of sudden cardiac death in patients with hypertrophic cardiomyopathy. Int J Cardiol 2015; 186:3–4. doi:10.1016/j.ijcard.2015.03.207
  33. Ingles J, Burns C, Bagnall RD, et al. Nonfamilial hypertrophic cardiomyopathy: prevalence, natural history, and clinical implications. Circ Cardiovasc Genet 2017; 10(2)e001620. doi:10.1161/CIRCGENETICS.116.001620
  34. Ko C, Arscott P, Concannon M, et al. Genetic testing impacts the utility of prospective familial screening in hypertrophic cardiomyopathy through identification of a nonfamilial subgroup. Genet Med 2017; 20(1):69–75. doi:10.1038/gim.2017.79
  35. Maron BJ, Chaitman BR, Ackerman MJ, et al. Recommendations for physical activity and recreational sports participation for young patients with genetic cardiovascular diseases. Circulation 2004; 109(22):2807–2816. doi:10.1161/01.CIR.0000128363.85581.E1
  36. Saberi S, Wheeler M, Bragg-Gresham J, et al. Effect of moderate-intensity exercise training on peak oxygen consumption in patients with hypertrophic cardiomyopathy: a randomized clinical trial. JAMA 2017; 317(13):1349–1357. doi:10.1001/jama.2017.2503
  37. Bourmayan C, Razavi A, Fournier C, et al. Effect of propranolol on left ventricular relaxation in hypertrophic cardiomyopathy: an echographic study. Am Heart J 1985; 109(6):1311–1316. pmid:4039882
  38. Spoladore R, Maron MS, D’Amato R, Camici PG, Olivotto I. Pharmacological treatment options for hypertrophic cardiomyopathy: high time for evidence. Eur Heart J 2012; 33(14):1724–1733. doi:10.1093/eurheartj/ehs150
  39. Choudhury L, Elliott P, Rimoldi O, et al. Transmural myocardial blood flow distribution in hypertrophic cardiomyopathy and effect of treatment. Basic Res Cardiol 1999; 94(1):49–59. pmid:10097830
  40. Kaltenbach M, Hopf R, Kober G, Bussmann WD, Keller M, Petersen Y. Treatment of hypertrophic obstructive cardiomyopathy with verapamil. Br Heart J 1979; 42(1):35–42. doi:10.1136/hrt.42.1.35
  41. Sherrid MV, Shetty A, Winson G, et al. Treatment of obstructive hypertrophic cardiomyopathy symptoms and gradient resistant to first-line therapy with beta-blockade or verapamil. Circ Heart Fail 2013; 6(4):694–702. doi:10.1161/CIRCHEARTFAILURE.112.000122
  42. Guttmann OP, Rahman MS, O’Mahony C, Anastasakis A, Elliott PM. Atrial fibrillation and thromboembolism in patients with hypertrophic cardiomyopathy: systematic review. Heart 2014; 100(6):465–472. doi:10.1136/heartjnl-2013-304276
  43. Olivotto I, Cecchi F, Casey SA, Dolara A, Traverse JH, Maron BJ. Impact of atrial fibrillation on the clinical course of hypertrophic cardiomyopathy. Circulation 2001; 104(21):2517–2524. pmid:11714644
  44. Maron BJ, Olivotto I, Spirito P, et al. Epidemiology of hypertrophic cardiomyopathy-related death: revisited in a large non-referral-based patient population. Circulation 2000; 102(8):858–864. pmid:10952953
  45. Harris KM, Spirito P, Maron MS, et al. Prevalence, clinical profile, and significance of left ventricular remodeling in the end-stage phase of hypertrophic cardiomyopathy. Circulation 2006; 114(3):216-225. doi:10.1161/CIRCULATIONAHA.105.583500
  46. Maron MS, Kalsmith BM, Udelson JE, Li W, DeNofrio D. Survival after cardiac transplantation in patients with hypertrophic cardiomyopathy. Circ Heart Fail 2010; 3(5):574–579. doi:10.1161/CIRCHEARTFAILURE.109.922872
  47. Smedira NG, Lytle BW, Lever HM, et al. Current effectiveness and risks of isolated septal myectomy for hypertrophic obstructive cardiomyopathy. Ann Thorac Surg 2008; 85(1):127–133. doi:10.1016/j.athoracsur.2007.07.063
  48. Robbins RC, Stinson EB. Long-term results of left ventricular myotomy and myectomy for obstructive hypertrophic cardiomyopathy. J Thorac Cardiovasc Surg 1996; 111(3):586–594. pmid:8601973
  49. Heric B, Lytle BW, Miller DP, Rosenkranz ER, Lever HM, Cosgrove DM. Surgical management of hypertrophic obstructive cardiomyopathy. Early and late results. J Thorac Cardiovasc Surg 1995; 110(1):195–208. pmid:7609544
  50. Ommen SR, Maron BJ, Olivotto I, et al. Long-term effects of surgical septal myectomy on survival in patients with obstructive hypertrophic cardiomyopathy. J Am Coll Cardiol 2005; 46(3):470–476. doi:10.1016/j.jacc.2005.02.090
  51. Desai MY, Bhonsale A, Smedira NG, et al. Predictors of long-term outcomes in symptomatic hypertrophic obstructive cardiomyopathy patients undergoing surgical relief of left ventricular outflow tract obstruction. Circulation 2013; 128(3):209–216. doi:10.1161/CIRCULATIONAHA.112.000849
  52. McLeod CJ, Ommen SR, Ackerman MJ, et al. Surgical septal myectomy decreases the risk for appropriate implantable cardioverter defibrillator discharge in obstructive hypertrophic cardiomyopathy. Eur Heart J 2007; 28(21):2583–2588. doi:10.1093/eurheartj/ehm117
  53. Veselka J, Tomasov P, Zemanek D. Long-term effects of varying alcohol dosing in percutaneous septal ablation for obstructive hypertrophic cardiomyopathy: a randomized study with a follow-up up to 11 years. Can J Cardiol 2011; 27(6):763–767. doi:10.1016/j.cjca.2011.09.001
  54. Veselka J, Jensen MK, Liebregts M, et al. Low procedure-related mortality achieved with alcohol septal ablation in European patients. Int J Cardiol 2016; 209:194–195. doi:10.1016/j.ijcard.2016.02.077
  55. Veselka J, Krejci J, Tomašov P, Zemánek D. Long-term survival after alcohol septal ablation for hypertrophic obstructive cardiomyopathy: a comparison with general population. Eur Heart J 2014; 35(30):2040–2045. doi:10.1093/eurheartj/eht495
  56. Sorajja P, Ommen SR, Holmes DR Jr, et al. Survival after alcohol septal ablation for obstructive hypertrophic cardiomyopathy. Circulation 2012; 126(20):2374–2380. doi:10.1161/CIRCULATIONAHA.111.076257
  57. Agarwal S, Tuzcu EM, Desai MY, et al. Updated meta-analysis of septal alcohol ablation versus myectomy for hypertrophic cardiomyopathy. J Am Coll Cardiol 2010; 55(8):823–834. doi:10.1016/j.jacc.2009.09.047
  58. Leonardi RA, Kransdorf EP, Simel DL, Wang A. Meta-analyses of septal reduction therapies for obstructive hypertrophic cardiomyopathy: comparative rates of overall mortality and sudden cardiac death after treatment. Circ Cardiovasc Interv 2010; 3(2):97–104. doi:10.1161/CIRCINTERVENTIONS.109.916676
  59. Liebregts M, Vriesendorp PA, Mahmoodi BK, Schinkel AF, Michels M, ten Berg JM. A systematic review and meta-analysis of long-term outcomes after septal reduction therapy in patients with hypertrophic cardiomyopathy. JACC Heart Fail 2015; 3(11):896–905. doi:10.1016/j.jchf.2015.06.011
  60. Panaich SS, Badheka AO, Chothani A, et al. Results of ventricular septal myectomy and hypertrophic cardiomyopathy (from Nationwide Inpatient Sample [1998-2010]). Am J Cardiol 2014; 114(9):1390–1395. doi:10.1016/j.amjcard.2014.07.075
  61. Maron BJ, Dearani JA, Ommen SR, et al. Low operative mortality achieved with surgical septal myectomy: role of dedicated hypertrophic cardiomyopathy centers in the management of dynamic subaortic obstruction. J Am Coll Cardiol 2015; 66(11):1307–1308. doi:10.1016/j.jacc.2015.06.1333
  62. Kwon DH, Smedira NG, Thamilarasan M, Lytle BW, Lever H, Desai MY. Characteristics and surgical outcomes of symptomatic patients with hypertrophic cardiomyopathy with abnormal papillary muscle morphology undergoing papillary muscle reorientation. J Thorac Cardiovasc Surg 2010; 140(2):317–324. doi:10.1016/j.jtcvs.2009.10.045
  63. Schaff HV, Brown ML, Dearani JA, et al. Apical myectomy: a new surgical technique for management of severely symptomatic patients with apical hypertrophic cardiomyopathy. J Thorac Cardiovasc Surg 2010; 139(3):634–640. doi:10.1016/j.jtcvs.2009.07.079
References
  1. Maron BJ. Hypertrophic cardiomyopathy: an important global disease. Am J Med 2004; 116(1):63–65. pmid:14706671
  2. Semsarian C, Ingles J, Maron MS, Maron BJ. New perspectives on the prevalence of hypertrophic cardiomyopathy. J Am Coll Cardiol 2015; 65(12):1249–1254. doi:10.1016/j.jacc.2015.01.019
  3. Maron BJ, Maron MS, Semsarian C. Genetics of hypertrophic cardiomyopathy after 20 years: clinical perspectives. J Am Coll Cardiol 2012; 60(8):705–715. doi:10.1016/j.jacc.2012.02.068
  4. Shirani J, Pick R, Roberts WC, Maron BJ. Morphology and significance of the left ventricular collagen network in young patients with hypertrophic cardiomyopathy and sudden cardiac death. J Am Coll Cardiol 2000; 35(1):36–44. pmid:10636256
  5. Sherrid MV, Chu CK, Delia E, Mogtader A, Dwyer EM Jr. An echocardiographic study of the fluid mechanics of obstruction in hypertrophic cardiomyopathy. J Am Coll Cardiol 1993; 22(3):816–825. pmid:8354817
  6. Ro R, Halpern D, Sahn DJ, et al. Vector flow mapping in obstructive hypertrophic cardiomyopathy to assess the relationship of early systolic left ventricular flow and the mitral valve. J Am Coll Cardiol 2014; 64(19):1984–1995. doi:10.1016/j.jacc.2014.04.090
  7. Kwon DH, Setser RM, Thamilarasan M, et al. Abnormal papillary muscle morphology is independently associated with increased left ventricular outflow tract obstruction in hypertrophic cardiomyopathy. Heart 2008; 94(10):1295–1301. doi:10.1136/hrt.2007.118018
  8. Patel P, Dhillon A, Popovic ZB, et al. Left ventricular outflow tract obstruction in hypertrophic cardiomyopathy patients without severe septal hypertrophy: implications of mitral valve and papillary muscle abnormalities assessed using cardiac magnetic resonance and echocardiography. Circ Cardiovasc Imaging 2015; 8(7):e003132. doi:10.1161/CIRCIMAGING.115.003132
  9. Gersh BJ, Maron BJ, Bonow RO, et al. 2011 ACCF/AHA guideline for the diagnosis and treatment of hypertrophic cardiomyopathy: a report of the American College of Cardiology Foundation/American Heart Association Task Force on Practice Guidelines. J Thorac Cardiovasc Surg 2011; 142(6):e153-e203. doi:10.1016/j.jtcvs.2011.10.020
  10. Elliott PM, Gimeno JR, Thaman R, et al. Historical trends in reported survival rates in patients with hypertrophic cardiomyopathy. Heart 2006; 92(6):785–791. doi:10.1136/hrt.2005.068577
  11. Maron MS, Olivotto I, Zenovich AG, et al. Hypertrophic cardiomyopathy is predominantly a disease of left ventricular outflow tract obstruction. Circulation 2006; 114(21):2232–2239. doi:10.1161/CIRCULATIONAHA.106.644682
  12. Ho CY, Carlsen C, Thune JJ, et al. Echocardiographic strain imaging to assess early and late consequences of sarcomere mutations in hypertrophic cardiomyopathy. Circ Cardiovasc Genet 2009; 2(4):314–321. doi:10.1161/CIRCGENETICS.109.862128
  13. Wasfy MM, Weiner RB. Differentiating the athlete’s heart from hypertrophic cardiomyopathy. Curr Opin Cardiol 2015; 30(5):500–505. doi:10.1097/HCO.0000000000000203
  14. Palka P, Lange A, Fleming AD, et al. Differences in myocardial velocity gradient measured throughout the cardiac cycle in patients with hypertrophic cardiomyopathy, athletes and patients with left ventricular hypertrophy due to hypertension. J Am Coll Cardiol 1997; 30(3):760–768. pmid:9283537
  15. Eriksson MJ, Sonnenberg B, Woo A, et al. Long-term outcome in patients with apical hypertrophic cardiomyopathy. J Am Coll Cardiol 2002; 39(4):638–645. pmid:11849863
  16. Rickers C, Wilke NM, Jerosch-Herold M, et al. Utility of cardiac magnetic resonance imaging in the diagnosis of hypertrophic cardiomyopathy. Circulation 2005; 112(6):855–861. doi:10.1161/CIRCULATIONAHA.104.507723
  17. Kwon DH, Setser RM, Popovic ZB, et al. Association of myocardial fibrosis, electrocardiography and ventricular tachyarrhythmia in hypertrophic cardiomyopathy: a delayed contrast enhanced MRI study. Int J Cardiovasc Imaging 2008; 24(6):617–625. doi:10.1007/s10554-008-9292-6
  18. Rubinshtein R, Glockner JF, Ommen SR, et al. Characteristics and clinical significance of late gadolinium enhancement by contrast-enhanced magnetic resonance imaging in patients with hypertrophic cardiomyopathy. Circ Heart Fail 2010; 3(1):51–58. doi:10.1161/CIRCHEARTFAILURE.109.854026
  19. O’Hanlon R, Grasso A, Roughton M, et al. Prognostic significance of myocardial fibrosis in hypertrophic cardiomyopathy. J Am Coll Cardiol 2010; 56(11):867–874. doi:10.1016/j.jacc.2010.05.010
  20. Bruder O, Wagner A, Jensen CJ, et al. Myocardial scar visualized by cardiovascular magnetic resonance imaging predicts major adverse events in patients with hypertrophic cardiomyopathy. J Am Coll Cardiol 2010; 56(11):875–887. doi:10.1016/j.jacc.2010.05.007
  21. Authors/Task Force members, Elliott PM, Anastasakis A, Borger MA, et al. 2014 ESC guidelines on diagnosis and management of hypertrophic cardiomyopathy: the Task Force for the Diagnosis and Management of Hypertrophic Cardiomyopathy of the European Society of Cardiology (ESC). Eur Heart J 2014; 35(39):2733–2779. doi:10.1093/eurheartj/ehu284
  22. Olivotto I, Maron BJ, Montereggi A, Mazzuoli F, Dolara A, Cecchi F. Prognostic value of systemic blood pressure response during exercise in a community-based patient population with hypertrophic cardiomyopathy. J Am Coll Cardiol 1999; 33(7):2044–2051. pmid:10362212
  23. Sadoul N, Prasad K, Elliott PM, Bannerjee S, Frenneaux MP, McKenna WJ. Prospective prognostic assessment of blood pressure response during exercise in patients with hypertrophic cardiomyopathy. Circulation 1997; 96(9):2987–2991. pmid:9386166
  24. Elliott PM, Poloniecki J, Dickie S, et al. Sudden death in hypertrophic cardiomyopathy: identification of high risk patients. J Am Coll Cardiol 2000; 36(7):2212–2218. pmid:11127463
  25. Masri A, Pierson LM, Smedira NG, et al. Predictors of long-term outcomes in patients with hypertrophic cardiomyopathy undergoing cardiopulmonary stress testing and echocardiography. Am Heart J 2015; 169(5):684–692.e1. doi:10.1016/j.ahj.2015.02.006
  26. Coats CJ, Rantell K, Bartnik A, et al. Cardiopulmonary exercise testing and prognosis in hypertrophic cardiomyopathy. Circ Heart Fail 2015; 8(6):1022–1031. doi:10.1161/CIRCHEARTFAILURE.114.002248
  27. Desai MY, Bhonsale A, Patel P, et al. Exercise echocardiography in asymptomatic HCM: exercise capacity, and not LV outflow tract gradient predicts long-term outcomes. JACC Cardiovasc Imaging 2014; 7(1):26–36. doi:10.1016/j.jcmg.2013.08.010
  28. Spirito P, Seidman CE, McKenna WJ, Maron BJ. The management of hypertrophic cardiomyopathy. N Engl J Med 1997; 336(11):775–785. doi:10.1056/NEJM199703133361107
  29. Wang W, Lian Z, Rowin EJ, Maron BJ, Maron MS, Link MS. Prognostic implications of nonsustained ventricular tachycardia in high-risk patients with hypertrophic cardiomyopathy. Circ Arrhythm Electrophysiol 2017; 10(3)e004604. doi:10.1161/CIRCEP.116.004604
  30. O’Mahony C, Jichi F, Pavlou M, et al. A novel clinical risk prediction model for sudden cardiac death in hypertrophic cardiomyopathy (HCM risk-SCD). Eur Heart J 2014; 35(30):2010–2020. doi:10.1093/eurheartj/eht439
  31. Maron BJ, Casey SA, Chan RH, Garberich RF, Rowin EJ, Maron MS. Independent assessment of the European Society of Cardiology sudden death risk model for hypertrophic cardiomyopathy. Am J Cardiol 2015; 116(5):757–764. doi:10.1016/j.amjcard.2015.05.047
  32. Jahnlová D, Tomašov P, Zemánek D, Veselka J. Transatlantic differences in assessment of risk of sudden cardiac death in patients with hypertrophic cardiomyopathy. Int J Cardiol 2015; 186:3–4. doi:10.1016/j.ijcard.2015.03.207
  33. Ingles J, Burns C, Bagnall RD, et al. Nonfamilial hypertrophic cardiomyopathy: prevalence, natural history, and clinical implications. Circ Cardiovasc Genet 2017; 10(2)e001620. doi:10.1161/CIRCGENETICS.116.001620
  34. Ko C, Arscott P, Concannon M, et al. Genetic testing impacts the utility of prospective familial screening in hypertrophic cardiomyopathy through identification of a nonfamilial subgroup. Genet Med 2017; 20(1):69–75. doi:10.1038/gim.2017.79
  35. Maron BJ, Chaitman BR, Ackerman MJ, et al. Recommendations for physical activity and recreational sports participation for young patients with genetic cardiovascular diseases. Circulation 2004; 109(22):2807–2816. doi:10.1161/01.CIR.0000128363.85581.E1
  36. Saberi S, Wheeler M, Bragg-Gresham J, et al. Effect of moderate-intensity exercise training on peak oxygen consumption in patients with hypertrophic cardiomyopathy: a randomized clinical trial. JAMA 2017; 317(13):1349–1357. doi:10.1001/jama.2017.2503
  37. Bourmayan C, Razavi A, Fournier C, et al. Effect of propranolol on left ventricular relaxation in hypertrophic cardiomyopathy: an echographic study. Am Heart J 1985; 109(6):1311–1316. pmid:4039882
  38. Spoladore R, Maron MS, D’Amato R, Camici PG, Olivotto I. Pharmacological treatment options for hypertrophic cardiomyopathy: high time for evidence. Eur Heart J 2012; 33(14):1724–1733. doi:10.1093/eurheartj/ehs150
  39. Choudhury L, Elliott P, Rimoldi O, et al. Transmural myocardial blood flow distribution in hypertrophic cardiomyopathy and effect of treatment. Basic Res Cardiol 1999; 94(1):49–59. pmid:10097830
  40. Kaltenbach M, Hopf R, Kober G, Bussmann WD, Keller M, Petersen Y. Treatment of hypertrophic obstructive cardiomyopathy with verapamil. Br Heart J 1979; 42(1):35–42. doi:10.1136/hrt.42.1.35
  41. Sherrid MV, Shetty A, Winson G, et al. Treatment of obstructive hypertrophic cardiomyopathy symptoms and gradient resistant to first-line therapy with beta-blockade or verapamil. Circ Heart Fail 2013; 6(4):694–702. doi:10.1161/CIRCHEARTFAILURE.112.000122
  42. Guttmann OP, Rahman MS, O’Mahony C, Anastasakis A, Elliott PM. Atrial fibrillation and thromboembolism in patients with hypertrophic cardiomyopathy: systematic review. Heart 2014; 100(6):465–472. doi:10.1136/heartjnl-2013-304276
  43. Olivotto I, Cecchi F, Casey SA, Dolara A, Traverse JH, Maron BJ. Impact of atrial fibrillation on the clinical course of hypertrophic cardiomyopathy. Circulation 2001; 104(21):2517–2524. pmid:11714644
  44. Maron BJ, Olivotto I, Spirito P, et al. Epidemiology of hypertrophic cardiomyopathy-related death: revisited in a large non-referral-based patient population. Circulation 2000; 102(8):858–864. pmid:10952953
  45. Harris KM, Spirito P, Maron MS, et al. Prevalence, clinical profile, and significance of left ventricular remodeling in the end-stage phase of hypertrophic cardiomyopathy. Circulation 2006; 114(3):216-225. doi:10.1161/CIRCULATIONAHA.105.583500
  46. Maron MS, Kalsmith BM, Udelson JE, Li W, DeNofrio D. Survival after cardiac transplantation in patients with hypertrophic cardiomyopathy. Circ Heart Fail 2010; 3(5):574–579. doi:10.1161/CIRCHEARTFAILURE.109.922872
  47. Smedira NG, Lytle BW, Lever HM, et al. Current effectiveness and risks of isolated septal myectomy for hypertrophic obstructive cardiomyopathy. Ann Thorac Surg 2008; 85(1):127–133. doi:10.1016/j.athoracsur.2007.07.063
  48. Robbins RC, Stinson EB. Long-term results of left ventricular myotomy and myectomy for obstructive hypertrophic cardiomyopathy. J Thorac Cardiovasc Surg 1996; 111(3):586–594. pmid:8601973
  49. Heric B, Lytle BW, Miller DP, Rosenkranz ER, Lever HM, Cosgrove DM. Surgical management of hypertrophic obstructive cardiomyopathy. Early and late results. J Thorac Cardiovasc Surg 1995; 110(1):195–208. pmid:7609544
  50. Ommen SR, Maron BJ, Olivotto I, et al. Long-term effects of surgical septal myectomy on survival in patients with obstructive hypertrophic cardiomyopathy. J Am Coll Cardiol 2005; 46(3):470–476. doi:10.1016/j.jacc.2005.02.090
  51. Desai MY, Bhonsale A, Smedira NG, et al. Predictors of long-term outcomes in symptomatic hypertrophic obstructive cardiomyopathy patients undergoing surgical relief of left ventricular outflow tract obstruction. Circulation 2013; 128(3):209–216. doi:10.1161/CIRCULATIONAHA.112.000849
  52. McLeod CJ, Ommen SR, Ackerman MJ, et al. Surgical septal myectomy decreases the risk for appropriate implantable cardioverter defibrillator discharge in obstructive hypertrophic cardiomyopathy. Eur Heart J 2007; 28(21):2583–2588. doi:10.1093/eurheartj/ehm117
  53. Veselka J, Tomasov P, Zemanek D. Long-term effects of varying alcohol dosing in percutaneous septal ablation for obstructive hypertrophic cardiomyopathy: a randomized study with a follow-up up to 11 years. Can J Cardiol 2011; 27(6):763–767. doi:10.1016/j.cjca.2011.09.001
  54. Veselka J, Jensen MK, Liebregts M, et al. Low procedure-related mortality achieved with alcohol septal ablation in European patients. Int J Cardiol 2016; 209:194–195. doi:10.1016/j.ijcard.2016.02.077
  55. Veselka J, Krejci J, Tomašov P, Zemánek D. Long-term survival after alcohol septal ablation for hypertrophic obstructive cardiomyopathy: a comparison with general population. Eur Heart J 2014; 35(30):2040–2045. doi:10.1093/eurheartj/eht495
  56. Sorajja P, Ommen SR, Holmes DR Jr, et al. Survival after alcohol septal ablation for obstructive hypertrophic cardiomyopathy. Circulation 2012; 126(20):2374–2380. doi:10.1161/CIRCULATIONAHA.111.076257
  57. Agarwal S, Tuzcu EM, Desai MY, et al. Updated meta-analysis of septal alcohol ablation versus myectomy for hypertrophic cardiomyopathy. J Am Coll Cardiol 2010; 55(8):823–834. doi:10.1016/j.jacc.2009.09.047
  58. Leonardi RA, Kransdorf EP, Simel DL, Wang A. Meta-analyses of septal reduction therapies for obstructive hypertrophic cardiomyopathy: comparative rates of overall mortality and sudden cardiac death after treatment. Circ Cardiovasc Interv 2010; 3(2):97–104. doi:10.1161/CIRCINTERVENTIONS.109.916676
  59. Liebregts M, Vriesendorp PA, Mahmoodi BK, Schinkel AF, Michels M, ten Berg JM. A systematic review and meta-analysis of long-term outcomes after septal reduction therapy in patients with hypertrophic cardiomyopathy. JACC Heart Fail 2015; 3(11):896–905. doi:10.1016/j.jchf.2015.06.011
  60. Panaich SS, Badheka AO, Chothani A, et al. Results of ventricular septal myectomy and hypertrophic cardiomyopathy (from Nationwide Inpatient Sample [1998-2010]). Am J Cardiol 2014; 114(9):1390–1395. doi:10.1016/j.amjcard.2014.07.075
  61. Maron BJ, Dearani JA, Ommen SR, et al. Low operative mortality achieved with surgical septal myectomy: role of dedicated hypertrophic cardiomyopathy centers in the management of dynamic subaortic obstruction. J Am Coll Cardiol 2015; 66(11):1307–1308. doi:10.1016/j.jacc.2015.06.1333
  62. Kwon DH, Smedira NG, Thamilarasan M, Lytle BW, Lever H, Desai MY. Characteristics and surgical outcomes of symptomatic patients with hypertrophic cardiomyopathy with abnormal papillary muscle morphology undergoing papillary muscle reorientation. J Thorac Cardiovasc Surg 2010; 140(2):317–324. doi:10.1016/j.jtcvs.2009.10.045
  63. Schaff HV, Brown ML, Dearani JA, et al. Apical myectomy: a new surgical technique for management of severely symptomatic patients with apical hypertrophic cardiomyopathy. J Thorac Cardiovasc Surg 2010; 139(3):634–640. doi:10.1016/j.jtcvs.2009.07.079
Issue
Cleveland Clinic Journal of Medicine - 85(5)
Issue
Cleveland Clinic Journal of Medicine - 85(5)
Page Number
399-411
Page Number
399-411
Publications
Publications
Topics
Article Type
Display Headline
Hypertrophic cardiomyopathy: A complex disease
Display Headline
Hypertrophic cardiomyopathy: A complex disease
Legacy Keywords
hypertrophic cardiomyopathy, HCM, sarcomere, sudden cardiac death, left ventricular hypertrophy, LVH, left ventricular outflow tract obstruction, LVOT, mitral valve, papillary muscle, Laura Young, Nicholas Smedira, Albree Tower-Rader, Harry Lever, and Milind Desai
Legacy Keywords
hypertrophic cardiomyopathy, HCM, sarcomere, sudden cardiac death, left ventricular hypertrophy, LVH, left ventricular outflow tract obstruction, LVOT, mitral valve, papillary muscle, Laura Young, Nicholas Smedira, Albree Tower-Rader, Harry Lever, and Milind Desai
Sections
Inside the Article

KEY POINTS

  • Obstruction of the left ventricular outflow tract is a key pathophysiologic mechanism in HCM.
  • Because most of the genetic variants that contribute to HCM are autosomal dominant, genetic counseling and testing are suggested for patients and their first-degree relatives.
  • Transthoracic echocardiography is the first-line imaging test, followed by magnetic resonance imaging.
  • Beta-blockers are the first-line drugs for treating symptoms of HCM.
  • An implantable cardioverter-defibrillator can be considered for patients at risk of sudden cardiac death.
  • When medical therapy fails or is not tolerated in patients with severe symptoms of obstructive HCM, surgery to reduce the size of the ventricular septum can be considered. Alcohol septal ablation is an alternative.
Disallow All Ads
Content Gating
No Gating (article Unlocked/Free)
Alternative CME
Disqus Comments
Default
Gate On Date
Wed, 04/25/2018 - 15:15
Un-Gate On Date
Wed, 04/25/2018 - 15:15
Use ProPublica
CFC Schedule Remove Status
Wed, 04/25/2018 - 15:15
Hide sidebar & use full width
render the right sidebar.
Article PDF Media

Improving Strength and Balance for Long-Term Care Residents At Risk for Falling: Suggestions for Practice

Article Type
Changed
Wed, 04/29/2020 - 11:27

From the Geriatric Education and Research in Aging Sciences Centre, McMaster University Hamilton, ON (Dr. McArthur) and the University of Waterloo and Research Institute for Aging, Waterloo, ON (Dr. Giangregorio), Canada

 

Abstract

  • Objective: To synthesize the available literature on exercise and falls reduction interventions in long-term care (LTC) and provide practical information for clinicians and other decision makers.
  • Methods: Review of positive trials included in systematic reviews.
  • Results: Falls are a major concern for residents, families, clinicians, and decision-makers in LTC. Exercise is recommended as part of a multifactorial falls prevention program for residents in LTC. Strength and balance exercises should be incorporated into the multifactorial falls prevention program. They should be challenging and progressed as the residents’ abilities improve. Evidence suggests that exercises should be completed 2 to 3 times per week for a period longer than 6 months. Exercise programs in LTC should be resident-centered and should consider residents’ potential physical and cognitive impairments. Exercises in standing should be prioritized where appropriate.
  • Conclusion: Appropriately challenging and progressive strength and balance exercises should be included in a multifactorial falls prevention program for residents in LTC.

Key words: long-term care; nursing homes; falls reduction; exercise.

 

Falls are common in long-term care (LTC) homes: the estimated falls rate is 1.5 falls per bed per year, which is 3 times greater than that for older adults living in the community [1]. Falls can have significant consequences for residents in LTC, including functional disability, fractures, pain, reduced quality of life, and death [1–6]. Indeed, 25% of residents who are hospitalized after a fall die within 1 year [3]. Consequently, falls prevention programs are important to help in reducing falls and averting the associated negative consequences.

Exercise may address the circumstances and physical deconditioning that often contribute to falls in LTC residents. Weight shifting [7], walking, and transferring [8–10], are common activities that precede falls, suggesting that balance, gait, and functional mobility training may be possible targets for prevention. Additionally, it is estimated that LTC residents spend three quarters of their waking time in sedentary activities [11,12] and have a high prevalence of sarcopenia [13–16]. Challenging balance training and resistance exercise are well-known intervention for reducing falls [17] and improving muscle strength for community-dwelling older adults [18]. However, evidence around balance and strength training for preventing falls in LTC is mixed [17,19,20], and careful planning and modification of exercises is necessary to meet the needs of LTC residents.

Residents in LTC are often medically complex, with multiple comorbidities [21] that can affect their ability to meaningfully participate in exercise. In Canada, 56.3% of residents have a diagnosis of Alzheimer’s or other dementias, 25.0% have diabetes, 14.4% have chronic obstructive pulmonary disease, and 21.2% have experienced a stroke [21]. Residents also often have significant functional impairments. For example, 97% of residents require assistance with basic activities of daily living [21]. Therefore, the lack of effect of exercise as a single falls prevention strategy observed in previous studies may be because the often complex, multimorbid LTC population likely requires a multifactorial approach to fall prevention [17]. Additionally, organizational aspects of LTC homes (eg, specific funds dedicated to employing exercise professionals and to support exercise programming) can affect residents’ engagement in exercise [22,23]. Subsequently, prescribing exercises in the LTC context must consider both resident characteristics and organizational features of the LTC home (eg, professionals available to support exercise programming).

A comprehensive exercise prescription describes the elements of an appropriate exercise program to facilitate implementation of that program. The exercise prescription should include a description of the type (eg, balance, strength) and intensity of exercises (eg, subjective or objective measurement of how hard the resident is working) included in the program [24]. The prescription should also include a description of the dose of exercise: frequency of exercise participation (eg, 2 days per week), duration of individual exercise sessions (eg, 30-minute sessions), and duration of exercise program (eg, 12-week program) [24]. Lastly, the prescription should describe the setting of the exercise program (eg, group or individual basis) and the professional delivering the program (eg, physiotherapist, fitness instructor) [24].

Therefore, the objectives of this article are to (1) synthesize studies demonstrating a positive effect of exercise on reducing falls for residents in LTC; (2) provide an overview of the principles of balance and strength training to guide clinicians in designing appropriate exercise prescription; and (3) make suggestions for clinical practice regarding an appropriate strength and balance exercise protocol by considering the influence of the LTC context.

Methods

To provide clinicians and other policy-makers with a description of which balance and strength exercises may be effective for preventing falls, we synthesized trials that demonstrated a positive effect on reducing falls or falls risk for residents in LTC. Studies were identified through a database search for systematic reviews in PubMed, Ovid, and Google Scholar using the keywords falls, long-term care, nursing homes, exercise, strength, balance, and systematic reviews. Our purpose was to provide practical information on what works to prevent falls through balance and strength training for residents in LTC rather than to evaluate the available evidence. Therefore, only positive trials from systematic reviews were discussed, as we wanted to present exercises that seem to have a positive effect on decreasing falls. Positive trials were defined as those included in identified systematic reviews with a risk or rate ratio and confidence intervals below 1.0.

 

 

We first provide an overview of the conclusions of the systematic reviews found in our search. Next, for each positive trial we describe the following elements of the exercise component of the intervention: frequency, time of sessions, length of program, intensity, type of exercise including a description of the specific exercises performed, whether the intervention was delivered in a group or on an individual basis, the professional delivering the intervention, and any other features of the intervention aside from the exercise component. We used the ProFaNE taxonomy definitions [25] to identify and describe each element of the exercise interventions. Frequency is the number of times per week that residents engage in sessions, time of sessions is the amount allocated to each exercise session, duration of program is how long the resident participates in the exercise program, and intensity is the subjective or objective report of how hard the resident is working [25]. The types of exercises described were those targeting balance defined as “...the efficient transfer of bodyweight from one part of the body to another or challenges specific aspects of the balance systems (eg, vestibular system)” [25], and strength defined as “...contracting the muscles against a resistance to ‘overload’ and bring about a training effect in the muscular system” [25]. Strength could be either an external resistance (eg, dumbbell) or using body weight against gravity (eg, squat) [25].

Results

We found 3 systematic reviews that include exercise programs to reduce falls in LTC homes [17,19,20]. Overall, evidence suggests that exercise should be included as part of a multifactorial falls prevention program for residents in LTC. There is limited evidence that exercise as a single intervention prevents falls, and some trials, albeit underpowered, even demonstrate an increased risk of falling in the exercise group compared to control [19]. With regards to specific exercise programs, the Cochrane review found that gait, balance, and functional training decrease the rate of falls but not the risk of falling [26–28], and the 2013 review by Silva et al [20] concluded that combined exercise programs (ie, multiple types of exercise) that include balance tasks, are completed frequently (2–3 times per week), and over a long term (greater than 6 months) were most effective at preventing falls [20].

 

A more recent systematic review and meta-analysis [17] also concluded that there was no evidence that exercise as a single intervention can prevent falls for residents in LTC. Table 1 provides a description of the exercise component of the seven positive trials [29–35] that were included in the 3 systematic reviews we identified in our search.

jcom02501028_t1a.jpg
jcom02501028_t1b.jpg

Type of Exercise

Balance Exercises

There were 4 positive trials that included balance exercises in their intervention [31,33–35]. Trials that had a positive effect on reducing falls and included balance training employed mostly dynamic balance exercises in standing (Table 1). However, only 2 of the 7 trials provided a detailed description of their balance exercises (Table 1) [26,34]. Jensen et al [30] and Dyer et al [31] did not include a description of the balance training performed but stated that balance was part of the multicomponent exercise program. Becker et al [36] stated that participants performed standing balance exercises, while Schnelle et al [39] and Huang et al [32] did not include balance training in their trial.

Strength Exercises

Of the 7 positive trials included in this review, 6 included strength exercises [29–32,34,35]. The strength activities used in trials where exercise had a positive effect on decreasing falls included functional activities [29,31] and progressive resistance training [31,36] (Table 1). Functional activities are those that replicate what a resident might be required to do in their everyday life, such as performing sit-to-stands out of a chair (Figure

jcom02501028_f1.jpg
or practicing bed mobility (eg, rolling from side to side, transitioning from lying to sitting and vice versa) [25]. The exercise program in Schnelle et al’s [29] trial had residents performing sit-to-stands every 2 hours, and arm curls or arm raises once a day. Participants in Jensen et al [30] at Dyer et al’s [31] trials performed progressive resistance training but did not describe which particular exercises participants performed. Similarly, Becker et al [36] described progressive resistance training with dumbbells and ankle weights but did not describe any specific exercises. Participants in the trial by Lord et al [34] performed strengthening exercises of ankle dorsiflexors, knee extensors, hip abductors, and hip side flexors. One trial stated their exercise program focused on muscle endurance rather than strength (eg, higher repetitions with lower resistance), and participants did grip ball exercises, ankle dorsiflexion and plantar flexion, upper limb elevation, knee elevation, and sit-to-stands [32].

 

 

Frequency, Time of Sessions, Duration of Program

In our description of positive trials, exercise was performed on 2 to 3 days per week for 20 to 75 minutes per session, for periods ranging from 4 to 52 weeks (Table 1).

Intensity

For the trials including balance exercises, one trial described the intensity as resident-specific [37] and another as individualized [33]. Two studies did not describe the intensity of their balance exercises [31,34]. The intensity of strength exercises included in the positive trials was individualized for one of the trial [29]. Two trials had participants complete 2 to 3 sets of 10 repetitions [32,35], with one indicating an intensity of 12–13 or “somewhat difficult” on the Borg Rating of Perceived Exertion Scale [32] and the other using a 10-rep max [35]. Two studies described their strength exercises as progressive [31,37], and one at a moderate to high intensity [30]. Lord et al prescribed 30 repetitions of each strength exercise [34].

Delivery of Intervention

Exercise was delivered in a group setting for 4 of the trials [31,32,34,36], individually for 2 of the trials [26,29], and the setting was not described for one of the trials (Table 1) [30]. Finally, only 3 of the 7 articles reported the professional delivering the intervention: one was research staff [29], one was geriatric nurses [32], and one was exercise assistants supported by a physiotherapist [31].

 

Discussion

There is limited evidence to support the use of strength and balance exercise as a single intervention to prevent falls in LTC. However, exercise should be included as part of a multifactorial falls prevention program. Trials that had a positive effect on decreasing falls training used dynamic balance exercises in standing, functional training, and progressive resistance training on 2 to 3 days per week, for 20 to 75 minutes per session, over 4 to 52 weeks. The intensity of balance exercises was individualized, and strength exercises were described as somewhat difficult or performed at a moderate to high intensity. Exercise was performed in a group or individually, and was delivered by research staff, geriatric nurses, exercise assistants supervised by physiotherapists, or more frequently, it was not reported who delivered the intervention.

Balance Training

Our work suggests that standing, dynamic balance exercises may be best to decrease falls. Example balance exercises include reducing the base of support (eg, standing with feet together instead of apart, or tandem with one foot in front), moving the center of gravity and control body position while standing (eg, reaching, weight shifting, stepping up or down), and standing without using arms for support or reducing reliance on the upper limbs for support (eg, use one hand on a handrail instead of two, or two fingers instead of the whole hand) [17]. It is well established that balance training programs, especially those including challenging exercises, can prevent falls in community-dwelling older adults [17]. However, the relationship is not as clear in LTC.

Strength Training

Reduced muscle strength has been identified as an important risk factor for falls [38]. There are also many psychological and metabolic benefits to strength training [39]. To induce change in muscular strength, resistance exercises need to be challenging and progressive. Our work suggests that strength training that is effective at decreasing falls is functional and progressive, and is completed at a moderate to high intensity. A resident should be able to do a strength exercise for one to two sets of 6 to 8 repetitions before being fatigued [40]. Once the resident can complete two sets of 13 to 15 repetitions easily the exercise should be progressed. Residents who are particularly deconditioned may need to begin with lower intensity strength exercises (eg, only do one set, with a lower resistance and progress to a higher resistance) [40]. Residents should perform resistance exercises for all major muscle groups [40]. Progression could include increasing the number of sets (eg, increase from one to two sets), the resistance (eg, holding dumbbells while squatting), or the intensity of the exercise (eg, squat lower or faster) [41].

Implementing Exercise Programs in LTC

Implementation of exercise programs into LTC homes should consider the dose of exercise (eg, time and frequency of sessions, duration of program), if they are delivered in a group or individual setting, and who is delivering them. First, trials included in this paper suggest that strength and balance exercises to prevent falls were delivered 2 to 3 times per week, for 20 to 75 minutes per session, over 4 to 52 weeks. Second, previous work has established that exercise programs delivered on 2 to 3 days per week over a period of more than 6 months are most effective at reducing falls in LTC [20]. Finally, a recent task force report from an international group of clinician researchers in LTC recommends twice weekly exercise sessions lasting 35 to 45 minutes each [40]. Therefore, strength and balance exercises to prevent falls in LTC should be delivered at least twice per week, for at least 20 minutes, for greater than 6 weeks’ duration.

 

 

Whether exercise should be performed in a group or individual setting remains unclear. Two of the 6 positive trials in this paper were completed individually, while 3 were in a group. The aforementioned task force also recommended that every resident who does not have contraindications to exercise must have an individualized exercise program as part of their health care plan [40]. However, whether the exercise program is provided on an individual basis or in a group setting was not delineated. Indeed, there are currently no recommendations concerning prioritizing group or individual exercise programs. Therefore, exercise programs being implemented into LTC homes should consider the residents’ preferences, the social benefits of group exercise, and the feasibility of individualizing exercises for the complex needs of residents in LTC in large group settings.

Finally, which professionals should deliver the exercise program is also uncertain. Only 3 of the positive trials in this paper described the professional delivering the intervention, with one being research staff, one geriatric nurses, and one exercise assistants supported by a physiotherapist. We suggest that professionals delivering an exercise program should be trained in exercise planning, delivery, and progression, be familiar with the principles of balance and strength training, and have training in working with older adults in LTC.

 

Modifications for Physical Impairments

Residents in LTC often have complex health needs, with multiple comorbidities (eg, stroke, Parkinson’s disease, multiple sclerosis) [21]. Modifications of strength and balance exercises may be required to accommodate for physical impairments (eg, hemiplegia, drop foot, freezing gait). For example, if a resident has hemiplegia and cannot fully activate the muscles of one arm, one can do resistance exercises with a dumbbell on the functioning side and active assisted range of motion (ie, the exercise provider assists the resident to achieve full range of motion against gravity) on the hemiparetic side. A resident with Parkinson’s disease who has freezing gait may need visual or rhythmical verbal cues to be able to accomplish standing balance tasks such as altered walking patterns (eg, wide or narrow stepping) [42].

Modifications for Cognitive Impairments

More than 80% of residents in LTC have some degree of cognitive impairment [21]. Cognitive impairment may be the result of stroke, depression, traumatic injuries, medications, and degenerative diseases such as Parkinson’s and Alzheimer’s disease [43]. A common misconception is that residents with cognitive impairment cannot benefit from exercise because they cannot learn new skills and have difficulty following directions. On the contrary, evidence suggests that exercise can improve functional mobility for residents with cognitive impairment [44,45].

Residents with cognitive impairment may require a different approach to facilitate participation in the desired exercises because of difficulty following multi-step directions, responsive behaviors, or increased distractibility [46]. Clear communication is key in improving the quality of interaction for residents with cognitive impairment. The Alzheimer Society of Ontario suggests 10 strategies for communicating with people with dementia [47], and we have provided suggestions of how to apply these communication strategies to the exercise context in LTC (Table 2). Other suggestions for engaging residents with cognitive impairment in strength and balance training include making the exercises functional (eg, ask them to pick something up of the floor to perform a squat, or reach a point on the wall to do calf raises) and playful (eg, toss a ball back and forth or sing a song about rowing to promote weight shifting) [48].

jcom02501028_t2.jpg

Standing versus Seated Exercises

Residents may not be able to participate in standing exercises for several reasons: perhaps the resident cannot stand or has severe balance impairments and a high falls risk; the resident may have poor insight into which exercises are safe to perform in standing versus sitting; or there may be limited supervision of a large group exercise class where the risk of falls is a concern. If balance impairments are a concern, where the risk of injury or falling while completing exercises in standing outweighs the benefit of doing the exercises, then seated exercises are appropriate. However, when residents are able, we recommend encouraging some or all exercises in standing, to facilitate carry over of strength gains into functional tasks such as being able to rise from a chair and walking. A recent study, comparing standing versus seated exercises for community dwelling older adults, saw greater functional gains for those who completed the standing exercises [49]. Therefore, strength and balance exercises should be performed in standing, where appropriate.

Resident-Centered Exercise for Falls Prevention

Putting the resident at the center of falls prevention is important. Previous work has found that older adults have expressed a strong preference for care that transcends traditional biomedical care and that values efficiency, consistency, and hierarchical decision making [50]. On the contrary, resident-centered care emphasizes well-being and quality of life as defined by the resident, values giving residents greater control over the nature of services they receive, and respects their rights to be involved in every day decision making [51,52]. Indeed, residents may choose to engage in risky behaviors that increase their risk of falls but also increases their quality of life. Previous work has found disconnects between residents’ perceived frailty and the potential ability of protective devices to prevent adverse events, such as falls and fractures [53]. Additionally, one study identified that older residents feared being labelled, so instead hid impairments and chose to refuse assistance and assistive devices [54]. For example, a resident with impaired balance and gait may choose to walk independently when they have been deemed as requiring a gait aid (eg, rollator walker). However, they may value walking without a gait aid and accept the increased risk of falling. Therefore, it is essential to find the delicate balance between respecting a resident’s right to make their own decisions and preventing adverse events, such as falls [52]. An example of this would be respecting a resident’s right to refuse to attend exercise programming even though the team may think they can benefit from strength and balance training.

 

 

There is limited evidence around falls prevention and resident-centered care. A recent systematic review [55] revealed that resident-centered care may increase falls rates [56,57]. However, the authors of the review attributed the increase in falls to differences in frailty between the control and intervention group [56], and to environmental factors (eg, slippery flooring material, lack of handrails) [57]. Additionally, these trials did not include an exercise program as part of the resident-centered care program. On the other hand, resident-centered care has been associated with reduction of boredom, helplessness, and depression [58,59]. Most studies included in the review were quasi-experimental, which significantly limits the evidence quality [55]. At this point in time, the evidence suggests that resident-centered care is important for mood and quality of life but may have a negative or no effect on reducing falls.

Multifactorial Falls Prevention Programs

While there are mixed results about the effect of exercise as a single intervention for reducing falls for residents in LTC, the literature clearly supports exercise as part of a multifactorial falls prevention program [17,20,60–62]. A 2015 umbrella review [62] of meta-analyses of randomized controlled trials of falls prevention interventions in LTC concluded that multifactorial interventions were the most effective at preventing falls in LTC. Additionally, recently developed recommendations for fracture prevention in LTC [61] suggest that balance, strength, and functional training should be included for residents who are not at high risk of fracture, while for those at high risk, exercise should be provided as part of a multifactorial falls prevention intervention. Clinicians must therefore incorporate elements aside from exercise into their falls prevention strategies. Interventions that have shown positive effects on reducing falls when delivered as part of multifactorial interventions include: staff and resident education [31,35,37], environmental modifications [31,35], supply/repair/provision of assistive devices [30], falls problem-solving conferences [30], urinary incontinence management [29], medication review [30], optician review [31], and cognitive behavioral therapy [32].

Conclusion and Suggestions for Clinical Practice

We suggest incorporating strength and balance exercises as part of a multifactorial falls prevention program for residents in LTC. Balance exercises should be challenging and dynamic (eg, weight shifting). Strength exercises should be of a moderate to high intensity (eg, can complete one to sets of 6 to 8 repetitions) and need to be progressed as the residents’ abilities improve. Residents should participate in strength and balance training on 2 to 3 days per week, for 30- to 45-minute sessions, for at least 6 months. Exercises in standing should be prioritized where appropriate. Exercise could be delivered in a group or individual format, but should consider the residents’ preferences, the social benefits of group exercise, and the feasibility of individualizing exercises for the complex needs of residents in LTC in large group settings. Professionals delivering an exercise program should be trained in exercise planning, delivery, and progression, be familiar with the principles of balance and strength training, and have training in working with older adults in LTC. Exercise programs in LTC should be resident-centered and consider residents’ potential physical and cognitive impairments.

Funding/support: Dr. Giangregorio was supported by grants from the Canadian Frailty Network and Canadian Institutes of Health Research.

References

1. Harris IA, Yong S, McEvoy L, Thorn L. A prospective study of the effect of nursing home residency on mortality following hip fracture. ANZ J Surg 2010;80:447–50.

2. Ooms ME, Vlasman P, Lips P, et al. The incidence of hip fractures in independent and institutionalized elderly people. Osteoporos Int 1994;4:6–10.

3. Ayoung-Chee P, McIntyre L, Ebel BE, et al. Long-term outcomes of ground-level falls in the elderly. J Trauma Acute Care Surg 2014;76:498–503.

4. Heinrich S, Rapp K, Rissmann U, et al. Cost of falls in old age: a systematic review. Osteoporos Int 2010;21: 891–902.

5. Rubenstein LZ, Josephson KR, Robbins AS. Falls in the nursing home. Ann Intern Med 1994;121:442–51.

6. Hartholt KA, van Beeck EF, Polinder S, et al. Societal consequences of falls in the older population: injuries, healthcare costs, and long-term reduced quality of life. J Trauma
2011;71:748–53.

7. Robinovitch SN, Feldman F, Yang Y, et al. Video capture of the circumstances of falls in elderly people residing in long-term care: an observational study. Lancet 2013;381:
47–54.

8. Rapp K, Becker C, Cameron ID, et al. Epidemiology of falls in residential aged care: analysis of more than 70,000 falls from residents of bavarian nursing homes. J Am Med Dir Assoc 2012;13:187.

9. Büchele G, Becker C, Cameron ID, et al. Predictors of serious consequences of falls in residential aged care: analysis of more than 70,000 falls from residents of Bavarian nursing homes. J Am Med Dir Assoc 2014;15:559–63.

10. McArthur C, Gonzalez DA, Roy E, Giangregorio L. What are the circumstances of falls and fractures in long-term care? Can J Aging / La Rev Can du Vieil 2016;35:491–8.

11. Chin A Paw MJM, van Poppel MNM, van Mechelen W. Effects of resistance and functional-skills training on habitual activity and constipation among older adults living in long-term care facilities: a randomized controlled trial. BMC Geriatr 2006;6:9.

12. Ikezoe T, Asakawa Y, Shima H, et al. Daytime physical activity patterns and physical fitness in institutionalized elderly women: an exploratory study. Arch Gerontol Geriatr 2013;57:221–5.

13. Senior HE, Henwood TR, Beller EM, et al. Prevalence and risk factors of sarcopenia among adults living in nursing homes. Maturitas 2015;82:418–23.

14. Smoliner C, Sieber CC, Wirth R. Prevalence of sarcopenia in geriatric hospitalized patients. J Am Med Dir Assoc 2014;15:267–72.

15. Landi F, Liperoti R, Fusco D, et al. Sarcopenia and mortality among older nursing home residents. J Am Med Dir Assoc 2012;13:121–6.

16. Yalcin A, Aras S, Atmis V, et al. Sarcopenia prevalence and factors associated with sarcopenia in older people living in a nursing home in Ankara Turkey. Geriatr Gerontol Int
2016;16:903–10.

17. Sherrington C, Michaleff ZA, Fairhall N, et al. Exercise to prevent falls in older adults: an updated systematic review and meta-analysis. Br J Sports Med October 2016.

18. Liu C, Latham NK. Progressive resistance strength training for improving physical function in older adults. In: Liu C, ed. Cochrane Database Syst Rev;2009:CD002759.

19. Cameron ID, Gillespie LD, Robertson MC, et al. Interventions for preventing falls in older people in care facilities and hospitals. Cochrane Database Syst Rev;2012:CD005465.

20. Silva RB, Eslick GD, Duque G. Exercise for falls and fracture prevention in long term care facilities: a systematic review and meta-analysis. J Am Med Dir Assoc 2013;14:685–9.

21. Hirdes JP, Mitchell L, Maxwell CJ, White N. Beyond the “iron lungs of gerontology”: Using evidence to shape the future of nursing homes in Canada. Can J Aging 2011;30: 371–90.

22. Benjamin K, Edwards N, Guitard P, et al. Factors that influence physical activity in long-term care: Perspectives of residents, staff, and significant others. Can J Aging 2011;30:247–58.

23. Benjamin K, Edwards N, Ploeg J, Legault F. Barriers to physical activity and restorative care for residents in long-term care: A review of the literature. J Aging Phys Act 2014;22:154–65.

24. American College of Sports Medicine. ACSM’s guidelines for exercise testing and prescription. 9th ed. American College of Sports Medicine; 2013.

25. Prevention of Falls Network Europe. Prevention of Falls Network Europe. Accessed 27 Nov 2017 at www.profane.eu.org/.

26. Sihvonen SE, Sipilä S, Era PA. Changes in postural balance in frail elderly women during a 4-week visual feedback training: a randomized controlled trial. Gerontology 2004;50:87–95.

27. Sakamoto K, Nakamura T, Hagino H, et al. Effects of unipedal standing balance exercise on the prevention of falls and hip fracture among clinically defined high-risk elderly individuals: a randomized controlled trial. J Orthop Sci 2006;11:467–72.

28. Shimada H, Obuchi S, Furuna T, Suzuki T. New intervention program for preventing falls among frail elderly people: the effects of perturbed walking exercise using a bilateral separated treadmill. Am J Phys Med Rehabil 2004;83:493–9.

29. Schnelle JF, Kapur K, Alessi C, et al. Does an exercise and incontinence intervention save healthcare costs in a nursing home population? J Am Geriatr Soc 2003;51:161–8.

30. Jensen J, Lundin-Olsson L, Nyberg L, Gustafson Y. Fall and injury prevention in older people living in residential care facilities: A cluster randomized trial. Ann Intern Med 2002;136:733–41.

31. Dyer CAE. Falls prevention in residential care homes: a randomised controlled trial. Age Ageing 2004;33:596–602.

32. Huang T-T, Chung M-L, Chen F-R, Chin Y-F, Wang B-H. Evaluation of a combined cognitive-behavioural and exercise intervention to manage fear of falling among elderly residents in nursing homes. Aging Ment Health 2016;20:2–12.

33. Sihvonen S, Sipilä S, Taskinen S, Era P. Fall incidence in frail older women after individualized visual feedback-based balance training. Gerontology 2004;50:411–6.

34. Lord SR, Castell S, Corcoran J, et al. The effect of group exercise on physical functioning and falls in frail older people living in retirement villages: a randomized, controlled trial. J Am Geriatr Soc 2003;51:1685–92.

35. Becker C, Kron M, Lindemann U, et al. Effectiveness of a multifaceted intervention on falls in nursing home residents. J Am Geriatr Soc 2003;51:306–13.

36. Becker C, Kron M, Lindemann U, et al. Effectiveness of a multifaceted intervention on falls in nursing home residents. J Am Geriatr Soc 2003;51:306–13.

37. Jensen J, Lundin-Olsson L, Nyberg L, Gustafson Y. Fall and injury prevention in older people living in residential care facilities. A cluster randomized trial. Ann Intern Med 2002;136:733–41.

38. Moreland JD, Richardson JA, Goldsmith CH, Clase CM. Muscle weakness and falls in older adults: a systematic review and meta-analysis. J Am Geriatr Soc 2004;52: 1121–9.

39. Chodzko-Zajko WJ, Proctor DN, Fiatarone Singh MA, et al. Exercise and physical activity for older adults. Med Sci Sport Exerc 2009;41:1510–30.

40. de Souto Barreto P, Morley JE, Chodzko-Zajko W, et al. Recommendations on physical activity and exercise for older adults living in long-term care facilities: a taskforce report. J Am Med Dir Assoc 2016;17:381–92.

41. American College of Sports Medicine. Progression models in resistance training for healthy adults. Med Sci Sport Exerc 2009;41:687–708.

42. Fietzek UM, Schroeteler FE, Ziegler K, et al. Randomized cross-over trial to investigate the efficacy of a two-week physiotherapy programme with repetitive exercises of cueing to reduce the severity of freezing of gait in patients with Parkinson’s disease. Clin Rehabil 2014;28:902–11.

43. Patterson C, Feightner J, Garcia A, MacKnight C. General risk factors for dementia: A systematic evidence review. Alzheimer Dement 2007;3:341–7.

44. Roach KE, Tappen RM, Kirk-Sanchez N, et al. A randomized controlled trial of an activity specific exercise program for individuals with alzheimer disease in long-term care settings. J Geriatr Phys Ther 2011;34:50–6.

45. Christofoletti G, Oliani MM, Gobbi S, et al. A controlled clinical trial on the effects of motor intervention on balance and cognition in institutionalized elderly patients with dementia. Clin Rehabil 2008;22:618–26.

46. van Alphen HJM, Hortobágyi T, van Heuvelen MJG. Barriers, motivators, and facilitators of physical activity in dementia patients: A systematic review. Arch Gerontol Geriatr 2016;66:109–18.

47. Alzheimer Society of Ontario. Rethink Dementia. Accessed 18 Sep 2017 at http://rethinkdementia.ca/.

48. Roach KE, Tappen RM, Kirk-Sanchez N, et al. A randomized controlled trial of an activity specific exercise program for individuals with Alzheimer disease in long-term care settings. J Geriatr Phys Ther 2011;34:50–6.

49. Brach JS, Perera S, Gilmore S, et al. Effectiveness of a timing and coordination group exercise program to improve mobility in community-dwelling older adults. JAMA Intern Med August 2017.

50. Rosher RB, Robinson S. Impact of the Eden alternative on family satisfaction. J Am Med Dir Assoc 2005;6:189–93.

51. Crandall LG, White DL, Schuldheis S, Talerico KA. Initiating person-centered care practices in long-term care facilities. J Gerontol Nurs 2007;33:47–56.

52. Sims-Gould J, McKay HA, Feldman F, et al. Autonomy, choice, patient-centered care, and hip protectors: the experience of residents and staff in long-term care. J Appl Gerontol 2014;33:690–709.

53. Robinovitch SN, Cronin T. Perception of postural limits in elderly nursing home and day care participants. J Gerontol A Biol Sci Med Sci 1999;54:B124-30.

54. Perkins MM, Ball MM, Whittington FJ, Hollingsworth C. Relational autonomy in assisted living: a focus on diverse care settings for older adults. J Aging Stud 2012;26:214–25.

55. Brownie S, Nancarrow S. Effects of person-centered care on residents and staff in aged-care facilities: a systematic review. Clin Interv Aging 2013;8:1–10.

56. Coleman MT, Looney S, O’Brien J, et al. The Eden Alternative: findings after 1 year of implementation. J Gerontol A Biol Sci Med Sci 2002;57:M422–7.

57. Chenoweth L, King MT, Jeon Y-H, et al. Caring for Aged Dementia Care Resident Study (CADRES) of personcentred care, dementia-care mapping, and usual care in dementia: a cluster-randomised trial. Lancet Neurol 2009;8: 317–25.

58. Bergman-Evans B. Beyond the basics. Effects of the Eden Alternative model on quality of life issues. J Gerontol Nurs 2004;30:27–34.

59. Robinson SB, Rosher RB. Tangling with the barriers to culture change: creating a resident-centered nursing home environment. J Gerontol Nurs 2006;32:19–25.

60. Cameron ID, Gillespie LD, Robertson MC, et al. Interventions for preventing falls in older people in care facilities and hospitals. Cochrane Database Syst Rev 2012;12.

61. Papaioannou A, Santesso N, Morin SN, et al. Recommendations for preventing fracture in long-term care. Can Med Assoc J 2015;187:1135–44.

62. Stubbs B, Denkinger MD, Brefka S, Dallmeier D. What works to prevent falls in older adults dwelling in long term care facilities and hospitals? An umbrella review of meta-analyses of randomised controlled trials. Maturitas 2015;81:335–42.

Article PDF
Issue
Journal of Clinical Outcomes Management - 25(1)
Publications
Topics
Sections
Article PDF
Article PDF

From the Geriatric Education and Research in Aging Sciences Centre, McMaster University Hamilton, ON (Dr. McArthur) and the University of Waterloo and Research Institute for Aging, Waterloo, ON (Dr. Giangregorio), Canada

 

Abstract

  • Objective: To synthesize the available literature on exercise and falls reduction interventions in long-term care (LTC) and provide practical information for clinicians and other decision makers.
  • Methods: Review of positive trials included in systematic reviews.
  • Results: Falls are a major concern for residents, families, clinicians, and decision-makers in LTC. Exercise is recommended as part of a multifactorial falls prevention program for residents in LTC. Strength and balance exercises should be incorporated into the multifactorial falls prevention program. They should be challenging and progressed as the residents’ abilities improve. Evidence suggests that exercises should be completed 2 to 3 times per week for a period longer than 6 months. Exercise programs in LTC should be resident-centered and should consider residents’ potential physical and cognitive impairments. Exercises in standing should be prioritized where appropriate.
  • Conclusion: Appropriately challenging and progressive strength and balance exercises should be included in a multifactorial falls prevention program for residents in LTC.

Key words: long-term care; nursing homes; falls reduction; exercise.

 

Falls are common in long-term care (LTC) homes: the estimated falls rate is 1.5 falls per bed per year, which is 3 times greater than that for older adults living in the community [1]. Falls can have significant consequences for residents in LTC, including functional disability, fractures, pain, reduced quality of life, and death [1–6]. Indeed, 25% of residents who are hospitalized after a fall die within 1 year [3]. Consequently, falls prevention programs are important to help in reducing falls and averting the associated negative consequences.

Exercise may address the circumstances and physical deconditioning that often contribute to falls in LTC residents. Weight shifting [7], walking, and transferring [8–10], are common activities that precede falls, suggesting that balance, gait, and functional mobility training may be possible targets for prevention. Additionally, it is estimated that LTC residents spend three quarters of their waking time in sedentary activities [11,12] and have a high prevalence of sarcopenia [13–16]. Challenging balance training and resistance exercise are well-known intervention for reducing falls [17] and improving muscle strength for community-dwelling older adults [18]. However, evidence around balance and strength training for preventing falls in LTC is mixed [17,19,20], and careful planning and modification of exercises is necessary to meet the needs of LTC residents.

Residents in LTC are often medically complex, with multiple comorbidities [21] that can affect their ability to meaningfully participate in exercise. In Canada, 56.3% of residents have a diagnosis of Alzheimer’s or other dementias, 25.0% have diabetes, 14.4% have chronic obstructive pulmonary disease, and 21.2% have experienced a stroke [21]. Residents also often have significant functional impairments. For example, 97% of residents require assistance with basic activities of daily living [21]. Therefore, the lack of effect of exercise as a single falls prevention strategy observed in previous studies may be because the often complex, multimorbid LTC population likely requires a multifactorial approach to fall prevention [17]. Additionally, organizational aspects of LTC homes (eg, specific funds dedicated to employing exercise professionals and to support exercise programming) can affect residents’ engagement in exercise [22,23]. Subsequently, prescribing exercises in the LTC context must consider both resident characteristics and organizational features of the LTC home (eg, professionals available to support exercise programming).

A comprehensive exercise prescription describes the elements of an appropriate exercise program to facilitate implementation of that program. The exercise prescription should include a description of the type (eg, balance, strength) and intensity of exercises (eg, subjective or objective measurement of how hard the resident is working) included in the program [24]. The prescription should also include a description of the dose of exercise: frequency of exercise participation (eg, 2 days per week), duration of individual exercise sessions (eg, 30-minute sessions), and duration of exercise program (eg, 12-week program) [24]. Lastly, the prescription should describe the setting of the exercise program (eg, group or individual basis) and the professional delivering the program (eg, physiotherapist, fitness instructor) [24].

Therefore, the objectives of this article are to (1) synthesize studies demonstrating a positive effect of exercise on reducing falls for residents in LTC; (2) provide an overview of the principles of balance and strength training to guide clinicians in designing appropriate exercise prescription; and (3) make suggestions for clinical practice regarding an appropriate strength and balance exercise protocol by considering the influence of the LTC context.

Methods

To provide clinicians and other policy-makers with a description of which balance and strength exercises may be effective for preventing falls, we synthesized trials that demonstrated a positive effect on reducing falls or falls risk for residents in LTC. Studies were identified through a database search for systematic reviews in PubMed, Ovid, and Google Scholar using the keywords falls, long-term care, nursing homes, exercise, strength, balance, and systematic reviews. Our purpose was to provide practical information on what works to prevent falls through balance and strength training for residents in LTC rather than to evaluate the available evidence. Therefore, only positive trials from systematic reviews were discussed, as we wanted to present exercises that seem to have a positive effect on decreasing falls. Positive trials were defined as those included in identified systematic reviews with a risk or rate ratio and confidence intervals below 1.0.

 

 

We first provide an overview of the conclusions of the systematic reviews found in our search. Next, for each positive trial we describe the following elements of the exercise component of the intervention: frequency, time of sessions, length of program, intensity, type of exercise including a description of the specific exercises performed, whether the intervention was delivered in a group or on an individual basis, the professional delivering the intervention, and any other features of the intervention aside from the exercise component. We used the ProFaNE taxonomy definitions [25] to identify and describe each element of the exercise interventions. Frequency is the number of times per week that residents engage in sessions, time of sessions is the amount allocated to each exercise session, duration of program is how long the resident participates in the exercise program, and intensity is the subjective or objective report of how hard the resident is working [25]. The types of exercises described were those targeting balance defined as “...the efficient transfer of bodyweight from one part of the body to another or challenges specific aspects of the balance systems (eg, vestibular system)” [25], and strength defined as “...contracting the muscles against a resistance to ‘overload’ and bring about a training effect in the muscular system” [25]. Strength could be either an external resistance (eg, dumbbell) or using body weight against gravity (eg, squat) [25].

Results

We found 3 systematic reviews that include exercise programs to reduce falls in LTC homes [17,19,20]. Overall, evidence suggests that exercise should be included as part of a multifactorial falls prevention program for residents in LTC. There is limited evidence that exercise as a single intervention prevents falls, and some trials, albeit underpowered, even demonstrate an increased risk of falling in the exercise group compared to control [19]. With regards to specific exercise programs, the Cochrane review found that gait, balance, and functional training decrease the rate of falls but not the risk of falling [26–28], and the 2013 review by Silva et al [20] concluded that combined exercise programs (ie, multiple types of exercise) that include balance tasks, are completed frequently (2–3 times per week), and over a long term (greater than 6 months) were most effective at preventing falls [20].

 

A more recent systematic review and meta-analysis [17] also concluded that there was no evidence that exercise as a single intervention can prevent falls for residents in LTC. Table 1 provides a description of the exercise component of the seven positive trials [29–35] that were included in the 3 systematic reviews we identified in our search.

jcom02501028_t1a.jpg
jcom02501028_t1b.jpg

Type of Exercise

Balance Exercises

There were 4 positive trials that included balance exercises in their intervention [31,33–35]. Trials that had a positive effect on reducing falls and included balance training employed mostly dynamic balance exercises in standing (Table 1). However, only 2 of the 7 trials provided a detailed description of their balance exercises (Table 1) [26,34]. Jensen et al [30] and Dyer et al [31] did not include a description of the balance training performed but stated that balance was part of the multicomponent exercise program. Becker et al [36] stated that participants performed standing balance exercises, while Schnelle et al [39] and Huang et al [32] did not include balance training in their trial.

Strength Exercises

Of the 7 positive trials included in this review, 6 included strength exercises [29–32,34,35]. The strength activities used in trials where exercise had a positive effect on decreasing falls included functional activities [29,31] and progressive resistance training [31,36] (Table 1). Functional activities are those that replicate what a resident might be required to do in their everyday life, such as performing sit-to-stands out of a chair (Figure

jcom02501028_f1.jpg
or practicing bed mobility (eg, rolling from side to side, transitioning from lying to sitting and vice versa) [25]. The exercise program in Schnelle et al’s [29] trial had residents performing sit-to-stands every 2 hours, and arm curls or arm raises once a day. Participants in Jensen et al [30] at Dyer et al’s [31] trials performed progressive resistance training but did not describe which particular exercises participants performed. Similarly, Becker et al [36] described progressive resistance training with dumbbells and ankle weights but did not describe any specific exercises. Participants in the trial by Lord et al [34] performed strengthening exercises of ankle dorsiflexors, knee extensors, hip abductors, and hip side flexors. One trial stated their exercise program focused on muscle endurance rather than strength (eg, higher repetitions with lower resistance), and participants did grip ball exercises, ankle dorsiflexion and plantar flexion, upper limb elevation, knee elevation, and sit-to-stands [32].

 

 

Frequency, Time of Sessions, Duration of Program

In our description of positive trials, exercise was performed on 2 to 3 days per week for 20 to 75 minutes per session, for periods ranging from 4 to 52 weeks (Table 1).

Intensity

For the trials including balance exercises, one trial described the intensity as resident-specific [37] and another as individualized [33]. Two studies did not describe the intensity of their balance exercises [31,34]. The intensity of strength exercises included in the positive trials was individualized for one of the trial [29]. Two trials had participants complete 2 to 3 sets of 10 repetitions [32,35], with one indicating an intensity of 12–13 or “somewhat difficult” on the Borg Rating of Perceived Exertion Scale [32] and the other using a 10-rep max [35]. Two studies described their strength exercises as progressive [31,37], and one at a moderate to high intensity [30]. Lord et al prescribed 30 repetitions of each strength exercise [34].

Delivery of Intervention

Exercise was delivered in a group setting for 4 of the trials [31,32,34,36], individually for 2 of the trials [26,29], and the setting was not described for one of the trials (Table 1) [30]. Finally, only 3 of the 7 articles reported the professional delivering the intervention: one was research staff [29], one was geriatric nurses [32], and one was exercise assistants supported by a physiotherapist [31].

 

Discussion

There is limited evidence to support the use of strength and balance exercise as a single intervention to prevent falls in LTC. However, exercise should be included as part of a multifactorial falls prevention program. Trials that had a positive effect on decreasing falls training used dynamic balance exercises in standing, functional training, and progressive resistance training on 2 to 3 days per week, for 20 to 75 minutes per session, over 4 to 52 weeks. The intensity of balance exercises was individualized, and strength exercises were described as somewhat difficult or performed at a moderate to high intensity. Exercise was performed in a group or individually, and was delivered by research staff, geriatric nurses, exercise assistants supervised by physiotherapists, or more frequently, it was not reported who delivered the intervention.

Balance Training

Our work suggests that standing, dynamic balance exercises may be best to decrease falls. Example balance exercises include reducing the base of support (eg, standing with feet together instead of apart, or tandem with one foot in front), moving the center of gravity and control body position while standing (eg, reaching, weight shifting, stepping up or down), and standing without using arms for support or reducing reliance on the upper limbs for support (eg, use one hand on a handrail instead of two, or two fingers instead of the whole hand) [17]. It is well established that balance training programs, especially those including challenging exercises, can prevent falls in community-dwelling older adults [17]. However, the relationship is not as clear in LTC.

Strength Training

Reduced muscle strength has been identified as an important risk factor for falls [38]. There are also many psychological and metabolic benefits to strength training [39]. To induce change in muscular strength, resistance exercises need to be challenging and progressive. Our work suggests that strength training that is effective at decreasing falls is functional and progressive, and is completed at a moderate to high intensity. A resident should be able to do a strength exercise for one to two sets of 6 to 8 repetitions before being fatigued [40]. Once the resident can complete two sets of 13 to 15 repetitions easily the exercise should be progressed. Residents who are particularly deconditioned may need to begin with lower intensity strength exercises (eg, only do one set, with a lower resistance and progress to a higher resistance) [40]. Residents should perform resistance exercises for all major muscle groups [40]. Progression could include increasing the number of sets (eg, increase from one to two sets), the resistance (eg, holding dumbbells while squatting), or the intensity of the exercise (eg, squat lower or faster) [41].

Implementing Exercise Programs in LTC

Implementation of exercise programs into LTC homes should consider the dose of exercise (eg, time and frequency of sessions, duration of program), if they are delivered in a group or individual setting, and who is delivering them. First, trials included in this paper suggest that strength and balance exercises to prevent falls were delivered 2 to 3 times per week, for 20 to 75 minutes per session, over 4 to 52 weeks. Second, previous work has established that exercise programs delivered on 2 to 3 days per week over a period of more than 6 months are most effective at reducing falls in LTC [20]. Finally, a recent task force report from an international group of clinician researchers in LTC recommends twice weekly exercise sessions lasting 35 to 45 minutes each [40]. Therefore, strength and balance exercises to prevent falls in LTC should be delivered at least twice per week, for at least 20 minutes, for greater than 6 weeks’ duration.

 

 

Whether exercise should be performed in a group or individual setting remains unclear. Two of the 6 positive trials in this paper were completed individually, while 3 were in a group. The aforementioned task force also recommended that every resident who does not have contraindications to exercise must have an individualized exercise program as part of their health care plan [40]. However, whether the exercise program is provided on an individual basis or in a group setting was not delineated. Indeed, there are currently no recommendations concerning prioritizing group or individual exercise programs. Therefore, exercise programs being implemented into LTC homes should consider the residents’ preferences, the social benefits of group exercise, and the feasibility of individualizing exercises for the complex needs of residents in LTC in large group settings.

Finally, which professionals should deliver the exercise program is also uncertain. Only 3 of the positive trials in this paper described the professional delivering the intervention, with one being research staff, one geriatric nurses, and one exercise assistants supported by a physiotherapist. We suggest that professionals delivering an exercise program should be trained in exercise planning, delivery, and progression, be familiar with the principles of balance and strength training, and have training in working with older adults in LTC.

 

Modifications for Physical Impairments

Residents in LTC often have complex health needs, with multiple comorbidities (eg, stroke, Parkinson’s disease, multiple sclerosis) [21]. Modifications of strength and balance exercises may be required to accommodate for physical impairments (eg, hemiplegia, drop foot, freezing gait). For example, if a resident has hemiplegia and cannot fully activate the muscles of one arm, one can do resistance exercises with a dumbbell on the functioning side and active assisted range of motion (ie, the exercise provider assists the resident to achieve full range of motion against gravity) on the hemiparetic side. A resident with Parkinson’s disease who has freezing gait may need visual or rhythmical verbal cues to be able to accomplish standing balance tasks such as altered walking patterns (eg, wide or narrow stepping) [42].

Modifications for Cognitive Impairments

More than 80% of residents in LTC have some degree of cognitive impairment [21]. Cognitive impairment may be the result of stroke, depression, traumatic injuries, medications, and degenerative diseases such as Parkinson’s and Alzheimer’s disease [43]. A common misconception is that residents with cognitive impairment cannot benefit from exercise because they cannot learn new skills and have difficulty following directions. On the contrary, evidence suggests that exercise can improve functional mobility for residents with cognitive impairment [44,45].

Residents with cognitive impairment may require a different approach to facilitate participation in the desired exercises because of difficulty following multi-step directions, responsive behaviors, or increased distractibility [46]. Clear communication is key in improving the quality of interaction for residents with cognitive impairment. The Alzheimer Society of Ontario suggests 10 strategies for communicating with people with dementia [47], and we have provided suggestions of how to apply these communication strategies to the exercise context in LTC (Table 2). Other suggestions for engaging residents with cognitive impairment in strength and balance training include making the exercises functional (eg, ask them to pick something up of the floor to perform a squat, or reach a point on the wall to do calf raises) and playful (eg, toss a ball back and forth or sing a song about rowing to promote weight shifting) [48].

jcom02501028_t2.jpg

Standing versus Seated Exercises

Residents may not be able to participate in standing exercises for several reasons: perhaps the resident cannot stand or has severe balance impairments and a high falls risk; the resident may have poor insight into which exercises are safe to perform in standing versus sitting; or there may be limited supervision of a large group exercise class where the risk of falls is a concern. If balance impairments are a concern, where the risk of injury or falling while completing exercises in standing outweighs the benefit of doing the exercises, then seated exercises are appropriate. However, when residents are able, we recommend encouraging some or all exercises in standing, to facilitate carry over of strength gains into functional tasks such as being able to rise from a chair and walking. A recent study, comparing standing versus seated exercises for community dwelling older adults, saw greater functional gains for those who completed the standing exercises [49]. Therefore, strength and balance exercises should be performed in standing, where appropriate.

Resident-Centered Exercise for Falls Prevention

Putting the resident at the center of falls prevention is important. Previous work has found that older adults have expressed a strong preference for care that transcends traditional biomedical care and that values efficiency, consistency, and hierarchical decision making [50]. On the contrary, resident-centered care emphasizes well-being and quality of life as defined by the resident, values giving residents greater control over the nature of services they receive, and respects their rights to be involved in every day decision making [51,52]. Indeed, residents may choose to engage in risky behaviors that increase their risk of falls but also increases their quality of life. Previous work has found disconnects between residents’ perceived frailty and the potential ability of protective devices to prevent adverse events, such as falls and fractures [53]. Additionally, one study identified that older residents feared being labelled, so instead hid impairments and chose to refuse assistance and assistive devices [54]. For example, a resident with impaired balance and gait may choose to walk independently when they have been deemed as requiring a gait aid (eg, rollator walker). However, they may value walking without a gait aid and accept the increased risk of falling. Therefore, it is essential to find the delicate balance between respecting a resident’s right to make their own decisions and preventing adverse events, such as falls [52]. An example of this would be respecting a resident’s right to refuse to attend exercise programming even though the team may think they can benefit from strength and balance training.

 

 

There is limited evidence around falls prevention and resident-centered care. A recent systematic review [55] revealed that resident-centered care may increase falls rates [56,57]. However, the authors of the review attributed the increase in falls to differences in frailty between the control and intervention group [56], and to environmental factors (eg, slippery flooring material, lack of handrails) [57]. Additionally, these trials did not include an exercise program as part of the resident-centered care program. On the other hand, resident-centered care has been associated with reduction of boredom, helplessness, and depression [58,59]. Most studies included in the review were quasi-experimental, which significantly limits the evidence quality [55]. At this point in time, the evidence suggests that resident-centered care is important for mood and quality of life but may have a negative or no effect on reducing falls.

Multifactorial Falls Prevention Programs

While there are mixed results about the effect of exercise as a single intervention for reducing falls for residents in LTC, the literature clearly supports exercise as part of a multifactorial falls prevention program [17,20,60–62]. A 2015 umbrella review [62] of meta-analyses of randomized controlled trials of falls prevention interventions in LTC concluded that multifactorial interventions were the most effective at preventing falls in LTC. Additionally, recently developed recommendations for fracture prevention in LTC [61] suggest that balance, strength, and functional training should be included for residents who are not at high risk of fracture, while for those at high risk, exercise should be provided as part of a multifactorial falls prevention intervention. Clinicians must therefore incorporate elements aside from exercise into their falls prevention strategies. Interventions that have shown positive effects on reducing falls when delivered as part of multifactorial interventions include: staff and resident education [31,35,37], environmental modifications [31,35], supply/repair/provision of assistive devices [30], falls problem-solving conferences [30], urinary incontinence management [29], medication review [30], optician review [31], and cognitive behavioral therapy [32].

Conclusion and Suggestions for Clinical Practice

We suggest incorporating strength and balance exercises as part of a multifactorial falls prevention program for residents in LTC. Balance exercises should be challenging and dynamic (eg, weight shifting). Strength exercises should be of a moderate to high intensity (eg, can complete one to sets of 6 to 8 repetitions) and need to be progressed as the residents’ abilities improve. Residents should participate in strength and balance training on 2 to 3 days per week, for 30- to 45-minute sessions, for at least 6 months. Exercises in standing should be prioritized where appropriate. Exercise could be delivered in a group or individual format, but should consider the residents’ preferences, the social benefits of group exercise, and the feasibility of individualizing exercises for the complex needs of residents in LTC in large group settings. Professionals delivering an exercise program should be trained in exercise planning, delivery, and progression, be familiar with the principles of balance and strength training, and have training in working with older adults in LTC. Exercise programs in LTC should be resident-centered and consider residents’ potential physical and cognitive impairments.

Funding/support: Dr. Giangregorio was supported by grants from the Canadian Frailty Network and Canadian Institutes of Health Research.

From the Geriatric Education and Research in Aging Sciences Centre, McMaster University Hamilton, ON (Dr. McArthur) and the University of Waterloo and Research Institute for Aging, Waterloo, ON (Dr. Giangregorio), Canada

 

Abstract

  • Objective: To synthesize the available literature on exercise and falls reduction interventions in long-term care (LTC) and provide practical information for clinicians and other decision makers.
  • Methods: Review of positive trials included in systematic reviews.
  • Results: Falls are a major concern for residents, families, clinicians, and decision-makers in LTC. Exercise is recommended as part of a multifactorial falls prevention program for residents in LTC. Strength and balance exercises should be incorporated into the multifactorial falls prevention program. They should be challenging and progressed as the residents’ abilities improve. Evidence suggests that exercises should be completed 2 to 3 times per week for a period longer than 6 months. Exercise programs in LTC should be resident-centered and should consider residents’ potential physical and cognitive impairments. Exercises in standing should be prioritized where appropriate.
  • Conclusion: Appropriately challenging and progressive strength and balance exercises should be included in a multifactorial falls prevention program for residents in LTC.

Key words: long-term care; nursing homes; falls reduction; exercise.

 

Falls are common in long-term care (LTC) homes: the estimated falls rate is 1.5 falls per bed per year, which is 3 times greater than that for older adults living in the community [1]. Falls can have significant consequences for residents in LTC, including functional disability, fractures, pain, reduced quality of life, and death [1–6]. Indeed, 25% of residents who are hospitalized after a fall die within 1 year [3]. Consequently, falls prevention programs are important to help in reducing falls and averting the associated negative consequences.

Exercise may address the circumstances and physical deconditioning that often contribute to falls in LTC residents. Weight shifting [7], walking, and transferring [8–10], are common activities that precede falls, suggesting that balance, gait, and functional mobility training may be possible targets for prevention. Additionally, it is estimated that LTC residents spend three quarters of their waking time in sedentary activities [11,12] and have a high prevalence of sarcopenia [13–16]. Challenging balance training and resistance exercise are well-known intervention for reducing falls [17] and improving muscle strength for community-dwelling older adults [18]. However, evidence around balance and strength training for preventing falls in LTC is mixed [17,19,20], and careful planning and modification of exercises is necessary to meet the needs of LTC residents.

Residents in LTC are often medically complex, with multiple comorbidities [21] that can affect their ability to meaningfully participate in exercise. In Canada, 56.3% of residents have a diagnosis of Alzheimer’s or other dementias, 25.0% have diabetes, 14.4% have chronic obstructive pulmonary disease, and 21.2% have experienced a stroke [21]. Residents also often have significant functional impairments. For example, 97% of residents require assistance with basic activities of daily living [21]. Therefore, the lack of effect of exercise as a single falls prevention strategy observed in previous studies may be because the often complex, multimorbid LTC population likely requires a multifactorial approach to fall prevention [17]. Additionally, organizational aspects of LTC homes (eg, specific funds dedicated to employing exercise professionals and to support exercise programming) can affect residents’ engagement in exercise [22,23]. Subsequently, prescribing exercises in the LTC context must consider both resident characteristics and organizational features of the LTC home (eg, professionals available to support exercise programming).

A comprehensive exercise prescription describes the elements of an appropriate exercise program to facilitate implementation of that program. The exercise prescription should include a description of the type (eg, balance, strength) and intensity of exercises (eg, subjective or objective measurement of how hard the resident is working) included in the program [24]. The prescription should also include a description of the dose of exercise: frequency of exercise participation (eg, 2 days per week), duration of individual exercise sessions (eg, 30-minute sessions), and duration of exercise program (eg, 12-week program) [24]. Lastly, the prescription should describe the setting of the exercise program (eg, group or individual basis) and the professional delivering the program (eg, physiotherapist, fitness instructor) [24].

Therefore, the objectives of this article are to (1) synthesize studies demonstrating a positive effect of exercise on reducing falls for residents in LTC; (2) provide an overview of the principles of balance and strength training to guide clinicians in designing appropriate exercise prescription; and (3) make suggestions for clinical practice regarding an appropriate strength and balance exercise protocol by considering the influence of the LTC context.

Methods

To provide clinicians and other policy-makers with a description of which balance and strength exercises may be effective for preventing falls, we synthesized trials that demonstrated a positive effect on reducing falls or falls risk for residents in LTC. Studies were identified through a database search for systematic reviews in PubMed, Ovid, and Google Scholar using the keywords falls, long-term care, nursing homes, exercise, strength, balance, and systematic reviews. Our purpose was to provide practical information on what works to prevent falls through balance and strength training for residents in LTC rather than to evaluate the available evidence. Therefore, only positive trials from systematic reviews were discussed, as we wanted to present exercises that seem to have a positive effect on decreasing falls. Positive trials were defined as those included in identified systematic reviews with a risk or rate ratio and confidence intervals below 1.0.

 

 

We first provide an overview of the conclusions of the systematic reviews found in our search. Next, for each positive trial we describe the following elements of the exercise component of the intervention: frequency, time of sessions, length of program, intensity, type of exercise including a description of the specific exercises performed, whether the intervention was delivered in a group or on an individual basis, the professional delivering the intervention, and any other features of the intervention aside from the exercise component. We used the ProFaNE taxonomy definitions [25] to identify and describe each element of the exercise interventions. Frequency is the number of times per week that residents engage in sessions, time of sessions is the amount allocated to each exercise session, duration of program is how long the resident participates in the exercise program, and intensity is the subjective or objective report of how hard the resident is working [25]. The types of exercises described were those targeting balance defined as “...the efficient transfer of bodyweight from one part of the body to another or challenges specific aspects of the balance systems (eg, vestibular system)” [25], and strength defined as “...contracting the muscles against a resistance to ‘overload’ and bring about a training effect in the muscular system” [25]. Strength could be either an external resistance (eg, dumbbell) or using body weight against gravity (eg, squat) [25].

Results

We found 3 systematic reviews that include exercise programs to reduce falls in LTC homes [17,19,20]. Overall, evidence suggests that exercise should be included as part of a multifactorial falls prevention program for residents in LTC. There is limited evidence that exercise as a single intervention prevents falls, and some trials, albeit underpowered, even demonstrate an increased risk of falling in the exercise group compared to control [19]. With regards to specific exercise programs, the Cochrane review found that gait, balance, and functional training decrease the rate of falls but not the risk of falling [26–28], and the 2013 review by Silva et al [20] concluded that combined exercise programs (ie, multiple types of exercise) that include balance tasks, are completed frequently (2–3 times per week), and over a long term (greater than 6 months) were most effective at preventing falls [20].

 

A more recent systematic review and meta-analysis [17] also concluded that there was no evidence that exercise as a single intervention can prevent falls for residents in LTC. Table 1 provides a description of the exercise component of the seven positive trials [29–35] that were included in the 3 systematic reviews we identified in our search.

jcom02501028_t1a.jpg
jcom02501028_t1b.jpg

Type of Exercise

Balance Exercises

There were 4 positive trials that included balance exercises in their intervention [31,33–35]. Trials that had a positive effect on reducing falls and included balance training employed mostly dynamic balance exercises in standing (Table 1). However, only 2 of the 7 trials provided a detailed description of their balance exercises (Table 1) [26,34]. Jensen et al [30] and Dyer et al [31] did not include a description of the balance training performed but stated that balance was part of the multicomponent exercise program. Becker et al [36] stated that participants performed standing balance exercises, while Schnelle et al [39] and Huang et al [32] did not include balance training in their trial.

Strength Exercises

Of the 7 positive trials included in this review, 6 included strength exercises [29–32,34,35]. The strength activities used in trials where exercise had a positive effect on decreasing falls included functional activities [29,31] and progressive resistance training [31,36] (Table 1). Functional activities are those that replicate what a resident might be required to do in their everyday life, such as performing sit-to-stands out of a chair (Figure

jcom02501028_f1.jpg
or practicing bed mobility (eg, rolling from side to side, transitioning from lying to sitting and vice versa) [25]. The exercise program in Schnelle et al’s [29] trial had residents performing sit-to-stands every 2 hours, and arm curls or arm raises once a day. Participants in Jensen et al [30] at Dyer et al’s [31] trials performed progressive resistance training but did not describe which particular exercises participants performed. Similarly, Becker et al [36] described progressive resistance training with dumbbells and ankle weights but did not describe any specific exercises. Participants in the trial by Lord et al [34] performed strengthening exercises of ankle dorsiflexors, knee extensors, hip abductors, and hip side flexors. One trial stated their exercise program focused on muscle endurance rather than strength (eg, higher repetitions with lower resistance), and participants did grip ball exercises, ankle dorsiflexion and plantar flexion, upper limb elevation, knee elevation, and sit-to-stands [32].

 

 

Frequency, Time of Sessions, Duration of Program

In our description of positive trials, exercise was performed on 2 to 3 days per week for 20 to 75 minutes per session, for periods ranging from 4 to 52 weeks (Table 1).

Intensity

For the trials including balance exercises, one trial described the intensity as resident-specific [37] and another as individualized [33]. Two studies did not describe the intensity of their balance exercises [31,34]. The intensity of strength exercises included in the positive trials was individualized for one of the trial [29]. Two trials had participants complete 2 to 3 sets of 10 repetitions [32,35], with one indicating an intensity of 12–13 or “somewhat difficult” on the Borg Rating of Perceived Exertion Scale [32] and the other using a 10-rep max [35]. Two studies described their strength exercises as progressive [31,37], and one at a moderate to high intensity [30]. Lord et al prescribed 30 repetitions of each strength exercise [34].

Delivery of Intervention

Exercise was delivered in a group setting for 4 of the trials [31,32,34,36], individually for 2 of the trials [26,29], and the setting was not described for one of the trials (Table 1) [30]. Finally, only 3 of the 7 articles reported the professional delivering the intervention: one was research staff [29], one was geriatric nurses [32], and one was exercise assistants supported by a physiotherapist [31].

 

Discussion

There is limited evidence to support the use of strength and balance exercise as a single intervention to prevent falls in LTC. However, exercise should be included as part of a multifactorial falls prevention program. Trials that had a positive effect on decreasing falls training used dynamic balance exercises in standing, functional training, and progressive resistance training on 2 to 3 days per week, for 20 to 75 minutes per session, over 4 to 52 weeks. The intensity of balance exercises was individualized, and strength exercises were described as somewhat difficult or performed at a moderate to high intensity. Exercise was performed in a group or individually, and was delivered by research staff, geriatric nurses, exercise assistants supervised by physiotherapists, or more frequently, it was not reported who delivered the intervention.

Balance Training

Our work suggests that standing, dynamic balance exercises may be best to decrease falls. Example balance exercises include reducing the base of support (eg, standing with feet together instead of apart, or tandem with one foot in front), moving the center of gravity and control body position while standing (eg, reaching, weight shifting, stepping up or down), and standing without using arms for support or reducing reliance on the upper limbs for support (eg, use one hand on a handrail instead of two, or two fingers instead of the whole hand) [17]. It is well established that balance training programs, especially those including challenging exercises, can prevent falls in community-dwelling older adults [17]. However, the relationship is not as clear in LTC.

Strength Training

Reduced muscle strength has been identified as an important risk factor for falls [38]. There are also many psychological and metabolic benefits to strength training [39]. To induce change in muscular strength, resistance exercises need to be challenging and progressive. Our work suggests that strength training that is effective at decreasing falls is functional and progressive, and is completed at a moderate to high intensity. A resident should be able to do a strength exercise for one to two sets of 6 to 8 repetitions before being fatigued [40]. Once the resident can complete two sets of 13 to 15 repetitions easily the exercise should be progressed. Residents who are particularly deconditioned may need to begin with lower intensity strength exercises (eg, only do one set, with a lower resistance and progress to a higher resistance) [40]. Residents should perform resistance exercises for all major muscle groups [40]. Progression could include increasing the number of sets (eg, increase from one to two sets), the resistance (eg, holding dumbbells while squatting), or the intensity of the exercise (eg, squat lower or faster) [41].

Implementing Exercise Programs in LTC

Implementation of exercise programs into LTC homes should consider the dose of exercise (eg, time and frequency of sessions, duration of program), if they are delivered in a group or individual setting, and who is delivering them. First, trials included in this paper suggest that strength and balance exercises to prevent falls were delivered 2 to 3 times per week, for 20 to 75 minutes per session, over 4 to 52 weeks. Second, previous work has established that exercise programs delivered on 2 to 3 days per week over a period of more than 6 months are most effective at reducing falls in LTC [20]. Finally, a recent task force report from an international group of clinician researchers in LTC recommends twice weekly exercise sessions lasting 35 to 45 minutes each [40]. Therefore, strength and balance exercises to prevent falls in LTC should be delivered at least twice per week, for at least 20 minutes, for greater than 6 weeks’ duration.

 

 

Whether exercise should be performed in a group or individual setting remains unclear. Two of the 6 positive trials in this paper were completed individually, while 3 were in a group. The aforementioned task force also recommended that every resident who does not have contraindications to exercise must have an individualized exercise program as part of their health care plan [40]. However, whether the exercise program is provided on an individual basis or in a group setting was not delineated. Indeed, there are currently no recommendations concerning prioritizing group or individual exercise programs. Therefore, exercise programs being implemented into LTC homes should consider the residents’ preferences, the social benefits of group exercise, and the feasibility of individualizing exercises for the complex needs of residents in LTC in large group settings.

Finally, which professionals should deliver the exercise program is also uncertain. Only 3 of the positive trials in this paper described the professional delivering the intervention, with one being research staff, one geriatric nurses, and one exercise assistants supported by a physiotherapist. We suggest that professionals delivering an exercise program should be trained in exercise planning, delivery, and progression, be familiar with the principles of balance and strength training, and have training in working with older adults in LTC.

 

Modifications for Physical Impairments

Residents in LTC often have complex health needs, with multiple comorbidities (eg, stroke, Parkinson’s disease, multiple sclerosis) [21]. Modifications of strength and balance exercises may be required to accommodate for physical impairments (eg, hemiplegia, drop foot, freezing gait). For example, if a resident has hemiplegia and cannot fully activate the muscles of one arm, one can do resistance exercises with a dumbbell on the functioning side and active assisted range of motion (ie, the exercise provider assists the resident to achieve full range of motion against gravity) on the hemiparetic side. A resident with Parkinson’s disease who has freezing gait may need visual or rhythmical verbal cues to be able to accomplish standing balance tasks such as altered walking patterns (eg, wide or narrow stepping) [42].

Modifications for Cognitive Impairments

More than 80% of residents in LTC have some degree of cognitive impairment [21]. Cognitive impairment may be the result of stroke, depression, traumatic injuries, medications, and degenerative diseases such as Parkinson’s and Alzheimer’s disease [43]. A common misconception is that residents with cognitive impairment cannot benefit from exercise because they cannot learn new skills and have difficulty following directions. On the contrary, evidence suggests that exercise can improve functional mobility for residents with cognitive impairment [44,45].

Residents with cognitive impairment may require a different approach to facilitate participation in the desired exercises because of difficulty following multi-step directions, responsive behaviors, or increased distractibility [46]. Clear communication is key in improving the quality of interaction for residents with cognitive impairment. The Alzheimer Society of Ontario suggests 10 strategies for communicating with people with dementia [47], and we have provided suggestions of how to apply these communication strategies to the exercise context in LTC (Table 2). Other suggestions for engaging residents with cognitive impairment in strength and balance training include making the exercises functional (eg, ask them to pick something up of the floor to perform a squat, or reach a point on the wall to do calf raises) and playful (eg, toss a ball back and forth or sing a song about rowing to promote weight shifting) [48].

jcom02501028_t2.jpg

Standing versus Seated Exercises

Residents may not be able to participate in standing exercises for several reasons: perhaps the resident cannot stand or has severe balance impairments and a high falls risk; the resident may have poor insight into which exercises are safe to perform in standing versus sitting; or there may be limited supervision of a large group exercise class where the risk of falls is a concern. If balance impairments are a concern, where the risk of injury or falling while completing exercises in standing outweighs the benefit of doing the exercises, then seated exercises are appropriate. However, when residents are able, we recommend encouraging some or all exercises in standing, to facilitate carry over of strength gains into functional tasks such as being able to rise from a chair and walking. A recent study, comparing standing versus seated exercises for community dwelling older adults, saw greater functional gains for those who completed the standing exercises [49]. Therefore, strength and balance exercises should be performed in standing, where appropriate.

Resident-Centered Exercise for Falls Prevention

Putting the resident at the center of falls prevention is important. Previous work has found that older adults have expressed a strong preference for care that transcends traditional biomedical care and that values efficiency, consistency, and hierarchical decision making [50]. On the contrary, resident-centered care emphasizes well-being and quality of life as defined by the resident, values giving residents greater control over the nature of services they receive, and respects their rights to be involved in every day decision making [51,52]. Indeed, residents may choose to engage in risky behaviors that increase their risk of falls but also increases their quality of life. Previous work has found disconnects between residents’ perceived frailty and the potential ability of protective devices to prevent adverse events, such as falls and fractures [53]. Additionally, one study identified that older residents feared being labelled, so instead hid impairments and chose to refuse assistance and assistive devices [54]. For example, a resident with impaired balance and gait may choose to walk independently when they have been deemed as requiring a gait aid (eg, rollator walker). However, they may value walking without a gait aid and accept the increased risk of falling. Therefore, it is essential to find the delicate balance between respecting a resident’s right to make their own decisions and preventing adverse events, such as falls [52]. An example of this would be respecting a resident’s right to refuse to attend exercise programming even though the team may think they can benefit from strength and balance training.

 

 

There is limited evidence around falls prevention and resident-centered care. A recent systematic review [55] revealed that resident-centered care may increase falls rates [56,57]. However, the authors of the review attributed the increase in falls to differences in frailty between the control and intervention group [56], and to environmental factors (eg, slippery flooring material, lack of handrails) [57]. Additionally, these trials did not include an exercise program as part of the resident-centered care program. On the other hand, resident-centered care has been associated with reduction of boredom, helplessness, and depression [58,59]. Most studies included in the review were quasi-experimental, which significantly limits the evidence quality [55]. At this point in time, the evidence suggests that resident-centered care is important for mood and quality of life but may have a negative or no effect on reducing falls.

Multifactorial Falls Prevention Programs

While there are mixed results about the effect of exercise as a single intervention for reducing falls for residents in LTC, the literature clearly supports exercise as part of a multifactorial falls prevention program [17,20,60–62]. A 2015 umbrella review [62] of meta-analyses of randomized controlled trials of falls prevention interventions in LTC concluded that multifactorial interventions were the most effective at preventing falls in LTC. Additionally, recently developed recommendations for fracture prevention in LTC [61] suggest that balance, strength, and functional training should be included for residents who are not at high risk of fracture, while for those at high risk, exercise should be provided as part of a multifactorial falls prevention intervention. Clinicians must therefore incorporate elements aside from exercise into their falls prevention strategies. Interventions that have shown positive effects on reducing falls when delivered as part of multifactorial interventions include: staff and resident education [31,35,37], environmental modifications [31,35], supply/repair/provision of assistive devices [30], falls problem-solving conferences [30], urinary incontinence management [29], medication review [30], optician review [31], and cognitive behavioral therapy [32].

Conclusion and Suggestions for Clinical Practice

We suggest incorporating strength and balance exercises as part of a multifactorial falls prevention program for residents in LTC. Balance exercises should be challenging and dynamic (eg, weight shifting). Strength exercises should be of a moderate to high intensity (eg, can complete one to sets of 6 to 8 repetitions) and need to be progressed as the residents’ abilities improve. Residents should participate in strength and balance training on 2 to 3 days per week, for 30- to 45-minute sessions, for at least 6 months. Exercises in standing should be prioritized where appropriate. Exercise could be delivered in a group or individual format, but should consider the residents’ preferences, the social benefits of group exercise, and the feasibility of individualizing exercises for the complex needs of residents in LTC in large group settings. Professionals delivering an exercise program should be trained in exercise planning, delivery, and progression, be familiar with the principles of balance and strength training, and have training in working with older adults in LTC. Exercise programs in LTC should be resident-centered and consider residents’ potential physical and cognitive impairments.

Funding/support: Dr. Giangregorio was supported by grants from the Canadian Frailty Network and Canadian Institutes of Health Research.

References

1. Harris IA, Yong S, McEvoy L, Thorn L. A prospective study of the effect of nursing home residency on mortality following hip fracture. ANZ J Surg 2010;80:447–50.

2. Ooms ME, Vlasman P, Lips P, et al. The incidence of hip fractures in independent and institutionalized elderly people. Osteoporos Int 1994;4:6–10.

3. Ayoung-Chee P, McIntyre L, Ebel BE, et al. Long-term outcomes of ground-level falls in the elderly. J Trauma Acute Care Surg 2014;76:498–503.

4. Heinrich S, Rapp K, Rissmann U, et al. Cost of falls in old age: a systematic review. Osteoporos Int 2010;21: 891–902.

5. Rubenstein LZ, Josephson KR, Robbins AS. Falls in the nursing home. Ann Intern Med 1994;121:442–51.

6. Hartholt KA, van Beeck EF, Polinder S, et al. Societal consequences of falls in the older population: injuries, healthcare costs, and long-term reduced quality of life. J Trauma
2011;71:748–53.

7. Robinovitch SN, Feldman F, Yang Y, et al. Video capture of the circumstances of falls in elderly people residing in long-term care: an observational study. Lancet 2013;381:
47–54.

8. Rapp K, Becker C, Cameron ID, et al. Epidemiology of falls in residential aged care: analysis of more than 70,000 falls from residents of bavarian nursing homes. J Am Med Dir Assoc 2012;13:187.

9. Büchele G, Becker C, Cameron ID, et al. Predictors of serious consequences of falls in residential aged care: analysis of more than 70,000 falls from residents of Bavarian nursing homes. J Am Med Dir Assoc 2014;15:559–63.

10. McArthur C, Gonzalez DA, Roy E, Giangregorio L. What are the circumstances of falls and fractures in long-term care? Can J Aging / La Rev Can du Vieil 2016;35:491–8.

11. Chin A Paw MJM, van Poppel MNM, van Mechelen W. Effects of resistance and functional-skills training on habitual activity and constipation among older adults living in long-term care facilities: a randomized controlled trial. BMC Geriatr 2006;6:9.

12. Ikezoe T, Asakawa Y, Shima H, et al. Daytime physical activity patterns and physical fitness in institutionalized elderly women: an exploratory study. Arch Gerontol Geriatr 2013;57:221–5.

13. Senior HE, Henwood TR, Beller EM, et al. Prevalence and risk factors of sarcopenia among adults living in nursing homes. Maturitas 2015;82:418–23.

14. Smoliner C, Sieber CC, Wirth R. Prevalence of sarcopenia in geriatric hospitalized patients. J Am Med Dir Assoc 2014;15:267–72.

15. Landi F, Liperoti R, Fusco D, et al. Sarcopenia and mortality among older nursing home residents. J Am Med Dir Assoc 2012;13:121–6.

16. Yalcin A, Aras S, Atmis V, et al. Sarcopenia prevalence and factors associated with sarcopenia in older people living in a nursing home in Ankara Turkey. Geriatr Gerontol Int
2016;16:903–10.

17. Sherrington C, Michaleff ZA, Fairhall N, et al. Exercise to prevent falls in older adults: an updated systematic review and meta-analysis. Br J Sports Med October 2016.

18. Liu C, Latham NK. Progressive resistance strength training for improving physical function in older adults. In: Liu C, ed. Cochrane Database Syst Rev;2009:CD002759.

19. Cameron ID, Gillespie LD, Robertson MC, et al. Interventions for preventing falls in older people in care facilities and hospitals. Cochrane Database Syst Rev;2012:CD005465.

20. Silva RB, Eslick GD, Duque G. Exercise for falls and fracture prevention in long term care facilities: a systematic review and meta-analysis. J Am Med Dir Assoc 2013;14:685–9.

21. Hirdes JP, Mitchell L, Maxwell CJ, White N. Beyond the “iron lungs of gerontology”: Using evidence to shape the future of nursing homes in Canada. Can J Aging 2011;30: 371–90.

22. Benjamin K, Edwards N, Guitard P, et al. Factors that influence physical activity in long-term care: Perspectives of residents, staff, and significant others. Can J Aging 2011;30:247–58.

23. Benjamin K, Edwards N, Ploeg J, Legault F. Barriers to physical activity and restorative care for residents in long-term care: A review of the literature. J Aging Phys Act 2014;22:154–65.

24. American College of Sports Medicine. ACSM’s guidelines for exercise testing and prescription. 9th ed. American College of Sports Medicine; 2013.

25. Prevention of Falls Network Europe. Prevention of Falls Network Europe. Accessed 27 Nov 2017 at www.profane.eu.org/.

26. Sihvonen SE, Sipilä S, Era PA. Changes in postural balance in frail elderly women during a 4-week visual feedback training: a randomized controlled trial. Gerontology 2004;50:87–95.

27. Sakamoto K, Nakamura T, Hagino H, et al. Effects of unipedal standing balance exercise on the prevention of falls and hip fracture among clinically defined high-risk elderly individuals: a randomized controlled trial. J Orthop Sci 2006;11:467–72.

28. Shimada H, Obuchi S, Furuna T, Suzuki T. New intervention program for preventing falls among frail elderly people: the effects of perturbed walking exercise using a bilateral separated treadmill. Am J Phys Med Rehabil 2004;83:493–9.

29. Schnelle JF, Kapur K, Alessi C, et al. Does an exercise and incontinence intervention save healthcare costs in a nursing home population? J Am Geriatr Soc 2003;51:161–8.

30. Jensen J, Lundin-Olsson L, Nyberg L, Gustafson Y. Fall and injury prevention in older people living in residential care facilities: A cluster randomized trial. Ann Intern Med 2002;136:733–41.

31. Dyer CAE. Falls prevention in residential care homes: a randomised controlled trial. Age Ageing 2004;33:596–602.

32. Huang T-T, Chung M-L, Chen F-R, Chin Y-F, Wang B-H. Evaluation of a combined cognitive-behavioural and exercise intervention to manage fear of falling among elderly residents in nursing homes. Aging Ment Health 2016;20:2–12.

33. Sihvonen S, Sipilä S, Taskinen S, Era P. Fall incidence in frail older women after individualized visual feedback-based balance training. Gerontology 2004;50:411–6.

34. Lord SR, Castell S, Corcoran J, et al. The effect of group exercise on physical functioning and falls in frail older people living in retirement villages: a randomized, controlled trial. J Am Geriatr Soc 2003;51:1685–92.

35. Becker C, Kron M, Lindemann U, et al. Effectiveness of a multifaceted intervention on falls in nursing home residents. J Am Geriatr Soc 2003;51:306–13.

36. Becker C, Kron M, Lindemann U, et al. Effectiveness of a multifaceted intervention on falls in nursing home residents. J Am Geriatr Soc 2003;51:306–13.

37. Jensen J, Lundin-Olsson L, Nyberg L, Gustafson Y. Fall and injury prevention in older people living in residential care facilities. A cluster randomized trial. Ann Intern Med 2002;136:733–41.

38. Moreland JD, Richardson JA, Goldsmith CH, Clase CM. Muscle weakness and falls in older adults: a systematic review and meta-analysis. J Am Geriatr Soc 2004;52: 1121–9.

39. Chodzko-Zajko WJ, Proctor DN, Fiatarone Singh MA, et al. Exercise and physical activity for older adults. Med Sci Sport Exerc 2009;41:1510–30.

40. de Souto Barreto P, Morley JE, Chodzko-Zajko W, et al. Recommendations on physical activity and exercise for older adults living in long-term care facilities: a taskforce report. J Am Med Dir Assoc 2016;17:381–92.

41. American College of Sports Medicine. Progression models in resistance training for healthy adults. Med Sci Sport Exerc 2009;41:687–708.

42. Fietzek UM, Schroeteler FE, Ziegler K, et al. Randomized cross-over trial to investigate the efficacy of a two-week physiotherapy programme with repetitive exercises of cueing to reduce the severity of freezing of gait in patients with Parkinson’s disease. Clin Rehabil 2014;28:902–11.

43. Patterson C, Feightner J, Garcia A, MacKnight C. General risk factors for dementia: A systematic evidence review. Alzheimer Dement 2007;3:341–7.

44. Roach KE, Tappen RM, Kirk-Sanchez N, et al. A randomized controlled trial of an activity specific exercise program for individuals with alzheimer disease in long-term care settings. J Geriatr Phys Ther 2011;34:50–6.

45. Christofoletti G, Oliani MM, Gobbi S, et al. A controlled clinical trial on the effects of motor intervention on balance and cognition in institutionalized elderly patients with dementia. Clin Rehabil 2008;22:618–26.

46. van Alphen HJM, Hortobágyi T, van Heuvelen MJG. Barriers, motivators, and facilitators of physical activity in dementia patients: A systematic review. Arch Gerontol Geriatr 2016;66:109–18.

47. Alzheimer Society of Ontario. Rethink Dementia. Accessed 18 Sep 2017 at http://rethinkdementia.ca/.

48. Roach KE, Tappen RM, Kirk-Sanchez N, et al. A randomized controlled trial of an activity specific exercise program for individuals with Alzheimer disease in long-term care settings. J Geriatr Phys Ther 2011;34:50–6.

49. Brach JS, Perera S, Gilmore S, et al. Effectiveness of a timing and coordination group exercise program to improve mobility in community-dwelling older adults. JAMA Intern Med August 2017.

50. Rosher RB, Robinson S. Impact of the Eden alternative on family satisfaction. J Am Med Dir Assoc 2005;6:189–93.

51. Crandall LG, White DL, Schuldheis S, Talerico KA. Initiating person-centered care practices in long-term care facilities. J Gerontol Nurs 2007;33:47–56.

52. Sims-Gould J, McKay HA, Feldman F, et al. Autonomy, choice, patient-centered care, and hip protectors: the experience of residents and staff in long-term care. J Appl Gerontol 2014;33:690–709.

53. Robinovitch SN, Cronin T. Perception of postural limits in elderly nursing home and day care participants. J Gerontol A Biol Sci Med Sci 1999;54:B124-30.

54. Perkins MM, Ball MM, Whittington FJ, Hollingsworth C. Relational autonomy in assisted living: a focus on diverse care settings for older adults. J Aging Stud 2012;26:214–25.

55. Brownie S, Nancarrow S. Effects of person-centered care on residents and staff in aged-care facilities: a systematic review. Clin Interv Aging 2013;8:1–10.

56. Coleman MT, Looney S, O’Brien J, et al. The Eden Alternative: findings after 1 year of implementation. J Gerontol A Biol Sci Med Sci 2002;57:M422–7.

57. Chenoweth L, King MT, Jeon Y-H, et al. Caring for Aged Dementia Care Resident Study (CADRES) of personcentred care, dementia-care mapping, and usual care in dementia: a cluster-randomised trial. Lancet Neurol 2009;8: 317–25.

58. Bergman-Evans B. Beyond the basics. Effects of the Eden Alternative model on quality of life issues. J Gerontol Nurs 2004;30:27–34.

59. Robinson SB, Rosher RB. Tangling with the barriers to culture change: creating a resident-centered nursing home environment. J Gerontol Nurs 2006;32:19–25.

60. Cameron ID, Gillespie LD, Robertson MC, et al. Interventions for preventing falls in older people in care facilities and hospitals. Cochrane Database Syst Rev 2012;12.

61. Papaioannou A, Santesso N, Morin SN, et al. Recommendations for preventing fracture in long-term care. Can Med Assoc J 2015;187:1135–44.

62. Stubbs B, Denkinger MD, Brefka S, Dallmeier D. What works to prevent falls in older adults dwelling in long term care facilities and hospitals? An umbrella review of meta-analyses of randomised controlled trials. Maturitas 2015;81:335–42.

References

1. Harris IA, Yong S, McEvoy L, Thorn L. A prospective study of the effect of nursing home residency on mortality following hip fracture. ANZ J Surg 2010;80:447–50.

2. Ooms ME, Vlasman P, Lips P, et al. The incidence of hip fractures in independent and institutionalized elderly people. Osteoporos Int 1994;4:6–10.

3. Ayoung-Chee P, McIntyre L, Ebel BE, et al. Long-term outcomes of ground-level falls in the elderly. J Trauma Acute Care Surg 2014;76:498–503.

4. Heinrich S, Rapp K, Rissmann U, et al. Cost of falls in old age: a systematic review. Osteoporos Int 2010;21: 891–902.

5. Rubenstein LZ, Josephson KR, Robbins AS. Falls in the nursing home. Ann Intern Med 1994;121:442–51.

6. Hartholt KA, van Beeck EF, Polinder S, et al. Societal consequences of falls in the older population: injuries, healthcare costs, and long-term reduced quality of life. J Trauma
2011;71:748–53.

7. Robinovitch SN, Feldman F, Yang Y, et al. Video capture of the circumstances of falls in elderly people residing in long-term care: an observational study. Lancet 2013;381:
47–54.

8. Rapp K, Becker C, Cameron ID, et al. Epidemiology of falls in residential aged care: analysis of more than 70,000 falls from residents of bavarian nursing homes. J Am Med Dir Assoc 2012;13:187.

9. Büchele G, Becker C, Cameron ID, et al. Predictors of serious consequences of falls in residential aged care: analysis of more than 70,000 falls from residents of Bavarian nursing homes. J Am Med Dir Assoc 2014;15:559–63.

10. McArthur C, Gonzalez DA, Roy E, Giangregorio L. What are the circumstances of falls and fractures in long-term care? Can J Aging / La Rev Can du Vieil 2016;35:491–8.

11. Chin A Paw MJM, van Poppel MNM, van Mechelen W. Effects of resistance and functional-skills training on habitual activity and constipation among older adults living in long-term care facilities: a randomized controlled trial. BMC Geriatr 2006;6:9.

12. Ikezoe T, Asakawa Y, Shima H, et al. Daytime physical activity patterns and physical fitness in institutionalized elderly women: an exploratory study. Arch Gerontol Geriatr 2013;57:221–5.

13. Senior HE, Henwood TR, Beller EM, et al. Prevalence and risk factors of sarcopenia among adults living in nursing homes. Maturitas 2015;82:418–23.

14. Smoliner C, Sieber CC, Wirth R. Prevalence of sarcopenia in geriatric hospitalized patients. J Am Med Dir Assoc 2014;15:267–72.

15. Landi F, Liperoti R, Fusco D, et al. Sarcopenia and mortality among older nursing home residents. J Am Med Dir Assoc 2012;13:121–6.

16. Yalcin A, Aras S, Atmis V, et al. Sarcopenia prevalence and factors associated with sarcopenia in older people living in a nursing home in Ankara Turkey. Geriatr Gerontol Int
2016;16:903–10.

17. Sherrington C, Michaleff ZA, Fairhall N, et al. Exercise to prevent falls in older adults: an updated systematic review and meta-analysis. Br J Sports Med October 2016.

18. Liu C, Latham NK. Progressive resistance strength training for improving physical function in older adults. In: Liu C, ed. Cochrane Database Syst Rev;2009:CD002759.

19. Cameron ID, Gillespie LD, Robertson MC, et al. Interventions for preventing falls in older people in care facilities and hospitals. Cochrane Database Syst Rev;2012:CD005465.

20. Silva RB, Eslick GD, Duque G. Exercise for falls and fracture prevention in long term care facilities: a systematic review and meta-analysis. J Am Med Dir Assoc 2013;14:685–9.

21. Hirdes JP, Mitchell L, Maxwell CJ, White N. Beyond the “iron lungs of gerontology”: Using evidence to shape the future of nursing homes in Canada. Can J Aging 2011;30: 371–90.

22. Benjamin K, Edwards N, Guitard P, et al. Factors that influence physical activity in long-term care: Perspectives of residents, staff, and significant others. Can J Aging 2011;30:247–58.

23. Benjamin K, Edwards N, Ploeg J, Legault F. Barriers to physical activity and restorative care for residents in long-term care: A review of the literature. J Aging Phys Act 2014;22:154–65.

24. American College of Sports Medicine. ACSM’s guidelines for exercise testing and prescription. 9th ed. American College of Sports Medicine; 2013.

25. Prevention of Falls Network Europe. Prevention of Falls Network Europe. Accessed 27 Nov 2017 at www.profane.eu.org/.

26. Sihvonen SE, Sipilä S, Era PA. Changes in postural balance in frail elderly women during a 4-week visual feedback training: a randomized controlled trial. Gerontology 2004;50:87–95.

27. Sakamoto K, Nakamura T, Hagino H, et al. Effects of unipedal standing balance exercise on the prevention of falls and hip fracture among clinically defined high-risk elderly individuals: a randomized controlled trial. J Orthop Sci 2006;11:467–72.

28. Shimada H, Obuchi S, Furuna T, Suzuki T. New intervention program for preventing falls among frail elderly people: the effects of perturbed walking exercise using a bilateral separated treadmill. Am J Phys Med Rehabil 2004;83:493–9.

29. Schnelle JF, Kapur K, Alessi C, et al. Does an exercise and incontinence intervention save healthcare costs in a nursing home population? J Am Geriatr Soc 2003;51:161–8.

30. Jensen J, Lundin-Olsson L, Nyberg L, Gustafson Y. Fall and injury prevention in older people living in residential care facilities: A cluster randomized trial. Ann Intern Med 2002;136:733–41.

31. Dyer CAE. Falls prevention in residential care homes: a randomised controlled trial. Age Ageing 2004;33:596–602.

32. Huang T-T, Chung M-L, Chen F-R, Chin Y-F, Wang B-H. Evaluation of a combined cognitive-behavioural and exercise intervention to manage fear of falling among elderly residents in nursing homes. Aging Ment Health 2016;20:2–12.

33. Sihvonen S, Sipilä S, Taskinen S, Era P. Fall incidence in frail older women after individualized visual feedback-based balance training. Gerontology 2004;50:411–6.

34. Lord SR, Castell S, Corcoran J, et al. The effect of group exercise on physical functioning and falls in frail older people living in retirement villages: a randomized, controlled trial. J Am Geriatr Soc 2003;51:1685–92.

35. Becker C, Kron M, Lindemann U, et al. Effectiveness of a multifaceted intervention on falls in nursing home residents. J Am Geriatr Soc 2003;51:306–13.

36. Becker C, Kron M, Lindemann U, et al. Effectiveness of a multifaceted intervention on falls in nursing home residents. J Am Geriatr Soc 2003;51:306–13.

37. Jensen J, Lundin-Olsson L, Nyberg L, Gustafson Y. Fall and injury prevention in older people living in residential care facilities. A cluster randomized trial. Ann Intern Med 2002;136:733–41.

38. Moreland JD, Richardson JA, Goldsmith CH, Clase CM. Muscle weakness and falls in older adults: a systematic review and meta-analysis. J Am Geriatr Soc 2004;52: 1121–9.

39. Chodzko-Zajko WJ, Proctor DN, Fiatarone Singh MA, et al. Exercise and physical activity for older adults. Med Sci Sport Exerc 2009;41:1510–30.

40. de Souto Barreto P, Morley JE, Chodzko-Zajko W, et al. Recommendations on physical activity and exercise for older adults living in long-term care facilities: a taskforce report. J Am Med Dir Assoc 2016;17:381–92.

41. American College of Sports Medicine. Progression models in resistance training for healthy adults. Med Sci Sport Exerc 2009;41:687–708.

42. Fietzek UM, Schroeteler FE, Ziegler K, et al. Randomized cross-over trial to investigate the efficacy of a two-week physiotherapy programme with repetitive exercises of cueing to reduce the severity of freezing of gait in patients with Parkinson’s disease. Clin Rehabil 2014;28:902–11.

43. Patterson C, Feightner J, Garcia A, MacKnight C. General risk factors for dementia: A systematic evidence review. Alzheimer Dement 2007;3:341–7.

44. Roach KE, Tappen RM, Kirk-Sanchez N, et al. A randomized controlled trial of an activity specific exercise program for individuals with alzheimer disease in long-term care settings. J Geriatr Phys Ther 2011;34:50–6.

45. Christofoletti G, Oliani MM, Gobbi S, et al. A controlled clinical trial on the effects of motor intervention on balance and cognition in institutionalized elderly patients with dementia. Clin Rehabil 2008;22:618–26.

46. van Alphen HJM, Hortobágyi T, van Heuvelen MJG. Barriers, motivators, and facilitators of physical activity in dementia patients: A systematic review. Arch Gerontol Geriatr 2016;66:109–18.

47. Alzheimer Society of Ontario. Rethink Dementia. Accessed 18 Sep 2017 at http://rethinkdementia.ca/.

48. Roach KE, Tappen RM, Kirk-Sanchez N, et al. A randomized controlled trial of an activity specific exercise program for individuals with Alzheimer disease in long-term care settings. J Geriatr Phys Ther 2011;34:50–6.

49. Brach JS, Perera S, Gilmore S, et al. Effectiveness of a timing and coordination group exercise program to improve mobility in community-dwelling older adults. JAMA Intern Med August 2017.

50. Rosher RB, Robinson S. Impact of the Eden alternative on family satisfaction. J Am Med Dir Assoc 2005;6:189–93.

51. Crandall LG, White DL, Schuldheis S, Talerico KA. Initiating person-centered care practices in long-term care facilities. J Gerontol Nurs 2007;33:47–56.

52. Sims-Gould J, McKay HA, Feldman F, et al. Autonomy, choice, patient-centered care, and hip protectors: the experience of residents and staff in long-term care. J Appl Gerontol 2014;33:690–709.

53. Robinovitch SN, Cronin T. Perception of postural limits in elderly nursing home and day care participants. J Gerontol A Biol Sci Med Sci 1999;54:B124-30.

54. Perkins MM, Ball MM, Whittington FJ, Hollingsworth C. Relational autonomy in assisted living: a focus on diverse care settings for older adults. J Aging Stud 2012;26:214–25.

55. Brownie S, Nancarrow S. Effects of person-centered care on residents and staff in aged-care facilities: a systematic review. Clin Interv Aging 2013;8:1–10.

56. Coleman MT, Looney S, O’Brien J, et al. The Eden Alternative: findings after 1 year of implementation. J Gerontol A Biol Sci Med Sci 2002;57:M422–7.

57. Chenoweth L, King MT, Jeon Y-H, et al. Caring for Aged Dementia Care Resident Study (CADRES) of personcentred care, dementia-care mapping, and usual care in dementia: a cluster-randomised trial. Lancet Neurol 2009;8: 317–25.

58. Bergman-Evans B. Beyond the basics. Effects of the Eden Alternative model on quality of life issues. J Gerontol Nurs 2004;30:27–34.

59. Robinson SB, Rosher RB. Tangling with the barriers to culture change: creating a resident-centered nursing home environment. J Gerontol Nurs 2006;32:19–25.

60. Cameron ID, Gillespie LD, Robertson MC, et al. Interventions for preventing falls in older people in care facilities and hospitals. Cochrane Database Syst Rev 2012;12.

61. Papaioannou A, Santesso N, Morin SN, et al. Recommendations for preventing fracture in long-term care. Can Med Assoc J 2015;187:1135–44.

62. Stubbs B, Denkinger MD, Brefka S, Dallmeier D. What works to prevent falls in older adults dwelling in long term care facilities and hospitals? An umbrella review of meta-analyses of randomised controlled trials. Maturitas 2015;81:335–42.

Issue
Journal of Clinical Outcomes Management - 25(1)
Issue
Journal of Clinical Outcomes Management - 25(1)
Publications
Publications
Topics
Article Type
Sections
Disallow All Ads
Content Gating
No Gating (article Unlocked/Free)
Alternative CME
Disqus Comments
Default
Use ProPublica
Hide sidebar & use full width
render the right sidebar.
Article PDF Media
Image
Teambase ID
18000D03.SIG
Disable zoom
Off